You are on page 1of 401

ii  Theory of Counting

• Head Office : B-32, Shivalik Main Road, Malviya Nagar, New Delhi-110017

• Sales Office : B-48, Shivalik Main Road, Malviya Nagar, New Delhi-110017
Tel. : 011-26691021 / 26691713

Price : ` 280

Typeset by Disha DTP Team

DISHA PUBLICATION
All RightS Reserved

© Copyright Publisher
No part of this publication may be reproduced in any form without prior permission of the
publisher. The author and the publisher do not take any legal responsibility for any errors
or misrepresentations that might have crept in. We have tried and made our best efforts to
provide accurate up-to-date information in this book.

For further information about the books from DISHA,


Log on to www.dishapublication.com or email to info@dishapublication.com
On the Cards  iii
PREFACE
In my teaching career, whenever I asked my students “Which part of mathematics are you afraid of?”
most of the time I got the answer “Permutation and Combination/ Probability”. In various cases, we see
decisions depend on chances, which in turn depend on various factors. In a situation when certain choices
are made to get to a final selection of a desired outcome, technically it is then where the concept of Per-
mutation & Combination and Probability comes to light.
The book “Theory of Counting” delves deep into the basics of Permutation & Combination and Probability
then advances into the IIT JEE level and ends up high with the International Maths Olympiad level.
Each chapter has been well explained and illustrated with numerous examples for the better under-
standing of the reader. To be precise most of the time students get confused between Permutation and
Combination, so I thought to make it easier by eliminating Permutation. Just concentrate on Selection/
Combination and you can solve problems easily.
Each Chapter is divided in two parts -
Part A: Basic, concept building and Board level Questions
Part B: Nightmare, IIT JEE and Maths Olympiad Questions
It is advisable to students that before attempting Part B of any Chapter please solve Part A of all the
chapters.
Quantitative problems in this book has been divided into 5 difficulty levels viz.
• Concept Applicator: Basic concept testing questions.
• Concept Builder: Concept strengthening questions.
• Concept Cracker: Challenging level questions.
• Concept Deviator: IIT JEE equivalent level questions.
• Concept Eliminator: International Maths Olympiad standard questions.
The best bet is that this book is self-sufficient in itself. For the aspirants of any competitive examina-
tion like IIT JEE Main & Advanced, RMO and also Board exams etc. their quest for an overall adaptive
book ends here. Once the reader is able to successfully complete this book, he can be rest assured of his
excellent performance on the exam day. What’s more is that this is India’s first book with video links/
QR codes.
I would like to express my sincere thanks to Mr. Deepak Agarwal, Founder-Director of Disha Publication
for consistently guiding me to bring this book as immaculate.
To understand the fact that a book is not an individual work, it incorporates the efforts of other people, I
am thankful to all my students who have helped me throughout the compiling of this book and also proof-
reading it. These include but not limited to Kushal Dey, Bhupesh Kumar Srivastava, Abhishek Gourav,
Ashish Jha, Gaurav Jain and Rahul Roy.
Last but not the least, I would like to express my sincere gratitude to my father (my only Maths teacher),
mother, siblings and have no words for my better half ‘Suvra’, who is my inspiration and partner in all
ventures. Finally, thanks to the Hero of my life, my son – ‘Sanyukt’ (meaning summation in Math). He is
the great source of inspiration for this book.
I have taken extreme care while writing the theories, compiling the questions and preparing the solu-
tions, however, some errors may have crept in, so hereby I invite all readers to bring it to our notice.
Gajendra Kumar
iv  Theory of Counting

index
1. Introduction 1 - 34

2. Selection (Combination) 35 - 70

3. Geometrical Construction 71 - 102

4. Arrangement 103 - 124

5. Circular Arrangement 125 - 144

6. Word/ Number Formation 145 - 180

7. Distribution 181 - 206

8. Number of Integral Solution 207 - 238

9. Probability Introduction 239 - 264

10. Dependent and independent Events 265 - 290

11. Conditional Probability 291 - 310

12. Binomial Probability 311 - 332

13. On the Cards 333 - 348

14. Probability Miscellaneous 349 - 370

15. Puzzles 371 - 396


1
Introduction  1

Introduction

Topics Covered
 Factorials
 Highest power of a number in N!
 1st Non-zero, 1st two non-zero digits in N!
 Fundamental principle of counting
 Properties of nCr and nPr
 Properties of Factorials

Part A: Topic Number of Questions


Solved Example 25
Concept Applicator 20
Concept Builder 20
Concept Cracker 20
Part B: Topic Number of Questions
Concept Deviator 16
Concept Eliminator 10
Total 111
2  Theory of Counting
Part A

Concept 1 Introduction
Factorial of a natural number is defined as between largest and smallest possible
the products of all the integers from 1 to n. It value of a.
is represented as ! or ∠. Solution:  Since summation of two factorial
From the definition 1! = 1 is a three digit number hence maximum
possible value of a is 6 while minimum
2! = 1.2 = 2
possible value is 0 hence required difference
3! = 1 × 2 × 3 = 6 is 6 – 0 = 6
4! = 1 × 2 × 3 × 4 = (3!)(4) = 24
Example 2:  Find the unit digit of summation
5! = 1 × 2 × 3 × 4 × 5 = (3!)(4 × 5) = (4!)(5) = 120 of 1! + 2! + 3! + 4! + 5! + … + 10!.

6! = 1 × 2 × 3 × 4 × 5 × 6 = (4!)(5 × 6) = (5!)(6) Solution:  Since unit digit of all the factorial


= 720 from 5! is 0, hence required unit digit is given
Or N! = 1 × 2 × 3 × 4 × 5 × ..(N – 2) × (N – 1) × (N) by 1! + 2! + 3! + 4! Which is 1 + 2 + 6 + 24 and
= (N – 1)! × N unit digit of this is 3.

0! is defined as 1 and we will see the proof of Example 3:  Find the unit digit of product
this at the end of this chapter. of 1! × 2! × 3! × …. × 10!

Example 1:  If (a!) + (b!) is a three digit Solution:  Since unit digit of 5! is 0 hence
number then find the difference required unit digit of product is 0.

QR Code / Video Link

For Smart phone/ Tablet users

Video Link for Desktop/Laptops users http://dishapublication.com/video-resources


Introduction  3

2 Highest power of a
Concept number in N!
Most of the questions in the numbers related to Example 5:  Find highest power of 3 in
factorial are of the type to find out the largest 100!.
power of a number in a factorial n (n!). Solution:  The required power is given by
Highest power of a PRIME NUMBER
p that divides n! exactly that means  100   100   100   100 
 3  +   2  +  3  +  4 
without leaving any remainder is given by 3  3  3 
n  n   n  = 33 + 11 + 3 + 1 = 48
 p  +  2  +  3  ……. where, [x] represents Another way to represent the same concept is
   p   p 
described as below –
greatest integer function, that means greatest
integer less than or equal to x. We will see 3 100
that with some examples.
3 33 … 1st quotient
Example 4:  Find the largest power of 5 3 11 … 2nd quotient
that divides 27! Perfectly that means
without leaving any remainder. 3 3 … 3rd quotient
Solution:  The required number of powers is 1 … 4th quotient
 27   27 
given by   +    =  5 + 1 = 6 . and last quotient since 1 < 3. So highest
 5   52 
power of 3 in 100! = 33 + 11 + 3 + 1= 48.
The same question can be solved by another
method. Let we need to find highest power But above rule is valid only for prime
numbers not for composite numbers. If we
of a prime number p in factorial n (n!). then
need to find the same for composite number
1st divide n/ p, we will get some quotient, then 1st convert that composite number as
divide that quotient again by p and continue product of primes and then find out the
the process till we get quotient less than highest power of each prime numbers in
the p. then add all the quotients that will that factorial, and least of those highest
give us the required number of power of p powers is the required answer. We will see
that divide n!. some examples –
QR Code / Video Link

For Smart phone/ Tablet users

Video Link for Desktop/Laptops users http://dishapublication.com/video-resources


4  Theory of Counting
Example 6:  Find highest power of 6 in 200! Example 7:  Find highest power of 15 in
Solution:  Here given number 6 is not a prime 150!
number so 1st convert that as product of Solution:  Since 15= 3 × 5 and 5 > 3 so highest
primes, power of 5 is same as that of 15 in 150!. So
6 = 2 × 3, so now we will find out the highest we will find out the highest power of 5 in
powers of 2 and 3. 100!.
To find highest power of 2 in 200! 5 150
2 200 5 30 … 1st quotient
2 100 … 1st quotient 5 6 … 2nd quotient
2 50 … 2nd quotient 1 … 3rd quotient

2 25 … 3rd quotient So highest power of 5 or 15 in 150!
2 12 … 4th quotient = 30 + 6 + 1= 37.
Consider a number 5 and its highest power
2 6 … 5th quotient
in N!
2 3 … 6th quotient If 0 ≤ N < 5 highest power of 5 (or number of
1 … last quotient zeros) is 0
If 5 ≤ N < 10 highest power of 5 (or number of
So highest power of 2 in 200! is 100 + 50 + 25 zeros) is 1
+ 12 + 6 + 3 + 1= 197
If 10 ≤ N < 15 highest power of 5 (or number
Similarly highest power of 3 in 200! is of zeros) is 2
3 200 If 15 ≤ N < 20 highest power of 5 (or number
of zeros) is 3
3 66 … 1st quotient
If 20 ≤ N < 25 highest power of 5 (or number
3 22 … 2nd quotient of zeros) is 4
3 7 … 3rd quotient If 25 ≤ N < 30 highest power of 5 (or number
of zeros) is 6
2 … 4th quotient If 30 ≤ N < 35 highest power of 5 (or number

of zeros) is 7
So highest power of 3 in 200! is 66 + 22 + 7 +
2 = 97, If number is a multiple of 5 then highest
power of 5 will increase by 1,
So 200! Contains (2)197 × (3) 97 or it contains
97 pairs of 2 and 3 hence required power is If number is a multiple of 52 then highest
(6)97 or directly we can say the smaller in power of 5 will increase by 2,
between the two powers is the answer. If number is a multiple of 53 then highest
For a composite number, convert the number power of 5 will increase by 3,
as product of primes and find the highest If number is a multiple of 5k then highest
power of the prime factors then highest power power of 5 will increase by k,
of prime (or its power), lowest of these values So if N is a multiple of ak and if highest
will give us the highest power of composite power of ‘a’ in (N – 1)! is ‘p’ then highest
number. power of ‘a’ in N! is ‘p + k’.
Introduction  5

Example 8:  If highest power of 8 in N! is Solution:  As we discussed above the highest


19 find highest power of 8 in (N + 1)!. power of 10 in 120! is same as that of 5. So
Solution:  Since highest power of 8 in N! is 19 we will find the highest power of 5 in 120!
hence that of 2 in N! is 57, 58 or 59, 5 120
Since highest power of 2 in 62! and 63! is
57 and that in 64! is 63, and hence N = 62 5 24 … 1st quotient
or 63. 4 … 2nd quotient
If N = 62 then N + 1 = 63 and highest power
hence highest power of 5 in 120! is
of 2 is 57 and that of 8 is 19,
24 + 4 = 28. So highest power of 10 or
If N = 63 then N + 1 = 64 and highest power
number of zeros in 120! = 28.
of 2 is 63 and that of 8 is 21,
Number Of Zeros In A Number Example 10:  Find the highest power of
Number of zeros in a number is same as the 100 in 100!.
highest power of 10 in that number, and we Solution:  Since 100 = 22 × 52 hence, we
know that 10 = 2 × 5 then we needs to find the will find out highest power of 2 and
highest power of 2 & 5, and smaller in the two that of 5, since highest power of 5
is equal to highest powers of 10. Since, 5 > 2
has to be less than that of 2 hence we
hence we will find highest power of 5, as highest
power of 2 must be more than that of 5. will calculate only highest power of 5.
Highest power of 5 in 100! is 24 hence that
Example 9:  Find the highest power of 10 of 52 is 12, so highest power of 100 in 100!
in 120! or the number of trailing zeros Is 12.
in the 120!

Concept 3 1st Non-Zero Digit in N!


Consider 5! = 120 here last digit is 0 and Example 11:  Find the 1st non-zero digit of
1stnon-zero digit is 2. Then consider 10! = 34!
3628800 it ends with 2 zeros and then 1st Solution:  34! = 1 × 2 × 3 × 4 × 5 × 6 × 7 × 8 ×
non-zero digit is 8. 9 × 10 × 11 × …….31 × 32 × 33 × 34
If we have to find 1st non-zero digit of factorial We will segregate it in a group of 5 then we
of big numbers then we have to find an easy will get
method. 34! = (1 × 2 × 3 × 4 × 5) × (6 × 7 × 8 × 9 × 10) ×
Base Rule : – Unit digit of (5k + 1)(5k + 2) (11 × 12 × 13 × 14 × 15) × (16 × 17 × 18 × 19 ×
(5k + 3)(5k + 4) = U[4!] 20) …. (31 × 32 × 33 × 34)
Consider 1st group 1 × 2 × 3 × 4 × 5 = 24 × 5
Let us understand this by some
= 12 × (10) × (1) or 1st non-zero unit digit is
examples.
2×1
6  Theory of Counting
2nd group (6 × 7 × 8 × 9 × 10) = (3024 × 10) = Hence 1st non-zero digit will be given by the
1512 × (10) × (2) or 1st non-zero unit digit is product of
2×2 (2 × 1) × (2 × 2) × (2 × 3) × … × (2 × 18) × (2!)
3rd group 11 × 12 × 13 × 14 × 15 = 24024 ×
= Unit digit of U[218] × U[18!] × U[2!]
15 = 12012 × (2 × 5) × (3) or 1stnon-zero unit
digit is 2 × 3 Now from similar concept U[18!] = U[23] ×
Similarly, 2nd last group is (26 × 27 × 28 × 29 U[3!] × [3!] = U[8 × 6 × 6] = 8
× 30) = or1stnon-zero unit digit is 2 × 6 Hence non-zero unit digit of 92! = U[218] ×
And then last group is (31 × 32 × 33 × 34) unit U[18!] × U[2!] = U[4 × 8 × 2] = 4
digit is that of 4! Now from formula if we divide 92/5 quotient
Hence 1st non-zero digit will be given by the is Q = 18 and remainder is R = 2 then
product of U[92!] = U[2Q] × U[Q!] × U[R!] = U[218] ×
(2 × 1) × (2 × 2) × (2 × 3) × ….. × (2 × 6) × (4!) U[18!] × U[2!] = U[4 × 8 × 2] = 4
= Unit digit of U[26] × U[6!] × U[4!]
= Unit digit of 4 × 2 × 4 = 2 A New Approach to Find 1st Non-Zero Digit
Now lets sum up the concept by a formula Let highest power of 5 in N! is P and R1, R2,
Here we need to find unit digit of N! etc are remainders when N is successively
Divide N by 5 and let quotient is Q and divided by 5
remainder is R then unit digit of N! = U[N!] = U[2P] × U[R1!] × U[R2!] ….
U[2Q] × U[Q!] × U[R!]
Lets take one more example We will understand this by taking one
example: –
Example 12:  Find the 1st non-zero digit
of 92! Example 13:  Find 1st non-zero digit of 14!
Solution:  92! = 1 × 2 × 3 × 4 × 5 × 6 × 7 × 8 × 9 Solution:  With normal method we will find
× 10 × 11 × … 87 × 88 × 89 × 90 × 91 × 92 highest power of 5 in 14!
We will make it in a group of 5 then we will get
92! = (1 × 2 × 3 × 4 × 5) × (6 × 7 × 8 × 9 × 10) × 5 14
(11 × 12 × 13 × 14 × 15) × (16 × 17 × 18 × 19 × 5 2 → R1 = 4
20) …. (85 × 86 × 87 × 88 × 90) × (91 × 92)
Consider 1st group 1 × 2 × 3 × 4 × 5 = 24 × 5 0 → R2 = 2

= 12 × (10) × (1) or non-zero unit digit is 2 × 1
Hence highest power of 5 in 14! is P = 2 + 0 = 2
2nd group (6 × 7 × 8 × 9 × 10) = (3024 × 10) =
1512 × (10) × (2) or non-zero unit digit is 2 × 2 So U(14!) = U[22] × U[4!] × U[2!] × U[0!]
3rd group 11 × 12 × 13 × 14 × 15 = 24024 × 15
= U[4 × 4 × 2 × 1] = 2
= 12012 × (2 × 5) × (3) or non-zero unit digit
is 2 × 3
Similarly 2nd last group is (86 × 87 × 88 × 89 × 90) Example 14:  Find 1stnon-zero digit of 137!
= or non-zero unit digit is 2 × 18
Solution:  With normal method we will find
And then last group is (91 × 92) unit digit is highest power of 5 in 137!
2 or that of 2!
Introduction  7
Solution: 
5 137
5 27 → R1 = 2 5 222

5 5 → R2 = 2 5 44 → R1 = 2

5 1 → R3 = 0 5 8 → R2 = 4

0 → R4 = 1 5 1 → R3 = 3

0 → R4 = 1
Hence highest power of 5 in 137! is
Hence highest power of 5 in 222! is P = 44
P = 27 + 5 + 1 + 0 = 33
+ 8 + 1 + 0 = 53
So U(137!) = U[233] × U[2!] × U[2!] × U[0!] × Hence U[222!] = U[253] × U[2!] × U[4!] × U[3!]
U[1!] × U[0!] = U[2 × 2 × 2 × 1 × 1 × 1] = 8
× U[1!] × U[0!]
Example 15:  Find 1stnon-zero digit of 222! = U[2 × 2 × 4 × 6 × 1 × 1] = 6

1st Two Non-Zero


Concept 4 Digit in N!
Consider 5! = 120 here last digit is 0 and Consider 1st group 1 × 2 × 3 × 4 × 5 = 24 × 5
1st two non-zero digit is 2. Then consider = 12 × (10) × (1) or 1st two non-zero unit digit
10! = 3628800 it ends with 2 zeros and then is 12 × 1
1st two non-zero digits is 88. 2nd group (6 × 7 × 8 × 9 × 10) = (3024 × 10)
If we have to find 1sttwo
non-zero digit of = 1512 × (10) × (2) or 1st 2 non-zero unit digit
factorial of big numbers then we have to find is 12 × 2
an easy method.
3rd group 11 × 12 × 13 × 14 × 15 = 24024 × 15
Example 16:  Find 1sttwo non-zero digit of = 12012 × (2 × 5) × (3) or 1st 2 non-zero unit
34! digit is 12 × 3
Solution:  Consider 34! 4th group (16 × 17 × 18 × 19 × 20) = (93024 ×
34! = 1 × 2 × 3 × 4 × 5 × 6 × 7 × 8 × 9 × 10 × 20) = 46512 × (10) × (4) or 1st 2 non-zero unit
11 × …….31 × 32 × 33 × 34 digit is 12 × 4
We will make it in a group of 5 then we will get Similarly 2nd last group is 4th group (26 × 27
34! = (1 × 2 × 3 × 4 × 5) × (6 × 7 × 8 × 9 × 10) × × 28 × 29 × 30) = (570024 × 30) = (570024 × 5
(11 × 12 × 13 × 14 × 15) × (16 × 17 × 18 × 19 × × 6) = 285012 × 10 × 6 or 1st 2 non-zero unit
20) …. (31 × 32 × 33 × 34) digit is 12 × 6
8  Theory of Counting
And then last group is (31 × 32 × 33 × 34) and Solution:  With normal method we will find
1st 2 unit digit is 1113024 or 24. highest power of 5 in 137!
Hence 1st 2 non-zero digit will be given by the
5 137
product of
(12 × 1) × (12 × 2) × (12 × 3) × ….. × (12 × 6) × (24) 5 27 → R1 = 2 Q1 = T[136 × 137] = 32
Hence required 1st 2 non zero digit is T[126] × 5 5 → R2 = 2 Q2 = T[26 × 27] = 02
T [6!] × (24) = T[84 × 72 × 24] = 52
1 → R3 = 0
Divide N by 5 and let quotient is Q and remainder
is R then 1st two non-zero digit of N!
Hence highest power of 5 in 137! is P = 27 +
= T[12Q] × T[Q!] × T[ N × (N – 1)…R terms] 5 + 1 = 33
Now lets use this concept to find 1st 2 non- So T(137!) = T[1233] × T[32 × 02]
zero digit of 92!
= T[72 × 64] = 08
When 92 divided by 5 quotient is 18 and
remainder is 2, hence Example 19:  Find 1st two non-zero digit
of 222!
T[92!] = T[1218] × T[18!] × T[91 × 92] = T [04
× 28 × 72] = 64 Solution: 
Example 17:  Find 1st two non-zero digit
5 222
of 137!
Solution:  if we divide 137/5 then quotient is 5 44 → R1 = 2 Q1= T[221 × 222] = 62
Q = 27, and remainder R = 2 5 8 → R2 = 4 Q2 = T[41×42×43×44]=24
Hence, T[137!] = T[1227] × T[27!] × T[136
1 → R3 = 3 Q3 = T[6 × 7 × 8] = 36
× 137]
1st term is T[1227] = T[ {254} × {327}] Hence highest power of 5 in 222! is P = 44 +
= T[{24 × 16} × {( 816)(33)}] 8 + 1 = 53
Hence, T[222!] = T[1253] × T[62 × 24 × 36]
= T[84 × 81 × 27] = 08
2nd term is T[27!] = T[125] × T[5!] = T[72 × 68] = 96
× T [26 × 27] = T[32 × 12 × 26 × 27] = 68 Example 20:  Find 1sttwo non-zero digit of
3rd term is T[136 × 137] = T[36 × 37] = 32 444!
Hence T[137!] = T[08 × 68 × 32] = 08 Solution: 

A new approach to find last 2 5 444


non- zero digit of N!
5 88 → R1=4; Q1= T[441×442×443×444]=24
Let highest power of 5 in N! is P and sum of
all the remainders of [N/5] + [N/52] + .. is R1, 5 17 → R2=3; Q2 = T[86 × 87 × 88] = 16
R2, etc then 3 → R3=2; Q3 = T[16 × 17] = 72
T[N!] = T[12P] × T[Q1 × Q2…]
→ R4=3; Q4 = T[1 × 2 × 3] = 6
Here Q1, Q2, are defined as below with example
Hence highest power of 5 in 444! is P = 88 +
We will understand this by taking one 17 + 3 = 108
example:
Hence, T[444!] = T[12108] × T[24 × 16 × 72 × 6]
Example 18:  Find 1st two non-zero digit
= T[96 × 88] = 48
of 137!
Introduction  9

Fundamental
Concept 5 Principle of Counting
Fundamental principle of summation: If her food if she is allowed to have either
a work A can be done in m ways and another Indian or Chinese food.
work B can be done in n ways then work A or Solution:  Number of ways that she can select
B can be done in m + n ways. Indian food is 10 and number of ways that
Consider total n works, if 1st work can be she can select Chinese food is 12 but she
done in a1 ways, 2nd work can be done in a2 is allowed to have either Indian or Chinese
ways, and so on nth can be done in an ways food hence number of ways that she can
then work a1 or a2 or …or an can be done in select food is 10 + 12 = 22 ways.
a1 + a2 + … + an ways. Fundamental principle of multiplication:
If a work A can be done in m ways and another
Example 21:  If Suvro has one sandal,
work B can be done in n ways then work A &
one formal shoe, one sports shoe,
B can be done in m × n ways.
in how many ways he can select his
footwear? Consider total n works, if 1st work can be
done in a1 ways, 2nd work can be done in a2
Solution:  Since a person can wear only one
ways, and so on nth can be done in an ways
footwear at a time, so he can wear either
then work a1, a2, …,an–1, an can be done in
sandal or formal shoe or sports shoe, that
a1 × a2 × … × an ways.
means he has total 3 ways to select his foot
wear. Example 23:  If Rajesh has 4 formal shirts
and 5 formal trousers then in how
Example 22:  Trina went to a famous many ways he can wear his dress?
restaurant for dinner, there in menu
Solution:  Rajesh can select shirts in 4 ways
card she saw 10 Indian and 12 Chinese
and trousers in 5 ways hence number of
food, in how many ways she can select
ways he can select his dress is 4 × 5 = 20

QR Code / Video Link

For Smart phone/ Tablet users

Video Link for Desktop/Laptops users http://dishapublication.com/video-resources


10  Theory of Counting
Example 24:  In a hostel mess, for breakfast Solution:  Student can select breakfast in 4
4 different items, for lunch 3 different ways, lunch in 3 ways and dinner in 5 ways
items and for dinner 5 different items so required number of ways is 4 × 3 × 5 =
was available, In how many different 60 ways.
ways a student can select food?

Concept 6 Properties of nCr and nPr


n!
From the definition nCr = nC + nCr = n + 1Cr
(n − r )!(r !) r–1

nC = nCy, then x = y or x + y = n
n! x
And nPr = = (r!)(nCr)
(n − r )! nC
r = nCn – r
Some other properties of binomial
coefficients (nCr)/(r + 1) = nCr + 1/(n – r)
nC + nC1 + nC2 + … + nCn nP
0 Example 25:  If 3= 1716, then find the
= (1 + 1)n = 2n value of n

nC nC + nC2 + … ( – 1)n – 1nCn


0– 1 n!
Solution:  Since, nPr =
= (1 – 1)n = 0
(n − r )!
nC + nC2 + nC4 + … n!
0 Hence, =  n (n − 1)(n − 2 ) = 1716
(n − 3)!
= nC1 + nC3 + nC5 + …. = 2n – 1
= 11 × 12 × 13 hence n = 13
Introduction  11

1
Concept Applicator (CA)
Ideal Time Apply your concepts with easy and
20 Min. conceptual questions

1. If a! + b! + c! + d! + e! is a two digit number 12!


find the maximum value of e. 7. Find the value of
{(8!)(4!)}
(a) 1 (b) 2 (a) 220 (b) 320
(c) 4 (d) 5 (c) 450 (d) 495
2. If unit digit of a! + b! + c! is 9 then find 8. If sum of factorials of “K” consecutive
the value of {(a!)(b!)(c!)} natural numbers is a three digit number
(a) 8 (b) 5040 then find the maximum value of “K”.

(c) 0 (d) 12 (a) 3 (b) 4


(c) 5 (d) 6
3. Find the unit digit of 1! + 2! + 3! + 4! + ..
+ 10! 9. If sum of factorials of “K” consecutive
natural numbers is a single digit number
(a) 0 (b) 1
then find the maximum value of “K”.
(c) 3 (d) 7
(a) 3 (b) 4
4. If N = a! + b! + c! is a two digit prime
(c) 5 (d) 6
number then how many values of N
exist? 10. If a two digit number ‘ab’ = a! + b! then
how many such two digit number exist?
(a) 0 (b) 1
(a) 0 (b) 1
(c) 2 (d) 3
(c) 2 (d) 3
5. If product of factorials of n consecutive
positive integers is a single digit number 11. Find the highest power of 2 in 50!.
then find the maximum value of n. (a) 42 (b) 44
(a) 0 (b) 1 (c) 47 (d) 49
(c) 2 (d) 3 12. Find the highest power of 3 in 50!.
6. Find the value of (10!)/(7!) (a) 19 (b) 22
(a) 72 (b) 720 (c) 25 (d) 27
(c) 840 (d) 120
12  Theory of Counting

13. Find the highest power of 5 in 100!. 18. If 12Pr = 11880 then find the value of r.
(a) 19 (b) 22 (a) 3 (b) 4
(c) 5 (d) None of these
(c) 25 (d) None of these
19. ICICI Bank has a vacancy for the post of
14. Find the highest power of 7 in 77!
a brand manager. List of applicants are
(a) 12 (b) 13 as follows – Enrolled

(c) 11 (d) 16 IIMA – 4 applicants


IIMB – 5 applicants
15. Find the highest power of 6 in 50!. IIMC – 6 applicants
(a) 47 (b) 23 In how many ways ICICI bank can
recruit its brand manager?
(c) 22 (d) 24
(a) 120 (b) 15
16. If 20Pr = 6840 then find the value of r
(c) 14 (d) None of these
(a) 1 (b) 2
20. Amit, a final year student of B school, has
(c) 3 (d) None of these to go for job interview of PWC. He has 4
15P formal shirts and 3 formal trousers. In
17. If r = 32760 then find the value of
15C how many ways he can select his outfit
r for interview?
(a) 120 (b) 210 (a) 6 (b) 7
(c) 1365 (d) None of these (c) 12 (d) None of these

Response Grid
1. a b c d 2. a b c d 3. a b c d 4. a b c d 5. a b c d
6. a b c d 7. a b c d 8. a b c d 9. a b c d 10. a b c d
11. a b c d 12. a b c d 13. a b c d 14. a b c d 15. a b c d
16. a b c d 17. a b c d 18. a b c d 19. a b c d 20. a b c d
Introduction  13

2
Concept Builder (CB)
Ideal Time Revise your concepts with questions
30 Min. medium difficulty level questions.

1. If (a!) + (b!) is a three digit number and c!


‘K’ is the difference between largest and 5. If N = ( a !)( b !) is a single digit number
smallest possible value of a. Then which then how many values of N exist?
one of the following is true about K. (a) 0 (b) 1
(i) Maximum value of K is 6 (c) 2 (d) 3
(ii) Minimum value of K is 1 6. If product of factorials of n consecutive
(iii) Maximum value of K is 5 positive integers is a single digit number
(a) Only (i) (b) Only (ii) then find the maximum value of n.
(c) Only (iii) (d) Only (i) & (ii) (a) 0 (b) 1
2. If N is product of factorials of three (c) 2 (d) 3
consecutive positive integers then which 7. Let N = 1! + 2! + 3! + 4! + …. + 200! Which
of the following is correct about N? one about N is true?
(i) N may be a single digit number (i) N is even
(ii) If N is a two digit number then there (ii) N is odd
exist only one value of N.
(iii) N/2 is Odd
(iii) If N is a three digit number then
(a) Only (i) (b) Only (ii)
there exist only one value of N.
(c) Only (i) & (iii) (d) Only (iii)
(iv) N may be a four digit number
8. If a(b!) is completely divisible by 511
(a) Only (i) & (ii)
where a is a single digit no. then find the
(b) Only (ii) & (iii)
minimum value of b.
(c) Only (i), (ii) & (iii)
(a) 40 (b) 45
(d) Only (i) & (iv)
(c) 50 (d) 46
3. If N is sum of factorials of all the prime
9. A three digit no abc is such that a + b + c
numbers less than 100, find the last two
digits of N = 25. Find the sum of all possible values
of a!.b!.c!
(a) 48 (b) 58
(a) 8!7! × !7 (b) 9!.8!.7!
(c) 68 (d) 78
(c) 9!8! × 2 (d) None of these
4. Find the smallest perfect square number
divisible by 9!. 10. Find the highest power of 44 in 100!
(a) 25401600 (b) 3628800 (a) 9 (b) 18
(c) 2540160 (d) 50803200 (c) 22 (d) 31
14  Theory of Counting

11. Find the highest power of 10 in 50! + 60! 16. Find the value of 10C1 + 10C2 + … + 10C10
+ 70! (a) 29 (b) 210
(a) 12 (b) 14 (c) 28 (d) None of these
(c) 16 (d) 17 17. Find the value of 10C1 + 10C3 + … + 10C9
12. Find the highest power of 10 in 50! × 60! (a) 29 (b) 210
× 70! (c) 28 (d) None of these
(a) 40 (b) 42 18. Find the value of 10C
0 + 10C1 + 10C2 + …
(c) 38 (d) 44 + 10C10
13. Find the highest power of 10 in 10! + 20! (a) 29 (b) 210
+ 30! + 40! + 50! + 60! + 70! + 80! + 90! + (c) 28 (d) None of these
100! 19. Bipasha has to go to a party. She has a
(a) 140 (b) 142 choice to wear either traditional Indian
(c) 138 (d) None of these dress (Saree) or modern outfit (Gown).
She has 5 Sarees and 6 Gowns in how
14. Find the highest power of 10 in 10! × 20!
many ways can she select her dress?
× 30! × 40! × 50! × 60! × 70! × 80! × 90! ×
(a) 30 (b) 11
100!
(c) 18 (d) None of these
(a) 140 (b) 142
20. Priyanka, a friend of Bipasha also has
(c) 138 (d) 128
to go to the same party and she decided
15. If highest power of 10 in N! is16 then to wear either a Saree or a skirt and top.
what could be the highest power of 10 in She has 6 sarees, 4 skirts and 6 tops. In
(N + 1)! how many ways she can select her outfit?
(a) 16 (b) 17 (a) 16 (b) 144

(c) 18 (d) None of these (c) 30 (d) None of these

Response Grid
1. a b c d 2. a b c d 3. a b c d 4. a b c d 5. a b c d
6. a b c d 7. a b c d 8. a b c d 9. a b c d 10. a b c d
11. a b c d 12. a b c d 13. a b c d 14. a b c d 15. a b c d

16. a b c d 17. a b c d 18. a b c d 19. a b c d 20. a b c d


Introduction  15

3
Concept Cracker (CC)
Ideal Time Boost up your confidence with good
40 Min. questions.

1. Find the value of 1(1!) + 2(2!) + 3(3!) + … (iii) The remainder when summation is
20(20!) divided by 5 is 3
(a) 20! – 1 (b) 21! – 1 (a) Only (i)
(c) 22! – 2 (d) 21! (b) Only (ii)
2. Find the value of 9(2!) + 16(3!) + 25(4!) + (c) Only (iii)
36(5!) + 49(6!) + … 2601(50!)
(d) All three are correct
(a) 52! – 6 (b) 52! – 5
5. Find the unit digit of summation of two
(c) 53! – 4 (d) 55! – 7
Fns taken at a time i.e ∑( Fi )( F j ) here
3. Find the sum of the series till 100 terms i, j ≤ 20 and i ≠ j,
1 2 3 100 (a) 1 (b) 0
+ + + …. +
2! 3! 4! 101!
(c) 3 (d) 7
101! − 1 102! − 1
(a) (b) 6. Find the number of zeros at the end of
101! 101!
(F1)(F2)(F3)….. (F20)
101! − 5 101! − 1 (a) 241 (b) 242
(c) (d)
101! 102!
(c) 220 (d) 244
Directions (Qs. 4 to 6): Consider a set 7. If N(N + 1) and (N + 1)N is divisible by
of 100 elements S = {1, 2, 3, 4,… 100}, 100! Then find the maximum possible
consider 20 subsets with 5 elements e.g value of N
S1 {1, 2, 3, 4, 5}, S2 = {6, 7, 8, 9, 10} and so on.
(a) 12 (b) 24
Lets us define Fn as sum of factorials of all the
elements of subset Sn, e.g F1 = 1! + 2! + 3! + (c) 20 (d) 15
4! + 5!, F2 = 6! + 7! + 8! + 9! + 10! And so on.  8. If a8 and 8a is completely divisible by 50!
4. Which one of the following is true about Then which one of the following is true
sum= F1 + F2 + … + F20 about ‘ highest value of a’?
(i) Unit digit of summation is 3 (a) 10 < a < 14 (b) 14 < a < 16
(ii) Last two digit odd summation is 13 (c) 16 < a < 18 (d) 18 < a < 20
16  Theory of Counting

9. Find the last two digits of (a) 3 (b) 6


1! 2! 3! 4! (c) 10 (d) None of these
+ + + ………100 terms
1 2 3 4
18. Which one of the following represents
(a) 14 (b) 24
the ratio between (nCr): nCr + 1?
(c) 34 (d) 64
10. st
Find 1 non-zero digit of 85! (a) (r + 1)/(n – r) (b) (r – 1)/(n – r)
(a) 2 (b) 4 (c) r/(n – r) (d) None of these
(c) 6 (d) 8 19. Rajesh can reach Patna from Hajipur
11. st
Find 1 non-zero digit of 100! either by a combination of Train and
(a) 2 (b) 4 Auto or by a combination of Bus, Taxi
(c) 6 (d) 8 and Auto. If he has total options as
12. Find 1st non-zero digit of 165! follows – Trains – 5, Auto – 6, Bus – 7
(a) 2 (b) 4 and Taxi – 8. In how many ways he can
(c) 6 (d) 8 reach Patna from Hajipur?
13. st
Find the 1 non-zero digit of 100! + 165!
(a) 316 (b) 336
(a) 2 (b) 4
(c) 6 (d) 8 (c) 366 (d) None of these
14. st
Find the 1 non-zero digit of 100! × 165! 20. Suresh, a school going boy has three
(a) 2 (b) 6 dress codes, official, formal and ethnic in
(c) 4 (d) 8 official he has to wear shirt, trousers, and
15. Find the 1st non-zero digit of 100! + 110! tie along with shoe, in formal dress he
+ 120! + 130! + 140! + 150! has to wear jeans, Tee and sandle, while
(a) 4 (b) 6 in ethnic dress he has to wear kurta and
(c) 2 (d) 8 paijama with shoe or sandle, If he has
following items available – Shirt – 5,
16. Find the value of 1P1 + 2.2P2 + 3.3P3 + …
Trousers – 4, Tie – 5, Shoe – 4, Jeans – 5,
+ n.nPn
Tee – 4, Sandle – 5, Kurta – 4, Paijama
(a) (n + 1)! – 1! (b) (n + 1)! + 1!
– 5. If on a particular day in how many
(c) (n + 2)! – (n – 1)! (d) None of these ways he can select his dress?
17. If 2n + 1Pn – 1 :2n – 1Pn= 3 : 5 then find the (a) 316 (b) 336
value of nC (c) 366 (d) None of these
2

Response Grid
1. a b c d 2. a b c d 3. a b c d 4. a b c d 5. a b c d
6. a b c d 7. a b c d 8. a b c d 9. a b c d 10. a b c d
11. a b c d 12. a b c d 13. a b c d 14. a b c d 15. a b c d

16. a b c d 17. a b c d 18. a b c d 19. a b c d 20. a b c d


Introduction  17

Solutions
Concept Applicator (CA)
1. (c) 2. (d) 3. (c) 4. (b) 5. (c) 6. (b)
7. (d) 8. (d) 9. (a) 10. (a) 11. (c) 12. (b)
13. (d) 14. (a) 15. (c) 16. (c) 17. (c) 18. (b)
19. (b) 20. (c)

1. (c) We know that 5! = 120 and 4! is 24 10. (a) From the given condition a! + b! =
hence maximum value of e is 4. 10a + b Since 0! = 1, 1! = 1, 2! = 2, 3! = 6
2. (d) Since unit digit of all the factorials and 4! = 24 since summation is a two
more than 4 is 0 hence unit digit 9 is digit number hence one of the number
given by either 0! + 2! + 3! or 1! + 2! + must be 4, from trail and error method
3! In both the cases the value of {(a!) no such two digit number exist.
(b!)(c!)} = 1 × 2 × 6 = 12 11. (c) Divide successively 50 by 2 and
3. (c) Since unit digit of all the factorials keep on writing the quotient and
more than 4 is 0 hence required then find the summation of all the
unit digit is given by 1! + 2! + 3! + 4! quotient this summation will give
Which is 1 + 2 × 6 + 24 = 33, hence us the highest power of 2 in 50!.
required unit digit is 3
4. (b) Since 0! = 1, 1! = 1, 2! = 2, 3! = 6, 50
4! = 24 2 25
Since, a! + b! + c! is a two digit prime 2 12
number hence possible cases are – 2 6
0! + 3! + 4! = 31, 1! + 3! + 4! = 31, 2 3
hence only one number 31 exist.
2 1
5. (c) For maximum value of n we have (1!)
(2!) which is a single digit number Sum of all the quotient is 25 + 12 + 6
hence n = 2, here point to note that + 3 + 1 = 47, hence highest power of
0! = 1 but 0 is not a positive integers. 2 in 50! is 47.
6. (b) Since, (10!)/(7!) = 8 × 9 × 10 = 720 Alternately : –
7. (d) The required value is (9 × 10 × 11 × Required highest power of 2 is
12) /(1 × 2 × 3 × 4) = 495  50   50   50   50   50 
8. (d) Since, 1! + 2! + 3! + 4! + 5! + 6! = 1 +  1  +   2  +   3  +   4  +   5  = 25
2  2  2  2  2 
2 + 6 + 24 + 120 + 720 = 873
9. (a) Since, 1! + 2! + 3! = 1 + 2 + 6 = 9, + 12 + 6 + 3 + 1 = 47, hence highest
hence maximum value of ‘K’ is 3. power of 2 in 50! is 47.
18  Theory of Counting
12. (b) Divide successively 50 by 3 and  100   100 
keep on writing the quotient and  1  +   2  = 20 + 4 = 24
5  5 
then find the summation of all the
hence highest power of 5 in 100! is
quotient this summation will give us
24.
the highest power of 3 in 50!.
14. (a) Same method as above highest
50
 77   77 
3 16 power of 7 in 77! is  1  +   2 
7  7 
3 5 = 11 + 1 = 12
3 1 15. (c) Since 6 is a composite number hence
Sum of all the quotient is 16 + 5 + 1 we cant use above method.
= 22, hence highest power of 3 in 50! We know that 6 = 2×3 hence we will
is 22. get 6 when we have pair of 2 and 3,
so we will find out highest power of
Alternately:
2 and that of 3 in 50!
Required highest power of 3 is Since highest power of 2 and 3 in 50!
is 47 and 22 hence highest power of
 50   50   50  6 in 50! is 22
 1  +   2  +   3  = 16 + 5 + 1 = 22,
3  3  3  n! 20!
16. (c) Since, nPr =
hence highest power of 3 in 50! is 22. (n − r )! Hence, (20 − r )!
13. (d) Divide successively 100 by 5 and = 6840 = 20 × 19 × 18 so r = 3
keep on writing the quotient and n! 15!
17. (c) Since nPr =
then find the summation of all the (n − r )! . Hence (15 − r )!
quotient this summation will give us
the highest power of 5 in 100!. = 32760 = 16 × 14 × 13 × 12 so r = 4
So 15Cr = (15Pr)/(r!) = 32760/24 = 1365
100
5 20 18. (b) Since 12Pr = (12!)/(12 – r)! = 11880
5 4 = 12 × 11 × 10 × 9, then r = 4

Sum of all the quotient is 20 + 4 = 19. (b) ICICI bank can select either from
24, hence highest power of 5 in 100! IIMA, or IIMB or IIMC so total
is 24. number of ways = 4 + 5 + 6 = 15 ways
20. (c) Amit has to wear both a formal shirt
Alternately
and a trouser hence number of ways
Required highest power of 5 is is 4 × 3 = 12

Concept Builder (CB)


1. (a) 2. (b) 3. (c) 4. (a) 5. (d) 6. (c)
7. (b) 8. (b) 9. (d) 10. (a) 11. (a) 12. (b)
13. (d) 14. (d) 15. (c) 16. (d) 17. (a) 18. (b)
19. (b) 20. (c)
Introduction  19
1. (a) Since summation of two factorial is a If a = 3 then b = 0 or 1 then c can
three digit number hence maximum take any value and N = 6
possible value of a is 6 while Hence three values of N exist.
minimum possible value is 0 hence
required difference is 6 – 0 = 6, hence 6. (c) Since (1!)(2!)(3!) = 12 hence there is
maximum value of K is 6. no three consecutive numbers exist.
Now we have to find the minimum 7. (b) Since all the factorials except 1!
value of K, for minimum value of is even number hence required
K, largest value of a should be the summation must be odd.
smallest (largest possible values of a 8. (b) Since 45! contains 510 if a = 5 then we
is 5 & 6 and out of these two smallest can get 5(45!) is completely divisible
is 5), and smallest value of a should by 511(50! Contains 512).
be maximum ( smallest possible 9. (d) Since, a + b + c = 25.
value of a is 0, 1, 2, 3, 4, and 5, out of
these largest value is 5) Hence possible values of
Hence smallest value of K is 0. abc = 9,9,7& 9,8,8
2. (b) As per the given condition N = (a!)(b!) There are 3 numbers with 9, 9 & 7
(c!) here a, b, and c are consecutive these numbers are 997, 979 and 799,
integers. similarly there are 3 numbers with
If a = 1 then N = 1 × 2 × 6 = 12 9, 8 and 8.
If a = 2 then N = 2 × 6 × 24 = 288 So required summation is 3 × 9! × 9!
× 7! + 3 × 9! × 8! × 8! = 3 × 9! × 8! ×
If a = 3 then N = 6 × 24 × 120 = 17280
7!(9 + 8) = 51 × 7! × 8! × 9!
If a = 4 then N = 24 × 120 × 720 =
10. (a) Since 44 = 22 × 11 so we just need to
2073600
find the highest power of 11 in 100!
Hence only statement (ii) & (iii) are & that will be same as the highest
correct. power of 44 and is equal to 9.
3. (c) Since last two digits of all the 11. (a) Since highest power of 10 in 50! is
factorials more than 9 is 00 hence less than that of 60! and 70! hence
required last two digits is given by required highest power of 10 is same
2! + 3! + 5! + 7! = 2 + 6 + 120 + 5040 as that of 50! Which is equal to 12.
= 5168, For further explanation–since
Hence required last two digits is 68. highest power of 10 in 50! is 12 hence
4. (a) Since, 9! =  (27 ) × (34 ) × (51 ) × (71 ) we can write 50! = P × 1012
Similarly highest power of 10 in 60!
To get smallest perfect square we is 14 hence we can write 60! = Q ×
have to multiply 9! With 2 × 5 × 7
1014
hence smallest such perfect square
Similarly highest power of 10 in 70!
is (28 ) × (34 ) × (52 ) × (72 ) = 25401600
is 16 hence we can write 70! = R ×
5. (d) Since, 0! = 1, 1! = 1, 2! = 2, 3! = 6, 1016
If a = 0, or 1 then b and c can take Hence we can write 50! + 60! + 70!
any value but N = 1, = P × 1012 + Q × 1014 + R × 1016 =
If a = 2 then b = 0 or 1 then c can 1012(P + Q × 102 + R × 104) hence
take any value and N = 2 highest power of 10 is 12.
20  Theory of Counting
12. (b) Similar to above explanation – 16. (d) Since nC0 + nC1 + nC2 + … + nCn
50! × 60! × 70! = (P × 1012) = (1 + 1)n = 2n, hence 10C0 + 10C +
1
× (Q × 1014) × (R × 1016) 10C + … + 10C = 210
2 10
= (PQR) × (1042) hence highest or 10C1 + 10C2 + … + 10C10
power of 10 is 42.
= 210 – 10C0 = 210 – 1
13. (d) Since highest power of 10 in 10! is n n n n n
17. (a) C0 + C2 + C4 + … = C1 + C3 +
smallest hence required highest
nC + …. = 2n – 1
power of 10 is same as that of 10! 5

Which is equal to 2. Hence 10C1 + 10C3 + … + 10C9


14. (d) Required highest power of 10 is = 210 – 1 = 29
summation of highest power of 10 in 18. (b) Since nC0 + nC1 + nC2 + … + nCn
all the given factorial. = (1 + 1)n = 2n hence 10C + 10C +
0 1
Required highest power is 2 + 4 + 7 10C +…+ 10C 210
2 10=
+ 9 + 12 + 14 + 16 + 19 + 21 + 24
19. (b) Since Bipaha can wear either
= 128
traditional Indian dress (Saree)
15. (c) Highest power of 10 in N! is 16 hence or modern outfit (Gown). Hence
70 ≤ N < 74, number of ways is 5 + 6 = 11 ways.
If N = 70, 71, 72, 73 then (N + 1)! will 20. (c) Priyanka can select Saree in in
have highest power of 10 as 16, and 6 ways and skirt and top in 6 × 4 =
if N = 74 then N + 1 = 75 and 75! has 24 ways so total number of ways is
highest power of 10 as 18. 6 + 24 = 30

Concept Cracker (CC)


1. (b) 2. (a) 3. (a) 4. (d) 5. (b) 6. (c)
7. (d) 8. (b) 9. (a) 10. (a) 11. (b) 12. (c)
13. (b) 14. (c) 15. (a) 16. (a) 17. (b) 18. (a)
19. (c) 20. (d)

1. (b) (1!) = (2 – 1)! = 2! – 1! 2601(50!) = 51.51.50! = 51.51!


2(2!) = (3 – 1)2! = 3! – 2! = (52 – 51)51! = 52! – 51!
3(3!) = (4 – 1)3! = 4! – 3! Sum = 52! – 3! = 52! – 6
20(20!) = (21 – 1)20! = 21! – 20! 1 1
3. (a) Consider the 1st term =1
  −
Hence sum = 21! – 1 2! 2!
2. (a) 9(2!) = 3.3.2! = 3.3! = (4 – 1)3! 2 3 −1 1 1
2nd term = = –
= 4! – 3! 3! 3! 2! 3!
16(3!) = 4.4.3! = 4.4! = (5 – 1)4! 3 4 −1 1 1
3rd term = = = –
= 5! – 4! 4! 4! 3! 4!
25(4!) = 5.5.4! = 5.5! = (6 – 1)5! And so on 2nd last term
= 6! – 5!
Introduction  21

99 100 –1  1 1 Which is equal to 0 + 1 + 2 + 3 + 4 +


= = = –
100! 100! 99! 100! 6 + 7 + 8 + 9 + 10 + 12 + 13 + 14 + 15
Hence last term + 16 + 18 + 19 + 20 + 21 + 22 = 220
97 48 24 1
100 100 − 1 1 1 7. (d) Since 100! = 2 × 3 × 5 × … 97
= = = –
101! 101! 100! 101! From options we will find that
When we add all these then we all highest value of N is 15.
the terms get cancelled except 1st 8. (b) Since highest power of 2 in 50! is 47
and last hence required summation hence highest power of 8 in 50! is
1 101!− 1 [47/3] = 15
is 1 − =
101! 101! If we assume a = 15 then 158 or 38 and 58
4. (d) Required summation is 1! + 2! + 3! + So we have to check that whether
.. + 100! 38 and 58 are divisible by 50! or not,
Its unit digit is given by 1! + 2! + 3! + Since highest power of 3 in 50! is 22
4! Which is 3. Hence statement (i) is and that of 5 is 12, hence a =15 will
correct. satisfy the condition.
Its last two digits is given by 1! + 2! + n!
3! + 4! + 5! + 6! + 7! + 8! + 9! Which is 9. (a) Since = (n − 1)! hence given sum is
n
13. Hence statement (ii) is correct.
1! + 1! + 2! + 3! + 4! + … 100 terms
Remainder when summation is since all the numbers after 9!
divided by 5 is given by 1! + 2! + 3!
(i.e from 10! To 99!) contains at least
+ 4! Which is equal to 3, hence this
statement (iii) is also correct. two zeros hence last two digit is
given by sum of last two digits from
5. (b) Since unit digit of F1 is 3 and unit 0! To 9!
digit of all others is 0 hence unit digit
of (FiFj) is always 0 hence required Required last two digit is same as
unit digit is 0. last two digits of 1! + 1! + 2! + 3! + 4!
6. (c) Consider F1 = 1! + 2! + 3! + 4! + 5! + 5! + 6! + 7! + 8! + 9!
Highest power of 10 in this is 0. Last two digits of 1! = 01
F2 = 6! + 7! + 8! + 9! + 10! Highest Last two digits of 1! = 01
power of 10 in this is 1 (Same as that Last two digits of 2! = 02
of 6!)
Last two digits of 3! = 06
F3 = 11! + 12! + 13! + 14! + 15! Highest
Last two digits of 4! = 24
power of 10 in this is 2 (Same as that
of 11!) Last two digits of 5! = 20
Last two digits of 6! = 20
F4 = 16! + 17! + 18! + 19! + 20! Highest
power of 10 in this is 3 (Same as that Last two digits of 7! = 40
of 16!) Last two digits of 8! = 20
And so on hence required highest Last two digits of 9! = 80
power of 10 is same as that of (1!)(6!) Sum of all last two digits is 214
(11!)(16!)… (91!)(96!) hence last two digits = 14.
22  Theory of Counting
10. (a) 13. (b) Since number of zeros in 165! is more
5 85
than that in 100! Hence required
5 17 → R1 = 0 1stnon-zero digit is given by 1st
5 3 → R2 = 2 non-zero digit of 100! Which is equal
to 4.
0 → R3 = 3
14. (c) Since 1st non-zero digit of 100! is
Hence highest power of 5 in 85! is P 4 and that of 165! is 6 hence 1st
= 17 + 3 + 0 = 20 non-zero digit of their product is
given by 4 × 6 which is 4.
Hence U[85!] = U[220] × U[0!] × U[2!]
15. (a) Since number of zeros in 100! is
× U[3!] = U[6 × 1 × 2 × 6] = 2
smallest hence 1st non-zero digit of
11. (b) With normal method we will find this summation is same as that of
highest power of 5 in 100! 100! which is 4.
16. (a) Since, n. nPn = n(n!) = (n + 1) ! – n!
5 100
Hence, 1P1 = 2! – 1!
5 20 → R1 = 0
2. 2P2 = 3! – 2!
5 4 → R2 = 0
3. 3P3 = 4! – 3!
0 → R3 = 4
………………..
Hence highest power of 5 in 100! is P …………………
= 20 + 4 = 24 n. nPn= (n + 1)! – 1!
So U(100!) = U[224] × U[0!] × U[0!] × Hence 1P1 + 2.2P2 + 3.3P3 + … + n.
nP = (n + 1)! – 1!
U[4!] = U[6 × 1 × 1 × 4] = 4 n

12. (c) With normal method we will find (2n + 1)!


17 . (b) Consider 2n + 1Pn – 1 = n + 2 !
highest power of 5 in 165! ( )
5 165 2n – 1P (2n − 1)!
n=
5 33 → R1 = 0 (n − 1)!
5 6 → R2 = 3 From the given condition
(2n + 1)! (n − 1)! 3
5 1 → R3 = 1  × = , 
(n + 2)! (2n − 1)! 5
0 → R4 = 1 (2n + 1)(2n) 3
or =
(n + 2)(n + 1)(n) 5
Hence highest power of 5 in 165! is
P = 33 + 6 + 1 + 0 = 40 On solving we will get 3n2 – 11n – 4 = 0
So U(165!) = U[240] × U[0!] × U[3!] × or n = – 1/3 or 4 but only acceptable
U[1!] × U[1!] × U[0!] = U[6 × 1 × 6 × value is n = 4.
1 × 1 × 1] = 6 So, 4C2 = 6
Introduction  23
18. (a) From formula we know that (nCr)/(r 20. (d) Suresh can select Official dress in
+ 1) = nCr + 1/(n – r), hence option (a) 5 × 4 × 5 × 4 = 400 ways
is correct. Can select formal dress in 5 × 4 × 5
19. (c) Number of ways for 1st combination = 100 ways
is 5 × 6 = 30
And ethnic dress in 4 × 5(4 + 5) = 20
And number of ways for 2nd combina- × 9 = 180 ways
tion is 7 × 8 × 6 = 336
So total number of ways is 400 + 100
So total number of ways is 30 + 336 + 180 = 680 ways
= 366
24  Theory of Counting

Part B
 Highest power of a PRIME NUMBER p that  Fundamental principle of multiplication:
divides n! exactly that means without leaving any If a work A can be done in m ways
    and another work B can be done in n
remainder is given by  n  +  n  +  n  …….
 p   p 2   p 3  ways then work A & B can be done in
m × n ways.
where, [x] represents greatest integer function,
that means greatest integer less than or equal Consider total n works, if 1st work can be
to x. done in a1 ways, 2nd work can be done in
a2 ways, and so on nth can be done in an
ways then work a1, a2, …,an–1, an can be
 Number of zeros in a number is same as the
highest power of 10 in that number, and we done in a1× a2×…×an ways.
know that 10 = 2 × 5 then we needs to find the
highest power of 2 & 5, and smaller in the two n!
 From the definition nCr = n − r ! r !
is equal to highest powers of 10. Since 5 > 2 ( )( )
hence we will find highest power of 5, as highest
power of 2 must be more than that of 5. n!
And nPr = n
(n − r )! = (r!)( Cr)
 A new approach to find 1st non-zero
digit: Let highest power of 5 in N! is P Some other properties of binomial
coefficients-
and R1, R2, etc are remainders when n is
successively divided by 5 →nC0 + nC1 + nC2 + … + nCn
U[N!] = U[2P] × U[R1!] × U[R2!] …. = (1+1)n = 2n
→nC0 –nC1 + nC2 + … (–1)n–1nCn = (1–1)n = 0
 Fundamental principle of summation:
If a work A can be done in m ways and →nC0 + nC2 + nC4 + …
another work B can be done in n ways then
= nC1 + nC3 + nC5 + …. = 2n–1
work A or B can be done in m + n ways.
→nCr–1 + nCr = n+1Cr
Consider total n works, if 1st work can be
done in a1 ways, 2nd work can be done in →nCx = nCy then x = y or x + y = n
a2 ways, and so on nth can be done in an
ways then work a1 or a2 or …or an can be →nCr = nCn–r
done in a1+a2+…+an ways. →(nCr)/(r+1) = nCr+1/(n–r)
Introduction  25

Concept Deviator (CD)


1 Ideal Time
80 Min.
Revise your concept (IIT advance or
tougher questions) Ideal Time – 5 min
per question

1. Find the no. of zeroes in1!2!3!4!......100! 7. If a27 and 27a is divisible by 100! Then
(a) 525 (b) 825 which one of the following is true about
(c) 1124 (d) 850 ‘highest value of a’?

2. If 390 does not divide n! then find the (a) 8 < a < 10 (b) 10 < a < 12
maximum value of n.
(c) 12 < a < 14 (d) 14 < a < 17
(a) 13 (b) 12
x y z
(c) 17 (d) 19 8. If 100! =  K  (2!) (5!) (7 !) . Then find the
maximum possible value of x + y + z
3. If ‘abc’ is a three digit no such that
a! + b! + c! < abc.How many 3 digit no (a) 122 (b) 47
exist that satisfy the above condition
(c) 99 (d) None of these
given that a, b & c are different digits.
(a) 40 (b) 72 9. In the previous question find the
(c) 84 (d) 98 maximum value of product x y z

4. How many two digit number ab exists (a) 1872 (b) 1728
such that (a! + b!)/ab < 10.
(c) 144 (d) 2016
(a) 22 (b) 30
(c) 15 (d) None of these 10. N is the product of first 100 multiple of
K. If N is divisible by 10100 then find the
5. The quotient and remainder of division
minimum number of zeros at the end of
(a! + b! + c!)/abc are 7 and 434 where
abc is a 3 digit no. What could be the N
value of a? (a) 24 (b) 124
(a) 5 (b) 6 (c) 97 (d) 121
(c) 7 (d) 8 or 7
11. Find the number of zeros at the end of
6. Lets define a set S ={ Pi | set of all 3 digit
(510)!
natural number that has odd number of
factors}. Find the highest power of 12 in 59 − 1 510 − 1
(a) (b)
the product of all the elements of set S. 4 4
(a) 16 (b) 18 11 8
(c) 5 − 1 (d) 5 − 1
(c) 20 (d) None of these 4 4
26  Theory of Counting
12. Find the number of zeros at the end of 14. N is product of the first 100 multiples of
(1515)! 5. Find highest power of 10 in N.
 1515 − 315   515 − 1  (a) 124 (b) 107
(a)   (b)  
 4   4  (c) 97 (d) 149
 1515 − 1   1515 − 515  15. If N = 10! + 20! + 30! + ......+ 100! Then
(c)   (d)   find the highest power of 10 in NN
4   4 

(a) N (b) 2N
13. Find the number of zeros at the end of
(1000........100 times zeros)! (c) 4N (d) 124
16. In a college of 300 students, every student
 10100 − 5100   10100 − 550  read 5 newspapers and every newspaper
(a)   (b)  
 4   4  is read by 60 students. The number of
news papers is- [IIT JEE 1998]
 10100 − 2 50   10100 − 2 100 
(c)  (a) at least 30 (b) at most 20
 (d) 
4

 4    (c) exactly 25 (d) None of these

Response Grid
1. a b c d 2. a b c d 3. a b c d 4. a b c d 5. a b c d
6. a b c d 7. a b c d 8. a b c d 9. a b c d 10. a b c d
11. a b c d 12. a b c d 13. a b c d 14. a b c d 15. a b c d
16. a b c d

Concept Eliminator (CE)


2 Ideal Time
80 Min.
Revise your concept (Maths Olympiad or
tougher questions)

Direction (Qs. 1-5): Let us define noctorial 1. Find the number of trailing zeros at the
(denoted as !!) as product of ‘n’ consecutive end of 25!!
integers starting from ‘n’. (a) 5 (b) 6
Hence from definition (n!!) = (n)(n + 1) (c) 8 (d) 17
(n +2).....(2n –1)
2. Find the highest power of 100 in 100!!
Or 3!! = 3 × 4 × 5 = 60
(a) 18 (b) 17
4!! = 4 × 5 × 6 × 7 = 840 and so on. (c) 21 (d) 12
Introduction  27
3. Find the 1st non-zero digit of 90!! 8. Consider a two digit prime number P such
that exponent of P in number N = (300!)/
(a) 8 (b) 7
(100!)(200!) is 1. How many values of P
(c) 6 (d) 5
more than 50 satisfy above condition?
4. Find the number of trailing zeros in (1!!)
(a) 5 (b) 6
(2!!)(3!!)....(20!!)
(c) 7 (d) 9
(A) 42 (b) 48
9. Let us consider three numbers A, B and
(c) 64 (d) None of these
C, A is product of 1st 200 multiples of 36,
5. Find the minimum value of k such that B is product of 1st 100 multiple of 6, C is
(k!!) is completely divisible by all two product of 1st 100 multiples of 12, How
digit prime numbers. many prime two digit numbers more
(a) 49 (b) 97  A
than 50 is a factor of N  =   .
(c) 32 (d) 16  BC 

6. Find 1sttwo non-zero digits of 147! (a) 1 (b) 2


(c) 3 (d) 5
(a) 42 (b) 48
(c) 64 (d) 84 333!
10. Find 1st two non zero digit of
222!
7. Find 1sttwo non-zero digit of (222!)(333!)
(a) 24 (b) 64 (a) 24 (b) 64
(c) 84 (d) 94 (c) 04 (d) 84

Response Grid
1. a b c d 2. a b c d 3. a b c d 4. a b c d 5. a b c d
6. a b c d 7. a b c d 8. a b c d 9. a b c d 10. a b c d
28  Theory of Counting

Solutions
Concept Deviator (CD)
1. (c) 2. (b) 3. (d) 4. (d) 5. (c) 6. (b)
7. (c) 8. (d) 9. (a) 10. (b) 11. (b) 12. (a)
13. (d) 14. (c) 15. (b) 16. (c)

1.
(c) No of zeroes means highest power of From 45! To 49! all contain highest
10. power of 5 as 10
Since 10 = 2 × 5 so we need to From 50! To 54! all contain highest
find highest power of 5. Divide power of 5 as 12
the given multiplication in 4 From 55! To 59! all contain highest
parts(1!2!3!.......24!)(25!26!......49!) power of 5 as 13
(50! 51!...74!)(75!76!....99!) × 100!
From 60! To 64! all contain highest
Till 4!highest power of 5 is zero. So power of 5 as 14
we will start with 5!
From 65! To 70! all contain highest
5!→1 10!→2 15!→3 20!→4 power of 5 as 15
6!→1 11!→2 16!→3 21!→4 From 70! To 74! all contain highest
7!→1 12!→2 17!→3 : power of 5 as 16
8!→1 13!→2 18!→3 : From 75! To 79! all contain highest
9!→1 14!→2 19!→3 24!→4 power of 5 as 18
5 10 15 20 From 80! To 84! all contain highest
power of 5 as 19
So highest power of 5 in 1!2!.......24!
is 5+10+15+20 = 50 From 85! To 89!all contain highest
power of 5 as 20
Now consider the 2nd part
From 90! To 94! all contain highest
(25!26!.....49!)
power of 5 as 21
From 25! To 29! all contain highest
From 95! To 99! all contain highest
power of 5 as 6
power of 5 as 22
From 30! To 34! all contain highest
100! contain highest power of 5 as 24
power of 5 as7
Hence total power of 5
From 35! To 39! all contain highest
power of 5 as8 50 +(6 + 7 + 8 + 9 + 10)5 + (12 + 13 +
14 + 15 +16)× 5 + (18 + 19 + 20 + 21
From 40! To 44! all contain highest
+ 22) × 5 = 50 + 200 + 350 + 500 + 24
power of 5 as 9
= 1124
Introduction  29
2. (b) Since 390 = 65 × 6 = 2 × 3 × 5 × 13 It is easy to note down these no as for
So, if n = 13 then 390 is divisible by all the nos formed when a = 2,3,or 4
n! will satisfy the above condition

But if n = 12 then 390 is not divisible Now let none of a,b or c is 5 then
by 13 total no of 3 digit of no formed with
digits 1,2,3, & 4 = 4 × 4 × 3 = 48
Hence maximum value of n = 12
So total nos formed
3. (d) Let one of the digit is 6 then 6! = 720
hence a has to be 7 or 8 or 9 but 7! = 20 = 16 + 14 + 48 = 98 nos
is a four digit no so none of the digit 4. (d) Since a! + b! < 10 (two digit no).
are 6,7,8 or 9 Hence, a! + b! < 1000, so no of the a
Let one of the digit is 5 or b is 7, 8 or 9.
If a = 5 then abc > 500 but a! + b! + c! Let assume one of them is 6 then
< 500 minimum value of a! + b!/ab = 61 +
Number of numbers formed when a a!/6a > 10
= 5 then b can be any one of 0,1,2,3,4, Hence if any one of a or b is 6 then
(5 ways) and can be chosen from the given ratio is > 10. Hence, we can
remaining 4 digits conclude non of a or b is equal to 6.
So, if 1st digit is 5 then number of Remaining digits that we can use
numbers formed = 5 × 4 = 20 are 0,1,2,3,4,5 then maximum value
If b = 5 then maximum value of a! + of (a! + b!)/ab is 1! + 5!/!5 < 10
b! + c! = 4! + 5! + 3! = 24 + 120 + 6 = 5. (c) Since a! + b! + c! = 7(abc) + 434
150. So maximum value of a! + b! + c!
And smallest 3 digit number with b = 7(3digit no) + 434 and value of
= 5 is 153 hence all values of a & c a! + b! + c! is a 4 digit no.
will satisfy the given condition. Now The minimum value is when a = 1
we can select a in 4 ways. then also the value of a! + b! + c! is a
1,2,3 or 4 and c also in 4 ways. So 4 digit no.
total number of numbers formed To get a! + b! + c! as a 4 digit no. one
with b = 5 = 4 × 4 = 16 of them has to be 7 as 7! = 5040 are
If c = 5 then it’s better to write all none of them is equal to 8 or 9.
numbers such that a! + b!c! < abc Now we can know that one of a,b or
125,135,205,215,235,245,305,315, c is 7.
325,345,405,415,425,435 = 134 nos Hence a! + b! + c!. 5040
30  Theory of Counting
If we assume minimum value of a! + 7. (c) Since highest power of 3 in 100! is
b! + c! as 5040 only then abc = 5040 48 hence that of 27 is 16 so assume
–434/7 = 4606/7 > 600 a = 16
So a can either 6 or 7 (it can not be 8 Now 1627 = 2108 but highest power
or 9)
of 2 in 100! is 97 hence a can not be
Let a = 6 and one of b or c is 7 then equal to 16.
minimum value of a! + b! + c! = 6! +
Let us assume a = 15, then we have
7! + 0! = 5761,
to check 1527 or 327 and 527 since
abc = 5761 – 434/7 = 700
highest power of 5 in 100! is 24 hence
hence for this condition a has to be 7
a can not be equal to 15.
so we only have one option left with
a = 7 that satisfy the given condition. If a =14 then we have to check 1427
Hence a = 7 or 227 and 727 since highest power
For your eagerness to know that of 7 in 100! is 16 hence a can not be
abc is 761. equal to 14.

To solve completely the question If a =13 then we have to check 1327


requires a lot of time so it is since highest power of 13 in 100! is 7
beyond the preview of syllabus. hence a can not be equal to 13.
6. (d) We know that only perfect squares If a =12 then we have to check 1227
can have odd number of factors. or 254 and 327 since highest power
Hence set S can be defined as = {100, of 2 in 100! is 97 and that of 3 is 48
121, 144, …., 961} hence a = 12.
Product of all the elements is 100 ×
8. (d) (2!)x = 2x
121 × 144 ×….× 961 = 102 × 112 × 122
(5!)y= 23y, 3y, 5y
× ….× 312 = (10 ×11 × 12 ×..×31)2 =
[(31!)/(9!)]2 (7!)z = 24z, 32z, 5z,7z
Since, 12 = 22×31 Hence (2!)x × (5!)y × (7!)z = 2x + 3y +4z
Highest power of 2 in 31! is 26 and × 3y + 2z × 5y + z × 7z
that in 9! is 7 hence highest power
Since 100! = 297 × 348 × 524 × 716
of 2 in (31!)/(9!) is 26 – 7 = 19 and so
highest power of 22 in (31!)/(9!) is 19. Maximum value of z = 16
Similarly highest power of 3 in 31! is Maximum value of y + z = 24
14 and that in 9! is 4 hence highest Maximum value of z = x + 3y + 4z
power of 3 in (31!)/(9!) is 14 – 4 = 10
= 97; hence for maximum value of
and so highest power of 3 in [(31!)/
x + y + z, y = 0, z = 0 and x = 97, then
(9!)]2 is 20.
x + y + z = 97
Therefore, highest power of 12 is 19
Introduction  31
9. (a) From the solution of the previous  1515   1515   1515 
question:   +  2  +  3  +
 5   5   5 
Maximum value of z = 16
 1515   1515 
Maximum value of y + z = 24 ……  14  +  15 
 5   5 
Maximum value of y + 2z = 48
Maximum value of x + 3y + 4z = 97 = 315[ 514 + 513 + 512 + ....... 51 + 1]
For the maximum value of x.y.z each
 15   15 15 
of them should be as close as possible. = 315  5 − 1     =        15 − 3 
 4   4 
Y = z = 12; then x = 97 – 3y – 42 = 13;
13. (d) Here given number is 10100
hence the maximum value of x.y.z =
Number of zeros is same as that of
12 × 12 × 13 = 1872.
highest power of 5. Highest power of
10. (b) From the given condition N = 5 in (1515)! is given by
K100(100!)
 10100   10100   10100   10100   10100 
Since, 100! has the highest power of 2  1  +  2   +   3  +…   .  99  +   100 
 5   5   5   5   5 
and 5 as 97 and 24, but N is divisible
by 10100 i.e. 2100 and 5100. Hence = 2100[599 + 598 + 597+..... + 51 + 50]
K must be the multiple of 2 and 5.
 5100 − 1   10100 − 2 100 
For the minimum value K = 10, = 2100     =   
 4   4 
then N = 10100(1001). Hence the
minimum number of zeros at the 14. (c) N = 5 × 10 × 15 ×......× 500
end of N is 124. = 5100 [1 × 2 × 3 ......× 100]
11. (b) Number of zeros is same as that
= 5100(100!)
of highest power of 5. Highest
power of 5 in (510)! is given by Hence, N = 5100 (100!)

 510   510   510   510  Highest power of 5 in n is 124 and


  +   2  +   3  +……
  . +  10  that of 2 in is 97.Hence the highest
 5   5   5   5 
power of 10 in N is 97
= 59 + 58 + 57 + ….. + 1
15. (b) Highest power of 10 in 10! is 2, hence
10 10
= 5 −1
− 1  =
5 −1 the required highest power is 2 N
5 4
16. (c) Required number of news papers is
12. (a) Number of zeros is same as that of
(300 × 5)/60 = 25
highest power of 5. Highest power of
5 in (1515)! is given by
32  Theory of Counting

Concept Eliminator (CE)


1. (b) 2. (d) 3. (a) 4. (d) 5. (a) 6. (d)
7. (b) 8. (a) 9. (c) 10. (c)

Solution from question number 1-5 Hence 1!! = (1!)


From the definition (n!!) = (n)(n + 1)(n + 2)..... 2!! = 2 × 3 = (3!)/(1!)

(2n – 1) =
{(2n − 1)!} 3!! = 3 × 4 × 5 = (5!)/(2!)
{(n − 1)!} 4!! = 4 × 5 × 6 × 7 = (7!)/(3!)
1. (b) Hence 25!! = (49!)/(24!) Hence (1!!)(2!!)(3!!)....(20!!)
Since, number of trailing zeros (or  {(1!)(3!)(5!)(7!)(9!)……. (39!)}
=   
highest power of 10) in 49! is 10  {(0!)(1!)(2!)….(19!)} 
and that in 24! is 4 hence number  {(21!)(23!)…(39!)}
=    
of trailing zeros in (49!)/(24!) is  {(2!)(4!)…(18!)} 
10 – 4 = 6 Since number of trailing zeros is
199! same as highest power of 10 or that
2. (d) From the definition 100!! =
99!
of 5.
Highest power of 10 in 199! is 47 and
Consider numerator–Highest power
that in 99! is 22 hence highest power
of 5 in 21!, 23! Is 4
199!
of 10 in is 47 – 22 = 25 and
99! Highest power of 5 in 25!, 27!, and
therefore required highest power of 29! is 6
100 in 100!! is 12. Highest power of 5 in 31! and 33!
179! is 7
3. (a) From the definition 90!! =
89!
Highest power of 5 in 35!, 37!, and
Since last two non-zero digits of 179!
39! is 8
Is 24 and it has 43 zeros at the end,
similarly last two digits of 89! Is 68 Hence highest power of 5 in numerator
and it has 20 zeros at the end. is 4 × 2 + 6 × 3 + 7 × 2 + 8 × 3 = 8 +
18 + 14 + 24 = 64
Hence when we divide 179! by 89!
We will have 43 – 20 = 23 zeros at Now consider denominator – Highest
the end and then non-zero digit is power of 5 in 2! And 4! is 0.
given by unit digit of 24/68 which is 8 Highest power of 5 in 6!, and 8! is 1
4. (d) From the definition (n!!) = (n)(n + 1) Highest power of 5 in 10!, 12!, and
14! is 2
{(2n − 1)!}
(n + 2).....(2n – 1) = Highest power of 5 in 16!, and 18! is 3
{(n − 1)!}
Introduction  33
Hence highest power of 5 in Now consider 333!
numerator is 1 × 2 + 2 × 3 + 3 × 2 = 14 5 333 R1 =3 andQ1=T[331 ×332 ×333=36
Hence required highest power of 5 5 66 R2 = 1 and Q2= T[66] = 66
or highest power of 10 or number of 5 13 R3 = 3 and Q3= T[11 × 12 ×13]=16
trailing zeros is 64 – 14 = 50 2
5. (a) Since largest two digit prime number Hence highest power of 5 in 333! is
is 97, P = 66 + 13 + 2 = 81
Consider 4!! = 4 × 5 × 6 × 7 Hence T[333!] = T[1281] × T[36 × 66
= (7!)/(3!) × 16 × 2] = T[12 × 32] = 84
Similarly, 5!! = 5 × 6 × 7 × 8 × 9 Hence require last two digit of 96 ×
= (9!)/(4!) 84 = 64

To be divisible by 97 minimum value 8. (a) Consider largest two digit prime


of k would be 49 number 97, highest power of 97
in 300!, 200!, and 100! is 3, 1 and
49!! = (97!)/(47!) and it is divisible by 97.
2 respectively hence (300!)/(200!)
6. (d) With normal method we will find
(100!) is not divisible by 97. Any
highest power of 5 in 137!
prime number more than 75 will
5 127 R1 = 2 Q1= T[146 ×147] = 62 have similar condition, now consider
5 29 R2 = 4 Q2= T[26×27×28×29] = 24 a prime number less than 75, i.e 73.
5 5 R3 = 0 Now, highest power of 73 in 300!,
1 200!, and 100! is 4, 2 and 1 hence
Hence highest power of 5 in 147! is P exponent of 73 in N is 1. This condition
= 29 + 5 + 1 = 35 is satisfied by all the prime numbers
So T(137!) = T[1235] × T[62 × 24] in the range of 66 < N < 75, these
= T[68 × 62 × 24] = 84 prime numbers are 67, 71 and 73. So
total 3 prime numbers in this range.
7. (b) Consider 222!, we will find last two
non-zero digit of 222! Any prime number more than 66 will
have similar conditions, now consider
5 222
a prime number less than 66, i.e 61.
5 44 R1 = 2 and Q1= T[221× 222] = 62
Highest power of 61 in 300!, 200!,
5 8 R2 = 4 and Q2 = T[41 × 42 ×43 ×44] =24
1 R3 = 3 and Q3 = T[6 × 7 × 8] = 36 and 100! is 4, 3 and 1 hence exponent
of 61 in N is 1.This condition is
Hence highest power of 5 in 222! is P
satisfied by all the prime numbers
= 44 + 8 + 1 = 53
in the range of 60 < N < 66, 61 is the
Hence, T[222!] = T[1253] × T[62 × 24
only prime numbers in this range.
× 36] = T[72 × 68] = 96
34  Theory of Counting
Now consider prime number in the
=  (2400 )(3400 )(200!) 
range of 50- 59, Highest power of 59  100 100 200 100

 {(2 )(3 )(100!)}{(2 )(3 )(100!)} 
in 300!, 200!, and 100! Is 5, 3 and 1
100 200
hence exponent is1. = (2 )(3 )(200!)
We can sum up this in a tabular (100!)(100!)
format. Now we have to find largest two digit
200!
Range Prime High- High- High- Expo- prime factor of N i.e of
Num- est est est nent of (100!)(100!)
bers power power power Prime since any prime number more
in in in num- than 66 will have highest power in
300! 200! 100! ber
200!, and 100! Is 2 and 1 hence its
75–99 3 2 1 0 exponent in the given number will
be 0, Now consider a prime number
66–75
67,
4 2 1 1 less than 66, i.e 61, its highest power
71, 73 in 200! and 100! Is 3 and 1 hence its
exponent in the given number will
60–65 61 4 3 1 0
be 1 and it is a factor of N.
This condition is satisfied by all
50–59 53, 59 5 3 1 1
the prime numbers in the range of
50-66, these prime numbers are 53,
Hence there are total 5 prime numbers exist. 59 and 61. So total three such prime
9. (c) From the given condition numbers exist.
10. (c) Since last two non-zero digit of 333!
A =  (36 200 )(200!) =  (2 400 )(3 400 )(200!) is 84 and that of 222! is 96,

Let us assume that required last two
And B = (6100 )(100!) = (2100 )(3100 )(100!) non-zero digit is k then last two digit
of 96 × k should be 84.
C = (12100 )(100!) = (2200 )(3100 )(100!) From the given options only 04
Hence the given number N = A/BC satisfies the condition.
2
Selection (Combination)  35

Selection
(Combination)

Topics Covered

 Introduction of Selection/Combination

 Selection of one or more

 Selection from identical articles

 Factor Theory

Part A: Topic Number of Questions


Solved Example 10
Concept Applicator 20
Concept Builder 20
Concept Cracker 30
Part B: Topic Number of Questions
Concept Deviator 25
Concept Eliminator 7
Total 112
36  Theory of Counting
Part A
Introduction of
Concept 1 Selection/Combination
Let us assume that Mayank has 4 friends A,  Number of ways of selecting ‘r’
B, C & D and he wants to invite 2 of them, so articles from ‘n’ distinct article if 1
he has to select 2 friend out of 4 friends, the article is always included is n-1Cr-1
question is in how many ways we can select ( n − 1)!
=
2 friends out of 4 friends. If we list down all { }
(n − r − 2)! (( r − 1)!)
the possible combination then we will get Example 2:  In how many ways Mayank
(A, B), (A, C), (A, D), (B, C), (B, D) & (C, D). Here can call 4 of his friends out of 10
selection of (A, B) is same as selection of (B, A). friends if one friend Rahim is always
In this case we had only 6 possible combination included?
hence we could list it down but if possibilities Solution:  Here Mayank has to select 4 friends
are more then it is not possible for us to list out of 10 but Rahim is always included hence
it down. he has to select only (4 – 1) = 3 friends from
Take another example: Consider ten points (10 – 1) = 9 friends, so he can do it in 9C3
lie on a circle. If we have to find the number ways = (9!)/(6!)(3!) =(7 × 8 × 9)/(6) = 84 ways.
of chords that can be drawn by joining these  Number of ways of selecting ‘r’
points pair wise. The number of chords is articles from ‘n’ distinct article if ‘k’
same as combination/selection of 2 out of 10 article is always included is n–kCr–k
points. ( n − k )!
=
 Number of ways of selecting ‘r’ { }
(n − r − 2k )! (( r − k )!)
articles from ‘n’ distinct article is
Example 3:  In how many ways Mayank
n!
nC = can call 4 of his friends out of 10
r
{
(n − r )! (r !)} friends if friends Rahim & Salim
Example 1:  In how many ways Mayank always included?
can call 4 of his friends out of 10 Solution:  Here Mayank has to select 4
friends? friends out of 10 but Rahim and Salim
always included hence he has to select only
Solution:  Here Mayank has to select 4 friends
(4 – 2) = 2 friends from (10 – 2) = 8 friends,
out of 10 so he can do it in 10C4 ways = (10!)/ so he can do it in 8C2 ways = (8!)/(6!)(2!)
(6!)(4!) =(7 × 8 × 9 × 10)/(24) = 210 ways. = (7 × 8)/(2) = 28 ways.
Selection (Combination)  37

Concept 2 Selection of One or More

 The total number of combinations ….


of ‘n’ different things taken one or ….. Similarly
more at a time is 2n – 1 ‘n’ articles out of ‘n’ distinct article can be
Proof: Consider the 1st thing there are two selected in nCn ways
ways we can deal with, it may be included or since we can select either 1 OR 2 OR 3…OR
excluded. Similarly there are 2 ways that we ‘n’ articles hence from fundamental principal
can deal with 2nd article and so on for 3rd and of summation total number of ways is nC1 +
next articles. nC + ….. + nC = 2n–1 (Property of binomial
2 n
Now from fundamental principal of multiplica- co-efficients)
tion total number of ways is 2 × 2 × 2 × 2 × ... Hence the total number of ways of
× 2 (n times) = 2n .
selecting one or more at a time out of ‘n’
Now consider the number of ways in which different things is 2n-1
nothing is selected is 1 × 1 × 1 × … × 1 = 1 way.
Hence the total number of ways of selecting Example 4:  Mr. Ramchandran has to
one or more at a time out of ‘n’ different things select one or more students from 10
is 2n – 1. students for a project, how many ways
he can make this selection?
Alternately Solution:  Consider how many options for
Total number of distinct article provided is ‘n’ each student he has, for 1st student he has
and we have to select one or more from these 2 option (either select or reject) similarly for
‘n’ article. 2nd student 2 ways and so on. so total number
One article out of ‘n’ distinct article can be of ways is 210 = 1024 ways, but out of these
selected in nC1 ways 1024 ways there is one way when he has
rejected all the students which is according
Two articles out of ‘n’ distinct article can be
to the given condition as he has to select
selected in nC2 ways
some students for his project, hence required
Three articles out of ‘n’ distinct article can be number of ways is 210 – 1 = 1023 ways.
selected in nC3 ways

QR Code / Video Link

For Smart phone/ Tablet users

Video Link for Desktop/Laptops users http://dishapublication.com/video-resources


38  Theory of Counting
Alternately ….
Total number of students is ‘10’ and we have to ….. Similarly
select one or more from these ‘10’ students. 10 students out of ‘10’ students can be selected
One student out of ‘10’ students can be selected in 10C1 ways
in 10C1 ways Since we can select either 1 OR 2 OR 3…OR
Two students out of ‘10’ students can be 10 students hence from fundamental principal
selected in 10C2 ways of summation total number of ways is 10C1 +
10C + ….. + 10C = 210 – 1 = 1023.
Three students out of ‘10’ students can be 2 n
selected in 10C3 ways

Selection from Identical


Concept 3 Articles
The total number of ways of selecting one Hence number of ways of selecting zero or
or more things from ‘p’ identical things more things out of p identical things = p + 1
of one type, ‘q’ identical things of another Similarly, number of ways of selecting zero or
type, ‘r’ identical things of 3rd type and ‘n’ more things out of q identical things = q + 1
distinct things is (p + 1)(q + 1)(r + 1)2n –1 Number of ways of selecting zero or more
things out of r identical things = r + 1
Since number of ways of selecting r things out
Also number of ways of selecting zero or more
of n identical things = 1 for all r ≤ n. things out of n different things = 2 × 2 × 2 ×
Hence number of ways of selecting one thing … n times = 2n
out of p identical things = 1 Therefore, number of ways of selecting zero or
Number of ways of selection two things out of more things out of given things = ( p + 1)(q + 1)
(r + 1 ) 2n.
p identical things = 1
But number of ways of selecting zero thing
Similarly number of ways of selecting p things out of given things = 1 × 1 × 1 × 1n = 1.
out of p identical things = 1 Thus total number of ways of selecting one or
Number of ways of selecting zero thing out of more things out of given = ( p + 1) (q + 1) (r + 1)
p identical things = 1 × 2n – 1.

QR Code / Video Link

For Smart phone/ Tablet users

Video Link for Desktop/Laptops users http://dishapublication.com/video-resources


Selection (Combination)  39
Example 5:  In how many ways one or And number of ways of selecting one or more
more fruits can be selected from a fruit from 3 distinct fruits is 23 ways.
basket that has 4 identical apples, 5
identical bananas and remaining 3 are Out of these there is one way where we have
of different types of fruits. not selected any fruit we have to subtract that
Solution:  Number of ways of selecting one or case from total number of case, hence required
more from 4 identical apples is (4 + 1) = 5.
number of ways is (4 + 1)(5 + 1)23 – 1 = 5 × 6
Number of ways of selecting one or more bananas
from 5 identical banana is (5 + 1) = 6 ways × 8 – 1 = 239 ways.

Concept 4 Factor Theory


NUMBER OF FACTORS OF A NUMBER (iii) A factor of above number have highest
To find number of factors of a composite power of 2, 3 and 5 is 3, 5 and 4
number write down the number in the form respectively.
of n = apbqcr…. where a, b and c are prime (iv) A factor of above number has power of 2 as
numbers , and p, q and r are natural number 20 or 21 or 22 or 23 (total 3 + 1 =4 ways)
then number of factors of n is given by (p + 1) (v) A factor of above number has power of 3
(q + 1)(r + 1)….. and it includes 1 and number
as 30 or 31 or 32 or 33 or 34 or 35 (total
itself.
5 + 1 = 6 ways)
Explanation of above formula:
(vi) A factor of above number has power of 5 as 50
Let us take an example of N = 233554 now 1st
or 51 or 52 or 53 or 54 (total 4 + 1 = 5 ways)
observe some facts about the factors of this
number:- Hence number of factors is given by (3 + 1)
(4 + 1)(5 + 1) = 4 × 5 × 6 = 120
(i) A factor of above number must have
prime factor either 2, 3 or 5 or none (In Now alternate way we can explain the same
case of 1) thing as:
(ii) A factor of above number is not divisible A Factor of given number must have prime
by any prime number other than 2, 3 or 5. factors either 2, 3, or 5

QR Code / Video Link

For Smart phone/ Tablet users

Video Link for Desktop/Laptops users http://dishapublication.com/video-resources


40  Theory of Counting
So a factor must be Hence a factor must be
2(0 or 1 or 3(0 or 1 or 2 5(0 or 1 or 2
2(1 or 3(0 or 1 5(0 or 1 7(0 or 1 or 2
2 or 3) × or 3 or 4 or 5) × or 3 or 4)
2 or 3) × or 2) × or 2 or 3) × or 3 or 4)
→

→

→

→

→

→

→

(3 + 1 = (5 + 1 = (4 + 1 = (3 ways) (2 + 1 = (3 + 1 = (4 + 1 =
4 ways) 6 ways) 5 ways) 3 ways) 4 ways) 5 ways)
Hence total number of factors is 4 × 6 × 5 = 120. Hence total number of factors = (3)(2 + 1)(3 + 1)
Example 6:  Find the number of factors of (4 + 1) = 180
23325374 Hence number of even factors of a number
Solution:  A factor of above number must N = 2paqbrcs is = p(q + 1)(r + 1)(s + 1)
have 2, 3, 5 or 7 as prime factors.
Example 8:  Find the number of odd
Hence a factor must be factors of 23325374
2(0 or 1 or 3(0 or 1 5(0 or 1 7(0 or 1 or 2 Solution:  From Example 6 we have seen that
2 or 3) × or 2) × or 2 or 3) × or 3 or 4)
total number of factors of N is 240 and from
→

→

→

→

example 7 total number of even factors is



180 hence number of odd factors = 240 –
(3+1= (2+1= (3 + 1 = (4 + 1 =
180 = 60
4 ways) 3 ways) 4 ways) 5 ways)
Hence number of odd factors = Total number
Hence total number of factors = (3 + 1)(2 + 1) of factors – Number of even factors
(3 + 1)(4 + 1) = 240
Alternate way:  since odd factors should
Number of factors have power of 2 as 0.
Hence a factor must be
Number of factors of number N = ap bqcrds …
is given by (p + 1)(q + 1)(r + 1)(s + 1). 2(0) × 3(0 or 1 × 5(0 or 1 × 7(0 or 1 or 2
or 2) or 2 or 3) or 3 or 4)

Different Types of Factors


→

→
→
→


Here in this section we will learn how to find
out some specific types of factors like even (1 way) (2+1= (3+1= (4+1=
factors, odd factors, factors divisible by 10, by 3 ways) 4 ways) 5 ways)
12 etc. We will discuss this concept with the
help of an example:- Hence total number of odd factors = (2 + 1)
(3 + 1)(4 + 1) = 60
Example 7:  Find the number of even Hence number of odd factors of a number
factors of 23325374 N = 2paqbrcs is = (q + 1)(r + 1)(s + 1).
Solution:  In this case we have to find number
of even factors, an even factor is divisible Example 9:  Find the number of factors of
by 2 or smallest power of 2 is 1 and not 0 as 23325374 that ends with 0 (Or divisible
in the case of total number of factors. by 10)
Selection (Combination)  41
Solution:  If a number is divisible by 10 then it Hence total number of factors divisible by 10
must have minimum power of 2 and 5 as1. is = (3)(2 + 1)(3)(4 + 1) = 135
Hence a factor divisible by 10 must be
Example 10:  Find the number of factors
2(1 or 2 × 3(0 or 1 × 5(1 or × 7(0 or 1 or 2 of 23325374 that are not divisible by 10.
or 3) or 2) 2 or 3) or 3 or 4)
Solution:  From example 6 total number of
→

→

→

→
factors is 240, oust of these 135 factors are

divisible by 10 hence remaining 240 – 135
(3 ways) (2+1= (3 ways) (4+1=
= 105.
3 ways) 5 ways)

1
Concept Applicator (CA)
Ideal Time Apply your concepts with easy and
20 Min. conceptual questions

1. In how many ways can I call 4 friends (a) 20C10 (b) 18C09
out of my 10 friends for dinner? 19
(c) C10 (d) None of these
(a) 210 (b) 250 6. In how many ways 11 players can be
(c) 180 (d) None of these selected out of 20 players if Sachin and
2. In how many ways 5+ students out of 10 Sourav are always selected?
students can be selected? (a) 20C10 (b) 18C09
19
(c) C10 (d) None of these
(a) 210 (b) 252
(c) 180 (d) None of these 7. In how many ways 11 players can be
3. In how many ways a team of 3 boys and selected out of 20 players if Sachin and
4 girls out of total 7 boys and 9 girls can Sourav are always rejected?
be formed? (a) 18C11 (b) 18C09
(a) 3150 (b) 4375 (c) 19C10 (d) None of these
(c) 4410 (d) None of these 8. In how many ways 11 players can be
selected out of 20 players if Sachin and
4. A work can be completed by 3 men or Sourav are always selected but Yuvi and
5 women in 5 days. In how many ways Dinesh always rejected?
workforce can be selected if work has
to be finished in 5 days and available (a) 18C11 (b) 16C09
19
(c) C10 (d) None of these
workforce is 10 men and 8 women? If
workforce has either only men or only 9. In class VII there are three sections A,
women. B and C with number of students are
(a) 210 (b) 240 40, 60 and 80 respectively while in class
(c) 180 (d) None of these VIII there are three sections A, B and
5. In how many ways 11 players can be C with number of students 60, 80, and
selected out of 20 players if Sachin is 40 respectively. In how many ways 30
always selected? students can to be selected either from
42  Theory of Counting
class VII or VIII equal from section A, B (a) 31 (b) 64
and C. (c) 63 (d) None of these
(a) (40C10 × 60C10 × 80C10) 15. Amit appeared in his board exam, In
× (60C10 × 80C10 × 40C10) his maths paper he got 10 subjective
(b) ( C30 × C30 × 80C30)
40 60 questions, what is the total number of
+ ( 60C30 × 80C30 × 40C30) ways that he can answer this paper?
(c) ( C10 × C10 × 80C10)
40 60 (a) 1024 (b) 1023
+ ( 60C10 × 80C10 × 40C10) (c) 512 (d) None of these
(d) None of these 16. In a class of 10 students 6 are boys and 4
10. In how many ways 5 distinct alphabets are girls, in how many ways class teacher
can be selected so that no vowels be can select student/s for a project?
selected? (a) 1024 (b) 1023
(a) 26C5 (b) 21C5
20 (c) 512 (d) None of these
(c) C5 (d) None of these
11. Out of 6 apples, 5 mangoes and 4 bananas 17. In a class of 10 students 6 are boys and 4
how many selection of fruits can be made? are girls, in how many ways class teacher
(a) 210 (b) 120 can select student/s for a project such that
(c) 119 (d) None of these the group has at least 1 boy and 1 girl?
12. Out of 6 apples, 5 mangoes and 4 bananas (a) 1024 (b) 1023
how many selection of fruits can be made (c) 512 (d) None of these
such that at least one mango is always 18. In a fruit basket 4 mangoes, 5 bananas,
included? and 4 different types of fruits are kept,
(a) 175 (b) 120 in how many ways one can select fruits
(c) 119 (d) None of these from this fruit basket?
13. Out of 6 apples, 5 mangoes and 4 bananas (a) 480 (b) 479
how many selection of fruits can be made (c) 240 (d) None of these
such that at least one fruit of each type is 19. Find the number of factors of 1200
always included?
(a) 30 (b) 24
(a) 175 (b) 120
(c) 32 (d) 36
(c) 119 (d) None of these
20. How many factors of 1800 other than 1800
14. In an examination a student has to write are 4 digit numbers?
6 papers and has to pass in each paper
to pass the exam. In how many ways a (a) 3 (b) 2
student can fail in the exam? (c) 1 (d) 0

Response Grid
1. a b c d 2. a b c d 3. a b c d 4. a b c d 5. a b c d
6. a b c d 7. a b c d 8. a b c d 9. a b c d 10. a b c d
11. a b c d 12. a b c d 13. a b c d 14. a b c d 15. a b c d
16. a b c d 17. a b c d 18. a b c d 19. a b c d 20. a b c d
Selection (Combination)  43

2
Concept Builder (CB)
Ideal Time Revise your concepts with questions
30 Min. medium difficulty level questions

1. In how many ways playing 11 can be (a) 20C10 – 19C9 – 18C8


selected from 15 players if out of 15 (b) 20C10 – 18C8 + 1
players, 2 players can play only as wicket
(c) 20C10 – 18C8 – 1
keeper?
(d) None of these
(a) 2C1 × 13C10 (b) 2C1 × 14C11
(c) 2C1 + 13C10 (d) None of these 7. Find the number of ways of selecting
10 students from a group of 30 students
2. A committee of 3 men and 3 women is to
such that the oldest and the youngest
be selected from 6 men and 7 women. In
students are always included in the
how many ways this can be done?
group.
(a) 6C3 × 7C3 (b) 6C3 × 13C3
(a) 30C10 – 28C8 (b) 28C8 – 1
(c) 6C3 + 7C3 (d) None of these
(c) 28C8 (d) None of these
3. A committee of 3 men and 3 women is
8. In how many ways at least 4 students can
to be selected from 6 men and 7 women.
be selected from a group of 6 students?
In how many ways this can be done if a
particular man is always selected? (a) 21 (b) 20
(a) 6C3 × 7C3 (b) 5C2 × 7C3 (c) 22 (d) None of these
(c) 6C3 + 7C3 (d) None of these 9. In how many ways a team of k students
can be selected from n students such
4. A committee of 3 men and 3 women is
that r students are always selected and
to be selected from 6 men and 7 women.
p students are always rejected?
In how many ways this can be done if a
particular man is always selected and a (a) (n – r – p)Ck–r
woman is always rejected? (b) (n – r – p)Ck–r –1
(a) 5C2 × 6C3 (b) 5C2 × 7C3 (c) (n – r – p)Ck–r +1 `
(c) 6C3 + 7C3 (d) None of these (d) None of these
5. There are n stations on a railway line. 10. In a party 10 students gave gift to each
If the number of types of tickets printed other then find the total number of
(for one way journey) is 210, then find the gifts?
number of stations on the railway line. (a) 10C2 (b) 210C2
10
(c) ( C2)/2 (d) None of these
(a) 16 (b) 20
(c) 25 (d) None of these 11. In a board exam there are two sections
6. In how many ways 10 students from a each section has 5 questions. As per the
group of 20 students can be selected given condition a candidate has to answer
if Rajesh and Dinesh are not selected any 6 questions out of 10 questions such
together? that not more than 4 questions selected
44  Theory of Counting
from one section. In how many ways a teacher can select student/s for a project
student can make up his choice to answer such that the group has at least 2 boys
the question paper? and 2 girls?
(a) 100 (b) 150 (a) 1024 (b) 1023
(c) 200 (d) None of these (c) 512 (d) None of these
12. From 8 boys and 4 girls a team of 5 is to 17. In a fruit basket 4 mangoes, 5 bananas,
be formed. In how many ways can this be and 4 apples are kept, in how many
done so as to include at least 1 girl in the ways one can select fruits from this fruit
team? basket such that at least one fruit of each
12C
type is always selected?
(a) 5 – 8C5 + 1 (b) 12C
5 – 8C 5
12C
(a) 80 (b) 79
(c) 5 – 8C5 – 1 (d) None of these
(c) 150 (d) None of these
13. From 8 boys and 4 girls a team of 5 is to
18. In a fruit basket 4 mangoes, 5 bananas,
be formed. In how many ways can this be
and 4 apples are kept, in how many ways
done so as to include at least 1 girl in the
one can select fruits from this fruit basket
team and exclude at least 1 boy from the
such that at least one 1 mango, 2 bananas
team?
and 3 apples are always selected?
(a) 12C – 8C5 + 1 (b) 12C5 – 8C5
5
12C – 8C – 1 (a) 80 (b) 79
(c) (d) None of these
5 5 (c) 32 (d) None of these
14. In how many ways 5 vowels and 5
19. There are 6 questions in an examination,
consonants can be selected from 26
a student has to answer at least three
letters?
question to pass the exam, in how many
(a) 26C (b) 5C121C5
10 ways student can fail the exam?
(c) 21C
5 (d) None of these (a) 20 (b) 31
15. What is the total number of ways of (c) 22 (d) None of these
selecting fruits with at least one apple
20. Let x be the number of factors of 2000
from 4 distinct apples and 3 distinct and y be the number of factors of 3000
bananas? and z be the number of common factors
(a) 120 (b) 105 of 2000 and 3000, then find the ratio of
(c) 119 (d) None of these (x – z):(y – z).
16. In a class of 10 students 6 are boys and (a) 2:3 (b) 3:2
4 are girls, in how many ways class (c) 1:2 (d) 1:4

Response Grid
1. a b c d 2. a b c d 3. a b c d 4. a b c d 5. a b c d
6. a b c d 7. a b c d 8. a b c d 9. a b c d 10. a b c d
11. a b c d 12. a b c d 13. a b c d 14. a b c d 15. a b c d
16. a b c d 17. a b c d 18. a b c d 19. a b c d 20. a b c d
Selection (Combination)  45

3
Concept Cracker (CC)
Ideal Time Boost up your confidence with good
60 Min. questions

1. Out of 6 boys and 4 girls a team of 5 is to at least 1 girl and two girl volunteers
be formed such that at least one girl is Mahima and Nita refuse to work
included in the team, in how many ways together. In how many ways committee
such a team can be formed? can be formed?
(a) 252 (b) 246 (a) 150 (b) 144
(c) 562 (d) None of these
(c) 190 (d) None of these
2. In Independence Day party every student
gave a gift to every other student. If total 7. For 1st stage of scholarship
number of gift distributed is 240 then 6 students has to be selected such that
how many students were present in the 3 students from mathematics group and
party? 3 students from biology group. Number
(a) 16 (b) 15 of participants in different group is as
(c) 32 (d) None of these follows-
3. In Independence Day party every student
Mathematics group: Total 7 students
gave a gift to every other student. If total
number of gift distributed is 240 then – 3 boys and 4 girls
how many gifts received by a participant Biology group: Total 7 students – 4
Rakesh? boys and 3 girls.
(a) 16 (b) 15
(c) 32 (d) None of these How many ways group of 6 students
4. In a party there are 10 men and 15 women is selected if it has 3 boys and 3 girl
danced such that in a particular dance students?
only 1 man and 1 woman participated. (a) 485 (b) 440
Total how many dance was performed?
(a) 150 (b) 144 (c) 490 (d) None of these
(c) 300 (d) None of these 8. There are 8 singers who is ready to
5. If a bus conductor has 190 different perform in a show that is scheduled
tickets then find the number of stoppage
for two timings, before lunch and after
that the bus has consider only one way
journey. lunch. If before lunch maximum 5 singers
can perform while after lunch maximum
(a) 19 (b) 20
4 singers can perform then in how many
(c) 22 (d) None of these
different ways organizer of the event can
6. At praxis business school Kolkata, a schedule the program?
committee of 5 has to be formed from 10
volunteers out of which 6 are boys and (a) 122 (b) 126
4 are girls. The committee should have (c) 142 (d) None of these
46  Theory of Counting
9. How many teams of 11 players can be girls is always more than the number of
made from 25 players if players Rajesh, boys?
Rajni and Riju can not be selected (a) (4C3 × 6C2) × (4C4 × 6C1)
together? [Two of them can be together]
(b) 4C3 × 6C1 + 4C4 × 6C1
(a) 25C 22C 25C 22C
11 – 8 (b) 11 – 11 (c) 4C3 × 6C2 + 4C4 × 6C1
(c) 25C 25C
11 – 8 (d) None of these (d) None of these
10. Consider a set S = { 100, 101, 102, …. 199} 14. 10 singers were selected to perform in a
a number machine has done following function, but at a time only 3 singers
operations- can perform, then total how many times
Step (i) select 2 numbers at random from performance was done if no same group
the set S of 3 students performed twice?
(a) 10C
Step (ii) Multiply the two numbers 3
selected in step (i) and the result R is the (b) 10C × 10C1 × 10C1
1
output of the machine. (c) 10C × 9C 1 × 8C 1
1
In how many ways selection can be (d) None of these
done at 1st step if the final result R is a
multiple of 3. 15. 10 singers were selected to perform in
33C
a function, but at a time only 3 singers
(a) 1 × 33C1 + 33C1 × 67C1 can perform, then how many times a
(b) 33C 67C + 32C × 66C
1× 1 1 1 particular singer performed if no same
(c) 33C × 33C1 + 33C1 × 67C1
1 group of 3 students performed twice?
(d) None of these (a) 10C (b) 9C2
2
11. In a group picnic 10 married couples (c) 9C × 8C 1 (d) None of these
1
planned to play mixed doubled game in 16. Mr. Raman has 10 friends and he wants
tennis. The condition is that no husband to invite his friends for a get together.
wife pair is present in the game. Then
At a time he can invite only 4 out of
what is the total number of game
10 friends. If a particular friend of Mr
played?
Raman, Mr. Singh came ‘p’ times and
(a) 10C × 8C2 (b) 2 × 10C2 × 8C2
2 out of those ‘p’ times Mr. Singh came ‘q’
(c) 2 × 10C2 × 6C2 (d) None of these times with Mr. Raj, another friend of Mr.
12. Playing 11 has to be selected from 15 Raman. By what percentage ‘p’ is more
players which has 5 bowlers what is the than ‘q’?
number of ways of selecting 11 players (a) 55% (b) 120%
such that at least 4 bowlers are always (c) 150% (d) None of these
selected?
17. Mr. Raman has 10 friends and he wants
(a) 5C4 × 10C7 + 5C
5 × 10C6 to invite his friends for a get together.
(b) 5C4 × 10C7 + 5C
3 × 10C6 At a time he can invite only 4 out of 10
(c) 5C
4 × 9C 7+ 5C
5 × 10C6 friends. How many times a particular
(d) None of these friend Ricky came with Susmit but not
with Soumyajyoti?
13. In a group of 10 students comprising 6
boys and 4 girls, in how many ways we (a) 18 (b) 28
can select a team of 5 so that number of (c) 21 (d) None of these
Selection (Combination)  47
18. ICICI bank has 12 vacancies out of (a) 32 (b) 41
which 3 seats are reserved for women
(c) 36 (d) None of these
applicants if there are total 25 applicants
20 of them are gentlemen and 5 of them 23. Total 16 people participated in a picnic,
are women then in how many ways 12 6 peoples are to be selected for a work
applicants can be selected? such that exactly one from Mr. & Mrs. A
(a) 5C3 × 22C9 is selected similarly one from Mr. & Mrs.
B and Mr. & Mrs. C is selected for the
(b) 5C3 × 20C9 + 5C4 × 20C8 + 5C5 × 20C7 work. How many ways this can be done?
(c) 5C × 22C9 + 5C4 × 21C8 + 5C5 × 20C7
3 (a) 960 (b) 480
(d) None of these
(c) 120 (d) None of these
19. At an election three wards of a town are
24. Consider a universal set S with n
canvassed by 4, 5 and 8 men respectively. If
elements, set S = { a1 , a2 , … , an} what
there are 20 volunteers, in how many ways
is the number of subsets of this set which
can they be allotted to different wards?
contain odd number of elements?
(a) 20C × 16C5 × 11C8
5
(a) 2n – 1 (b) 2n – 1
(b) 20C 16C × 11C
4× 5 8
(c) 2n – 2 (d) None of these
(c) 20C 16C + 11C
4+ 5 8
25. In a class the number of students is
(d) None of these
(2n + 1), in how many ways at most n
20. Given 5 different green dyes, 4 different students can be selected from these
blue dyes and 3 different red dyes. How students?
many combinations of dyes can be chosen
(a) 2n (b) 2n + 1
taking at least one green and one blue dyes.
(a) 54 (b) 3720 (c) 2n – 1 (d) 22n
(c) 4904 (d) None of these
26. A fruit basket has ‘n’ different types of
21. Mr. Rahim has one 5 rupee, one 10 rupee, fruits and each type has ‘k’ number of
one 20 rupee and one 50 rupee note, what fruits. What is the number of ways of
is the number of different sums that can selecting one or more fruits from the
be made from these notes? basket?
(a) 16 (b) 31 (a) kn – 1 (b) (k + 1)n –1
(c) 32 (d) None of these
(c) (n + 1)k – 1 (d) None of these
22. In Praxis Business school, during an
election of mess committee there 27. How many numbers less than 10000 has
are 3 seats of mess committee and 6 only 3 factors?
contestants. A student can not vote (a) 15 (b) 50
more than 3 votes, in how many ways a
student can vote? (c) 25 (d) None of these
48  Theory of Counting
28. How many 2 digit numbers exist whose (a) 576 (b) 288
number of factors is 8 (c) 625 (d) None of these
(a) 8 (b) 10 30. Find the sum of all the factors of 7200
(c) 12 (d) 14 that ends with 5.
29. How many factors of N = 25 × 35 × 55 × (a) 403 (b) 523
75` ends with 2 zeroes. (c) 925 (d) 3053

Response Grid
1. a b c d 2. a b c d 3. a b c d 4. a b c d 5. a b c d
6. a b c d 7. a b c d 8. a b c d 9. a b c d 10. a b c d
11. a b c d 12. a b c d 13. a b c d 14. a b c d 15. a b c d
16. a b c d 17. a b c d 18. a b c d 19. a b c d 20. a b c d
21. a b c d 22. a b c d 23. a b c d 24. a b c d 25. a b c d
26. a b c d 27. a b c d 28. a b c d 29. a b c d 30. a b c d
Selection (Combination)  49

Solutions
Concept Applicator (CA)
1. (a) 2. (b) 3. (c) 4. (d) 5. (c) 6. (b)
7. (a) 8. (b) 9. (c) 10. (b) 11. (d) 12. (a)
13. (b) 14. (c) 15. (b) 16. (b) 17. (d) 18. (b)
19. (a) 20. (d)

1. (a) Here we have to select 4 friends out Case (i): When work is completed
of 10 so it can be done in 10C4 ways, by Men then number of ways of
10! 7 × 8 × 9 × 10 selecting workforce (number of ways
10C = = =  210 of selecting 3 men out of 10 men) is
4 (6!)(4!) 1×2×3×4
10! 8 × 9 × 10
2. (b) Here we have to select 5 students out of 10C = = =1
  20
10 so it can be done in 10C5 ways, 10C5 3 (7!)(3!) 1×2×3
10! 6 × 7 × 8 × 9 × 10 Case (ii) When work is completed
= = =  252 by Women then number of ways of
( )( ) 1 × 2 × 3 × 4 × 5
5! 5!
selecting workforce (number of ways
3. (c) Here in this case we have to select of selecting 5 women out of 8 women)
3 boys and 4 girls out of total 7 boys
and 9 girls. 8! 6×7×8
is 8C5 = = =  56
Out of 7 boys, 3 boys can be selected in ( )( ) × 2 × 3
5! 3! 1
Now from fundamental principal
7C 7! 5×6×7 of addition total number of ways is
3 ways or = = = 35
(4!)(3!) 1 × 2 × 3 120 + 56 = 176
Out of 9 girls, 4 girls can be selected 5. (c) Here in this case it is given that
in 9C4 ways or total number of players is (n) =
20, since Sachin is always selected
9! 6×7×8×9
= = = 126 hence available number of player

(4!)(5!) 1 × 2 × 3 × 4 is 19 (About which selectors can
From fundamental principal of take decision) now from 19 we have
multiplication total number of ways to select only 10 (since one player
is 35 × 126 = 4410 Sachin is always selected so now
selectors have to select only 10),
4. (d) Since work can be completed by 3
hence number of ways is 19C10
men OR by 5 women [Here this OR
suggest that we have 2 cases and we 6. (b) Explanation is similar to the above
have to use fundamental principal of question option (b) 18C09 is the
summation] correct answer.
50  Theory of Counting
7. (a) Similar to question number 5, here 13. (b) Number of ways that we can deal
selectors have to select 11 out of with apples is 6.
available 18 players and that can be Number of ways that we can deal
done in 18C11 ways. with mangoes is 5.
8. (b) Here out of available 16 players, Number of ways that we can deal
selectors have to select only 9 players with bananas is 4.
and that can be done in 16C9
So number of ways of selecting fruits
9. (c) Here also we have two cases, is 6 × 5 × 4 = 120.
Equal number of students means
14. (c) Consider the total possibilities,
10 students from each section.
Let the 1st subject there are two
Case (i) If students are selected from
options either he can pass or he can
Class VII then number of ways is
40C × 60C × 80C fail, similarly there are two options
10 10 10 for 2nd subject and so on. So total
Case (i) If students are selected from number of ways is 2 × 2 × 2 × 2 ×
Class VII then number of ways is 2 × 2 = 64 and out of these 64 only
60C × 80C × 40C .
10 10 10 one case in which he can pass the
In this question either case(i) is exam so total number of ways that a
possible or case (ii) hence total student can fail is 64 – 1 = 63.
number of ways is 40C × 60C × 15. (b) Amit can deal with a particular
10 10
question in 2 ways hence total
80C + 60C 80C 40C
10 10 × 10 × 10 number of ways is 2 × 2 × 2 … × 2
10. (b) Out of 21 consonant we have to select = 210 = 1024 ways but out of these
5 that can be done in 21C5 ways. 1024 ways there is one way when he
11. (d) Number of ways that we can deal left all the questions unanswered so
with apples is (6 + 1) =7 total number of ways is 1023
Number of ways that we can deal Alternately:
with mangoes is (5 + 1) =6 Amit can answer 1 question in 10C1
Number of ways that we can deal ways, 2 questions in 10C2 ways and
with bananas is (4 + 1) =5
so on. 10 question in 10C10 ways hence
So number of ways of selecting fruits
is 7 × 6 × 5 = 210 out of these there is total number of ways is 10C1 + 10C2 +
one way when we have not selected …. + 10C10 = 210 – 1 = 1023 ways.
mangoes, bananas, or apples hence 16. (b) Consider how many options for each
total number of ways of selecting student he has, for 1st student he
one or more fruits is 210 – 1 = 219 has 2 option (either select or reject)
12. (a) Number of ways that we can deal similarly for 2nd student 2 ways and
with apples is (6 + 1) =7 so on. So total number of ways is 210
Number of ways that we can deal = 1024 ways, but out of these 1024
with mangoes is 5 ways there is one way when he has
rejected all the students which is not
Number of ways that we can deal
according to the given condition as
with bananas is (4 + 1) =5
he has to select some students for
So number of ways of selecting fruits his project, hence required number
is 7 × 5 × 5 = 175. of ways is 210 – 1 = 1023 ways.
Selection (Combination)  51
Alternately:  Total number of Hence required number of ways is
students is ‘10’ and teacher has to 63 × 15 = 945
select one or more from these ‘10’ 18. (b) We know that the total number of
article. ways of selecting one or more things
One student out of ‘10’ students can from ‘p’ identical things of one type,
be selected in 10C1 ways ‘q’ identical things of another type,
Two students out of ‘10’ students ‘r’ identical things of 3rd type and
can be selected in 10C2 ways ‘n’ distinct things is (p + 1)(q + 1)
Three students out of ‘10’ students (r + 1)2n
can be selected in 10C3 ways Hence here required number of ways
….. Similarly is 5 × 6 × 24 = 480
10 Student out of ‘10’ students can But there is one way where none of
be selected in 10C1 ways the fruits are selected and we have
Since we can select either 1 OR 2 to remove that case hence required
OR 3…OR 10 students hence from number of ways is 480 – 1 = 479
fundamental principal of summation 19. (a) Since 1200 = 12 × 100 = 4 × 3 × 4
total number of ways is 10C1 + 10C2 +
× 25 = 243152 hence its number of
….. + 10Cn = 210–1 = 1023.
factors is (4 + 1)(1 + 1)(2 + 1) = 5 × 2
17. (d) Number of ways of selecting one or × 3 = 30
more boys is (26 – 1) = 63 ways.
20. (d) The largest factor of 1800/2 = 900
Number of ways of selecting one or
hence number of four digit factor is 0.
more girls is (24 – 1) = 15 ways.

Concept Builder (CB)


1. (a) 2. (a) 3. (b) 4. (a) 5. (b) 6. (d)
7. (c) 8. (c) 9. (a) 10. (b) 11. (c) 12. (b)
13. (b) 14. (c) 15. (a) 16. (d) 17. (a) 18. (c)
19. (c) 20. (d)

1. (a) From 2 wicket keepers only 1 is 3. (b) We can select 2 men from 5 men
selected and this can be done in in 5C2 ways, and 3 women from 7
2C ways and remaining 10 can be women in 7C3 ways. Hence total
1
selected from remaining 13 players number of ways is 5C2 × 7C3
and that can be done in 13C10 so total 4. (a) We can select 2 men from 5 men
number of ways is 2C1 × 13C10. in 5C2 ways, and 3 women from 6
2. (a) We can select 3 men from 6 men women in 6C3 ways. Hence total
in 6C3 ways, and 3 women from 7 number of ways is 5C2 × 6C3.
women in 7C3 ways. Hence total 5. (b) A ticket will be between two stations
number of ways is 6C3 × 7C3 so 2 stations from n stations can be
52  Theory of Counting
selected in nC2 ways i.e n(n – 1)/2 12. (b) Without restriction we can select 5
which is given to us equal to 210 so members from 12 in 12C5 ways. Now
n(n – 1) = 210 × 2 = 420 or n = 20. with negative restriction (Means
6. (d) Without any restriction 10 students if no girl is selected in the team)
can be selected from 20 students in the number of ways is 8C5. So total
20C ways. With negative restriction number of ways with the given
10
that means when both Rajesh and restriction is 12C5 – 8C5.
Dinesh are selected the total number 13. (b) This question is exactly same as
of ways is 18C8. Hence required previous question, as at least one boy
number of ways is 20C10 – 18C8 . is always excluded from the team.
7. (c) As per the given condition from 28 14. (c) 5 vowels can be selected from 5 vowels
we have to select 8 and that can be in 5C5 = 1 way and 5 consonants can
done in 28C8 ways. be selected from 21 consonants in
21C ways.
5
8. (c) Here we have following cases- 15. (a) At least one apple from 4 distinct
Case (i) – when 4 students are apples can be selected in (24 – 1) = 15
selected then number of ways is ways and bananas can be selected in
6C = 15 ways.
4 (23 = 8 ways) hence total number of
Case (ii) – when 5 students are is 15 × 8 = 120.
selected then number of ways is 16. (d) Number of ways of selecting at least
6C = 6 ways.
5 two boys is 6C2 + 6C3 + 6C4 + 6C5 + 6C6.
Case (iii) – when 6 students are
selected then number of ways is Since we know that 6C0 + 6C1 + 6C2
6C = 1 way. + 6C3 + 6C4 + 6C5 + 6C6 = 26
6
Then total number of ways is 15 + 6 Hence 6C2 + 6C3 + 6C4 + 6C5 + 6C6
+ 1 = 22 ways. = 64 – 1 – 6 = 57 ways.
9. (a) From the given condition we have to Similarly number of ways of selecting
select (k – r) students from (n – r – p) at least two girls is 4C2 + 4C3 + 4C4
students and that can be done in Since we know that 4C0 + 4C1 + 4C2
(n – r – p)C
k – r ways. + 4C3 + 4C4 = 24
10. (b) Since total number of students is 10 Hence 4C2 + 4C3 + 4C4 = 16 – 1 – 4
so number of pairs is 10C2 and with = 11 ways.
each pair we have two gifts so total Hence total number of ways is
number of gifts is 210C2
57 × 11 = 627.
11. (c) We have these different cases- 17. (a) Mangoes can be selected in 4 ways
Section 1 Section 2 Number of ways (either 1 or 2 or 3 or 4 mangoes) similarly
Case 4 (in 5C4 2 (in 5C2 5C 5 Bananas and apples can be selected
4 × C2 = 5 × 10
(i) ways) ways) = 50 in 5 and 4 ways respectively, so total
number of ways is 4 × 5 × 4 = 80.
Case 3 (in 5C3 3 (in 5C3 5C 5
3 × C3 = 10 × 10 18. (c) Mangoes can be selected in 4 ways
(ii) ways) ways) = 100
(either 1 or 2 or 3 or 4 mangoes)
Case 2 (in 5C2 4 (in 5C4 5C × 5C2 = 5 × 10
4 similarly Bananas and apples
(iii) ways) ways) = 50 can be selected in 4 and 2 ways
Total number of ways is 50+100+50 respectively, so total number of
= 200 ways is 4 × 4 × 2 = 32.
Selection (Combination)  53
19. (c) Student will fail the exam if he y = 4 × 2 × 4 = 32.
has answered either 0, or 1, or 2 For common factors we have to find
questions, this can be done in 6C0 out HCF of 2000 and 3000, it is
+6C1 +6C2 = 1 + 6 + 15 = 22 ways. 1000.
20. (d) Since 2000 = 24 × 53 hence Since 1000 = 23 × 53 hence z = 4 × 4 = 16.
x = 5 × 4 = 20.
Hence (x – z) : (y – z) = 4 : 16 = 1 : 4.
Since 3000 = 23 × 31 × 53 hence

Concept Cracker (CC)


1. (b) 2. (a) 3. (b) 4. (a) 5. (b) 6. (c)
7. (a) 8. (b) 9. (a) 10. (d) 11. (b) 12. (a)
13. (c) 14. (a) 15. (b) 16. (d) 17. (c) 18. (b)
19. (b) 20. (b) 21. (d) 22. (b) 23. (a) 24. (a)
25. (d) 26. (b) 27. (c) 28. (b) 29. (a) 30. (a)

1.
(b) Total 6 boys and 4 girls and we have to number of ways of selecting 5 without
select 5 so we have following cases: restriction and the total number of
Case (i) team has 1 girl and 4 boys ways with no girl in the team i.e 252
the number of ways is (4C1)(6C4) = 4 – 6 = 246.
× 15 = 60 2. (a) Let us assume that total number of
Case (ii) team has 2 girls and 3 boys students in the party is ‘n’
the number of ways is (4C2)(6C3) = 6 Number of ways that 2 students can
× 20 = 120 be selected from n students is nC2. In
Case (iii) team has 3 girls and 2 each pair of students each student
boys the number of ways is (4C3) gave gift to other hence total number
(6C2) = 4 × 15 = 60 of gifts distributed is
Case (iv) team has 4 girls and 1 boy 2nC2 = 2n(n – 1)/2 = n(n – 1)
the number of ways is (4C4)(6C1) = 1 From the given condition
×6=6 n(n – 1) = 240 or n = 16
So total number of such ways is 60 + Hence number of students present
120 + 60+6 = 246 in the party is 16
Method 2: Total number of ways of 3. (b) From the solution of previous
selecting 5 out of 10 is 10C5 = 252 question, number of students present
Number of ways of selecting 5 out of in the party is 16 hence a particular
given 10 so that no girls in the team student has received 15 gifts.
(means only boys) is 6C1 = 6 4. (a) In a dance we have to select one man
But the given condition is that from 10 men in 10C1 = 10 ways, and
at least 1 girl in the team hence one woman from 15 woman in 15C1
required number of ways is given = 15 ways, hence total number of
by the difference between the total dance performed is 10 × 15 = 150.
54  Theory of Counting
5. (b) If bus has ‘K’ number of stoppages 6. (c) Here we will have 3 different cases
then to have a ticket we have to based on the number of girls in the
select 2 stoppages out of K and that committee.
K (K − 1)
can be done in KC2 = =1
  90
2
or K(K – 1) = 380 or K = 20.

No. of Girls No. of Boys No. of selection


Case (i) 1 4 4C × 6C4 = 4 x 15 = 60
1

Case (ii) 2 (But Mahima and Nita Cant be together) 3 (4C2 – 2C0) × 6C3 =5 x 20 = 100

Case (iii) 3 (But Mahima and Nita Cant be together) 2 (4C3 – 2C1) × 6C2 = 2 x 15 = 30

Total 190

4 girls can not be selected together as Mahima and Nita Can’t be together. So total
number of ways is 190

7. (a) We can solve this question by making different cases-


Mathemat- Biology No. of ways Hence
ics Group Group total
Girl Boy Girl Boy number
(4) (3) (3) (4) of ways
is 1+144+
Case (i) 0 3 3 0 4C 3C 3C 4C
0× 3× 3× 0 = 1×1×1×1 = 1 324+16 =
485
Case (ii) 1 2 2 1 4C 3C 3C 4C
1× 2× 2× 1 = 4×3×3×4 = 144

Case (iii) 2 1 1 2 4C 3C 3C 4C
2× 1× 1× 2 = 6×3×3×6 = 324

Case (iv) 3 0 0 3 4C 3C 3C 4C
3× 0× 0× 3 = 4×1×1×4 = 16

Total number of ways is 485 So total number of ways is 56 + 70 =


8. (b) Here we have two cases- 126.
Case (i) Before lunch 5 and after 9. (a) Without any restriction 11 players
lunch 3 then number of ways is 8C5 from 25 players can be selected
= 56 ways (We just have to select 5 in 25C11 ways and with negative
for before lunch program remaining restriction (means when the
three will automatically perform in
three mentioned players are
post lunch). 22
together) number of ways is C8
Case (ii) Before lunch 4 and after
hence required number of ways is
lunch 4 that can be done in 8C4
25C 22C
ways = 70 ways. 11– 8
Selection (Combination)  55
10. (d) Since product of two numbers is a 14. (a) Number of singers is 10.
multiple of three it is possible only Number of singers selected at a time
when at least one of them is multiple is 3.
of three so we have 2 cases:
So out of 10 singers 3 can be selected
Case (i): If both the number are in 10C3 ways.
multiple of three then number of
15. (b) From solution of previous question, if
ways is 33C2 since there are 33
a particular singer is selected always
numbers in the given set is multiple
then number of ways of selecting 2
of 3.
more players out of 9 players is 9C2
Case (ii): If only one number is 16. (d) Number of times that Mr. Singh
multiple of 3, then the number of ways came is 9C3 = 84 = p
is 33C1 × 67C1 And number of times Mr. Singh
Hence total number of ways is came with Mr. Raj is 8C2 = 28 = q
33C 33C × 67C .
2+ 1 1 So p is 200% more than q.
11. (b) Lets select ladies 1st, so two ladies 17. (c) Condition is same as Ricky and
can be selected in 10C2 ways then next Susmit selected but Soumyajyoti is
step is we have to select 2 gentlemen not selected that can be done in 7C2
from available 8 gentlemen and that ways = 21 ways.
can be done in 8C2 ways. Now we 18. (b) The question is similar to the
have 2 women and 2 men and they condition when we have to make
can be pair up in two ways hence selection with at least 3 women so
total number of ways is 2 × 10C2 × we have following cases-
8C .
2 Case (i) when 3 women and 9 men
12. (a) Here we have two cases: are selected then number of ways is
5C × 20C
Case (i) – 4 bowlers are selected- the 3 9
number of ways is 5C4 × 10C7 Case (ii) when 4 women and 8 men
Case (ii) – 5 bowlers are selected- the are selected then number of ways is
5C × 20C
number of ways is 5C5 × 10C6 4 8
So total number of ways is 5C4 × 10C7 Case (iii) when 5 women and 7 men
+ 5C5 × 10C6. are selected then number of ways is
5C × 20C .
13. (c) Since number of girls is always more 5 7
than number of boys hence we have So total number of ways is 5C ×
3
following cases- 20C + 5C × 20C + 5C × 20C
9 4 8 5 7
Case (i) when number of girls is 3 19. (b) 1st we will select 4 out of 20 in 20C4
and that of boys is 2 then number of ways then next 5 in 16C5 and finally
ways is 4C3 × 6C2 8 in 11C8 ways. Hence total number
Case (ii) when number of girls is 4 of ways is 20C4 × 16C5× 11C8
and that of boy is 1 then number of 20. (b) One or more green dyes can be
ways is 4C4 × 6C1 selected out of 5 different green dyes
Total number of ways is 4C

6C
2+
in 5C1 +5C2 +5C3 +5C4 +5C5 = 25 – 1
4C 6 = 31 ways.
4 × C 1.
56  Theory of Counting
One or more blue dyes can be selected similarly for 2nd fruit and so on, so
out of 4 different blue dyes in total number of selection is (k + 1)n
4C 4C 4C 4C but we have to select at least 1 fruit
1+ 2+ 3+= 24 – 1 = 15 ways.
4 hence required number of ways is
Zero or more red dyes can be selected (k + 1)n – 1 .
out of 3 different red dyes in 3C0 + 3C1 27. (c) Only square of a prime number has
+ 3C2 +3C3 = 23 = 8 ways. Required three factors, these are 1, p and p2.
number = 31 × 15 × 8 = 3720 Since the number or p2 is less than
21. (d) Since there are 4 different denomi- 10000 that means p or prime number
nations hence required summation is less than 100, we know that total 25
is 2 × 2 × 2 × 2 – 1 = 16 – 1 = 15 prime numbers less than 100 exist.
22. (b) A student can vote for either 1 or 28. (b) Since number of factors is 8, we have
2 or 3 contestants, hence required following cases-
number of ways is 6C1 + 6C2 + 6C3 = Case (i): if we take 8 = 2 × 2 × 2 i.e
6 + 15 + 20 = 41. number is in the format of a1 × b1
23. (a) Since out of 16 only 10 is available × c1, the two digit numbers in this
for selection, out of 10, 3 can be format are 2 × 3 × 5 = 30, 2 × 3 × 7 =
selected in 10C3 = 120 ways, and 42, 2 × 3 × 11 = 66, 2 × 3 × 13 = 78,
three from the group of Mr & Mrs A, 2 × 5 × 7 = 70 total 5 numbers in this
format.
B and C can be selected in 2 × 2 × 2
= 8 ways. Case (ii): If we take 8 = 2 × 4 i.e
number is in the format of a1b3, the
So total number of ways for selection
two digit numbers in this format 23
is 8 × 120 = 960 = 8(5!). × 31 = 24, 23 × 51 = 40, 23 × 71 = 56,
24. (a) Total number of its subset is 2n 23 × 111 = 88, 33 × 21 = 54 total 5
and out of these 50% is even hence numbers in this format.
required number is 2n – 1. Case (iii): If we take 8 = 1 × 8 i.e
25. (d) From the given condition require number is in the format of a7 but no
number of selection is two digit number exist that is in this
format.
K = 2n + 1C0 + 2n + 1C1 + 2n + 1C2 + …
+ 2n + 1Cn So total number of numbers is 10
29. (a) Given number is N = 25 × 35 × 55 ×
We have 2K = 2(2n +1C0 + 2n +1C1 + 2n 75, here we have to find the factors
+ 1C + … + 2n +1Cn) divisible by 100 = 22 × 52 that means
2
in all the concern factors 22 × 52
=(2n+1C0 + 2n +1C 2n +1C
2n +1) +( 1 + must be common. Required number
2n + 1 of factor is same as the number of
C2n )+ … + ( 2n +1 C + 2n + 1Cn +1)
n factors of 23 × 35 × 53 × 75, and it is
(  nCr = nCn – r). 4 × 6 × 4 × 6 = 576.
30. (a) Since 7200 = 25 × 32 × 52 its factors
Thus 2n – 1C0 + 2n + 1C1 + 2n + 1C2
will end with 5 only when it is odd
+… +
2n + 1C
2n + 1 = 22n + 1 factor, or all the factors of 32 × 52 will

satisfy the condition. Hence sum of
⇒  N = 22n. all the factors of 32 × 52 is given by
26. (b) Consider the 1st type, 1 or more
33 − 1 53 − 1
fruits can be selected in (k + 1) ways × = 13 × 31 = 403
3 −1 5 −1
Part B Selection (Combination)  57

Introduction of
Concept 1 Selection/Combination
• Number of ways of selecting ‘r’ ( n − 1)!
=
articles from ‘n’ distinct article is
{(n − r − 2 )! ((r − 1)!)}
nC = n!
• Number of ways of selecting ‘r’
r
{ r )!(r !)}
(n −
articles from ‘n’ distinct article if ‘k’
• Number of ways of selecting ‘r’ article is always included is n–kCr–k
articles from ‘n’ distinct article if 1
article is always included is n–1Cr–1 ( n − k )!
=

{(n − r − 2k )! ((r − k )!)}

Concept 2 Selection of one or more

The total number of combinations of ‘n’ Now consider the number of ways in which
different things taken one or more at a nothing is selected is 1 × 1 × 1 × … × 1 = 1
time is 2n–1 times
Proof: Consider the 1st thing there are two Hence the total number of ways of
ways we can deal with, it may be included or selecting one or more at a time out of ‘n’
excluded. Similarly, there are 2 ways that we different things is 2n–1
can deal with 2nd article and so on for 3rd and Alternately
nth articles.
Total number of distinct article provided is ‘n’
Now from fundamental principal of multiplica- and we have to select one or more from these
tion total number of ways is 2 × 2 × 2 × 2 × .. ‘n’ article.
× 2 (n times) = 2n .
58  Theory of Counting
One article out of ‘n’ distinct article can be ‘n’ articles out of ‘n’ distinct article can be
selected in nC1 ways. selected in nCn ways.
Two articles out of ‘n’ distinct article can be Since, we can select either 1 OR 2 OR 3…OR
selected in nC2 ways. ‘n’ articles hence from fundamental principal
Three articles out of ‘n’ distinct article can be of summation total number of ways is nC1 +
nC + ….. + nC = 2n–1
selected in nC3 ways. 2 n

…. Hence the total number of ways of


selecting one or more at a time out of ‘n’
….. Similarly,
different things is 2n–1

Concept 3 Selection from Identical Articles

The total number of ways of selecting Hence number of ways of selecting zero or
one or more things from ‘p’ identical more things out of p identical things = p + 1.
things of one type, ‘q’ identical things Similarly, number of ways of selecting zero or
of another type, ‘r’ identical things of more things out of q identical things = q + 1.
3rd type and ‘n’ distinct things is (p + 1)
Number of ways of selecting zero or more
(q + 1)(r + 1)2n
things out of r identical things = r + 1.
Since number of ways of selecting r things out Also number of ways of selecting zero or more
of n identical things = 1 for all r ≤ n. things out of n different things = 2 × 2 × 2 ×
Hence number of ways of selecting one thing … n times = 2n.
out of p identical things = 1. Therefore, number of ways of selecting zero
or more things out of given things = (p + 1)
Number of ways of selection two things out of
(q + 1)(r + 1 ) 2n.
p identical things = 1.
But number of ways of selecting zero thing
Similarly number of ways of selecting p things out of given things = 1 × 1 × 1 × 1n = 1.
out of p identical things = 1.
Thus total number of ways of selecting one
Number of ways of selecting zero thing out of or more things out of given = ( p + 1) (q + 1)
p identical things = 1. (r + 1) 2n–1.
Selection (Combination)  59

Concept 4 Factor Theory

To find number of factors of a composite given by (p + 1)(q + 1)(r + 1)(s + 1)


number write down the number in the form Number of even factors of a number N =
of n = apbqcr …. where a, b and c are prime 2paqbrcs is = p(q + 1)(r + 1)(s + 1).
numbers , and p, q and r are natural number Number of odd factors = Total number of
then number of factors of n is given by (p + 1) factors – Number of even factors
(q + 1)(r + 1) ….. and it includes 1 and number Alternate way: Number of odd factors of
itself. a number N = 2paqbrcs is = (q + 1)(r + 1)
(s + 1)
Explanation of above formula:
Number of factors of number N = apbqcrds…is
60  Theory of Counting

1
Concept Deviator (CD)
Ideal Time Revise your concept (IIT advance or
5 Min. tougher questions)
Per Question

1. What is the number of ways in which Marketing, HR and Operations students


three numbers in A.P can be selected such that at least 2 students must be
from a set of numbers { 1, 2, 3….100} selected from each marketing, HR and
Operations and number of students
(a) (50C2 ) × (50C2) (b) 100C
2 from marketing is more than number of
(c) 350C 2 (d) None of these students from HR which in turn is more
than number of students from operations.
2. What is the number of ways in which
In how many ways this selection can be
three numbers in A.P can be selected
done?
from a set of numbers { 1, 2, 3….101}
(a) 2125 (b) 1850
(a) (50C2 ) + (50C2) (b) (51C2 ) + (50C2)
(c) 2025 (d) None of these
(c) (51C2 ) + (52C2) (d) None of these 7. In an examination the question paper
Directions for question number 3 and 4: has three sections namely Physics,
Consider a set S = { 1, 2, 3, ……, n} Chemistry and Mathematics containing
3. In how many ways three numbers from 4, 5 and 6 questions respectively. As
the given set can be selected such that per the rule a students has to answer
all the three numbers are consecutive? minimum 2 questions from each section.
In how many ways a student can attempt
(a) nC3 (b) n – 2 7 questions?
n
(c) C3 – n (d) None of these (a) 2000 (b) 2700
4. In how many ways three numbers from (c) 3200 (d) None of these
the given set can be selected such that not
all the three numbers are consecutive? 8. In an election the number of contestants
is one more than the number of seats. If
(a) nC3– n + 2 (b) nC3 – n + 1 a particular voter can vote in ‘k’ different
(c) nC3 – n (d) None of these ways then what could be the value of
5. In Realty show of Dance Competition, k if a voter can give minimum 1 and
final round has 10 participants. In each maximum equal to the number of seats.
performance exactly two dancers will (a) 1024 (b) 1023
perform. Mr. kartick a fan of this show (c) 1022 (d) None of these
has decided to see exactly 10 dance
performance, in how many ways he can 9. In MERI Kolkata while voting for Co-
select the performances? operative society member, number of
seats are ‘n’ and number of participants
(a) 45C10 (b) 55C10 are ‘n+1’ as per the rule a student can
(c) 1 (d) None of these vote minimum 1 and maximum ‘n’ votes.
6. A delegation of 12 students has to be To vote for 1 candidate a student need 1
selected from 18 students 6 from each ballot paper which one of the following
Selection (Combination)  61
could be the total number of ballot papers answers is 2047, then what is the number
that management committee should be of questions in the test ?
ready with? (a) 11 (b) 12
(a) 899 (b) 1230 (c) 13 (d) None of these
(c) 1763 (d) 323 14. In a particular batch of Pioneer career
10. If ‘K’ is the number of ways in which Kolkata there are 4 boys and certain
three numbers in A. P. can be selected number of girls. In every mock test only
from ,1 2, 3, …, n then which one of the 5 students including at least 3 boys can
following is correct about ‘K’? appear. If different group of students
write the Mock exam every time, if
(i)
(n − 1)2
if n is odd number of times test conducted is 66
4 then find the total number of students in
n (n − 2 ) the class.
(ii) if n is even
4 (a) 10 (b) 12
(c) 8 (d) 5
(iii)
(n − 1)2
if n is even 15. Consider a set S = { 1, 2, . . . , 100 }, two
4 elements p and q are selected from this set
(a) Only (i) & (ii) (b) Only (ii) & (iii) S such that 7p + 7q is divisible by 5, How
(c) Only (i) & (iii) (d) None of these many ways this selection can be made?
11. In Praxis Business School Kolkata, roll (a) 1250 (b) 2200
number of N students are given from 1 to (c) 2500 (d) 5000
N. Three students are selected from these 16. In a class number of students is 4k. If
N students such that their roll numbers the number of ways of selecting ‘k’ boys
are not consecutive in how many ways and ‘k’ girls out of these 4k students is
this selection can be done? maximum possible then which of the
(a)
(N − 2 )(N − 3)(N + 3) following statements are correct?
6 (i) Number of boys is equal to number
(N − 2 )(N − 3)(N − 4) of girls.
(b)
6 (ii) Number of ways of selecting 4 boys is
(N − 2 )(N − 3)(N + 2 ) same as number of ways of selecting
(c) (k – 4) girls.
6
(d) None of these (iii) Number of boys can not be equal to
12. Consider infinite number of letter C, A number of girls.
and T, in how many ways we can choose (a) Only (i) & (ii) (b) Only (i) & (iii)
‘k’ letters such that word CAT can not be (c) Only (ii) & (iii) (d) None of these
formed? 17. In an election the number of contestants
(a) 4(2k – 1) (b) 2(2k – 1) is one more than the number of seats. If a
(c) 3(2k – 1) (d) None of these particular voter can vote in 126 different
ways then find the number of candidates if
13. In an Mock test at Pioneer Career
a voter can give minimum 1 and maximum
Kolkata, there are n questions. In this
equal to the number of seats.
test 2k students gave wrong answers to
at least ( n – k) questions, where k = 0, (a) 7 (b) 4
1, 2, . . . , n. If the total number of wrong (c) 8 (d) None of these
62  Theory of Counting
18. A committee of 12 is to be formed from 9 (a) 220 (b) 219
women and 8 men.
(c) 211 (d) 256
(i) In how many ways this can be done
if at least five women have to be 22. If r, s, t are prime numbers and p, q
included in a committee? are the positive integers such that the
(ii) In how many of these committees LCM of p, q is r2t4s2, then the number of
the women are in majority? ordered pair (p, q) is [IIT JEE 2006]
(ii) In how many of these committees (a) 252 (b) 254
the women are in majority?
(a) 6062, 2702, 1134 (c) 225 (d) 224
(b) 6062, 2702, 1008 23. In a certain test, ai students gave wrong
(c) 6062, 2520, 1134 answers to at least i question where
(d) None of these [IIT JEE 1994] i = 1, 2, …., k No student gave more that k
19. A student is allowed to select at most n wrong answers. The total number of wrong
books from n collection of (2n + 1) books. answer given is…. [IIT JEE 1982]
If the total number of ways in which he n n
can select at least one books is 63, find (a) 2 − 1 (b) 2 + 1
the value of n. [IIT JEE 1987] n+ 1
(c) 2 −1 (d) None of these
(a) 7 (b) 3
(c) 6 (d) None of these 24. A box contains two white balls, three
20. 7 relatives of a man comprises 4 ladies black balls and four red balls. In how
and 3 gentlemen; his wife has also 7 many ways can three balls be drawn from
relatives; 3 of them are ladies and 4 the box if at least one black ball is to be
gentlemen. In how many ways can they included in the draw?   [IIT JEE 1986]
invite a dinner party of 3 ladies and 3
(a) 64 (b) 54
gentlemen so that there are 3 of man’s
relative and 3 of the wife’s relative? (c) 25 (d) 42
[IIT JEE 1985] 25. Let E = {1, 2, 3, 4} and F= {1, 2} then the
(a) 485 (b) 440 number of onto function from E to F
(c) 490 (d) None of these
[IIT JEE 2001]
21. Let A and B be two sets containing 2
elements and 4 elements respectively. (a) 14 (b) 16
The number of subsets of A × B having 3 (c) 12 (d) 8
or more elements is- [IIT JEE 2013]

Response Grid
1. a b c d 2. a b c d 3. a b c d 4. a b c d 5. a b c d
6. a b c d 7. a b c d 8. a b c d 9. a b c d 10. a b c d
11. a b c d 12. a b c d 13. a b c d 14. a b c d 15. a b c d
16. a b c d 17. a b c d 18. a b c d 19. a b c d 20. a b c d
21. a b c d 22. a b c d 23. a b c d 24. a b c d 25. a b c d
Selection (Combination)  63

2
Concept Eliminator (CE)
Ideal Time Revise your concept (Maths Olympiad
Just solve it or tougher questions)

1. 30 players from India, China and Australia (a) 27:54 (b) 29:54
participated in world chess tournament. (c) 31:54 (d) None of these
Number of players from India is 3 more 5. In an election of Mess Committee
than the number of players from China. member at Praxis is business school,
In tournament players from same country number of vacancy was ‘n’ and number
are not allowed to play against each other. of participants was ‘n+k’ there are two
Find the maximum number of matches types of voting system as explained below
that was conducted in the tournament if Type 1: A voter can vote for minimum 1
no two player plays against each other and maximum ‘n’ candidates.
more than once.
Type 2: A voter can vote for minimum 1
(a) 297 (b) 292
and maximum ‘k–1’ candidates.
(c) 144 (d) None of these
If in type 1 a voter can vote in 3989
2. In the previous question if number of
different ways while in Type 2 a voter
matches is maximum then what is the
can vote in 105 different ways then find
ratio of number of players from India,
the value of ‘n–k’
Australia?
(a) 3:4 or 4:5 (b) 1:1 or 3:4 (a) 12 (b) 14
(c) 1:1 or 4:5 (d) None of these (c) 13 (d) None of these
3. In a fruit Basket there are n identical 6. Consider n identical toys of weight w,
apples, n identical bananas and n when it is weighed taken k number of
distinct fruits, If k is the number of ways toys at a time then sum of all the possible
in which n fruits can be selected from results is 11P, and when weighed in
the fruit basket then which one of the ‘k+1’ number of toys at a time then sum
following is the value of k? of all the possible results is 7P then find
(a) (n + 1)2n – 1 (b) (n2)(2n – 1) the minimum possible value of n.
(c) (n + 2)(2n – 1) (d) None of these (a) 19
4. During 1st stage of the round robin (b) 18
stage of Hockey world cup each country (c) 17
played against each other and scores (d) Can not be determine
of Pakistan, Srilanka, Bangladesh and
7. This question is based on Regional Maths
China was 4, 4, 4 and 4.5 respectively
Olympiad -
while other participants got equal points.
As per the rule of the tournament winner Consider a set X = { 1, 2, 3, …, 9, 10}.
gets 1 point, looser gets 0 while 0.5 points What is the number of pairs {A, B} such
awarded to both the teams in case of a tie, that A ⊆ X and B ⊆ X also A ≠ B and
then what is the ratio of average points A ∩ B = { 2, 3, 5, 7 }
scored by other countries to the number (a) 36 (b) 63
of countries? 6
(c) C3 (d) None of these
64  Theory of Counting

Solutions
Concept Deviator (CD)
1. (d) 2. (b) 3. (b) 4. (a) 5. (a) 6. (c)
7. (b) 8. (c) 9. (b) 10. (a) 11. (c) 12. (c)
13. (a) 14. (a) 15. (c) 16. (a) 17. (a) 18. (a)
19. (b) 20. (a) 21. (b) 22. (d) 23. (a) 24. (a)
25. (a)

1. (d) Given set of numbers is { 1, 2, Case (ii): when both of them are even
3….100} then these 2 can be selected from {2,
Let the 3 numbers in A.P be p, q and 4, 6, …. , 98, 100} i,e 50 even integers.
r then 2q = p + r, hence p + r must Number of ways of selecting a and c
be even that means both of them are out of 50 even integers is 50C2 .
either even or both of them are odd. So total number of ways is 51C2
Case (i): When both of them are odd +50C2
then these 2 can be selected from Now prove yourself:
{1, 3, 5,… 97, 99} i,e 50 odd integers.
The number of ways in which three
Number of ways of selecting a and c
numbers in A. P can be selected from a set
out of 50 odd integers is 50C2 .
of n integers {1, 2, 3, …..n} is given by-
Case (ii): When both of them are even
then these 2 can be selected from Case (i) when n is odd
(n – 1)2
{2, 4, 6, …. , 98, 100} i,e 50 even 4
integers. Number of ways of selecting (n)(n − 2 )
a and c out of 50 even integers is 50C2 . Case (ii) when n is even
4
So total number of ways is
50 C2 + 50C2 = 250C2 3. (b) Let us assume n numbers as a1, a2,
2. (b) Given set of numbers is …an
{1, 2, 3….100} Then possible selections are (a1,
Let the 3 numbers in A.P be p, q and a2,a3), (a2, a3,a4 ), … (an–2, an–1,an ),
r then 2q = p + r hence p + r must so there are n–2 ways we can select
be even that means both of them are such three numbers.
either even or both of them are odd. 4. (a) Without any restriction we can select
Case (i): When both of them are odd then 3 out of n in nC3ways , but from
these 2 can be selected from {1, 3, previous question with restriction
5,… 97, 99, 101} i,e 51 odd integers. we can select in n–2 ways, hence
Number of ways of selecting a and c in this case (with restriction) total
out of 51 odd integers is 51C2 . number of ways is nC3 – n + 2.
Selection (Combination)  65
5. (a) Total number of dance performance is 10C2 = 45, so Mr. Kartick has to select 10
performances from 45 that can be done in 45C10 ways.
6. (c) From the given conditions we have following cases:

Operations HR Marketing No. of ways

Case (i) 3 (in 6C3 = 20 ways ) 4(in 6C4 = 15 ways) 5(in 6C5 = 6 ways) 20 × 15 × 6 = 1800

Case (ii) 2 (in 6C2 = 15 ways ) 4(in 6C4 = 15 ways) 6(in 6C6 = 1 way) 15 × 15 × 1 = 225

So total number of ways is 1800 + 225 = 2025

7. (b) Here in this case we have following cases:

Physics (4) Chemistry (5) Maths (6) No of ways

Case (i) 2(in 4C2 = 6 ways) 2(in 5C2 = 10 ways) 3(in 6C3 = 20 ways) 6 × 10 × 20 = 1200

Case (ii) 2(in 4C2 = 6 ways) 3(in 5C3 = 10 ways) 2(in 6C2 = 15 ways) 6 × 10 × 15 = 900

Case (iii) 3(in 4C3 = 4 ways) 2(in 5C2 = 10 ways) 2(in 6C2 = 15 ways) 4 × 10 × 15 = 600

So total number of ways is 1200 + 900 + 600 = 2700

8. (c) Let the number of contestants is ‘n’ None of the other options are in this
then a particular voter can give 1 or format.
2 or 3 or … (n–1) votes, so number of 10. (a) Given number are 1 ,2 ,3 , . . . , n
ways a voter can vote is nC1 + nC2 + Let the three selected numbers in
….. + nCn–1 A.P. be a , b , c , then a + c = 2b
Since nC0+ nC1 + nC2 + ….. + nC
n–1 From this we can say that a + c
+nCn = 2n should be an even integer. This is
Hence nC1 + nC2 + ….. + nCn–1 possible only when both a and c are
= 2n – nC0 – nCn = 2n–2 odd or both are even.
So total number of ways in which a Case I: When n is even we can assume
voter can vote should be in 2n–2 form, n = 2m.
from the given options only 1022 is
Then number of odd numbers =
in this format and hence the answer
number of even numbers = m.
9. (b) From the solution of previous
question maximum number of ways Then number of selections of a and c
that a student can vote is 2n – 2 if from m odd numbers = mC2.
number of students is ‘k’ then total
And number of selections of a and c
number of ballot papers is k(2n – 2)
from m even numbers =mC2.
From the given options 1230 = 30 ×
41 here 30 is in the format of 2n – 2 Hence total number of ways in this
hence the correct answer. case mC2 + mC2= 2.mC2 = m ( m – 1)
66  Theory of Counting

 n   n   n(n  – 2 )  ‘k’ letters which are C or T = 2k


=    – 1  =   including the case when all the ‘k’
 2  2  4
letters are A or all are T.
Case II. When n is odd we can assume Similarly, number of ways of selecting
that n = 2m + 1 ‘k’ letters when all are A, C and T
Then number of odd numbers = m + are 1k in each case.
1 and number of even numbers = m Thus required number
Required number in this case = 2k + 2k + 2k – (1k + 1k + 1k)
= m + 1C2 + mC2 = 3.2k – 3.1k = 3(2k – 1)
=
( m +  1)m   +  m( m −1)  =  m2 13. (a) From the given condition the number
2 2 of students answering at least r
 1
2
2
questions incorrectly is 2 n–r.
=    n −  =  ¼  ( n  −1) Hence the number of students answering
 2
Hence statement (i) and (ii) is true. exactly r ( 1 ≤ r ≤ n – 1)
11. (c) 1st consider the number of ways questions incorrectly is 2n–1 – 2n– ( r + 1)
when all the three are consecutive And, the number of students
roll numbers. N roll numbers are 1, answering all questions wrongly is
2, 3…. N then possible selection will 20 = 1.
be (1, 2, 3), (2, 3,4)… (N–2, N–1, N) Then,the total number of wrong
Thus required number = n – 3 + 1 = n–2 answers is 1(2n–1 – 2n–2) + 2 (2n–2
Without restriction total number of – 2n–3) + 3 (2n–3–2n–4) + … + (n–1)
selections of 3 objects when there is (21–20) + n(20)
no restriction = NC3
= 2n–1 + 2n–2 + … + 20 = 2n – 1 = 2047
Number of selections of 3 objects
which are consecutive = N–2 ⇒ n = 11.
Required number = NC3–N – 2 14. (a) Let number of girls is ‘g’ then
Number of group having 4 boys and
N (N − 1)(N − 2 )
= – (N − 2 ) 1 girl = (4C4) × (gC1) = g
6
And number 4 of groups having 3 boys
(N − 2 )(N − 3)(N + 2 ) and 2 girls = (4C3) × (gC2) = 2g (g – 1 )
  = 6
12. (c) The word CAT cannot be spelt if the Thus, total number of tests is
‘k’ selected letters do not contain at g + 2g(g – 1) = 66
least one of A , T and C. or 2g2 – g – 66 = 0 only integral value
Number of ways of selecting ‘k’ letters of g is 6
which are A or T = 2k including the Hence total number of students is
case when all the ‘k’ letters are A or 4 + 6 = 10
all are T.
15. (c) Since unit digit of 7k ends in 7, 9, 3
Similarly, number of ways of selecting or 1 ( corresponding to k = 4x +1,
‘k’ letters which are A or C = 2k 4x +2, 4x +3 and 4x respectively.)
including the case when all the ‘k’
Thus, 7p + 7q cannot end in 5 for any
letters are A or all are C.
values of p, q. So for 7p + 7q to be
And Number of ways of selecting
Selection (Combination)  67
divisible by 5, it should end in 0. Thus there are 100 × 25 = 2500
For 7p + 7q to end in 0, the forms of p ordered pairs ( m, n) for which
and q should be as follows: 7p + 7q is divisible by 5.
16. (a) If number of boys is b and that of
p q girls is g then b+g = 4k and (bCk)
(gCk) is maximum that is possible
1 4x (Unit digit 1) 4y +2 (Unit digit 9) when b = g = 2k hence statement
(i) and (ii) is true.
2 4x + 1 (Unit digit 7) 4y + 3 (Unit digit 3) 17. (a) Let the number of contestants is ‘n’
then a particular voter can give 1 or
3 4x + 2 (Unit digit 9) 4y (Unit digit 1) 2 or 3 or … (n–1) votes, so number of
ways a voter can vote is nC1 + nC2 +
4 4x + 3 (Unit digit 3) 4y +1 (Unit digit 7)
….. + nCn–1
Thus, for a given value of m there Since nC0 + nC1 + nC2 + ….. + nCn–1
are just 25 values of n for which + nCn = 2n
7p + 7q ends in 0. Hence nC1 + nC2 + ….. + nCn–1
[For instance, if p = 4x, then q = 2, 6, = 2n – nC0 – nCn = 2n – 2 = 126
10, . . . , 98]
Hence n = 7.

18. (a) We have following cases-

Cases Men Women No. of ways

Case (i) 8 4 (8C8)(9C4) = 1 × 126 = 126

Case (ii) 7 5 (8C7)(9C5) = 8 × 126 = 1008

Case (iii) 6 6 (8C6)(9C6) = 28 × 84 = 2352

Case (iv) 5 7 (8C5)(9C7) = 56 × 36 = 2016

Case (v) 4 8 (8C4)(9C8) = 70 × 9 = 630

Case (vi) 3 9 (8C3)(9C9) = 56 × 1= 56

So
(i) At least five women number of ways = 1008 + 2352 + 2016 + 630 + 56 = 6062
(ii) When women are in majority = 2016 + 630 + 56 = 2702
(iii) When men are in majority = 126 + 1008 = 1134
19.
(b) As per the given condition the number of books that the student can select is 0, 1, 2…
or n so number of ways is given by (2n+1C1) + (2n+1C2) + … (2n+1Cn–1) + (2n+1Cn) = 63
We know that (2n+1C0) + (2n+1C1) + (2n+1C2) + … (2n+1Cn) + … (2n+1C2n) + (2n+1C2n+1)
= 22n+1
So (2n+1C1)+ (2n+1C2)+ … (2n+1Cn–1)+ (2n+1Cn) = 26 – 1
Hence n = 3
68  Theory of Counting
20. (a) We can solve this question by making different cases:
Man’s Woman’s
Relative Relative
No. of ways
Ladies Male Ladies Males
(4) (3) (3) (4)
Case (i) 0 3 3 0 4C 3C × 3C 3 × 4C 0 = 1 × 1 × 1 × 1 = 1
0× 3

Case (ii) 1 2 2 1 4C 3C × 3C2 × 4C1 = 4 × 3 × 3 × 4 = 144


1× 2

Case (iii) 2 1 1 2 4C 3C × 3C1 × 4C2 = 6 × 3 × 3 × 6 = 324


2× 1

Case (iv) 3 0 0 3 4C 3C × 3C0 × 4C3 = 4 × 1 × 1 × 4 = 16


3× 0

Hence total number of ways is 1 + 144 + 324 + 16 = 485


Total number of ways is 485

21. (b) Since A × B will have 2 × 4 = 8 Total number of wrong answers


elements so required answer is
= 1( 2 n − 1 – 2 n − 2  ) + 2 (2 n − 2 – 2 n − 3 )
28 – 8C0 – 8C1 – 8C2 = 256 – 1 – 8 – 28
= 219 + …. + (n–1)  (2 1 – 2 0 ) + n 2
0
()
22. (d) Required number of ordered pair (p,
= 2 n − 1 +  2 n − 2 + … 2 1 +  2 0 =  2 n − 1 
q) is (2 × 3 − 1) (2 × 5 −1) (2 × 3 − 1)
− 1 = 224. 24. (a) In this question we have 3 cases-

NOTE: Case (i): One black and two other then


number of ways is (3C1)(6C2) = 45
For details about this theory with variation
of questions refer to “Koncepts Of Numbers” Case (ii): One black and two other then
By Mr. Gajendra Kumar, Published by number of ways is (3C2)(6C1) = 18
Disha Publication.
Case (iii): All the three are blacks, then
23. (a) The number of students answering number of ways is (3C3) = 1
exactly k questions wrongly is Total number of ways is 45 + 18 + 1 = 64
2 n − k – 2 n − k −1 .
25. (a) Total number of function 42 = 16
The number of students answering
all the questions wrongly is 20 Number of onto function is 16 – 2 = 14
Selection (Combination)  69

Concept Eliminator (CE)


1. (a) 2. (b) 3. (c) 4. (b) 5. (d) 6. (a)
7. (d)

1. (a) Total number of players = 30 So the number of matches is


Let number of players from China is 2
−  3 p +  51 p  +  81 and it will attain a
‘p’ then that from India is ‘p+3’ and maximum value at
number of students from Australia 51
is ‘q’ then from the given condition p= − = 17/2 = 8.5
−6
P + (p + 3) + q = 30 or 2p+q = 27 …(i) But p must be integer hence possible
From the given condition number of value of p is 8 or 9
matches are as follows At p = 8 total number of matches
Number of matches between player of −  3 p 2 +  51 p  +  81 is 297
India and China is p+3C1 × pC1 = p(p+3)
At p = 9 total number of matches
Number of matches between player −  3 p 2 +  51 p  +  81 is 297
of India and Australia is p+3C1 × qC1
Hence maximum number of matches
= q(p+3)
is 297 when p = 8 or 9.
Number of matches between player of
China and Australia is pC1 × qC1 = pq 2. (b) Here we have 2 cases:

So total number of matches is Case (i): When p = 8, then players from


India, China and Australia is 11,
p(p + 3) + q(p + 3) + pq = p(p + 3) +
8 and 11 or required ratio is 1:1.
(27 – 2p)(p + 3) + p(27 – 2p)
Case (ii): When p = 9, then players from
=  p 2 +  3 p  +27 p  +81 – 2 p 2 − 6 p  +  27 p  −2 p 2 India, China and Australia is 12, 9
and 9 or required ratio is 4:3.
= −  3 p 2 +  51 p  +  81

3. (c) Given that there are n identical apples, n identical bananas and n distinct fruits, and
we have to select n articles, we have following cases
Number of
Apples & banana Banana
ways
Number of apples & banana Number of banana is 0
Case (i) (n+1)(nC0)
is n (n+1 ways) (nC0 Ways)
Number of apples & banana Number of banana is 1
Case (ii) (n)(nC1)
is n–1 (n ways) (nC1 Ways)
Number of apples & banana Number of banana is 2
Case (iii) (n–1)(nC2)
is n–2 (n–1 ways) (nC2 Ways)
Number of apples & banana Number of banana is n
Case (n+1) (1)(nCn)
is 0 (1 ways) (nCn Ways)

Total number of ways is (n+1)(nC0) + (n)(nC1) + (n–1)(nC2) + …… + (2)(nCn–1) + (1)(nCn)


From concept of binomial expansion this is equal to (n+2)(2n–1)
70  Theory of Counting
4. (b) Let ‘n’ is the number of teams Adding equations (i) and (ii) we will
participated in the tournament get n+kC + n+kC + n+kC + …..+
1 2 3
then number of matches are n+kC n+kC n+kC
nC = n(n–1)/2. Since each match n + n+1 + n+2 + ….. +
2 n+kC
will give 1 point hence total number n+k–2 + n+kCn+k–1
of points in the 1st stage is n(n–1)/2, = 3989 + 105 = 4094
given that 4 teams got points 4, 4,
Or (2n+k) –2 = 4094 or n+k = 14
4, and 4.5 i.e 17.5 points in total.
Let other members got k points the Now 14C1 = 14 and 14C2 = 91 hence
14C + 14C = 104 so k–1 = 2 or k = 3
as per the given condition n(n–1)/2 = 1 2
(n–4)k + 17.5 or n(n–1) = 2k(n–4) + 35 hence n = 14–3 = 11 and n–k = 11–3 = 8.
Here LHS is even hence RHS should 6. (a) Total weight when ‘k’ toys are taken
also be even it is possible when 2k is is (nCk)(kw) and total weight when
odd, now from slight trial and error ‘k+1’ toys are taken is (nCk+1)(k+1)w
we can find that n = 27 and k = 14.5 From the given condition (nCk)(kw)/
Hence required ratio is 14.5 : 27 or (nCk+1)(k+1)w =
k+1 k k
× =
29 : 54 n−k k+1 n−k
5. (d) As per the given condition – = 11/7
In type 1: n+kC1 + n+kC
2 + n+kC +
3
Or 7k = 11n – 11k or 18 k = 11n so n
…..+ n+kCn = 3989 ...(i) must be a multiple of 19 and hence
In type 2: n+kC1 + n+kC + n+kC + the minimum value of n is 19.
2 3
…..+ n+kCk–1 = 105 7. (d) Since it is given that A ∩ B = { 2, 3,
But we know that n+kC + n+kC + 5, 7 } so remaining numbers {1, 4, 6,
1 2
n+kC n+kC 8, 9, 10} can be a member of either
3 + ….. + k–1
= n+kCn+k–1 +n+kCn+k–2 + n+kCn+k–3 + set A or set B or neither in set A nor
….. + n+kCn+1 = 105 in set B, so each member of set {1,
4, 6, 8, 9, 10} has 3 options so total
or we can re-write it as
number of ways/ pairs is 36 but since
n+kC + n+kCn+2 + ….. + n+kCn+k–2
n+1 A ≠ B hence required number of
+ n+kCn+k–1 = 105 ...(ii) ways is 36 – 1
3
Geometrical Construction  71

Geometrical
Construction

Topics Covered

 Formation of geometrical figures

 Understanding of Chess Board

 Problems related to grid

Part A: Topic Number of Questions


Solved Example 10
Concept Applicator 15
Concept Builder 15
Concept Cracker 20
Part B: Topic Number of Questions
Concept Deviator 14
Concept Eliminator 9
Total 83
72  Theory of Counting
Part A
Formation on
Concept 1 Geometrical Figures
In this section we are going to learn, the number same straight lines hence we will not get kC2
of ways or the number of polygons that can be straight lines hence total number of straight
formed from the given points in the plane. lines is nC2 – kC2+1
Example 2:  How many straight lines
Number of Straight Lines from given ‘n’ can be drawn from the 15 points in a
Points plane if only 5 out of 15 points are in a
Consider ‘n’ points in a plane and we need to straight line?
find the number of straight lines that can be
Solution:  To draw a straight line we need to
draw from these ‘n’ points. In order to draw a
select 2 points, in this case total number of
straight line we have to select 2 points and join
them, so now question is in how many ways we straight lines is given by
15C – 5C + 1 = 105 – 10 + 1 = 96.
can select 2 points from ‘n’ points. In simple 2 2
line this question can not be answered unless it
is given that no three of which are collinear. Number of Triangles from given ‘n’ Points if
If no three of them are collinear then number no Three of Them are Collinear
of straight lines is nC2. To draw a triangles we need three non
Example 1:  How many straight lines can collinear points, and from ‘n’ points 3 points
be drawn from the 15 points in a plane can be selected in nC3 ways.
if no three points are in a straight line? Example 3:  How many triangles can be
Solution:  To draw a straight line we need to drawn from the 15 points in a plane if
select 2 points, so total number of straight no three points are in a straight line?
lines is 15C2 Solution:  To draw a triangle we need to select 3
points, so total number of triangle is 15C3.
Number of Straight Lines from given ‘n’
Points out of them Exactly ‘k’ Points are Number of Polygons (of k sides) from given
Collinear ‘n’ Points if no Three of Them are Collinear
If no three of them are collinear then number To draw a polygon (of k sides) we need ‘k’
of straight lines is nC2 but ‘k’ lines are in points as vertices such that no three of them

QR Code / Video Link

For Smart phone/ Tablet users

Video Link for Desktop/Laptops users http://dishapublication.com/video-resources


Geometrical Construction  73
are collinear points, and from ‘n’ points k straight lines are sides of polygon and that
points can be selected in nCk ways. can not be treated as diagonals hence number
Example 4:  How many hexagons can be n (n − 1) n (n − 3)
of diagonals is nC2 – n = –n= .
drawn from the 15 points in a plane if 2 2
no three points are in a straight line? Example 5:  What is the number of
Solution:  To draw a hexagons we need to diagonals of an octagon?
select 6 points, so total number of hexagons Solution:  Number of sides in an octagon is
is 15C6. 8 hence number of diagonals is given by
8 × 5/2 = 20
Number of Diagonals of a Polygon of ‘n’ Example 6:  If number of diagonals of
Sides polygon is 54 then how many sides
Since number of sides are ‘n’ hence number that polygon has?
of vertices is also ‘n’. To draw a diagonal we Solution:  Since number of diagonals of a polygon
have to select 2 points from ‘n’ points and that
can be done in nC2 ways, but out of those n with n sides is n (n − 3) =  54 or n = 12.
2

Concept 2 Chess Board


A chess board has 8 rows and 8 columns. Every Number of Squares in a Chess Board
alternate square is coloured with white and
To calculate the number squares in the chess
back. Figure below shows the chess board. board we have to see the size of squares:
Number of squares of size 1 × 1 is 8 × 8 = 64
Number of squares of size 2 × 2 is 7 × 7 = 49
and so on
Number of squares of size 8 × 8 is 1 × 1 = 1
Hence total number of squares in a chess
board is 12 + 22 + …. + 82 = 204.

Number of Rectangles in a Chess Board


To calculate the number rectangles in the
chess board we have to select 2 straight lines

QR Code / Video Link

For Smart phone/ Tablet users

Video Link for Desktop/Laptops users http://dishapublication.com/video-resources


74  Theory of Counting
from 9 straight line, this can be done in 9C2 Example 7:  Find the number of squares
ways similarly from another set parallel line in a chess board of 10 × 10.
we can select 2 straight lines from 9 straight Solution:  Number of squares in a chess board
line, this can be done in 9C2 ways hence total of 10 × 10 is given by
number of rectangles is 9C2 × 9C2 = 1296
10 × 11 × 21
12 + 22 + … + 102 = = 385.
In a chessboard of n × n: 6
Number of squares = 12 + 22 + ... + n2 Example 8:  Find the number of rectangles
in a chess b2oard of 10 × 10.
n(n + 1)(2n + 1)
= Solution:  Number of rectangles in a chess
6 board of 10 × 10 is given by
Number of rectangles = n + 1C2 × n + 1C2
2
 n(n + 1) 
2
10 × 11  = 552 = 3025.
=   

 2

  2 

Concept 3 Grid Based Problems

Generally two types of questions can be asked There are 3 + 2 + 1 = 6 rectangles


based on a grid, (3 rectangles of size 1 × 1, 2 rectangle of
Type 1: Number of rectangles, squares, etc. size 1 × 2 and 1 rectangle of size 1 × 3).
Type 2: Number of shortest path from one Similarly, if grid is n × 1: the number
corner to diagonally opposite corner. of rectangles is n + (n – 1) + (n – 2) ... + 1
n(n + 1)
= .
2
Number of Rectangles
Similarly consider a chessboard of n × m
with n rows and m columns:

• If the grid is 1 × 1, Since in a chessboard of n × m with n rows and


m columns, number of vertical lines is (m + 1)
and number of horizontal lines is (n + 1) to
there is 1 rectangle. draw a rectangle we have to select 2 vertical
• If it is 2 × 1, and 2 horizontal straight lines and that can
be done in m + 1C2 × n + 1C2 ways.
Number of rectangles = m + 1C2 × n + 1C2
There are 2 + 1 = 3 rectangles (2 rectangles m(m + 1) n(n + 1) mn(m + 1)(n + 1)
of size 1 × 1, and 1 rectangle of size 1 × 2) = × =
2 2 4
• If it is 3 × 1, Number of squares in a chessboard of n × m
with n rows and m columns (m > n).
Geometrical Construction  75
Lets consider an n × n chessboard, number of 8 × 10 × 9 × 11 = 1980
squares in this is
4
n(n + 1)(2n + 1) 2n3 + 3n2 + n
=
6 6 Number of shortest path
Now, if we add one row of squares at the
bottom, making it an n + 1 by n rectangle,
consider of the dots (or vertices) formed at the
Let there are h number of horizontal steps
corners of the little squares we have added
to make the bottom row. For every pair of and v number of vertical steps then number of
newly-added dots, we will get one new square. ways to reach from one corner to other corner
So the total number of new squares we can is h + vCh = h + vCv.
find in the larger rectangle is the number
Here in the below figure number of horizontal
of pairs of n + 1 dots (or vertices), which is
steps are h = 7 and number of vertical steps
n +1 n(n + 1)
C2 = . v = 5. Then number of ways to reach one corner
2
from the other is given by 7 + 5C7 = 7 + 5C5.
Alternately by adding a new row of n little
squares, we will get within the large rectangle The reason behind this is that if we have total
one new n × n square, two new n – 1 × n – 1 h + v steps, out of these steps if we fix all
squares, three new n – 2 × n – 2 squares, etc. horizontal steps then we have only one way to
So the number of new squares added this way select the vertical step, hence total number of
is the sum of the first n counting numbers, way is given by selecting h steps out of h = v
which is n +1n(n + 1) . steps OR by selecting v steps out of h = v steps
C2 =
2 or we can write h + vCh = h + vCv.
Now, add another row of squares at the bottom,
making it an n + 2 by n rectangle. This adds Example 10:  Consider a grid having 8
the same number of new squares,which is horizontal and 10 vertical paths what
n +1 n(n + 1) is the minimum number of ways a
C2 = .
2
person can travel from one corner to
So the total number of squares that can be
found within an m × n chessboard (m > n) is diagonally opposite corner travelling
given by this formula: minimum distance?

2n3 + 3n2 + n Solution:  Here h = 8 and v = 10, hence


+ (m − n)n + 1 C2
6 required number of ways is h + vCh = 18C8.
2n3 + 3n2 + n (m − n)(n)(n + 1)
= +
6 2
Example 9:  Find the number of rectangles
in a chess board of 8 × 10.
Solution:  Number of rectangles in a chess
board of 10 × 10 is given by
76  Theory of Counting

1
Concept Applicator (CA)
Ideal Time Apply your concepts with easy and
15 Min. conceptual questions

1. How many distinct lines can be drawn and remaining 4 are on other parallel
through 8 points, no three of which are line?
collinear? (a) 52 (b) 28
(a) 210 (b) 240 (c) 48 (d) None of these
(c) 120 (d) None of these 7. How many triangles can be formed by
2. How many distinct lines can be drawn joining the vertices of a polygon of 10
through 8 points, exactly 4 of which are sides?
collinear? (a) 210 (b) 240
(a) 23 (b) 24 (c) 120 (d) None of these
(c) 18 (d) None of these 8. What is the number of diagonals of a
3. How many distinct lines can be drawn polygon with 13 sides?
through 8 points, 4 of which are on a (a) 45 (b) 55
straight line and remaining 4 are on (c) 65 (d) None of these
other straight line which is parallel to
9. What is the number of quadrilaterals
the 1st straight line?
that can be formed by joining the vertices
(a) 23 (b) 24 of a polygon of side ‘K’?
(c) 18 (d) None of these (a) KC K – 2C
4 (b) 4
4. How many distinct triangles can be
(c) (K – 2C )(K – 2C 2) (d) None of these
drawn with their vertices selected from 2

8 points, no three of which are collinear? 10. Find the number of squares which can be
formed on a chessboard.
(a) 23 (b) 28
(c) 48 (d) None of these (a) 64 (b) 204
5. How many distinct triangles can be (c) 226 (d) None of these
drawn with their vertices selected from 8 Consider a set of 6 non-overlapping triangles
points, exactly 4 of which are collinear? in a plane such that no three points in the
(a) 52 (b) 28 plane is collinear.
(c) 56 (d) None of these 11. If all the possible triangles are drawn
taking vertices of these triangles such
6. How many distinct triangles can be
that not more than one point is selected
drawn with their vertices selected from 8
from a triangle then find the total number
points, exactly 4 of which are on one line
of triangles hence drawn?
Geometrical Construction  77
(a) 6C3 × 3C1 (a) 2 × 6C3 × 6C1 + 6C2 × 6C2
(b) 18C3 – 6C3 × 3C1 × 3C1 × 3C1 (b) 6C3 × 3C1 + 6C2 × 3C2
(c) 6C3 × 3C1 × 3C1 × 3C1 (c) 2 × 6C3 × 3C1 + 2 × 6C2 × 3C2
(d) None of these
(d) None of these
12. Find the total number of new triangles
that can be drawn from this system of 14. Consider a square along with its 2
triangles? diagonals, how many triangles is form
by this system?
(a) 18C3 (b) 18C3 – 6C1
(c) 18C3 – 6C3 (d) None of these (a) 6 (b) 9
13. Consider a set of two hexagons (Total 12 (c) 8 (d) None of these
points on vertices), what is the maximum 15. Find the number of diagonals of a polygon
number of quadrilaterals can be drawn with 10 sides.
from these 12 points as vertices such
that at least one point is selected from (a) 35 (b) 40
each hexagon? (c) 42 (d) None of these

Response Grid
1. a b c d 2. a b c d 3. a b c d 4. a b c d 5. a b c d
6. a b c d 7. a b c d 8. a b c d 9. a b c d 10. a b c d
11. a b c d 12. a b c d 13. a b c d 14. a b c d 15. a b c d

2
Concept Builder (CB)
Ideal Time Revise your concepts with questions
25 Min. medium difficulty level questions

1. Consider 2 parallel lines PQ and RS. Line can be formed taking these 10 points as
PQ has two points A and B while line RS vertices of triangle such that either A
has 8 points. How many triangles can be or B is selected?
formed taking these 10 points as vertices (a) 64 (b) 25
of triangle?
(c) 69 (d) None of these
(a) 64 (b) 25
(c) 69 (d) None of these 3. A polygon gas 54 diagonals then find its
number of sides.
2. Consider 2 parallel lines PQ and RS.
Line PQ has two points A and B while (a) 11 (b) 12
line RS has 8 points. How many triangles (c) 13 (d) None of these
78  Theory of Counting
4. Consider a polygon of ‘n’ sides if number (a) PC × QC2 + PC2 × RC2 + QC × RC2
2 2
of diagonals of polygon is 54 then how (b) PC × QC1 × RC1 + PC1 × QC2 × RC1
2
many triangles can be drawn from
taking vertices of polygon as vertices of + PC1 × QC1 × RC2
triangle? (c) (PC2 × QC1 × RC1 + PC1 × QC2 × RC1
(a) 144 (b) 72 + PC1 × QC1 × RC2) + ( PC2 × QC2
(c) 150 (d) None of these + PC2 × RC2 + QC2 × RC2)
5. Consider a set of ‘P’ parallel lines and (d) None of these
another set of ‘Q’ parallel lines then what Consider a set of parallel paths as shown
is the number of vertices of parallelograms in the figure below, with 6 horizontal and 8
thus form by these parallel lines? vertical paths.
(a) PC + QC2 (b) PC × QC2
2 2
(c) PQ (d) None of these
Consider three set of parallel lines having P, Q
and R points respectively and no three points
apart from the given points are collinear.
6. If K represents number of straight lines
that can pass through this system of
P + Q + R points excluding the three 9. Find the number of paths from one
original straight lines then how many of corner to diagonally opposite corner of
the following represents correct value of the network such that distance travelled
is minimum.
K?
P + Q + RC (a) 14C5 (b) 14C6
(i) – (PC2 + QC2 + RC2)
2
(c) 8C6 (d) None of these
(ii) PC1 × QC1 + PC1 × RC1 + RC1 × QC1
10. Let a new diagonal is constructed as
(iii) 2(PC1 × QC1 + PC1 × RC1 + RC1 × QC1) shown in the figure, find the number
(iv) PQ + PR + RQ of paths from one corner to diagonally
opposite corner (From A to B) of the
(a) 1 (b) 2
network such that distance travelled is
(c) 3 (d) 4 minimum.
7. What is the number of triangles that can A
be formed from the system of P + Q + R
points?
(a) P + Q + RC – (PC3 + QC3 + RC3) + 3
3
(b) P + Q + RC – 3(PC3 + QC3 + RC3)
3
(c) P + Q + RC – (PC3 + QC3 + RC3) B
3
(d) None of these (a) 12 (b) 24
8. What is the number of quadrilaterals (c) 36 (d) None of these
that can be formed from the system of Consider a square with 4 points on each side
P + Q + R points? (no points on the vertices).
Geometrical Construction  79

11. How many straight lines can be drawn (a) 1442 (b) 1624
from these 16 points such that each line (c) 1152 (d) None of these
passes through exactly 2 of the given 14. Consider vertices of a hexagon and mid
points? points of its sides (total 12 points). How
(a) 4C2 × 4C1 × 4C1 many triangles can be drawn taking these
(b) 4C2 × 4C2 × 4C1 12 points as vertices of the triangle?
4C (a) 12C – (3C3) (b) 12C – 6(3C3)
(c) 2 × 4C 2 × 4C 2 3 3
(c) 12C + 6(3C3) (d) None of these
(d) None of these 3

12. How many triangles can be drawn from 15. If number of triangles that can be drawn
these 16 points as vertices? from the given set of points as vertices is
(a) 144 (b) 256 56 then find the number of straight lines
that can be drawn from these points?
(c) 544 (d) None of these
(a) 28 (b) 21
13. How many quadrilaterals can be drawn
(c) 45 (d) None of these
from these 16 points as vertices?

Response Grid
1. a b c d 2. a b c d 3. a b c d 4. a b c d 5. a b c d
6. a b c d 7. a b c d 8. a b c d 9. a b c d 10. a b c d
11. a b c d 12. a b c d 13. a b c d 14. a b c d 15. a b c d

3
Concept Cracker (CC)
Ideal Time Boost up your confidence with good
40 Min. questions

1. There are 15 points in a plane out of 2. The sides AB, BC and CA of a triangle
which 6 of them are collinear. Find the ABC have, 3, 4 and 5 interior points
number of quadrilaterals formed by respectively on them. Find the number
joining these points as vertices? of triangles that can be constructed using
(a) 1350 (b) 1360 given interior points as vertices.
(c) 1372 (d) None of these (a) 215 (b) 205
(c) 219 (d) None of these
80  Theory of Counting
3. Consider 2 parallel lines PQ and RS. 9. A parallelogram is cut by two set of parallel
Line PQ has three points A, B and C lines parallel to sides of parallelogram.
while line RS has 7 points. How many Each set has ‘m’ parallel lines. What is
triangles can be formed taking these 10 the total number of parallelograms thus
points as vertices of triangle such that A formed?
is not selected with B or C? {(m + 1)2 }
(a)
(a) 70 (b) 25 4
(c) 69 (d) None of these {(m + 1)2 (m + 2)2 }
4. Consider a set of ‘P’ parallel lines and (b) 4
another set of ‘Q’ parallel lines then what
(c) mC × m + 1C2
is the number of parallelogram thus form 2
by these parallel lines? (d) None of these
(a) PC2 + QC2 (b) PC2 × QC2 10. Consider n points in a plane no three
(c) PQ (d) None of these of which are collinear and the ratio of
5. Consider three set of parallel lines in a number of hexagon and octagon that can
plane containing ‘a’ , ‘b’ and ‘c’ parallel be formed from these n points is 4:13
lines respectively. What is the highest then find the value of n.
number of parallelograms that can be (a) 14 (b) 20
formed with these set of parallel line? (c) 28 (d) None of these
(a) 14 [ab(a – 1)(b – 1) + bc(b – 1)(c – 1) 11. Consider n points in a plane no three
+ ac(a – 1)(c – 1)] of which are collinear and the number
(b) 14 [abc + (a – 1)(b – 1)(c – 1)] of polygon with 15 sides and number
of polygon with 30 sides are same then
(c) 14 [ab + bc + ca + (a – 1)(b – 1)(c – 1)]
which one of the following is correct?
(d) None of these (i) If K is the number of a polygon with
6. Let Tn denotes the number of triangles P sides then for maximum value of K
which can be formed by using the vertices the value of P is 23.
of a regular polygon of ‘n’ sides. Which (ii) Number of polygon with 10 sides
one of the following could be the value of and number of polygon with 25 sides
Tn + 1 – Tn – 1 ? are same.
(a) 142 (b) 144 (iii) Number of polygon with 11 sides
(c) 148 (d) one of these and number of polygon with 34 sides
7. Consider ‘n’ points in a plane, if number of are same.
heptagon (polygon with 7 sides) is equal (a) Only (i) and (ii)
to the number of Octagons (polygon with
(b) Only (i) and (iii)
8 sides) then find the number of triangles
that can be drawn from these n points. (c) Only (ii) and (iii)
(a) 142 (b) 455 (d) None of these
(c) 289 (d) None of these 12. Consider 35 points in a plane such that
8. Find the number of non-congruent no. 3 of them are collinear. A students is
rectangles on a chess board. told to 1st draw all the possible polygons
of same number of sides and then draw
(a) 8C2 – 8C8 (b) 8C1 + 8C2
all the possible diagonals. What is the
8 7
(c) C2 – C2 (d) None of these
Geometrical Construction  81
maximum number of such diagonals (c) 2 (d) None of these
possible? (If a particular line is diagonal 16. If ‘P’ is the maximum number of
of two polygons then count both of them.) quadrilaterals that can be drawn from
(a) 119(35C17) (b) (35C17) these nk points as vertices then how
(c) (35C17) × (35C17) (d) None of these many of the following best represents
13. Consider a set of n points in a plane, If the value of P ?
number of polygon of K sides that can be (i) nC (kC × kC1 × kC1 × kC1) + (nC3)(3 ×
4 1
formed from these n points as vertices kC × kC1 × kC1 ) + nC2 × kC2 × kC2
2
is more than that of polygon of (K – 3) nkC
sides and K + 3 sides. How many distinct (ii) 4 – n[kC4 + kC3 × k(n – 1)]
(iii) nkC – k[nC4 + kC3 × k(n – 1)]
values of n exist? [for a given set of n & K] 4

(a) 4 (b) 6 (a) 0 (b) 1


(c) 8 (d) None of these (c) 2 (d) None of these
17. Find the number of rectangles in a chess
Consider a polygon of n sides with K points on
board of 10×10 grid instead of 8×8 grid.
each side (no points on the vertices).
(a) 4225 (b) 3025
14. How many straight lines can be drawn
from these ‘nk’ points such that each line (c) 2025 (d) None of these
passes through exactly 2 of the given 18. Find the number of squares in a chess
points? board of 10 × 12 grid instead of 8 × 8 grid.
(a) nC2 × kC1 × kC1 (a) 4225 (b) 3025
(b) nC2 × kC2 × kC2 (c) 2025 (d) None of these
(c) nC2 (kC1 + kC1) 19. Find the number of rectangles in a chess
(d) None of these board of 10 × 12 grid instead of 8 × 8 grid.
15. If ‘P’ is the maximum number of triangles (a) 4225 (b) 3025
that can be drawn from these nk points as (c) 2025 (d) None of these
vertices then how many of the following
best represents the value of P ? 20. In how many ways can you place 2 rooks
on a chessboard such that they are not in
(i) (nC3 × kC1 × kC1 × kC1) + 2(nC2 )(kC2 × kC1) attacking positions, if rooks can attack
(ii) nkC – n(kC3) only in a same row or in a same column?
3
(iii) (nC3 × kC1 × kC1 × kC1) + 2(nC3 )(kC2 × kC1) (a) 1568 (b) 784
(c) 3136 (d) None of these
(a) 0 (b) 1

Response Grid
1. a b c d 2. a b c d 3. a b c d 4. a b c d 5. a b c d
6. a b c d 7. a b c d 8. a b c d 9. a b c d 10. a b c d
11. a b c d 12. a b c d 13. a b c d 14. a b c d 15. a b c d
16. a b c d 17. a b c d 18. a b c d 19. a b c d 20. a b c d
82  Theory of Counting

Solutions
Concept Applicator (CA)
1. (d) 2. (a) 3. (c) 4. (d) 5. (a) 6. (c)
7. (c) 8. (c) 9. (a) 10. (b) 11. (c) 12. (b)
13. (a) 14. (c) 15. (a)

1. (d) In order to draw a straight line we 3. (c) Same as previous question total
have to select 2 points from available number of ways is:
8 points and that can be done in 8C2 (4C1)(4C1) + 2 = 16 + 2 = 18
ways or 28 ways. 4. (d) From 8 points we have to select 3
2. (a) In counting we have different points and it can be done in 8C3 ways
process; this question can be solved
or in 56 ways.
by different methods:
5. (a) 8 points could have given us 8C3
Method 1: Without any restriction triangles if no three of them are in
number of straight lines is 8C2 ways a straight line, but 4 of them are
or 28. Now the restriction is that 4 in straight line so we will get 4C3
of them are collinear, since these 4 number of triangle less so required
could have given us 4C2 ways or 6 number of triangles is 8C3 – 4C3.
straight lines but instead of 6 they
56 – 4 = 52
are giving us only 1 straight line
6. (c) Same as previous question total
(Since these four point are collinear
number of ways is:
so they will form 1 straight line)
so total number of straight lines is (4C1)(4C2) + (4C2)(4C1) = 24 + 24 = 48
28 – 6 + 1 = 23 straight lines. 7. (c) Out of 10 vertices we have to select 3
and that can be done in 10C3 ways =
Method 2: Number of straight lines
120 ways.
from 4 non-collinear points is 4C2 or
6 straight lines. 8. (c) A polygon with ‘K’ sides has ‘K’
number of vertices. We can draw
Number of straight lines from 4 KC number of straight lines from K
collinear points is 1 straight line. 2
vertices but out of these there are ‘K’
Number of straight lines if we take number of sides and remaining are
one point from 4 non-collinear points diagonals.
(in 4C1 or 4 ways) is 4 and one from Number of diagonals
4 collinear points (in 4C1 or 4 ways)
is also 4, hence number of straight = K C − K = K(K − 3)
2
lines is 4 × 4 = 16. 2
Here in this case K = 13 hence
Total number of straight lines is
number of diagonals is 13 × 10/2 = 65
6 + 1 + 16 = 23
Geometrical Construction  83
9. (a) Here we have to select 4 points 13. (a) When 1 point selected from 1
out of K point in order to draw a Hexagon and 3 from another in
quadrilateral and it can be done in 2 × 6C 3 × 6C 1.
KC ways. And when 2 point selected from
4
10. (b) Number of squares on a chessboard 1st Hexagon and 3 from another in
6C × 6C .
is 12 + 22 + 32 + ... + 82 = 204. 2 2

11. (c) Total number of points in the plane So total number of quadrilaterals
is 6 × 3 = 18 points. We have to select 2 × 6C 3 × 6C 1 + 6C 2 × 6C 2.
3 triangles from 6 and that can be 14. (c) Consider the square along with two
done in 6C3 ways. Now from these diagonals:
triangles we have to select one point
from each of the triangles that can
be done in 3C1 × 3C1 × 3C1.
Hence total number of triangles is
6C 3 3 3
3 × C 1 × C 1 × C 1.
12. (b) Total number of triangles can be Number of smaller triangles is 4 and
drawn is 18C3 but out of these 6 are number of right angled triangle is
original triangles hence number 4 so total number of triangles is 8.
of new triangles is 18C3 – 6 or 15. (a) Since number of diagonals is
18C – 6C .
3 1 n(n – 3)/2 = 10 × 7/2 = 35.

Concept Builder (CB)


1. (a) 2. (d) 3. (b) 4. (d) 5. (c) 6. (b)
7. (c) 8. (c) 9. (b) 10. (b) 11. (a) 12. (c)
13. (b) 14. (b) 15. (a)

1. (a) We have following cases: 4. (d) From the given condition since
number of polygons is 54 hence
PQ RS n(n – 3)/2 = 54 or n = 12
(2 Points) (8 points) Number of triangles from these 12
points is 12C3 = (12 × 11 × 10)/6 = 220.
Case (i) 1 (in 2C1 2 (in 8C2 2 × 28
= 2 ways) = 28 ways) = 56 5. (c) Consider 1 line from P straight lines,
it is intersected by another set of
Case (ii) 2 (in 2C2 1 (in 8C1 1 × 8 = 8 parallel lines at Q points hence total
= 1 ways) = 8 ways) number of such points is PQ.
6. (b) Without restriction we can select
2 points from P + Q + R points
Total number of ways is 56 + 8 = 64.
in P+Q+RC2 ways but since points
2. (d) It is same as case (i) of previous are collinear hence we will not get
question, hence number of ways is 56. PC + QC + RC these straight lines
2 2 2
3. (b) Since number of diagonals is given hence required number of straight
by n(n – 3)/2 = 54 or n(n – 3) = 108 lines is given by:
= 12 × 9 hence n = 12. P+Q+RC – (PC + QC + RC ).
2 2 2 2
84  Theory of Counting
If we select 1 point from P points and 9. (b) Here H = 6 and V = 8 so total number
one point from Q points then number of such path is H+VCH or H+VCV = 14C6.
of straight lines is PC1 × QC1 = PQ. 10. (b) As we have done in previous question
Similarly from other two pairs we number of ways is
will get PC1 × RC1 = PR and 4C × 4C = 6 × 4 = 24 ways.
2 1
RC × QC = RQ.
1 1 11. (a) From four sides we have to select
So total number of ways is PC1 × QC1 2 sides that can be done in 4C2 = 6
+ PC1 × RC1 + RC1 × QC1 ways. Now from each side we have
= PQ+ PR + RQ to select 1 point and that can be done
So only two of the given expression in 4C1 × 4C1= 4 × 4 = 16 ways.
is true. So total number of such straight
lines is 4C2 × 4C1 × 4C1 = 96 ways.
7. (c) Without restriction we can select
3 points from P + Q + R points 12. (c) In order to draw a triangle we need
three non collinear points. Here in
in P+Q+RC3 ways but since points
this case we have two cases:
are collinear hence we will not get
PC + QC + RC these triangles Case (i): If three points selected
3 3 3 from three different sides: 4C3 × 4C1
hence required number of triangles
× 4C1 × 4C1 = 4 × 4 × 4 × 4 = 256.
is given by:
P+Q+RC – (PC + QC + RC ). Case (ii): If two points are selected
3 3 3 3 from one side while 1 point is selected
8. (c) In order to form a quadrilaterals from different side: (4C2)(4C2 × 4C1 +
we need four points and no three of 4C × 4C ) = 6(24 + 24) = 288.
2 1
them should be collinear so we have
to take 2 points from one straight So total number of such triangles is:
line and then 1 point from each of 256 + 288 = 544.
the remaining two straight lines or 2 From 16 points without any restriction
points from any of two straight lines- we can get maximum 16C3 = (16!)/(13!)
Case (i): 2 points from one straight (3!) = (14 × 15 × 16)/6 = 560.
line and then 1 point from each of From this we have to reduce the
number of triangles that we can
the remaining two straight lines:- in not get since 4 sides has 4 collinear
this case number of quadrilateral is points each, which is equal to 4(4C3)
PC × QC × RC + PC × QC × RC +
2 1 1 1 2 1
= 16.
PC × QC × RC So total number of such triangles is:
1 1 2
560 – 16 = 544
Case (ii): When 2 points from any of
13. (b) In order to draw a quadrilateral we
two straight lines:
need 4 points such that no 3 of them
PC × QC2 + PC2 × RC2 + QC2 × RC2.
2 are collinear points. Here in this
So total number of such quadri- case we have three cases:
laterals is: (PC2 × QC1 × RC1 + PC1 Case (i): If 4 points selected from 4
× QC2 × RC1 + PC1 × QC1 × RC2) + different sides: 4C1 × 4C1 × 4C1 × 4C1
= 4 × 4 × 4 × 4 = 256.
(PC2 × QC2 + PC2 × RC2+ QC2 × RC2)
Geometrical Construction  85
Case (ii): If two points are selected not get if three points are collinear
from one side while 1 point is selected this can happen in two cases:
from two different sides: Case (i): When all the four selected
(4C3)(3×4C2 × 4C1 × 4C1 ) = 4 (3 × 6 × points are collinear: 4 × 4C4 = 4.
4 × 4) = 1152.
Case (ii): When three points are
Case (iii): If two points are selected collinear 4C1 × 4C3 × 12 = 192.
from one side while 2 points are
selected from different side: So total number of such triangles is
4C × 4C × 4C = 6 × 6 × 6 = 216. 1820 – 4 – 192 = 1624
2 2 2
So total number of such quadrilaterals 14. (b) From 12 points we can draw 12C3
is: 256 + 1152 + 216 = 1624. triangles but there are 6 groups of
From 16 points without any 3 collinear points hence we will get
restriction we can get maximum 6(3C3) triangles less, hence required
16C = (16!)/(12!)(4!) = (13 × 14 × 15 number of triangles is 12C3 – 6(3C3).
4
× 16)/24 = 1820. 15. (a) From the given information
From this we have to reduce the nC
3 = (n – 2)(n – 1)(n)/6 = 56 or n = 8
number of quadrilateral that we can hence number of straight lines is 8C2=28.

Concept Cracker (CC)


1. (d) 2. (b) 3. (a) 4. (b) 5. (a) 6. (b)
7. (b) 8. (b) 9. (b) 10. (b) 11. (b) 12. (a)
13. (b) 14. (a) 15. (b) 16. (b) 17. (b) 18. (d)
19. (d) 20. (a)

1. (d) Here we have following cases-



No. of points from 6 collinear No. of points from other 9
Total no. of ways
points points

Case (i) 0 (in 6C0 = 1 way) 4 (in 9C4 = 126 ways) 1 × 126 = 126

Case (ii) 1(in 6C1 = 6 ways) 3 (in 9C3 = 84 ways) 6 × 84 = 504

Case (iii) 2 (in 6C2 = 15 way) 2 (in 9C2 = 36 way) 15 × 36 = 540

Total number of ways is 126 + 504 + 540 = 1170

2. (b) Total number of points is 3 + 4 + 5 = 12 points. If non of them are collinear then
number of triangles is 12C3 = 220. But actually we will get lesser number of triangles
as some points are collinear and this number is 3C3, 4C3 and 5C3 hence required
number of triangles is 12C3 – 3C3 – 4C3 – 5C3 = 220 – 1 – 4 – 10 = 205.
86  Theory of Counting
3. (a) We have following cases:

PQ (3 Points) RS (7 points)

Case (i) when A is selected 1 way 2 (in 7C2 = 21 ways) 1 × 21 = 21

Case (ii) when A is not selected 2 (in 2C2 = 1 ways) 1 (in 7C1 = 7 ways) 1×7=7

Case (iii) when A is not selected 1 (in 2C1 = 2 ways) 2 (in 7C2 = 21 ways) 2 × 21 = 42

Total number of ways is 42 + 7 + 21 = 70


Then Tn + 1 – Tn – 1
4. (b) To get a parallelogram we need to
= {
select 2 lines from P lines and that (n − 1)(n)(n + 1)} – {(n − 3)(n − 2)(n − 1)}
can be done in PC2 ways at the same 6 6
2
time 2 lines from Q lines and that = (n – 1) hence this value must be a
can be done in QC2 ways hence total perfect square, from the given options
number of parallelograms is only option (b) is a perfect square.
PC × QC . 7. (b) Number of heptagons with n points
2 2 as vertices is nC7 and number of
5. (a) Consider pair of parallel lines from octagons with n points as vertices
is nC8 from the given condition
‘a’ and ‘b’ parallel line. 1st we will nC = nC or (n!)/{(n – 7)! × 7!}
7 8
select 2 lines from ‘a’ straight lines
= (n!)/{(n – 8)! × 8!} = n – 7 = 8 or n = 15.
and 2 lines from ‘b’ straight lines,
Hence number of triangles from 15
this can be done in aC2 × bC2
points is
a(a − 1) b(b − 1) 1 15C = 13 × 14 × 15/6 = 455.
= × = [ab(a − 1)(b − 1)] 3
2 2 4 8. (b) 1st we will count the number of
Similarly, we can select pair from ‘b’ squares (since squares are also
and ‘c’ parallel lines and from ‘a’ and rectangles), size of square may be
‘c’ parallel lines. 1 × 1, 2 × 2 … or 8 × 8 hence 8 such
square exist = 8C1 = 8C7.
So total such parallelogram is
Number of rectangles with side m
1/4 [ab(a – 1)(b – 1) + bc(b – 1)(c – 1) and n such that m ≠ n is 8C2.
+ ac(a – 1)(c – 1)] Hence required number of rectangles
is 8C2 + 8C1.
6. (b) From the given condition
9. (b) In the system now total (m + 2) and
another set of (m + 2) parallel lines.
Tn+1 = (n+1)C3 From (m + 2) lines two lines can be
selected in m + 2C2 ways. So total
{(n + 1)!} {(n − 1)(n)(n + 1)} number of parallelogram is
= =
{(n − 2)! 3!} 6 {(m + 1)2 (m + 2)2 }
m + 2C m + 2C =
2× 2 4
and Tn – 1 = (n – 1)C3

{(n − 1)!} {(n − 3)(n − 2)(n − 1)} 10. (b) From n points number of hexagon is
= = n!
{(n − 4)! 3!} 6 nC = .
6
{(n − 6)!}{6!}
Geometrical Construction  87
From n points number of Octagons Hence total 5 values of n satisfy the
n! given condition.
is nC8 = .
{(n − 8)!}{8!} 14. (a) From n sides we have to select
Ratio of number of hexagon to 2 sides that can be done in nC2 ways.
number of octagon is Now from each side we have to select
{(n − 8)!}{8!} 7×8 4 1 point and that can be done in
= = kC × kC ways.
{(n − 6)!}{6!} (n − 7)(n − 6) 13 1 1
So total number of such straight
On solving this quadratic equation lines is nC2 × kC1 × kC1.
we will get n = 20 15. (b) In order to draw a triangle we need
11. (b) From the given condition nC15 = nC30 three non collinear points. Here in
then n = 15 + 30 = 45. this case we have two cases-
Hence statement (i) and (iii) are true Case (i) – if three points selected
since if n = 45 then nC23 will have from three different sides:
maximum value and since nC × kC × kC × kC .
3 1 1 1
11 + 34 = 45 hence nC11 = nC34 Case (ii) – if two points are selected
12. (a) We know that 35Cr will have from one side while 1 point is selected
maximum value when r = 17 so from different side:
number of polygons is 35C17 and from (n­C2 )(kC2 × kC1 + kC2 × kC1)
one polygon we will get 17(17 – 3)/2
= 119 diagonals hence maximum So total number of such triangles is
number of diagonals is 119(35C17). (nC3 × kC1 × kC1 × kC1) + 2(nC2 )(kC2
× kC1).
13. (b) From the given condition nCk is more
than nCk–3 and nCk + 3 we know that Alternately:
nC attains a maximum value if r is the From nk points without any restriction
r
middle term of Binomial expansion or we can get maximum nkC3.
Case (i) if n is odd then n = 2r + 1. From this we have to reduce the
Case (ii) if n is even then n = 2r or 2(r – 1). number of triangles that we can not
get form n sides has k collinear points
Consider the consecutive terms each, which is equal to n(kC3).
nC n n n nC
k – 3, Ck – 2, Ck – 1, Ck, k + 1 So total number of such triangles is
nC and n C . nkC – n(kC ).
k+2 k+3 3 3
If n = 2k then nCk, nCk+1 are maximum Statement (i) and (ii) is correct.
and the given conditions satisfy.
16. (b) In order to draw a quadrilateral we
If n = 2k+1 then nCk + 1 is maximum need 4 points such that no 3 of them
and the given conditions satisfy. are collinear points. Here in this
If n = 2k – 1 then nCk, is maximum case we have three cases:
and the given conditions satisfy. Case (i) – if 4 points selected from 4
If n = 2k + 2 then nCk + 1, nCk + 2 are different sides :
maximum and the given conditions nC (kC × kC1 × kC1 × kC1)
4 1
satisfy.
Case (ii) – if two points are selected
If n = 2k – 2 then nCk – 1, nCk are
from one side while 1 point is selected
maximum and the given conditions
satisfy. from two different sides
88  Theory of Counting
(nC3)(3 × kC2 × kC1 × kC1 ) another set of parallel lines has 11
Case (iii) – if two points are selected parallel lines. To get a rectangle we
from one side while 2 points are have to select 2 lines from 11 parallel
lines hence number of such rectangle
selected from different side
is 11C2 × 11C2 = (11 × 10/2)2 = 3025.
nC × kC2 × kC2.
2 18. (d) Since the total number of squares
So total number of such that can be found within an m × n
quadrilaterals is: chessboard (m > n) is given by this
nC (kC × kC1 × kC1 × kC1) + (nC3) formula:
4 1
(3 × kC2 × kC1 × kC1) + nC2 × kC2 × kC2 2n3 + 3n2 + n
+ (m − n)n + 1 C2
Alternately: 6
From nk points without any 2n3 + 3n2 + n (m − n)(n)(n + 1)
= +
restriction we can get maximum 6 2
nkC . Here m = 12 and n = 10 hence
4
required number of squares is
From this we have to reduce the
(2 × 1000 + 300 + 10)/6+(2)(10)(11)/2
number of quadrilaterals that we can = 385 + 110 = 495.
not get if three points are collinear
19. (d) Number of rectangles is 11C2 × 13C2
this can happen in two cases
20. (a) The first rook can be placed in 64
Case (i) – when all the four selected ways, and the second rook cannot
points are collinear— n × kC4. be placed in the same row or the
Case (ii) when three points are same column. So, it has 7 rows and
collinear n × kC3 × k(n – 1). 7 columns left for it. It can be placed
in 7 × 7 = 49 ways.
So total number of such triangles is
nkC k k But the order in which the rooks are
4 – n[ C4 + C3 × k(n – 1)].
placed is not important. So, it will be
Statement (i) and (ii) is correct. divided by 2!
17. (b) In the grid we have 11 parallel Total number of ways
lines in one set of parallel lines and
= 64 × 49/2 = 1568.
Part B Geometrical Construction  89

Formation on
Concept 1 Geometrical Figures
• Number of straight lines from given • Number of triangles from given ‘n’
‘n’ points:- Consider ‘n’ points in a points if no three of them are collinear:-
plane and we need to find the number To draw a triangles we need three non
of straight lines that can be draw from collinear points, and from ‘n’ points 3
these ‘n’ points. In order to draw a points can be selected in nC3ways.
straight line we have to select 2 points • Number of polygons (of k sides)
and join them, so now question is in how from given ‘n’ points if no three of
many ways we can select 2 points from them are collinear:-To draw a polygon
‘n’ points. In simple line this question (of k sides)we need ‘k’ points as vertices
can not be answered unless it is given such that no three of them are collinear
that no three of which are collinear. points, and from ‘n’ points k points can
If no three of them are collinear then be selected in nCkways.
number of straight lines is nC2. • Number of diagonals of a polygon
• Number of straight lines from of ‘n’ sides:-Since number of sides are
given ‘n’ points out of them exactly ‘n’ hence number of vertices is also ‘n’.
‘k’ points are collinear:-If no three to draw a diagonal we have to select 2
of them are collinear then number of points from ‘n’ points and that can be
straight lines is nC2 but ‘k’ lines are in done in nC2 ways, but out of those n
same straight lines hence we will not get straight lines are sides of polygon and
kC straight lines hence total number of that can not be treated as diagonals
2
straight lines is nC2 – kC2 +1 hence number of diagonals is nC2 – n
n (n − 1) n (n − 3)
= – n=
2 2

Concept 2 Chess Board


A chess board has 8 rows and 8 columns. Every • Number of squares in a chess board:- To
alternate square is coloured with white and calculate the number squares in the chess
back. Figure below shows the chess board. board we have to see the size of squares-
90  Theory of Counting
• Number of rectangles in a chess
board:- To calculate the number
rectangles in the chess board we have to
select 2 straight lines from 9 straight line,
this can be done in 9C2 ways similarly
from another set parallel line we can
select 2 straight lines from 9 straight
line, this can be done in 9C2 ways hence
total number of rectangles is 9C2 × 9C2 =
1296
• In a chessboard of n×n: Number of
squares = 12 + 22 + ..+n2

Number of squares of size 1 × 1 is 8 × 8 = 64 n (n + 1)(2 n + 1)


=
Number of squares of size 2 × 2 is 7 × 7 = 6
49 and so on Number of rectangles
Number of squares of size 8 × 8 is 1 × 1 = 1 2
 n (n + 1)
Hence total number of squares in a chess = n+1C2 × n+1C2=  
2 2
  . +  82 = 204  2 
board is 1 +  2 +…

Concept 3 Grid Based Problems


Generally two types of questions can be asked Since in a chessboard of n × m with n rows
based on a grid, and m columns, number of vertical lines is
Type 1:- Number of rectangles, squares etc. (m+1) and number of horizontal lines is (n+1)
to draw a rectangle we have to select 2 vertical
Type 2:- Number of shortest path from one
and 2 horizontal straight lines and that can
corner to diagonally opposite corner.
be done in m+1C2 × n+1C2ways
Number of rectangles Number of rectangles = m+1C2 × n+1C2
If grid is n × 1: the number of rectangles is m (m + 1) n (n + 1) mn (m + 1)(n + 1)
(n)(n + 1) = × =
n + (n – 1) + (n – 2) ... + 1 = 2 2 4
2
Number of squares in a chessboard of
Similarly consider a chessboard of n × m
n × m with n rows and m columns (m > n)
with n rows and m columns-
Geometrical Construction  91
Lets consider an n × n chessboard, number of v = 5. Then number of ways to reach one corner
squares in this is from the other is given by 7+5C7 = 7+5C5

n (n + 1)(2 n + 1) 2 n3 + 3n2 + n

6 6

The total number of squares that can be found


within an m × n chessboard (m > n) is given by
2 n3 + 3n2 + n
this formula: + (m–n)
6
n+1C = 2 n + 3n + n (m − n )(n )(n + 1)
3 2
2 +
6 2
Number of Shortest Path The reason behind this is that if we have total
Let there are h number of horizontal steps h + v steps, out of these steps if we fix all
and v number of vertical steps then number of horizontal steps then we have only one way to
ways to reach from one corner to other corner select the vertical step, hence total number of
is h+vCh = h+vCv way is given by selecting h steps out of h = v
Here in the below figure number of horizontal steps OR by selecting v steps out of h = v steps
or we can write h+vC = h+vCv
steps are h = 7 and number of vertical steps h
92  Theory of Counting

1
Concept Deviator (CD)
Ideal Time Revise your concept (IIT advance or
5 Min. tougher questions)
Per Question

1. Consider a polygon of k sides. What 4. In how many ways two identical kings
is the number of triangles that can be can be placed on a 8 × 8 chess board such
drawn taking vertices of these polygons they are not on adjacent squares?
as vertices of triangles and no sides of (a) 1442 (b) 3612
triangles is common with any sides of (c) 3422 (d) None of these
the polygon?
5. In how many ways two queens one black
k (k − 1)(k − 5) and one white can be placed on a 8×8
(a) chess board such they are not able to
6
attack each other (Queens can attack in
k (k − 4 )(k − 5) the same row/column/diagonal)?
(b)
6 (a) 2576 (b) 3612
( k − 3) (k − 4 )(k − 5) (c) 3422 (d) None of these
(c)
6 6. In how many ways two identical queens
(d) None of these can be placed on a 8×8 chess board such
that they are not able to attack each
2. There are ‘P’ collinear points and another other (Queens can attack in the same
set of ‘Q’ collinear points in a plane. All row/column/diagonal)?
the possible straight lines are drawn (a) 2576 (b) 3612
from the given (P + Q) points. What is
(c) 3422 (d) None of these
the number of intersecting points of
these straight lines (excluding the points Direction (Qs. 7 and 8):
on the two straight lines i.e excluding The basic property of a triangle is that sum
original P and Q points)? of two sides must be more than the 3rd side.
If triangles are formed with integral sides
(a) P2 × Q2
with minimum length of side is 1 unit and
(b) PC × QC2 – PQ
2 maximum length is n (an even number) units.
(c) 1 / 4 PQ (P − 1)(Q − 1) P, Q, R and S is defined as-
P: Number of scalene triangles with one
(d) None of these
side is 1 unit.
3. In how many ways two kings one black
Q: Number of scalene triangles with one
and one white can be placed on a 8×8
side is 2 units.
chess board such they are not on adjacent
squares? R: Number of isosceles triangles that can be
formed
(a) 1442 (b) 3612
S: Number of equilateral triangles that can
(c) 3422 (d) None of these be formed.
Geometrical Construction  93
7. Find the value of |P – Q| . (a) 4m+ n +1
(a) n (b) n – 2 (b) mn(m+1)(n+1)
(c) n + 2 (d) None of these (c) m2 n2
8. Find the value of |R – S| 2
(d) (m + n + 1)
n (n − 8) n (n − 4 ) 11. The sides AB, BC, & CA of a triangle
(a) (b)
4 4 ABC have 3, 4 and 5 interior points
n (n + 4 ) respectively on them. The number of
(c) (d) None of these
4 triangles that can be constructed using
9. Let Tn be the number of all possible these interior points as vertices is-
triangles formed by joining vertices of an [IIT JEE 1984]
n-sided regular polygon. If Tn+1 − Tn = 10, (a) 210 (b) 205
then the value of n is [IIT JEE 2013] (c) 310 (d) None of these
(a) 5 (b) 10 12. In how many ways can two squares be
(c) 8 (d) 7 chosen on a 8 × 8 chessboard such that
they have only one corner in common?
10. A rectangle with sides 2m – 1 and 2n – 1
is divided into squares of unit length by (a) 98 (b) 94
drawing parallel lines as shown in the (c) 108 (d) None of these
diagram, thenthe number of rectangles 13. There is a 36 sided regular polygon. How
possible with odd side lengths is many regular polygons can be formed
[IIT JEE 2005] by joining the vertices of the regular
polygon?
(a) 32 (b) 35
(c) 37 (d) None of these
14. If K is the minimum number of straight
lines that is required to get 16 non
overlapping straight lines then find the
value of K. (Here non overlapping means
two straight lines may be co-linear
without overlapping)
(a) 8 (b) 10
(c) 19 (d) 14

Response Grid
1. a b c d 2. a b c d 3. a b c d 4. a b c d 5. a b c d
6. a b c d 7. a b c d 8. a b c d 9. a b c d 10. a b c d
11. a b c d 12. a b c d 13. a b c d 14. a b c d
94  Theory of Counting

2
Concept Eliminator (CE)
Ideal Time Revise your concept (Maths Olympiad
Just solve it or tougher questions)

Direction (Qs. 1 and 2): (a) 9901 (b) 9902


There are ‘K’ points in a plane. No three of (c) 9903 (d) None of these
which are on same straight line. All the This question is based on International
possible straight lines are made by joining
Maths Olympiad:
these ‘K’ points.
5. Consider 5 points in a plane are situated
1. What is the maximum number of point of
so that no two of the straight lines joining
intersection of these straight lines?
them are parallel, perpendicular, or co
(a) KC × KC2 (b) K(KC2 )
2 incident. From each point perpendiculars
(c) K(KC2 × KC2) (d) None of these are drawn to all the lines joining
2. Taking point of intersection of these the other four points. Determine the
straight lines as vertices of triangles maximum number of intersections that
then what is the maximum number of these perpendiculars can have?
triangles that can be formed? (a) 435 (b) 415
(a) {k(k-1)/2C }C – {k(k–1)/2}{(k-2)(k+1)/2C3}
2 3 (c) 315 (d) None of these
(b) {k(k-1)/2C }C – {k(k-1)/2}{k(k+1)/2C3}
2 3 Question number 6 and 7 is based on
(c) {k(k-1)/2C }C – {(k-2)(k+1)/2C3}
2 3 International Maths Olympiad:
(d) None of these Consider a decomposition of an 8 × 8
3. Consider ‘k’ straight lines in a plane chessboard into p non overlapping rectangles
such that no two of which are parallel with the following conditions-
and no three of which pass through same Condition (i): Number of white and number
point. How many new straight lines can
of black squares are same.
be drawn from the point of intersection
of these straight lines? Condition (ii): If ai is the number of white
squares in the ith rectangle then
k (k − 1)(k − 2 )
(a) a1< a2< a3< … <ap.
8
k (k − 1)(k − 2 )( k − 3) 6. Find the maximum possible value of P
(b)
8 (a) 2 (b) 4
k (k − 1)(k − 2 )( k − 3) (c) 8 (d) None of these
(c)
4 7. How many such different cases are
(d) None of these
possible if P is maximum?
4. 100 circles are drawn on a plane what is
the maximum number of regions made (a) 2 (b) 4
by this system of 100 circles? (c) 8 (d) None of these
Geometrical Construction  95
This question is based on Regional This question is based on Maths Olympiad:
Maths Olympiad 9. In how many ways 2 ‘+’ and 2 ‘–’ signs are
8. Consider a 6 × 6 square which is dissected filled into 4 × 4 cell where each cell can
into 9 rectangles by lines parallel to its contain maximum 1 character such that
sides such that all the rectangles have each row and column can not contain
same sign?
integer sides. Out of 9 rectangles what
is the minimum number of congruent
rectangles?
(a) 0 (b) 1
(c) 2 (d) None of these

(a) 3960 (b) 3340


(c) 3255 (d) None of these

Response Grid
1. a b c d 2. a b c d 3. a b c d 4. a b c d 5. a b c d
6. a b c d 7. a b c d 8. a b c d 9. a b c d
96  Theory of Counting

Solutions

Concept Deviator (CD)


1. (b) 2. (c) 3. (b) 4. (d) 5. (a) 6. (d)
7. (b) 8. (a) 9. (a) 10. (c) 11. (b) 12. (a)
13. (c) 14. (b)

1.
(b) Total number of triangles formed lines and which in turn will
is kC3, now consider the following give us 1 point of intersection.
cases- So total number of points is
Case (i): Number of triangles having = PC2 × QC2
three sides common with the sides P (P − 1) Q (Q − 1) 1
of polygon is 0. = × = PQ (P − 1)(Q − 1)
2 2 4
Case (ii): Number of triangles having 3. (b) Here we have following three cases
two sides common with the sides of for placing the 1st king-
polygon is k. Case (i): Lets place 1st king at one of the
Case (iii): Number of triangles having corners (in 4 ways) then 2nd king can
one sides common with the sides be placed in 60 ways so number of
of polygon = number of ways of ways in this case is 4 × 60 = 240
selecting 3 points (vertices) out of Case (ii): If 1st king is placed at a square
which only two are consecutive is on edge (in 24 ways) then 2nd king
K(K–4C1) = K(K–4). can be placed in 58 ways so number
So number of required triangles is of ways in this case is 24 × 58 =
kC – k – k(k–4)
3
1392
= k(k–1)(k–2)/6 –k – k(k–4) Case (iii): If 1st king is placed at a
interior square (in 36 ways) then
k (k − 4 )(k − 5)
= 2nd king can be placed in 55 ways
6 so number of ways in this case is
2. (c) To get desired point of intersection 36 × 55 = 1980
we have to select two points from
1st straight line (i.e from P points So total number of ways is
in PC2 ways) and two points from 240+1392+1980 = 3612
2nd straight line (i.e from Q points 4. (d) From the solution of previous
in QC2 ways ). Such a selection of question since the two kings are
4 points will give us two straight identical hence total number of ways
is 3612/2 = 1806
Geometrical Construction  97
5.
(a) We will solve this question with In this case 2nd queen can be placed
help of diagram and taking different in 40 ways so total number of ways
cases- in this case is 20 × 40 = 800
Case (i): When 1st queen is placed at
one of the center from four center
squares as shown in the given figure x x x x x x
by ‘X’ . x x
In this case 2nd queen can be placed x x
in 36 ways so total number of ways
in this case is 4 × 36 = 144 x x
x x
x x x x x x

Case (iii): When 1st queen is placed at


x x
one of the squares as shown in the
x x given figure by ‘X’ that can be done
in 28 ways.
In this case 2nd queen can be placed
in 42 ways so total number of ways
in this case is 28 × 42 = 1176
Case (ii): When 1st queen is placed at
one of the squares as shown in the x x x x x x x x
given figure by ‘X’ that can be done x x
in 12 ways.
x x
In this case 2nd queen can be placed
in 36 ways so total number of ways x x
in this case is 12 × 38 = 456 x x
x x
x x
x x x x x x x x x x x x
x x Total number of ways is 144 + 456 +
800 + 1176 = 2576
x x
6. (d) From the solution of previous
x x x x question if identical queens then
required number is 2576/2 = 1288
Solution of 7 and 8:
Case (iii): When 1st queen is placed at Let three sides of the triangle be a, b and c,
one of the squares as shown in the then from the property a+b> c and |a–b| < c
given figure by ‘X’ that can be done Let a = 1 then |b – c| ≥ 1 = a which contradicts
in 20 ways. the basic property | b – c| < a so there exist
no such triangles so P =0
98  Theory of Counting
Now let one of the side is 2 units, since none 9. (a) As per the given condition
of the sides is of 1 unit hence side with length n+1C – nC = 10 or nC = 10 or n = 5
3 3 2
2 units must be the smallest side, say a = 2,
10. (c) Required number of rectangle is
let c > b then c – b < 2 or c < b +2, if b = 3 2 2
then c =4, if b = 4 then c = 5 and so on c can (mC ) (mC ) (nC ) (nC ) = m n
1 1 1 1
take all the values from 4 to n hence number 11. (b) We have following cases in this
of triangles is n – 3. question-
Q=n–2 Case (i) if one point is selected from
S:- Number of equilateral triangle which is each side then number of triangles is
equal to n. 3 × 4 × 5 = 60
R is the number of isosceles triangles:- Case (ii) If one point is selected from
If unequal side is 2 then two equal sides one side and 2 points from another
can take any value from 3 to n so number of side then number of ways is (3)
isosceles triangles is (n–2) (4 + 5) + 6(3 + 5) + 10(3 + 4)
If unequal side is 3 then two equal sides can = 27 + 48 + 70 = 145
take any value from 2 to n (except 3 as it will So total number of triangles is
become equilateral triangle) so number of 60 + 145 = 205
isosceles triangles is (n–2) 12. (a) Since the given condition is that
If unequal side is 4 then two equal sides can corner of two selected squares is
take any value from 3 to n (except 4 as it will common hence they must be on two
become equilateral triangle) so number of adjacent columns.
isosceles triangles is (n–3) Now 1st let us assume that 2 squares
If unequal side is 5 then two equal sides can that are chosen from the first 2
take any value from 3 to n (except 5 as it will columns.
become equilateral triangle) so number of When the first square is chosen
isosceles triangles is (n–3) either from the top row or the bottom
If unequal side is 6 then two equal sides can row in the first square is chosen,
take any value from 4 to n (except 6 as it will only one square can be chosen from
become equilateral triangle) so number of the 2nd column. Thus from the first
isosceles triangles is (n–4) two columns, 2 × 1 + 6 × 2 or 14
If unequal side is 7 then two equal sides can pairs of squares can be chosen. In 8
take any value from 4 to n (except 7 as it will x8 chessboards, 7 pairs of adjacent
become equilateral triangle) so number of columns can be chosen.
isosceles triangles is (n–4) Therefore, the total number of ways of
And so on, since n is even hence required choosing the squares = 7 × 14 = 98.
number of isosceles triangle is 2{(n–2) + (n–3) 13. (c) Let us solve this question by dividing
+ … + n/2 terms} = n(n–4)/4 on the number of sides-
Or R = n(n–4)/4 Say if number of sides is 18 then we
will have 36/18 = 2 polygons
7. (b) | P – Q| = n–2
If number of sides is 12 then number
n (n − 4 ) n (n − 8) of polygons is 36/12 = 3 and so on-
8. (a) | R – S| = – n =
4 4
Geometrical Construction  99

No. of sides in Hence, total 12 + 9 + 6 + 4 + 3 + 2 +


No. of polygons 1 = 37
polygon
14. (b) Since 16 = 4 × 4 so we need to draw
36 1
(4 + 1) = 5 parallel straight lines and
18 2 another set of 5 parallel straight
12 3 lines so total number of straight
9 4 lines is 5 + 5 = 10
6 6
4 9
3 12

Concept Eliminator (CE)


1. (d) 2. (a) 3. (b) 4. (b) 5. (c) 6. (d)
7. (b) 8. (c) 9. (a)

1. (d) Number of straight lines that can Total number of triangles (assuming
be drawn is kC2 = k(k–1)/2 = P (say). that all the intersecting points
Now from P straight lines for = k(k–1)/2C2 = m are non collinear ) is
maximum number of point of given by mC3 = {k(k–1)/2C2}C3.
intersection condition is , no two of
Now we will reduce the number of
them are parallel to each other and
triangles that we are not getting
no three of them are concurrent. So
because of collinear points which is
number of point of intersection of
these P straight lines is {k(k–1)/2}{(k–2)(k+1)/2C3}

PC k(k–1)/2C So required number of triangles is


2= 2
{k(k–1)/2C }C – {k(k–1)/2}{(k–2)(k+1)/2C3}
2. (a) From the solution of previous question 2 3

number of points of intersection is 3. (b) Number of intersection points is kC2


PC k(k–1)/2C = k(k–1)/2 = P (say)
2= 2 = m(say)

Out of these points some points are Number of straight lines from P
collinear, if we look at carefully then points is PC2
on each of P = kC2 = k(k–1)/2 straight
lines number of collinear points are = P (P − 1) = 1  k (k − 1)  k (k − 1) – 1 
2 2  2  2 
k (k − 1) (k − 2 )(k + 1)
P–1 =
2
– 1 =
2 =
(
k (k − 1) k 2 − k − 2 )
8
100  Theory of Counting
Since in the question we are asked 5. (c) Lets consider 5 points A, B, C, D
only the new lines and we also have and E. Consider a point A, number
to note that one particular line is of straight lines that can be drawn
intersected by (k–1) straight lines at through B, C, D and E is (4C2) = 6
(k–1) points and these points will not straight lines, so from A we can
give us new line hence from one line draw perpendicular to these 6
we will not get (k–1)C2 lines from one straight lines, similarly we will
straight line so from k lines we will get 6 perpendiculars from other
k (k − 1)(k − 2 ) points as well so total number of
not get k[(k–1)C2] =
2 perpendiculars is 6×5 = 30.
lines
The maximum number of
Hence number of new straight lines
intersections of these 30 straight
is
lines is (30C2) = 435, but these 435
k( k − 1)( k 2 − k − 2) k( k − 1)( k − 2) points are not distinct means out of

8 2 30 straight lines not all of them are
=
k( k − 1)( k − 2)( k − 3) non concurrent. We have to consider
8 the following cases-

4. (b) If 1 circle is on the plane then number Case (i) Consider one of the ten lines
of regions is 2 drawn from original 5 points,
say this line AB, there are three
If 2 circles is on the plane then
perpendiculars are drawn on AB
number of regions is 4
from points C, D and E, these 3
If 3 circles is on the plane then perpendiculars are parallel to each
number of regions is 8 other hence they will not intersect
If 4 circles is on the plane then each other, these three point if not
number of regions is 14 parallel could have intersected at
3 points hence we lost 3 × 10 = 30
From pattern we can say that if
points.
number of circles is n then number
of regions is n(n–1) + 2 (We can prove Case (ii) Since three altitudes of a
this by mathematical induction) triangle intersect each other at a
point (Called Ortho center). From
Hence when number of circles is 100
the original 5 points we will have
then maximum regions is
(10C3) = 10 triangles. Consider one
100(100–1) +2 = 9900 + 2 = 9902 of these 10 triangles, perpendicular
Geometrical Construction  101
from vertex to opposite side (altitude) Case (i) 1 + 2 + 3 + 4 + 5 + 6 + 11 = 32
will intersect at one point instead of
Case (ii) 1 + 2 + 3 + 4 + 5 + 7 + 10 = 32
3 points, hence from each triangle
we lost 2 points so total number of Case (iii) 1 + 2 + 3 + 4 + 5 + 8 + 9 = 32
points that we have to subtract is Case (iv) 1 + 2 + 3 + 4 + 6 + 7 + 9 = 32
2 × 10 = 20 points.
Case (v) 1 + 2 + 3 + 5 + 6 + 7 + 8 = 32
Case (iii) Consider one of the original
Now case (i) is ruled out as we can
5 points (say A) since from A we have
not get a rectangle with 11 white
drawn 6 perpendiculars and these
and 11 black squares, remaining all
6 perpendiculars are concurrent
cases are possible.
hence they intersect each other at
one point instead of (6C2) = 15 points Diagram of case (ii) is shown here
so from one of this point we lost remaining you can try on your own.

15–1 = 14 points, hence total we lost


14 × 5 = 70 points.

So total number of points is 435 – 30


– 20 – 70 = 315 Points

6. (d) Since a chess board has 32 white and


32 black squares hence a1+a2+..+ap
= 32

Given condition is

a1< a2< a3< … < ap.

hence a1≥ 1, a2 ≥ 2, …ap ≥ p 8. (c) Lets start with the minimum number
Or from the given condition of non congruent rectangles and for
that purpose with minimum area.
32 ≥ 1+2+..+p = p(p+1)/2 or p ≤ 7
If area is 1 sq. unit then we will get
Hence maximum value of p is 7
only 1 rectangle of 1 × 1
7. (b) Now we have to see in how many
If area is 2 sq. unit then we will get
ways summation of 7 integers can
only 1 rectangle of 1 × 2
give us 32, for this we have following
cases- If area is 3 sq. unit then we will get
only 1 rectangle of 1 × 3
102  Theory of Counting
If area is 4 sq. unit then we will get 9. (a) Let us consider 1st the ‘ + ’ sign, we
only 2 rectangle of 1 × 4 or 2 × 2 can select two columns in 4C2 = 6
ways and we can place two ‘+’ signs
If area is 5 sq. unit then we will get
in (4 × 3) = 12 ways so two ‘+’ sign
only 1 rectangle of 1 × 5
can be arranged in 72 ways.
If area is 6 sq. unit then we will get
Similarly two ‘–‘ sign can be arranged
2 rectangles of 1 × 6 or 2 × 3
in 72 ways
If area is 7 sq. unit then we will get
Now we have to exclude the cases in
only 1 rectangle of 1 × 7 but it is not
which both the signs are in same cell.
possible in 6 × 6 square-
When both the ‘–‘ sign occupy the
Last rectangle will be of area 8
cell of ‘+’ sign then number of ways
square unit
is 72.
Here we have considered all the non
When one of the ‘–‘ sign occupy the
congruent rectangles and summation
cell of ‘+’ sign then number of ways
of these areas is 1 + 2 + 3 + 4 + 4 +
is 4 × 4 × 72 = 16 × 72
5 + 6 + 6 + 8 = 39 > 36 hence we
must have at least 2 congruent So required number of ways
rectangles.
= 72 × 72 – 72 – 16 × 72 = 55 × 72 = 3960.
4 Arrangement
Arrangement  103

Topics Covered

 Methods of Arrangement

 SATA method- (Select and Then Arrange Method)

Part A: Topic Number of Questions


Solved Example 10
Concept Applicator 15
Concept Builder 15
Concept Cracker 20
Part B: Topic Number of Questions
Concept Deviator 16
Concept Eliminator 4
Total 80
QR Code / Video Link

For Smart phone/ Tablet users

Video Link for Desktop/Laptops users http://dishapublication.com/video-resources


104  Theory of Counting
Part A
Here is the main purpose of writing this Example 5:  In how many ways 20 students
book is to eliminate the confusion between can be arranged in a straight line if
Combination and Permutation, in order to two particular students A and B are
remove that confusion, I have decided to always together?
eliminate Permutation. In this chapter I am
Solution:  A and B can be tied together in (2!)
going to give you only one concept/formula –
ways then we will have 19 distinct elements
Number of ways in which ‘n’ distinct objects that can be arranged in (19!) ways hence
can be arranged is ‘n!’. total number of ways is 2(19!).
Example 1:  In how many ways 10 distinct
objects can be arranged in a straight Example 6:  In how many ways 20 students
line? can be arranged in a straight line if
two particular students A and B are
Solution:  Since we have to arrange 10 distinct
never together?
objects in a straight line that can be done in
10! ways. Solution:  Without restriction they can be
arranged in (20!) ways, and in previous
Example 2:  In how many ways 10 identical question we have seen that A and B are
objects can be arranged in a straight always together in 2(19!) hence number of
line? ways in which they are never together is
Solution:  Since we have to arrange 10 20! – 2(19!) = (18)(19!) ways.
identical objects in a straight line that can
be done in only 1 way. Example 7:  In how many ways 20 students
To solve questions related to Permutation can be arrange in a straight line such
just follow one rule SATA (Select and then that three students A, B and C are
Arrange) — always together.
“1st select and then arrange” Solution:  A, B and C can be tied together
Example 3:  In how many ways 7 students in (3!) ways then we will have 18 distinct
can be arranged on 10 chairs in a elements that can be arranged in (18!) ways
straight line? hence total number of ways is (3!)(18!).
Solution:  As per the rule SATA, selection of
7 chairs out of 10 chairs can be done in 10C7 Example 8:  In how many ways 20 students
ways and then on 7 chairs we can arrange can be arrange in a straight line such
7 students in 7! ways. So total number of that three students A, B and C are
ways is (7!)(10C7) = 10P7 ways. always together and C is always ahead
of A and B.
Example 4:  In how many ways 7 prizes
(from 1st to 7th ) can be awarded to 10 Solution:  A, B and C can be tied together in
students? (3!) ways but out of these (3!) ways only in
2 ways C is ahead of A and B and then we
Solution:  As per the rule SATA, selection of
7 students out of 10 students can be done will have 18 distinct elements that can be
in 10C7 ways and then on 7 prizes we can arranged in (18!) ways hence total number
be given to 7 students in 7! ways so total of ways is . (3!)(18!) (2) = 2(18!)
number of ways is (7!)(10C7) = 10P7 ways. (3!)
Arrangement  105
Example 9:  What is the number of total number of ways is (10!)(20C10) = 20P10
permutations of 10 distinct articles ways.
out of 20 articles?
Example 10:  How many 4 digit numbers
Solution:  As per the rule SATA, selection of can be formed from the digits 2, 3, 5, 7
10 distinct articles out of 20 articles can if repetition of digits is not allowed.
be done in 20C10 ways and then these 10 Solution:  We have to arrange 4 digits at 4
articles can be arranged in 10! ways so places that can be done in 4! = 24 ways.

1
Concept Applicator (CA)
Ideal Time Apply your concepts with easy and
15 Min. conceptual questions

1. In how many ways 10 students can be (a) (18C11)(11!) (b) (19C10)(11!)


arranged in a row? (c) 19P (d) None of these
10
(a) 10C
1 (b) 10! 6. In how many ways batting order of 11
(c) 10. 10C players can be made out of 20 players if
1 (d) None of these
2. In how many ways 10 students can be Sachin and Sourav are always selected
seated on 15 seats in a row? but Yuvi and Dinesh always rejected?

(a) 10C (a) (18C11)(11!) (b) (19C10)(11!)


1 (b) 10!
(c) 10! 15C10 (d) None of these (c) (16C9)(11!) (d) None of these
3. In how many ways batting order of 11 7. In how many ways batting order of 11
players can be made out of 20 players? players can be made out of 15 players if 2
20C players can play only as wicket keeper?
(a) 11 (b) (20C11)(11!)
20P (a) (11!)( 2C1 × 13C10)
(c) 10 (d) None of these
4. In how many ways batting order of 11 (b) (11!)( 2C1 × 14C11)
players can be made out of 20 players if (c) (11!)(2C1 + 13C10)
Sachin is always selected? (d) None of these
(a) 20P (19C
11 (b) 10)(11!) 8. In how many ways 4 distinct volumes of
(c) 19P (d) None of these English and 6 distinct volume of Maths
10
5. In how many ways batting order of 11 book can be arranged on a bookshelf ?
players can be made out of 20 players if (a) (4!)(6!) (b) 2(4!)(6!)
Sachin and Sourav are always rejected? (c) (10!) (d) None of these
106  Theory of Counting
9. In how many ways 4 distinct volumes of (a) (4!)(6!) (b) 7(4!)(6!)
English and 6 distinct volume of Maths (c) (6!)(7C4 × 4!) (d) None of these
book can be arranged on a bookshelf
13. In how many ways 4 distinct volumes of
such that all the English books and all
English and 6 distinct volume of Maths
the Maths books are together?
book can be arranged on a bookshelf such
(a) (4!)(6!) (b) 2(4!)(6!)
that all the Maths books are together?
(c) (10!) (d) None of these (a) (5)(4!)(6!) (b) 7(4!)(6!)
10. In how many ways 4 distinct volumes of
(c) (6!)(7C 4× 4!) (d) None of these
English and 6 distinct volume of Maths
book can be arranged on a bookshelf such 14. In how many ways ‘n’ boys and ‘n’ girls,
that no two English books are together? can be arranged in a straight line so that
no two boys are together?
(a) (4!)(6!) (b) 2(4!)(6!)
(a) (n!)2 (b) ½ (n!)2
(c) (6!)(7C4 × 4!) (d) None of these
11. In how many ways 4 distinct volumes of (c) (2n!)2 (d) None of these
English and 6 distinct volume of Maths 15. If P is the number of ways in which 7
book can be arranged on a bookshelf such students can be arranged on 10 chairs
that no two Maths books are together? in a straight lines and Q is the number
(a) (4!)(6!) (b) 2(4!)(6!) of ways in which 7 students out of 10
students can be arranged on 7 chairs
(c) (6!)(7C 4× 4!) (d) None of these
in a straight line, then which one of the
12. In how many ways 4 distinct volumes of
following is correct?
English and 6 distinct volume of Maths
book can be arranged on a bookshelf such (a) P > Q (b) Q < P
that all the English books are together? (c) P = Q (d) None of these

Response Grid
1. a b c d 2. a b c d 3. a b c d 4. a b c d 5. a b c d
6. a b c d 7. a b c d 8. a b c d 9. a b c d 10. a b c d
11. a b c d 12. a b c d 13. a b c d 14. a b c d 15. a b c d
Arrangement  107

2
Concept Builder (CB)
Ideal Time Revise your concepts with questions
25 Min. medium difficulty level questions

1. In how many ways 6 prizes from 1st to 6. In how many ways 10 boys and 8 girls can
6th prize can be given to a group of 6 be arranged in a straight line such that
students that includes 3 boys and 3 girls two girls Seema and Reema are together?
here a student can get only 1 prize? (a) (16)(17!)(2!) (b) (16!)(17)(2!)
(a) (6!) (b) 2(3!)(3!) (c) (16!)(17!)(2!) (d) None of these
(c) (3!)(3!) (d) None of these 7. In how many ways 10 boys and 8 girls
2. In how many ways 6 prizes from 1st to can be arranged in a straight line such
6th prize can be given to a group of 6 that two girls Seema and Reema are not
students that includes 3 boys and 3 girls together?
if total number of participants is 6 boys (a) (16!)(17C2 )
and 7 girls and a student can get only 1 (b) 18! – (16!)(17!)(2!)
prize?
(c) 18! – (16!)(17)
(a) 6C3 × 7C3 (b) (6!)(6C3 × 7C3)
(d) None of these
(c) (6!)(6C3 +7C3) (d) None of these
8. In how many ways 10 students of different
3. In how many ways 5 Indians, 6 Americans height can be arranged so that the tallest
and 4 Russians be seated in a row so that and shortest are never together?
all persons of the same nationality sit
together? (a) 8(9!)(10!) (b) 8(9!)
(a) (3!5!6!4!) (b) (5!6!4!) (c) 8(9!)(2!) (d) None of these
(c) (15!) (d) None of these 9. In how many ways 5 boys and 5 girls can
be seated in a row so that boys and girls
4. There are 15 books out of which 5 Science, are alternate?
4 Literature, 3 Arts and remaining are
different language books, these books (a) 2(5!)(5!) (b) (5!)(5!)
are arranged on a bookshelf such that all (c) 2(5! + 5!) (d) None of these
the Science, all the Literature and all the 10. Mr. Ricky has 10 friends to invite for
Arts books are together , how many ways his marriage. In how many ways can he
it can be done? send invitation card to them if he has 4
(a) (5!4!3!3!) (b) (5!4!3!6!) servants to carry the invitation card?
(c) (5!4!3!3!3!) (d) None of these (a) 104 (b) 410
5. In how many ways 10 boys and 8 girls (c) 10P4 (d) None of these
can be arranged in a straight line such 11. In how many ways 5 prizes can be given
that all the girls and all the boys are to 10 boys if each boy can get any number
together? of prizes
(a) (10!)(8!) (b) (10!) (a) 105 (b) 510
(c) (2!)(10!)(8!) (d) None of these (c) 10P5 (d) None of these
108  Theory of Counting
12. A telegraph on Merchant Ship MT (a) 2(10!) (b) (10!)/2
Cougar Ace has 4 arms and each arm is (c) (10!)(8!) (d) None of these
capable of 5 distinct position including
14. In how many ways 5 identical Apples
neutral (Rest). What is the total number
and 4 identical bananas be arranged in a
of signals that can be given to engine
row such that no 2 apples are together?
room from the bridge if all the arms
are at neutral then no signal can be (a) 0 (b) (5!)(4!)
generated? (c) (9!) – (4!)(5!) (d) None of these
(a) 45 (b) 54 15. In how many ways 5 distinct Apples and
(c) 5P4 (d) None of these 4 distinct bananas be arranged in a row
such that no 2 apples are together?
13. The total number of ways in which 10
students can be arranged in a row such (a) 0 (b) (5!)(4!)
that A is always ahead of B. (c) (9!) – (4!)(5!) (d) None of these

Response Grid
1. a b c d 2. a b c d 3. a b c d 4. a b c d 5. a b c d
6. a b c d 7. a b c d 8. a b c d 9. a b c d 10. a b c d
11. a b c d 12. a b c d 13. a b c d 14. a b c d 15. a b c d

3
Concept Cracker (CC)
Ideal Time Boost up your confidence with good
40 Min. questions

1. What is the total number of signals that (a) (10!)/6 (b) (10!)/6!
can be made by using 5 flags of different (c) (10!)/3 (d) None of these
colour when any number of them may be
used? 4. In how many ways 10 students can be
(a) 325 (b) 425 arranged in a row such that A and B are
(c) 525 (d) None of these always between C and D?
2. In how many ways 10 students can be (a) (10!)/6 (b) (10!)/6!
arranged in a row such that A is ahead (c) (10!)/4 (d) None of these
of B and who in turn is ahead of C? 5. What is the total number of permutations
(a) (10!)/6 (b) (10!)/6! of 10 different things taken not more than
(c) (10!)/3 (d) None of these 4 at a time? It is given that each thing may
3. In how many ways 10 students can be be repeated any number of times.
arranged in a row such that A is always (a) 11110 (b) 11111
between B and C? (c) 11100 (d) None of these
Arrangement  109
6. In a photography session 10 students has 18!
to stand in a row but out of 10 there are (c) –1
(5!)(6!)(7!)
three students A, B and C who want to
stand together while C and D do not want (d) None of these
to stand next to each other, In how many 10. In a Annual general meeting of XYZ
ways they can be arranged in a row? company, participants are ‘2a’ number
(a) (3!)(7!)(6!) (b) (3!)(7!)(6) of executives, ‘b’ number of managers,
‘c’ number of senior managers and
(c) (3!)(7!)(6!)(2!) (d) None of these
director of the company. The seating
7. There are 100 students with roll number arrangement is made in such a way that
1 to 100. If 10 students are selected out of all the executives are seated at the either
100 students and their names are listed end (‘a’ number of executives on one
down according to their roll numbers in end), All the managers will be together
increasing order then in how many times while Director don’t want to sit next to
rank 50 will be at 6th position? an executive or a manager, in how many
(a) (49P5)(50P4) (b) (49C5)( 50C4) ways they can be arranged in a straight
line?
(c) (49!)(50!) (d) None of these
(a) {(a)!}{(a)!}(b!){(c+1)!}{cP2}
8. In a U.N security council meeting 30
(b) {(2a)!}(b!){(c+1)!}{cP2}
members representing different nations,
out of these 30 members 10 members from (c) {(2a)!}(b!){(c)!}{cP2}
permanent members of the council, these (d) None of these
are USA, Russia, China, UK and France,
11. In an international convention participants
2 representative from each nations. For
from 10 different countries were arranged in
a voting purpose 10 members out of 30
a row such that all the participants from the
has to be selected such that exactly 5
same country were together. Each country
members one from each of permanent
has different number of participants with
member nations are selected. In how
maximum 10 participants from a country.
many ways this can be done?
If K is the number of ways that they can be
(a) 32(20P5) (b) 32(20C5) arranged in a row then find the highest
(c) (20P5) (d) None of these power of 10 in K.
(a) 5 (b) 7
9. In how many ways 5 identical apples, 6
identical bananas and 7 identical oranges (c) 8 (d) None of these
can be arranged in a straight line such 12. In how many ways 10 students can be
that at least one fruit is separated from arranged in a row such that A is always
similar fruits. before B and C is always before D?
(a) 18! (a) (10!) – (2!)(2!)(8!)
(5!)(6!)(7!) (b) (10!) – (2!)(2!)(7!)
18! (c) (10!) – (2!)(2!)(9!)
(b) – (5!)(6!)(7!)
(5!)(6!)(7!)
(d) None of these
110  Theory of Counting
13. There are 2 Indians, and 2 Americans, (2M + 2N) balls in a straight line such
In how many ways they can be arranged that arrangement of 1st (M + N) balls is
such that no two participants from the mirror image of last (M + N) balls, in how
same country are together? many ways he can do so?
(a) 8 (b) 12 (a) 2M+2NC2M (b) ½ (2M+2NC2M)
(c) M+N CM (d) None of these
(c) 16 (d) None of these
18. In how many ways ‘n’ boys and ‘n’ girls
14. There are 2 Indians, 2 Americans and 2 can be arranged in line such that all
Russians in a conference, In how many the boys and all the girls standing in
ways they can be arranged such that no increasing order of their weight assume
two participants from the same country that all boys and all girls are of different
are together? weight?
(a) (2)(6!) (a) ½ 2nCn (b) 2nCn
(b) (2)(5!) (c) 2n Cn(n!) (d) None of these
(c) (8!) – (2!)(2!)(2!)(3!) 19. Seven friends Ahaskar, Bhanu, Chandan,
(d) None of these Dripto, Esha, Fatima, and Garima are
15. A delegation has 5 Indians and 4 Chinese, standing in a line, in how many ways
In how many ways they can stand in a they can stand in a line such that Amar,
line such that 2 Chinese are together and Bimal, and Chandan are standing
other 2 Chinese are also together but no together but never standing according to
three Chinese are together? their increasing order of height (assume
(a) (5!)(2!)(2!)(6P2) all of them are of different height) and
(b) (5!)(2!)(2!)(6P2)(4C2) Esha, Fatima and Garima are standing
(c) (5!)(2!)(2!)(6P2)(4C2)/2 together but never standing according to
(d) None of these their increasing order of weight (assume
16. Total 7 boys and 6 girls are selected all of them are of different weight)?
to represent their school in science (a) 4812 (b) 4806
conference. In how many ways they can (c) 4818 (d) None of these
be arranged in a line such that 3 girls 20. Total 16 people participated in a picnic,
are together and other three are also 6 distinct work has to be assigned to 6
together but these two group of girls are people such that only one from Mr. & Mrs.
A is selected similarly one from Mr. &
not together?
Mrs. B and Mr. & Mrs. C is selected for the
(a) (5040)(8!) (b) (2520)(8!)
work. How many ways this can be done?
(c) (1260)(8!) (d) None of these
17. Susmit has 2M red and 2N black balls (a) 8(5!)(6!) (b) 4(5!)(6!)
(identical) he wants to arrange all these (c) 2(5!)(6!) (d) None of these

Response Grid
1. a b c d 2. a b c d 3. a b c d 4. a b c d 5. a b c d
6. a b c d 7. a b c d 8. a b c d 9. a b c d 10. a b c d
11. a b c d 12. a b c d 13. a b c d 14. a b c d 15. a b c d
16. a b c d 17. a b c d 18. a b c d 19. a b c d 20. a b c d
Arrangement  111

Solutions
Concept Applicator (CA)
1. (b) 2. (c) 3. (b) 4. (b) 5. (a) 6. (c)
7. (a) 8. (c) 9. (b) 10. (c) 11. (d) 12. (b)
13. (a) 14. (d) 15. (c)

1. (b) Since n distinct articles can be Step 2: Batting order of these 11


arranged in n! ways hence 10 students players can be made in 11! ways.
can be arranged in 10! ways. Hence total number of ways is
2. (c) Out of 15 seats we will select 10 in (19C10) (11!).
15C ways now 10 students can be
10 5. (a) We will solve this question by our
arranged on 10 seats in 10! ways. two step process
Hence total number of ways is
Step 1: Select
10!. 15C10.
Step 2: Arrange
3. (b) We will solve this question by our
Step 1: Out of 20 players 1st we
two step process
have to select 11 that can be done in
Step 1: Select 18C ways since Sachin and Sourav
11
Step 2: Arrange are always rejected.
Step 1: Out of 20 players 1st we have Step 2: Batting order of these 11
to select 11 that can be done in 20C11 players can be made in 11! ways.
ways Hence total number of ways is
(18C11) (11!).
Step 2: Batting order of these 11
players can be made in 11! ways. 6. (c) We will solve this question by our
two step process
Hence total number of ways is (20C11) Step 1: Select
(11!) which is also equal to 20P11.
Step 2: Arrange
4. (b) We will solve this question by our Step 1: Out of 20 players 1st we have
two step process to select 11 that can be done in 16C9
Step 1: Select ways since Sachin and Sourav are
always selected but Yuvi and Dinesh
Step 2: Arrange always rejected.
Step 1: Out of 20 players 1st we Step 2: Batting order of these 11
have to select 11 that can be done players can be made in 11! ways.
in 19C10 ways since Sachin is always Hence total number of ways is
selected. (16C9) (11!).
112  Theory of Counting
7. (a) We will solve this question by our keep the English books (Now we are
two step process using 2 step process). 4 places can be
Step 1: Select selected out of 7 in 7C4 ways. Now
at 4 English books can be arranged
Step 2: Arrange
at 4 places in 4! ways. Hence total
Step 1: From 2 wicket keepers only
number of ways is (6!)(7C4 × 4!)
1 is selected and this can be done in
which is also equal to (6!)(7P4 ).
2C ways and remaining 10 can be
1
selected from remaining 13 players 11. (d) Since number of Maths books are 6
and that can be done in 13C10 so total while that of English books are only
number of ways is 2C1 × 13C10. 4. Hence, we have no arrangement
where two maths books are not
Step 2: Batting order of these 11
together.
players can be made in 11! ways.
Hence total number of ways is 12. (b) 1st tie all the English books together,
(2C1 × 13C10)(11!). that can be done in 4! ways, then
arrange 6 Maths book and that can be
8. (c) Here we just have to arrange these
done in 6! ways, now this will create
10 distinct articles and that can be
7 places and out these 7 places select
done in 10! ways.
1 place, that can be done in 7 ways
9. (b) 1st ask a question to yourself, do we
and now put the bundle of English
need to select something? You will
books at that place.
get an answer no, we just have to
arrange in a particular fashion so here Total number of ways 7(4!)(6!).
we will not use our two step process. 13. (a) Similar to above question number of
Since all the English books are ways is (5)(4!)(6!).
together, they can be arranged in 4!
14. (d) Lets 1st person is a boy then boys
ways, similarly all the Maths book
can be arranged in (n!) ways and
can be arranged in 6! Ways and these
girls can be arranged in (n!) ways
two bundle of books can be arranged
so total number of ways is (n!)2,
in 2! ways. Hence total number of
similarly (n!)2 ways when girl is the
ways is 2!4!6!
1st person, hence the total number of
10. (c) 1st we will arrange 6 volumes of ways is 2(n!)2.
Math books in 6! Ways, this will
15. (c) Consider P it is the number of ways
create 7 spaces such that if we place
in which 7 students can be arranged
1 English book in each of the space
on 10 chairs in a straight lines hence
no English books can be together.
P = (10C7)(7!) = 10P7.
So we have now 7 places for 4
English books, so out of 7 places we Similarly,
will 1st select 4 places where we will Q = (10C7)(7!) = 10P7 so P = Q.
Arrangement  113

Concept Builder (CB)


1. (a) 2. (b) 3. (a) 4. (b) 5. (c) 6. (b)
7. (d) 8. (b) 9. (a) 10. (b) 11. (a) 12. (d)
13. (b) 14. (d) 15. (b)

1. (a) Here we have to arrange 6 distinct 6. (b) Total number students is 10 + 8 = 18


articles at six places and that can be out of these 18 students there is no
done in 6! ways. restriction with 18 – 2 = 16 students.
2. (b) Here 1st we need to select and then Hence 16 students can be arranged
arrange (two step process) selection in 16! ways, and it will create 17
of 3 boys and 3 girls can be done in spaces so Seema and Reema can be
6C × 7C ways, now after selecting placed in any of the 17 places in 17
3 3
these 6 we have to arrange these 6 ways and these two can be arranged
students and that can be done in 6! in 2! ways. Hence total number of
ways. Hence total number of ways is ways is (16!)(17)(2!)
(6!)(6C3 × 7C3). 7. (d) Similar to above question 16
3. (a) Since 5 Indians will be together so students can be arranged in 16!
tie them together, that can be done ways, and it will create 17 spaces.
in 5! ways, similarly 6 Americans So Seema and Reema can be
and 4 Russians can be tied together placed in any of the 17 places in
in 6! and 4! ways respectively. Now 17C × 2! ways, total number of ways
2
these three bunch of persons can be
is (16!)(17C2 × 2!).
arranged in 3! ways, so total number
of ways is (3!5!6!4!). Alternately: Without restriction 18
students can be arranged in 18! ways,
4. (b) Since 5 science books can be arranged
and from the answer of previous
in 5! ways, 4 Literature books can
question out of these 18! ways there
be arranged in 4! ways, and 3 Arts
books can be arranged in 3! are (16!)(17)(2!)ways where Reema
and Seema are always together, so
Number of different language books
the number of ways that they are
= 15 – 5 – 4 – 3 = 3 not together is 18! – (16!)(17)(2!).
Now total 3 + 3 = 6 units can be 8. (b) Without restriction we can arrange
arranged in 6! ways, 10 students in 10! ways, and the
Total number of ways is (5!4!3!6!) number of ways in which tallest and
5. (c) Similar question we have solved in shortest students are together is
Concept applicator (CA) , all the boys 2(9!), hence number of ways in which
can be arranged in 10! ways, while the tallest and shortest are never
8 girls can be arranged in 8! ways. together is (10!) – 2(9!) = 8(9!).
Now these two bunches of boys and 9. (a) In this case we have total 10 seats
girls can be arranged in 2! ways.
5 for boys and 5 for girls, so we will
Total number of ways is (2!)(10!)(8!). have two cases,
114  Theory of Counting
Case (i) when 1st place is occupied 11. (a) Here it is given that each boy can get
by a boy then situation is as shown any number of prizes hence 1st prize
below can be dealt in 10 ways, 2nd prize in 10
ways and so on, hence total number of
B G B G B G B G B G
ways is 10 × 10 ... × 10 = 105.
Number of arrangements in this 12. (d) Required number of ways is
case is (5!)(5!) 54 – 1 = 624.
Case (ii) when 1st place is occupied 13. (b) When there is no restriction 10
by a girl then situation is as shown students can be arranged in 10! ways
below out of these in 50% cases A is ahead
of B and in remaining 50% ways B
G B G B G B G B G B is ahead of A, hence number of ways
Number of arrangements in this when A is ahead of B is (10!)/2.
case is (5!)(5!). 14. (d) Here as per the given condition we
Total number of ways is 2(5!)(5!). have only one way to arrange them
10. (b) Logic says that one servant can in this particular fashion and this is
carry more than one card but one ABABABABA. Here A denotes apple
card can not be carried by more than and B denotes Banana.
1 servant. So 1st card can be dealt 15. (b) From the solution of previous
in 4 ways, 2nd card again in 4 ways question arrangement is like
and so on, so total number of ways is ABABABABA now since apples and
4 × 4 × 4 × ... × 4= 410. bananas are distinct so they can be
arranged in (4!)(5!).

Concept Cracker (CC)


1. (a) 2. (a) 3. (d) 4. (a) 5. (a) 6. (b)
7. (a) 8. (b) 9. (d) 10. (b) 11. (d) 12. (d)
13. (a) 14. (b) 15. (c) 16. (b) 17. (c) 18. (b)
19. (b) 20. (a)

1. (a) Here we have different cases: ways. Hence number of ways in this
Case 1: When one flag is used, case is (3!)(5C3) = (5P3) = 60.
number of ways 1 flag is selected Case 4: When 4 flags are used,
is 5C1 and it can be arranged in 1! number of ways 4 flags is selected
ways. Hence number of ways in this is 5C4 and it can be arranged in 4!
case is (1!)(5C1) = (5P1) = 5. ways. Hence number of ways in this
Case 2: When two flags are used, case is (4!)(5C4) = (5P4) = 120.
number of ways 2 flags is selected Case 5: When 5 flags are used,
is 5C2 and it can be arranged in 2! number of ways 5 flags is selected
ways. Hence number of ways in this is 5C5 and it can be arranged in 5!
case is (2!)(5C2) = (5P2) = 20. ways. Hence number of ways in this
Case 3: When three flags are used, case is (5!)(5C5) = (5P5) = 120.
number of ways 3 flags is selected Total number of ways is 120 + 120 +
is 5C3 and it can be arranged in 3! 60 + 20 + 5 = 325.
Arrangement  115
2. (a) When there is no restriction 10 Case (iii): When 4 items are taken
students can be arranged in 10! at a time then number of ways is 10
ways. Out of these consider the × 10 × 10 × 10 = 10000.
arrangements of A, B and C, the So total number of ways is 10000 +
possible arrangements are (A, B, C), 1000 + 100 + 10 = 11110.
(A, C, B), (B, C, A), (B, A, C), (C, A,
6. (b) Since three students A, B and C want
B), (C, B, A) in total 3! or 6 ways. Out
to stand together hence number of
of these 6 only one is (A, B, C) which
such arrangements is (3!)(8!). now
satisfy the given condition, hence we
C and D do not want to stand next
can say that every 6 only one case
to each other hence total number of
satisfy the given condition. Hence
ways is (3!)(8!) – (3!)(2!)(7!) = (3!)(7!)(6).
total number of ways is (10!)/6.
7. (a) If rank 50 is at 6th position that
3. (d) When there is no restriction 10 students
means 5 students are selected from
can be arranged in 10! ways. Out of
rank 1 to 49 and then they are
these consider the arrangements of A,
arranged, that can be done in (49C5)
B and C, the possible arrangements are
(5!) = 49P5 ways, similarly 4 has to
(A, B, C), (A, C, B), (B, C, A), (B, A, C),
selected and arranged from rank 51
(C, A, B), (C, B, A) in total 3! or 6
to 100, that can be done in (50C4)
ways. Out of these 6 only two are
(4!) = 50P4 ways. Hence total number
(B, A, C) and (C, A, B) which satisfy
of ways is (49P5)(50P4).
the given condition, hence we can say
that every 6 only two case satisfy the 8. (b) From permanent nations 5 can be
given condition. Hence total number selected in 2 × 2 × 2 × 2 × 2 = 32
of ways is 2(10!)/6 = (10!)/3. ways and then remaining 5 out of 20
can be selected in 20C5 ways. Hence
4. (a) When there is no restriction 10
total number of ways is 32(20C5).
students can be arranged in 10!
Ways out of these consider the In this question we don’t need
arrangements of A, B C, and D can to arrange, it is just a selection
arrange themselves in 4! = 24 ways, question or you can say it is wrong
out of these 24 the cases which placing of question it should be in
satisfy the condition is (C, A, B, D), chapter 2. I placed this question just
(C, B, A, D), (D, A, B, C), (D, B, A, C) to see that are u able to understand
so out of every 24 only 4 cases satisfy the difference between selection and
the given condition hence total arrangement.
number of ways is 9. (d) Without restriction 5 identical
(4/24)(10!) = 10!/6. apples, 6 identical bananas and 7
identical oranges can be arranged
5. (a) Here we have 4 different cases,
18!
Case (i): When 1 item is taken at a in a straight line in if
(5!)(6!)(7 !)
time then number of ways is 10.
Case (ii): When 2 items are taken all the identical fruits are arranged
at a time then number of ways is together then number of ways is 3!,
10 × 10 = 100. hence required number of ways is
18! 18!
Case (iii): When 3 items are taken − 3! or −6 .
at a time then number of ways is (5!)(6!)(7 !) (5!)(6!)(7 !)
10 × 10 × 10 = 1000.
116  Theory of Counting
10. (b) Since ‘a’ seats from both the end is Number of ways they can be
reserved for executives, they can be arranged such that 2 Indians as well
arranged in (2a!) ways. In between as 2 American are together is (2!)(2!)
we will have (b + c + 1) seats. (2!) = 8.
All the managers are together can Hence required number of ways is
be arranged in (b!) ways, now out 24 – { 2 × 12 – 8} = 8.
of (c+1) seats senior managers and 14. (b) Without restriction 6 persons can be
director can be seated in such a way arranged in 6! ways.
that director has senior manager in
Number of ways they can be arranged
both the sides. i.e (SM, D, SM) this
such that 2 Indians are together is
can be done in (cC2)(2!) = cP2 ways.
(2!)(5!) = (10)(4!) = (40)(3!).
Now bunch of managers, group of
director (which has three members) Number of ways they can be
and remaining (c – 2) senior managers arranged such that 2 American
can be arranged in (c + 1)!. are together is (2!)(5!) = (10)
(4!) = (40)(3!).
Hence total number of ways is {(2a)!}
(b!){(c+1)!}{cP2}. Number of ways they can be
arranged such that 2 Russians
11. (d) In this case number of participants
are together is (2!)(5!) = (10)
from 10 different countries are 1, 2,
(4!) = (40)(3!).
3… 10.
Number of ways they can be
Number of ways that they can be
arranged such that 2 Indians as well
arranged in a row such that all the
as 2 American are together is (2!)(2!)
participants from the same country
were together is (10!)(1!)(2!)(3!) (4!) = (4)(4!) = (16)(3!).
(4!)…. (10!). Number of ways they can be
Hence K = (10!)(1!)(2!)(3!)(4!)…. (10!) arranged such that 2 Indians as well
highest power of 10 or that of 5 can as 2 Russians are together is (2!)(2!)
be given by only (5!)(6!)(7!)(8!)(9!) (4!) = (4)(4!)= (16)(3!).
(10!)(10!) which is 1 + 1 + 1 + 1 + 1 + Number of ways they can be arranged
2 + 2 = 9. such that 2 American as well as
12. (d) Without restriction 10 students can 2 Russians are together is (2!)(2!)
be arranged in 10! Ways , now out (4!) = (4)(4!)= (16)(3!).
of these in ½ of them B is before C
and out of them ½ ways C is before Number of ways they can be arranged
D, Hence required number of ways such that 2 Indians ,2 American as
is (10!)/4. well as 2 Russians are together is
13. (a) Without restriction 4 persons can be (2!)(2!)(2!)(3!) = (8)(3!).
arranged in 4! = 24 ways. Hence required number of ways is
Number of ways they can be arranged [6!] – (80)(3!) = 40(3!) =2(5!).
such that 2 Indians are together is 15. (c) Let two Chinese are tied together
(2!)(3!) = 12. in 2! ways. Other 2 Chinese are tied
Number of ways they can be arranged together in 2! ways.
such that 2 American are together is Now this group of two Chinese can
(2!)(3!) = 12. be made in (4C2)/2 = 3 ways.
Arrangement  117
To understand this let us assume remaining N places N black balls
that the Chinese are A, B, C and D can be arranged in 1 way hence total
and these can be divided in a group number of ways is M+NCM.
of two as shown 18. (b) Let us assume that there are 2n
(A, B) and (C, D) chairs where we have to place these
(A, C) and (B, D) n boys and n girls. Out of 2n places
(A, D) and (B, C) we will select n places and arrange all
the boys in increasing order of weight
Total 3 or (4C2)/2 = 3 ways
(only in 1 way) then we can arrange
Next 5 Indians can be arranged in 5! girls in the increasing order in 1 way,
ways so total number of ways is 2nCn.
Now we have 6 spaces between them 19. (b) Since 7 friends can be arranged in
of which two places can be selected 7! ways = 5040 ways without any
and both bunches of Chinese can be restriction.
arranged in (6C2) × 2! = 6P2.
Now consider the number of ways
Hence total number of ways is (5!) when Amar, Bimal, and Chandan
(2!)(2!)(6P2)(4C2)/2 are standing together and according
16. (b) Since 7 boys can be arranged in (7!) to their increasing order of height
ways. Now 1st we will divide the 6 this can be done in (5!) = 120 ways.
girl students into two groups of 3 Similarly number of ways in which
students and that can be done in Esha, Fatima and Garima are
(6C3)/2 ways = 10 ways. These 3 girls standing together and according to
can be arranged in (3!) ways among their increasing order of weight is
themselves. Since 7 boys will create 5! = 120 ways.
8 separated space hence 1st group of
Then number of ways in which
girls can be placed in 8 ways while
Amar, Bimal, and Chandan are
2nd group can be placed in 7 ways
standing together and according to
hence total number of ways is (7!)
their increasing order of height and
(8 × 7)(6C3)(3!)(3!)/2 = (8!)(7 × 10 ×
also Esha, Fatima and Garima are
6 × 6) = 2520(8!).
standing together and according to
17. (c) As per the given condition there their increasing order of weight is
must be M red and N black balls in 3! = 6 ways.
1st (M + N) balls and similarly M
Hence required number of ways is
red and N black balls in last (M + N)
5040 – 120 – 120 + 6 = 4806.
balls, in order to arrange in the given
condition we just have to arrange 1st 20. (a) Since out of 16 only 10 is available for
(M + N) balls, next (M + N) will have selection, out of 10, 3 can be selected
only 1 way and that to image of 1st in 10C3 = 120 ways, and three from
(M + N) balls. the group of Mr. & Mrs. A, B and C
can be selected in 2 × 2 × 2 = 8 ways.
For 1st (M + N) balls we just have to
select M places out of (M + N) that So total number of ways for selection
can be done in M+NCM ways then we is 8 × 120 = 960 = 8(5!).
can arrange M red balls in these M Then 6 works can be assigned to 6
places in 1 way as all of them are persons in (6!) ways so total number
identical and remaining then in of ways is 8(5!)(6!).
118  Theory of Counting
Part B
Here is the main purpose of writing this Number of ways in which ‘n’ distinct
book is to eliminate the confusion between objects can be arranged is ‘n!’.
Combination and Permutation, in order To solve questions related to Permutation
to remove that confusion I have decided to just follow one rule SATA (Select and then
eliminate Permutation. In this chapter I am Arrange)
going to give you only one concept/formula –
“1st select and then arrange”

1
Concept Deviator (CD)
Ideal Time Revise your concept (IIT advance or
5 Min. tougher questions)
Per Question

1. What is the total number of ways of 5. In National Maths Olympiad 10


arranging N persons of different age in a participants 2 From Bihar, 2 From
straight line such that out of N persons Maharashtra, and 2 from WB and
there are K persons always standing remaining 4 from 4 different states are
according to their age (Increasing or seated in a straight line for exam such
decreasing). that no two participants from same state
(a) 2(NPN – K) (b) 2(NCN – K) are together. Find the number of ways in
(c) (NPN – K) (d) None of these which this can be done?
2. Amit a librarian has to arrange a book (a) (47)(8!) (b) (51)(8!)
shelf with 10 books for display. He has (c) (89)(8!) (d) None of these
books of 7 different subjects (Each subject 6. How many arrangements of A,B,C and D
has more than 10 books), in how many can be made such that sequence AB, BC
ways he can arrange book shelf such that
or CD never occur?
books from each subject is selected?
(a) 14 (b) 11
(a) (72)(10C7) (b) (73)(10C7)
(c) 16 (d) None of these
(c) (7)(10C7) (d) None of these
3. In how many ways batting order of 22 7. In MERI Kolkata 100 students are
players be decide if they are playing standing in a line for an inspection.
against each other. During inspection it was found that 5th
student is also 5th in height, 25th student
(a) (22P11)(11!)/2 (b) (22P11)(11!)
22
is also 25th in height, similarly 50, 75th
(c) (2!)( P11)(11!) (d) None of these and 100th students at 50th, 75th, and
4. In how many ways 10 students (A,B, C, 100th position respectively according to
D, …J)can be arranged in a row such their height. In how many ways students
that A is always before B and C is always can arrange themselves if above condition
before D and so on? is satisfied? (Assume that all of them are
10! 10! of different height)
(a) (b)
32 2 (a) (4!)(19!)(24!)4 (b) (4!)(19!)4
5
(c) (10!) (d) None of these (c) (4!)(24!)(19!) (d) None of these
Arrangement  119
Direction (Qs. 8 and 9): such that no two ‘–’ signs occur together
Consider 26 letters of the English alphabets is… [IIT JEE 1988]
they can be arranged in (26!) ways, out of (a) 21 (b) 31
these (26!) ways letter A and B are together
(c) 35 (d) None of these
in (2)(25!) ways.
13. Eight chairs are numbered 1 to 8. Two
8. In how many of the arrangements exactly
5 alphabets are between A and B? women and three men wish to occupy
one chair each. First the women choose
(a) 36(24!) (b) 38(24!)
the chairs from amongst the chairs
(c) 40(24!) (d) None of these
marked 1 to 4; and then the men select
9. In how many of the arrangements at the chairs from amongst the remaining.
most 20 alphabets are between A and The number of possible arrangements is
B?
[IIT JEE 1982]
(a) 360(24!) (b) 630(24!)
(a) 6C × 4C (b) 4P2 × 4P3
(c) 480(24!) (d) None of these 3 2
(c) 4C × 4P3 (d) None of these
10. If n students are standing in a straight 2
line such that 1st half (i.e for 1st n/2 if 14. m men and n women are to be seated
n is even or (n+1)/2 otherwise) students in a row so that no two women sit
are arranged according to the increasing together. If m > n then the number
order of their height and 2nd half students of ways in which they can be seated is
are arranged according to decreasing [IIT JEE 1983]
order of height. If total number of ways
that this arrangement can be done is m !(m + 1)! m!
a three digit number then how many (a) (b) (m − n + 1)!
(m − n + 1)!
values of n exist?
(a) 0 (b) 3 (m + 1)!
(c) (d) None of these
(c) 4 (d) None of these (m − n + 1)!
11. Let A be a set of n distinct elements. Then 15. In how many ways three girls and nine
the total number of distinct functions boys can be seated in two vans each
from A to A is…(X)..and out of these … having numbered seats, 3 in front and 4
(Y)… are onto functions, then the value at the back if 3 girls sit together at the
of (X, Y) is   [IIT JEE 1985] back row of a van? [IIT JEE 1996]
n
(a) 2(12!) (b) 4(12!)
(a) nn, ∑ ( −1)n−r  C(n, r )(r n )
r =1 (c) 12! (d) None of these
n 16. In a class of 10 students, there are 3 girls.
(b) nn–1, ∑ ( −1)n−r  C(n, r )(r n ) In how many different ways can they be
r =1
arranged in a row such that no two of the
n
three girls are consecutive?
(c) nn, ∑  C(n, r )(r n )
r =1 [IIT JEE 1961]
(d) None of these (a) 49(8!) (b) 42(7!)
12. Total number of ways in which six ‘+’ and (c) 42(8!) (d) None of these
four ‘–’ signs can be arranged in a line
120  Theory of Counting

Response Grid
1. a b c d 2. a b c d 3. a b c d 4. a b c d 5. a b c d
6. a b c d 7. a b c d 8. a b c d 9. a b c d 10. a b c d
11. a b c d 12. a b c d 13. a b c d 14. a b c d 15. a b c d
16. a b c d

2
Concept Eliminator (CE)
Ideal Time Revise your concept (Maths Olympiad
Just solve it or tougher questions)

1. Based on Chinese Maths Olympiad:- 2. What is the number of ways of arranging


In how many ways 4 symbols 2 ‘+’ and the n roses in these m vases if m is an
2 ‘–’ can be placed on a chess board such even number?
that each smaller square contains at mP
(a) 2n–1 (b) m/2P
n. 2
n
most 1 symbol and each row or column n

can not contain the same symbol? (c) m/2P n–1 m/2P n–1
n–1. 3 (d) n. 2
(a) 3136 (b) 9630656 3. What is the number of ways of arranging
(c) 96336792 (d) None of these the n roses in these m vases if m is odd
DIRECTIONS (Qs. 2 to 4): Answer the and the middle vass is empty?
questions on the basis of the information (m+1)/2P n–1 (m–1)/2P n
given below. (a) n. 2 (b) n. 2

(c) (m–1)/2P n–1


There are m numbers of flower Vases are n. 2 (d) None of these
placed in a row of a flower exhibition and
4. What is the number of ways of arranging
there are n (m>2n +1) distinct roses are to be
the n roses in these m vases if m is odd
decorated with these vases.
and middle vass is occupied?
These flower vases are arranged such that
(a) (m–1)/2P n–1 (m+1)/2 Pn–1.2n
out of any two vessels located symmetrically n–1. 2 (b)
about the middle of the row, at least one vass (m–1)/2P n–1
(c) n–1.n.2 (d) None of these
is empty.

Response Grid
1. a b c d 2. a b c d 3. a b c d 4. a b c d
Arrangement  121

Solutions
Concept Deviator (CD)

1. (a) 2. (a) 3. (a) 4. (a) 5. (c) 6. (b)


7. (d) 8. (c) 9. (b) 10. (c) 11. (a) 12. (c)
13. (d) 14. (a) 15. (c) 16. (c)

1. (a) Without restriction N persons can Case (3) 1 book from each of 4 distinct
be arranged in N! ways, while K subjects and 2 books from remaining
persons can be arranged in K! ways each subject , the selection can be
but instead they are arranging made in (7C3 = 35 ways) while the
themselves in only 2 ways, increasing subjects can be allotted to them in
or decreasing hence number of ways only 1 way as each from 2 subjects,
2(N!) and they can be arranged in
is = 2(NPN–K) ways
K! 10!
, hence total number of
2. (a) Here we have to arrange 10 books (2!)(2!)(2!)
from 7 subjects such that book from (35)(10!) = (10!)(105)
each subject is selected, the required ways is
cases are as follows- (2!)(2!)(2!) (3!)(4)
Case (1) 1 book from each of 6 distinct So total number of ways is
subjects and 4 books from remaining (10!)(105)

(7 )(10!) +  (10!)(21) +
subject, the selection can be made (3!)(4)
in (7C1 = 7 ways) and they can be 4! 3!
105  (10!)(7 )
2
arranged in (10!)/(4!), hence total 10!  7
= +  
2 1 
+ =
(7)(10!) 3!  4 4  3!
number of ways is
4! = (72)(10C7)
Case (2) 1 book from each of 5 distinct
subjects and 3 books from remaining 3. (a) 22 players can be divided into 2
one subject and 2 books from teams of 11 players in (22C11)/2 ways
remaining subject , the selection can and then batting order of each team
be made in (7C2 = 42 ways) while the can be done in (11!) ways. Hence
subjects can be allotted to them in 2! total number of ways is (22C11)(11!)
Ways, and they can be arranged in (11!)/2 = (22P11)(11!)/2.
(10!)/(3!)(2!) , hence total number of 4. (a) Since 10 students can be arranged
(42 )(10!)(2!) = (10!)(21) in (10!) ways out of this (10!) ways
ways is in 50% case A is before B, hence
(3!)(2!)(2!) 3!
122  Theory of Counting
number of ways that A is before B 7. (d) Since 1st 4 students at 1st 4 places
10! (According to height) they can be
is , similarly the number of ways
2! arranged in 4! Ways, similarly
that A is always before B and C is from position number 6th to 24th
10! are taken by all the students from
always before D is ways,
(2!)(2!) 6th highest height to 24th highest
height, they can be arrange in (19!)
similarly for all the pairs and the
ways similarly next each groups can
10! 10! be arranged in (24!) ways, so total
number of ways is = .
5
(2!) 32 number of ways is 4(19!)(24!)3.
5. (c) Without restriction 10 participants 8. (c) Since there are 5 alphabets between
can be arranged in 10! ways. A and B.
Number of ways that 2 participants One number from A or B can be
from each of the three states are selected in 2 ways.
together is (2!)(2!)(2!)(7!) = 8! 1st letter can be placed at 1st, 2nd , or
Number of ways that 2 participants at 20th position so in 20 ways,
from each of the two states are Then remaining 24 letters can be
together is (2!)(2!)(8!) = (4)(8!) arranged in (24!) ways.
Number of ways that exactly 2 So total number of ways is (2)(20)
participants from each of the two (24!) = 40(24!)
states are together is (4)(8!) – (8!) 9. (b) From the solution of previous
= (3)(8!) question
Number of ways that 2 participants 0 letters between A and B then
from one of the state are together is number of ways is
(2!)(9!) = (18)(8!)
2(25)(24!) = 50(24!)
Then Total number of ways in
which at least from one state two 1 letter between A and B then
participants are together is 3{(18) number of ways is
(8!)} – {(3)(8!)+ (3)(8!) + (3)(8!)}   2(24)(24!) = 48(24!)
– 2{(8!)} = 43(8!) 2 letters between A and B then
Hence required number of ways is 10! number of ways is
– 43(8!) = (90)(8!) – (43)(8!) = (47)(8!)
  2(23)(24!) = 46(24!)
6. (b) Total number of ways that A, B, C
And so on 20 letters between A and
and D can be arranged is 4! = 24,
B then number of ways is
Number of arrangements that AB
(or BC and CD) are together in the   2(5)(24!) = 10(24!)
given sequence is 3! = 6 So total number of ways is
Number of arrangements in which AB (24!)(50 + 48 + 46 + ….. + 10) = 630(24!)
and BC (i.e ABC) together is 2! = 2 10. (c) As per the given condition we have
Number of ways that AB, BC and two cases-
CD are together (i.e ABCD) is 1
Case (i) If n is even say 2k then a1< a2<…
So required number of ways is
ak-1<ak and ak> ak+1> ak+2> … > a2k
24 – 3 × 6 + 3 × 2 –1 = 11
Arrangement  123
Case (ii) If n is odd say 2k+1 then a1< a2<… will create 7 gaps and we have to
ak<ak+1 and ak+1> ak+2> ak+3>…> a2k select 4 gaps out of these 7 gaps that
In case (i) we have to just select k–1 can be done in 7C4 ways = 35 ways.
out of 2k–1 i.e., in 2k–1Ck–1 ways. 13. (d) 2 women select the chairs amongst 1
In case (ii) we have to just select k to 4 is 4P2
out of 2k–1 i.e., in 2k–1Ck ways From the remaining 6 chairs 3 men
So we have following results can be arranged in 6P3 ways
Total number of ways is (4P2)(6P3)
If n = 9 or k = 4 then number of ways
is 8C4 ways = 70 14. (a) First we will arrange m men and
that can be done in (m!) ways, this
If n = 10 or k = 5 then number of
will create (m+1) places for n women
ways is 9C4 ways = 126
{(m !)(m + 1)!}
If n = 11 or k = 5 then number of in m+1Pn =
ways is 10C5 ways = 252 {(m + 1 − n)!}
If n = 12 or k = 6 then number of 15. (c) 3 girls can be seated at the back row
ways is 11C5 ways = 462 in 2 × 2(3!) ways, and remaining 9
boys can be arranged on remaining
If n = 13 or k = 6 then number of 11 seats in (11P9) ways so total
ways is 12C6 ways = 924 number of ways is 2 × 2(3!) (11P9)
If n = 14 or k = 7 then number of = 12!
ways is 13C6 ways = 1716 16. (c) Since number of girls is 3 and that of
So n may be equal to 10, 11, 12, 13 boys is 7.
11. (a) The value of X = nn As per the given condition no two
And the value of Y girls are together so 7 boys can be
n arranged in (7!) ways now this will
   = ∑ ( −1)n−r  C(n, r )(r n ) create 8 separated places where girls
r =1 can be arranged , and the number
12. (c) Total number of characters are of ways for this is 8P3 or 8 × 7 × 6
6 + 4 = 10 ways.
6 signs of ‘+’ can be arranged in 1 So total number of ways is (6 × 7 × 8)
way (They are identical) then they (7!) = 42(8!).
124  Theory of Counting
Concept Eliminator (CE)

1. (b) 2. (b) 3. (b) 4. (c)

1. (b) 1st consider ‘+’ sign the two ‘+’ signs For each flower there are two ways
can be placed in 64 × 49 ways, it can be arranged. Hence n flowers
similarly 2 ‘–’ signs can be placed in can be arranged in 2n ways
64 × 49 ways. Hence a total of m/2Pn. 2n ways.
So total number of ways is 64 × 49 × 3. (b) If m is odd, then there will be a
64 × 49, but out of these there are 64 middle vass. If the vass is left
× 49 ways where both the ‘+’ and both vacant. Similar to above question
the ‘–’ signs are in the same square required number of ways is
and 64 × (64 × 49) ways where one of
    (m–1)/2Pn. 2n
the ‘+’ sign and one of the ‘–’ sign are
in same square. 4. (c) The solution to this question is
similar to the previous question
Required number of ways is
except that one rose is in the middle
(64 × 49)2 – (64 × 49) – 64(64 × 49)
vass. The flower to be assigned the
= 64 × 49(64 × 49 – 1 – 64)
middle chair can be selected out of
= 3136 × 3071 = 9630656 n boys in nC1 ways = n ways. Now
2. (b) As per the given condition m is even, the remaining n–1 roses have to be
consider the right half of the vases in arranged in m–1 vases satisfies the
the row, m/2 vases. Now we arrange given condition. That is (m–1)/2Pn–1.
the n roses only in these m/2 vases 2n–1 ways .as, selection of rose and
or we have to arrange n articles arrangement of rest n–1 roses are
at m/2 places, this can be done in independent events. Therefore, total
(m/2) P ways. number of ways = n. (m–1)/2Pn–1 2n–1
n
5
Circular Arrangement  125

Circular Arrangement

Topics Covered

 Circular Arrangement

 Formation of Garland

 Formation of Necklace/Bracelet

(Polya Counting/Burnside Lemma)

Part A: Topic Number of Questions


Solved Example 5
Concept Applicator 15
Concept Builder 15
Concept Cracker 15
Part B: Topic Number of Questions
Concept Deviator 6
Concept Eliminator 4
Total 60
126  Theory of Counting
Part A

Concept 1 Circular Arrangement


The main difference between a linear arrange- Hence, we can say that ‘n’ articles can be
ment and a circular arrangement is the point arranged in a straight line in n! ways.
of reference. To understand this let us take an And ‘n’ articles can be arranged in a circle in
example of three persons A, B and C, for linear (n – 1)! ways.
arrangement we have 3! = 6 ways and these 6
ways are ABC, ACB, BCA, BAC, CAB, CBA. Let us see the reason behind this. If we have
Now consider circular arrangement for this to arrange 10 students on 10 chairs in a
we have following case: straight line then how we will proceed, Lets
A call 1st student how many different seats are
A
available for him the answer is 10 as each
and every seat is distinguishable (Means we
can name each and every seat as 1st from
right end, 7th from left etc.) Similarly for 2nd
student 9 distinguishable seats are available
B C C B and so on. So total number of ways is 10 × 9 ×
8 × … × 2 × 1 = 10!.
Now consider 4 persons A, B, C and D they can
be arranged in a straight line in 4! = 24 ways Now consider the case when we have to
but when we have to arrange them around a arrange 10 students on 10 chairs around a
circle then we have following 6 cases: circle, again lets call 1st student now how
A A A many different seats are available the answer
is 1 as each seat is equivalent and they are not
distinguishable now for 2nd student there are
B D D B C D
9 options as taking the 1st student as reference
point 9 seats are distinct and distinguishable
C C B
then next student has 8 options and so on so
A A A total number of ways is 1 × 9 × 8 × ... × 2 × 1 = 9!
So the main difference is for the 1st person in
D C B C C B linear arrangement for 1st person we have ‘n’
seats while in a circular arrangement it is only 1.
B D D
QR Code / Video Link

For Smart phone/ Tablet users

Video Link for Desktop/Laptops users http://dishapublication.com/video-resources


Circular Arrangement  127
Example 1:  In how many ways 20 students 19 distinct elements that can be arranged
can be seated around a circular table around a circle in (18!) ways hence total
having 20 seats? number of ways is (2!)(18!).
Solution:  Since number of students is 20 Example 3:  In how many ways 20 students
hence number of ways is 19!. can be seated around a circular table
Example 2:  In how many ways 20 students having 20 seats such that two students
can be seated around a circular table A and B are never together?
having 20 seats such that two students (a) (9!) – (2!)(8!) (b) (2!)(8!)
A and B are always together? (c) 2(9!) (d) None of these
Solution:  To make this arrangement just Solution:  From the solution of previous two
assume that A and B are tied together, questions required answer is (19!) – (2!)
it can be done in (2!) ways, now we have (18!)

Concept 2 Formation of Garland


Consider 4 beads and we have to make a Now, if we look at carefully then figures (i)
necklace from those 4 beads, at 1st glance it and (iv) represents same necklace, if we
looks that we have to arrange A, B, C and D in turn figure (i) then we will get figure (iv) so
a circle and that can be done in 3! = 6 ways, and figure (i) and (iv) represents same necklace
these 6 arrangements are as shown below, similarly figure (ii) and figure (v) represents
A A A same necklace and figure (iii) and figure (vi)
represents same necklace so total number of
B D D C C B necklace is 6/2 = 3.
When clockwise and anticlockwise arrange-
C B D ments are not different i.e when observation
Fig. (i) Fig. (ii) Fig. (iii) can be made from both sides, then number of
circular arrangements of n different things is
(n – 1)!/2.
A A A
Example 4:  Find the number of ways in
D B C D B C which 20 different coloured beads can
be arranged to form a necklace.
C B D Solution:  Since number of beads is 20 hence
Fig. (iv) Fig. (v) Fig. (vi) number of necklace is 19!/2.

QR Code / Video Link

For Smart phone/ Tablet users

Video Link for Desktop/Laptops users http://dishapublication.com/video-resources


128  Theory of Counting
Example 5:  If number of garlands that Solution:  As per the given condition
can be formed from n distinct flowers (n – 1)!/2 = 60 or (n – 1)! = 120 or n – 1 = 5
is 60 then find the value of n. or n = 6.

1
Concept Applicator (CA)
Ideal Time Apply your concepts with easy and
15 Min. conceptual questions

1. In how many ways 10 students can be 7. In how many ways 7 boys and 8 girls can
seated round a circular table having 10 be seated around a circular table if all
seats? the boys do not sit together?
(a) (10!) – (9!) (b) 9! (a) (7!) × (8!) (b) 14!(7!)
(c) 2(9!) (d) None of these (c) (14!) – (7!)(8!) (d) None of these
2. In how many ways 10 students can be 8. In how many ways 7 boys and 8 girls can
seated around a circular table having 10 be seated around a circular table such
seats such that two students A and B are that no two boys sit together?
always together? (a) (7!) × (8!) (b) (9!)(7!)
(a) (9!) – (2!)(8!) (b) (2!)(8!) (c) (14!) – (7!)(8!) (d) None of these
(c) 2(9!) (d) None of these 9. In how many ways 7 boys and 8 girls can
3. In how many ways 10 students can be be seated around a circular table such
seated around a circular table having 10 that no two girls sit together?
seats such that two students A and B are (a) (7!) × (8!) (b) (9!)(7!)
never together?
(c) (14!) – (7!)(8!) (d) None of these
(a) (9!) – (2!)(8!) (b) (2!)(8!)
10. In how many ways 7 boys and 8 girls can
(c) 2(9!) (d) None of these be seated around a circular table having
4. In how many ways 7 boys and 8 girls can 15 chairs numbered from 1 to 15?
be seated around a circular table? (a) (7!) × (8!) (b) (9!)(7!)
(a) (7!) × (8!) (b) 14! (c) (14!)(15) (d) None of these
(c) 2(14!) (d) None of these 11. In how many ways 7 boys and 8 girls can
5. In how many ways 7 boys and 8 girls can be seated around a circular table having
be seated around a circular table if all 15 chairs, 1 wooden and 14 fiber chairs?
the boys sit together? (a) (7!) × (8!) (b) (9!)(7!)
(a) (7!) × (8!) (b) 14!(7!) (c) (14!)(15) (d) None of these
(c) (7!) × (9!) (d) None of these 12. A round table conference is to be held
6. In how many ways 7 boys and 8 girls can between 20 delegates of 20 countries. In
be seated around a circular table if all how many ways they can be seated if 2
the girls sit together? particular delegates always sit together.
(a) (7!) × (8!) (b) 14!(7!) (a) (18!) (b) (2!)(18!)
(c) (7!) × (9!) (d) None of these (c) (19!)/(2) (d) None of these
Circular Arrangement  129
13. A round table conference is to be held (a) (14!) (b) (14!)/2
between 20 delegates of 20 countries. In
(c) (15!)/2 (d) None of these
how many ways they can be seated if 2
particular delegates never sit together. 15. Find the number of ways in which 15
(a) (18!) (b) (2!)(18!) same coloured identical beads can be ar-
ranged to form a necklace.
(c) (18!)(17) (d) None of these
14. Find the number of ways in which 15 dif- (a) (14!) (b) (14!)/2
ferent coloured beads can be arranged to (c) (15!)/2 (d) None of these
form a necklace.

Response Grid
1. a b c d 2. a b c d 3. a b c d 4. a b c d 5. a b c d
6. a b c d 7. a b c d 8. a b c d 9. a b c d 10. a b c d
11. a b c d 12. a b c d 13. a b c d 14. a b c d 15. a b c d

2
Concept Builder (CB)
Ideal Time Revise your concepts with questions
25 Min. medium difficulty level questions

1. In how many ways 20 students can be (c) (3!)(17!) (d) None of these
seated around a circle so that 2 particu- 4. If the ratio of number of arrangements
lar students are always together? when n students are arranged in a
(a) (2!)(18!) (b) (2!)(19!) straight lines to the number of arrange-
(c) (2!)(2!)(8!) (d) None of these ments when n students are arranged in
2. In how many ways 20 students can be a circle is 9 then which one of the follow-
seated around a circle so that 3 students ing is the value of n?
A, B, C are always together? (a) 8 (b) 9
(a) (3!)(18!) (b) (2!)(19!)
(c) 10 (d) None of these
(c) (3!)(17!) (d) None of these
5. In how many ways 16 students can be
3. In how many ways 20 students can be
arranged around a square table having 4
seated around a circle so that 3 students
seats on each side.
A, B, C are always together and B is be-
tween A and C? (a) (2)(15!) (b) (4)(15!)
(a) (3!)(18!) (b) (2!)(17!) (c) (15!)/2 (d) None of these
130  Theory of Counting
6. In how many ways 20 students can be the number of arrangements when all
arranged around a square table having 5 the girls arranged in a circle is 56:1 then
seats on each side? find the number of ways of arranging all
of them in a circle?
(a) (5)(19!) (b) (4)(19!)
(a) 15! (b) 16!
(c) (19!)/2 (d) None of these
(c) 17! (d) None of these
7. In how many ways 15 students can be ar-
12. Which one of the following could be
ranged around an equilateral triangular
the ratio of number of arrangements of
table having 5 seats on each side. a group of students around a circular
(a) (5)(14!) (b) (4)(14!) table when all the boys are together to
(c) (15!)/2 (d) None of these the number of arrangements when all
8. In how many ways 12 students can be the girls together?
arranged around a isosceles triangular (a) 56 : 1 (b) 120 : 1
table having 5 seats on each of the equal (c) 1 : 12 (d) None of these
sides and 2 seats on 3rd side. 13. Which one of the following could be
(a) (2)(11!) (b) (5)(11!) the ratio of number of arrangements of
a group of students around a circular
(c) (12!) (d) None of these
table when no two boys are together to
9. In  an  international  convention,  5  par-
the number of arrangements when no
ticipants from each of USA, China and
Russia were arranged around a circle. In two girls together?
how many ways this arrangement can be (a) 0 (b) 120 : 1
made if participants from the same coun- (c) 1 : 12 (d) None of these
try are always together?
14. In a family gathering 4 couples were to
(a) (3!)(5!)(5!)(5!) (b) (2!)(5!)(5!)(5!) sit around a circular table such that no
(c) (3!)(5!)(5!)(5!) (d) None of these two males as well as no couple is togeth-
10. In  an  international  convention,  5  par- er, in how many ways this arrangement
ticipants from each of USA, China and can be made?
Russia were arranged around a circle. (a) 6 (b) 12
In how many ways this arrangement can
be made if no two participants from USA (c) 18 (d) None of these
are together? 15. In how many ways 2 identical red flow-
(a) (9!)(10P (9!)(10C ers and 2 identical yellow flowers can be
5) (b) 5)
used to make a garland?
(c) (3!)(5!)(5!)(5!) (d) None of these
(a) 6 (b) 5
11. The ratio of number of arrangements
when all the boys arranged in a circle to (c) 4 (d) None of these

Response Grid
1. a b c d 2. a b c d 3. a b c d 4. a b c d 5. a b c d
6. a b c d 7. a b c d 8. a b c d 9. a b c d 10. a b c d
11. a b c d 12. a b c d 13. a b c d 14. a b c d 15. a b c d
Circular Arrangement  131

3
Concept Cracker (CC)
Ideal Time Boost up your confidence with good
30 Min. questions

Directions for questions number 1 to 6 er next to him. In how many ways this
The participants of an annual general meeting arrangement can be made?
are 25 executives, 2 managers and a director. (a) (2!)(26!) (b) (2!)(25!)
The meeting is conducted around a circular (c) (2!)(26!) (d) None of these
table.
7. In how many ways 20 students can be
1. In how many ways this arrangement can arranged around two circular tables
be made? having 10 seats each?
(a) (2!)(26!) (b) (2!)(25!) (a) (20C10)(9!) (b) (20C10)(9!)(9!)
(c) (27!) (d) None of these (c) 2(20C10)(9!)(9!) (d) None of these
2. In how many ways this arrangement can 8. A group of students comprising 3 girls
be made if the director has 2 managers and 5 boys went for a picnic. During a
on his both sides? game they arranged in a circle such that
each boy has one girl on at least one
(a) (2!)(26!) (b) (2!)(25!)
side?
(c) (26!) (d) None of these
(a) 120 (b) 360
3. In how many ways this arrangement can
(c) 720 (d) None of these
be made if the director has a particular
manager Mr.Dhingchak next to him? 9. Rishita has 5 distinct red and 5 distinct
(a) (26!) (b) (2!)(25!) white flowers how many distinct gar-
lands she can make if flowers no flowers
(c) (2!)(26!) (d) None of these of same colour are together?
4. In how many ways this arrangement
(a) 12(5!) (b) (4!)(5!)
can be made if the director has at least 1
manager next to him? (c) (5!)(5!) (d) None of these
(a) (22!)(26!) (b) (102)(25!) 10. In the above question, how many gar-
lands she can make if all the flowers of
(c) (22!)(25!) (d) None of these
the same colour are together?
5. In how many ways this arrangement
(a) 12(5!) (b) 2(5!)(5!)
can be made if the director has exactly 1
manager next to him? (c) (5!)(5!) (d) None of these
(a) (100!)(26!) (b) (102!)(25!) 11. In Praxis Business school during a meet-
ing professors from different department
(c) (22!)(25!) (d) None of these
were to sit around a circular table. If
6. In an annual general meeting of a number of professors from Marketing,
company 25 executives, 2 managers and Finance, Operations, and HR are 4, 5, 6
a director are sitting around a circular and 7 in any order. In how many ways
table such that director has no manag- they can be seated around the table if
132  Theory of Counting
all the professors from the same depart- ways seating arrangements be made if
ment are together? two participants Mr. Susmit and Mr.
(a) (2!)(4!)(5!)(6!)(7!) Hasan do not sit next to each other?
(b) (3!)(4!)(5!)(6!)(7!) (a) (4n2 – 6n + 4)(2n – 2)!
(c) (4!)(5!)(6!)(7!) (b) (4n2 – 6n – 4)(2n – 2)!
(d) None of these (c) (4n2 + 6n + 4)(2n – 2)!
12. In Praxis Business school during a meet- (d) None of these
ing professors from different department 14. There are 40 distinct pearl, out of which
were to sit around a circular table. If 20 are of blue coloured and remaining 20
number of professors from Marketing, are of red coloured. How many different
Finance, Operations, and HR are 5 each. necklace can be formed such that no two
In how many ways they can be seated beads of same colour are together?
around the table if all the professors (a) (40!) – 2(19!)(20!)
from the same department are together
(b) (39!) – 2(19!)(20!)
and HOD of any department is exactly
between the professors of that depart- (c) 1/2 (19!)(20!)
ment? (d) None of these
(a) (3!)(4!)4 (b) (4!)4 15. In how many ways 32 students can be ar-
4
(c) (4)(4!) (d) None of these ranged around two square tables having
16 seats each (4 seats on each side of the
13. In an annual general meeting of a
table)?
company XYZ limited, President and
Chairman have fixed seats opposite (a) (32C16)(4)(15!)
to each other. There are 2n number of (b) (32C16)(4)(15!)(4)(15!)
participants in the meeting other than (c) (32C16)(15!)(15!)
President and Chairman. In how many
(d) None of these

Response Grid
1. a b c d 2. a b c d 3. a b c d 4. a b c d 5. a b c d
6. a b c d 7. a b c d 8. a b c d 9. a b c d 10. a b c d
11. a b c d 12. a b c d 13. a b c d 14. a b c d 15. a b c d
Circular Arrangement  133

Solutions
Concept Applicator (CA)
1. (b) 2. (b) 3. (a) 4. (b) 5. (a) 6. (a)
7. (c) 8. (a) 9. (d) 10. (c) 11. (c) 12. (b)
13. (c) 14. (b) 15. (d)

1. (b) Since n distinct articles can be number 5 the number of ways such
arranged around a circle in (n – 1)! that all the boys are together is (7!)
Hence 10 students can be arranged (8!) hence required number of ways
in 9! ways. is (14!) – (7!)(8!).
2. (b) To make this arrangement just 8. (a) Restriction is on boys so 1st we will
assume that A and B are tied arrange 8 girls and that can be done
together, it can be done in (2!) ways, in (7!) ways. Now this will create 8
now we have 9 distinct elements spaces between them in which 7 boys
that can be arranged around a circle
can be seated in (8!) ways, hence
in (8!) ways hence total number of
required number of ways is (7!)(8!).
ways is (2!)(8!).
9. (d) If we place boys 1st then it will create
3. (a) From the solution of previous
only 7 places so it is not possible
two questions required answer is
(9!) – (2!)(8!). that no two girls sit together. Total
number of ways is 0.
4. (b) Here we have total 7 + 8 = 15 distinct
elements that can be arranged in 10. (c) Since each of the seat is numbered
14! ways. and hence distinguishable so it is
equivalent to linear arrangement
5. (a) Since boys sit together so tie all
and number of ways is 15! or (14!)
the boys together it can be done in
(7!) ways, now we can treat them (15) as given in option (c).
as one unit then we have total 11. (c) Taking wooden chair as reference
8 + 1 = 9 distinct units and that can point, 1st person can be arranged in
be arranged in a circle in 8! ways. 15 ways and so on hence number of
Hence total number of ways is (7!)(8!). ways is 15!.
6. (a) Similar to the previous question 12. (b) Since 2 delegates are always together
total number of ways is (7!)(8!). so we can tie them in (2!) ways now
7. (c) From question number 4 without 19 elements can be arranged in (18!)
restriction they can be arranged ways hence required number of ways
in (14!) ways, and from question is (2!)(18!)
134  Theory of Counting
13. (c) Without restriction they can be 14. (b) Here in this case number of ways is
arranged in (19!) ways and with (14!)/2.
restriction in (2!)(18!) ways hence 15. (d) Since beads are identical so total
in this case number of ways is number of ways is 1.
(19!) – (2!)(18!) = (17)(18!).

Concept Builder (CB)


1. (a) 2. (c) 3. (b) 4. (b) 5. (b) 6. (a)
7. (a) 8. (c) 9. (b) 10. (a) 11. (a) 12. (d)
13. (a) 14. (b) 15. (d)

1. (a) If two students are together then 7. (a) For 1st student total number of ways
they can be arranged in 2! ways and is 5 [extreme right of a side, one from
remaining 19 units can be arranged extreme side, middle one, three from
in (18!) ways, hence total number of extreme right, extreme left] then for
ways is (2!)(18!). next student number of ways is 14
2. (c) If three students are together then and so on. So total number of ways
they can be arranged in 3! ways and is (5)(14!).
remaining 18 units can be arranged 8. (c) For 1st student total number of
in (17!) ways, hence total number of ways is 12 as each and every seat is
ways is (3!)(17!). distinguishable, hence this case is
3. (b) If three students are together then similar to linear arrangement, and
they can be arranged in 2 ways as per so number of ways is (12!).
the given condition and remaining 9. (b) As a 1st step tie all the participants
18 units can be arranged in (17!) from same nation this can be done in
ways, hence total number of ways is (5!) for each country, now we have 3
(2!)(17!). units that can be arranged in a circle
4. (b) Required ratio is (n!)/(n–1)! = n, in (2!) ways hence total number of
hence n = 9. ways is (2!)(5!)(5!)(5!).
5. (b) For 1st student total number of 10. (a) Since, there is no restriction with
ways is 4 [extreme right of a side, participants from China and Russia
one from extreme side, two from hence these 10 members can be
extreme right, extreme left] then for arranged around circle in (9!)
next student number of ways is 15 ways, this will create 10 spaces, so
and so on. So total number of ways now 5 participants from USA can
is (4)(15!). be arranged in these 10 places in
6. (a) For 1st student total number of ways (10C5×5!) = (10P5), hence total number
is 5 [extreme right of a side, one from of ways is (9!)(10P5).
extreme side, middle one, three from 11. (a) From the given condition (b – 1)!
extreme right, extreme left], then for /(g – 1)! = 56 = 7 × 8, hence b = 9 and
next student number of ways is 19 g = 7, so total number is 7 + 9 = 16 and
and so on. So total number of ways required number of ways is (15!)
is (5)(19!).
Circular Arrangement  135
12. (d) Let the number of boys is b 14. (b) Let 1st arrange 4 females that can
and number of girls is g then in be done in (3!) ways now this will
both the condition the number create 4 places in which males can
of arrangements is (b!)(g!), and be arranged in 2 ways hence total
required ratio is 1. number of ways is 2 × 3! = 12 ways.
13. (a) Exactly one of the two 15. (d) Two red flowers can be arranged in
arrangements must be zero. 1 way, and then 2 yellow flowers can
Refer questions number 8 and 9 of be arranged in 2 ways.
Concept Applicator. (Ratio becomes 1 Hence total number of ways is 2.
when b = g)

Concept Cracker (CC)


1. (c) 2. (b) 3. (c) 4. (b) 5. (d) 6. (d)
7. (b) 8. (c) 9. (a) 10. (c) 11. (b) 12. (a)
13. (a) 14. (c) 15. (b)

1. (c) This is a general case, since total managers are next to the Director
number of units is 25 + 2 + 1 = 28 That we have calculated in question
and these can be arranged around a number 2 and is equal to 2(25!).
circular table in (27!) ways.
Hence required number of ways is
2. (b) The director and two managers can 4(26!) – 2(25!) = 102(25!).
be arranged in (2!) ways and let they
are tied together then we have total For better understanding see the
26 units that can be arranged in (25!) Venn Diagram below:
ways hence total number of ways is
Total = (27!)
(2!)(25!).
3. (c) Manger Mr. Dhingchak and the M1 M2
52(25!) 52(25!)
director can be arranged in 2 ways.
If we tie this manager and director
then we can treat them as a unit then
50(25!) 2(25!) 50(25!)
we will have 27 units and that can be
arranged in a circle in (26!) ways. So
total number of ways is (2)(26!).
4. (b) A manager can be selected in 2 ways
now this manager and the director
can be arranged in 2 ways. If we tie 5. (d) From solution of previous questions
this manager and director, then we required number of ways is
can treat them as a unit then we   102(25!) – 2(25!) = 100(25!).
will have 27 units and that can be
arranged in a circle in (26!) ways. So 6. (d) Without restriction they can be
total number of ways is (4)(26!). arranged in (27!) ways, and the
number of ways in which at least
But this case also includes the
one manager is next to director is
number of ways in which both the
102(25!) [from question number 4].
136  Theory of Counting
So required number of ways is 12. (a) If we treat each department as one
(27!) – 102(25!) = 26 × 27(25!) – 102(25!) unit then we have 4 units and that
= 600(25!) can be arranged in a circle in (3!)
ways, and these faculties with their
Alternately: In this case the HOD at center can arranged in (4!)
Director has two executives next ways for each department.
to him, these two executives can Hence total number of ways is
be selected in 25C2 ways and these (3!)(4!)(4!)(4!)(4!) = (3!) (4!)4.
two can be arranged in (2!) ways. 13. (a) Here we have two cases:
Now treat these two executives and Case (i): If Mr. Susmit is next to
director as one unit then we have 26 President or Chairman, then he can
distinct units which can be arranged select his seat in 4 ways.
in (25!) ways. Hence total number of In this case Mr. Hasan can select his
ways is (25C2)(2!)(25!) = 600(25!). seat in (2n – 2) ways and remaining
7. (b) 1st we will select 10 out of 20 that (2n – 2) participants can be arranged
can be done in (20C10) ways, now in (2n – 2)! ways.
we have two groups of 10 students So total number of ways in this case
each group can be arranged around is 4(2n – 2)(2n – 2)!
a circular table in (9!) ways hence Case (ii): If Mr. Susmit do not sit next
total number of ways is (20C10)(9!) to President or Chairman, then he can
(9!) ways. select his seat in (2n – 4) ways.
8. (c) Since, 5 boys can be arranged in a In this case Mr. Hasan can select his
circle in (5 – 1)! = (4!) = 24 ways, seat in (2n – 3) ways and remaining
(2n – 2) participants can be arranged
Now girls can be arranged 5 × 3 × 2 in (2n – 2)! ways.
= 30 ways. So total number of ways in this case
Hence total number of ways is is (2n – 4)(2n – 3)(2n – 2)!
24 × 30 = 720. So total number of ways is 4(2n – 2)
9. (a) Since, 5 red and 5 white flowers can (2n – 2)! + (2n – 4)(2n – 3)(2n – 2)!
be arranged in a circle in (4!)(5!) if no = (4n2 – 6n + 4)(2n – 2)!
flowers of same colour are together 14. (c) Let us fix 1st a particular colour,
hence in case of garland it will be 20 beads of same colour (But beads
(4!)(5!)/2 = 12(5!). are distinct) can be arranged for
necklace in (20 – 1)!/2 = (19)!/2 ways.
10. (c) If they have to arrange in a circle There are 20 separated space in which
then number of ways is (2!)(5!)(5!), we can put beads of different colour,
so number of garlands is (2!)(5!)(5!)/2 this can be done in (20!) ways.
= (5!)(5!). Hence total number of such necklace
11. (b) If we treat each department as one is 1/2 (19!)(20!).
unit then we have 4 units and that 15. (b) 1st we will select 16 out of 32 that
can be arranged in a circle in (3!) can be done in (32C16) ways, now
ways, and these faculties themselves we have two groups of 16 students
can be arranged in (4!), (5!), (6!) and each group can be arranged around
(7!) ways. a square table in (4)(15!) ways hence
Hence total number of ways is (3!) total number of ways is (32C16)(4)
(4!)(5!)(6!)(7!). (15!)(4)(15!) ways.
Part B Circular Arrangement  137

Circular
Concept 1 Arrangement
The main difference between a linear arrange- but when we have to arrange them around a
ment and a circular arrangement is the point circle, then we have following 6 cases:
of reference. To understand this let us take an A A A
example of three persons A, B and C, for linear
arrangement we have 3! = 6 ways and these 6 B D D B C D
ways are ABC, ACB, BCA, BAC, CAB, CBA.
Now consider circular arrangement for this C C B
we have following case:
A A A A
A
D C B C C B

B D D
Hence we can say that ‘n’ articles can be
B C C B
arranged in a straight line in n! ways.

And ‘n’ articles can be arranged in a


Now consider 4 persons A, B, C and D they can
circle in (n – 1)! ways
be arranged in a straight line in 4! = 24 ways

Concept 2 Formation of Garland

A A A
Consider 4 beads and we have to make a
necklace from those 4 beads, at 1st glance it
B D D C C B
looks that we have to arrange A, B, C and D in
a circle and that can be done in 3! = 6 ways, and C B D
these 6 arrangements are as shown below: Fig. (i) Fig. (ii) Fig. (iii)
138  Theory of Counting
A A A (ii) and figure (v) represents same necklace
and figure (iii) and figure (vi) represents same
D B C D B C
necklace so total number of necklace is 6/2 = 3.
When clockwise and anti-clockwise arrange-
C B D
ments are not different i.e., when observation
Fig. (iv) Fig. (v) Fig. (vi)
can be made from both sides, then number of
Now, if we look at carefully then figures (i) and circular arrangements of n different things
(iv) represents same necklace, if we turn figure
(n – 1)!
(i) then we will get figure (iv) so figure (i) and is .
2
(iv) represents same necklace similarly figure

Concept 3 Formation of Necklace/Bracelet


(Polya Counting/Burnside Lemma)

Consider a regular hexagon, how many ways Case (iii): If both rotations and reflections
we can colour the six vertices of this with the are equivalent then number of
two available colour black and white. There are ways is 13.
mainly three ways in which it can be answered Case (i) is obvious but have a look at case
(i) “There are 64 ways of colouring the (ii) all the 14 arrangements are shown below:
hexagon.”
(ii) “There are 14 ways of colouring the
hexagon.”
(iii) “There are 13 ways of colouring the
hexagon.”
All the three answers are all valid, since the
question was not clear. Consider each sections Fig. 1 Fig. 2
separately
Case (i): If rotations and reflections are
considered to be distinct, then
number of ways is 26 = 64
Case (ii): If rotations are considered to be
equivalent, but reflections are
distinct, then number of ways is 14. Fig. 3 Fig. 4
Circular Arrangement  139
7 and Fig number 8 will represents the same
arrangement hence in this case total number
of cases is 14 – 1 = 13.

Burnside Lemma
The number of distinct objects is equal to the
mean number of objects fixed by each symmetry.
Fig. 5 Fig. 6 The counting we have done in case (ii) whose
diagram is shown by 14 figures are called
necklace and the counting we have done in
case (iii) is called bracelet.
The difference between a necklace and bracelet
is in bracelet reflections are equivalent.
The conclusion from the Burnside Lemma and
Polya Enumeration Theorem we can conclude
Fig. 7 Fig. 8 in simple terms that number of Necklace that
can be formed by n beads of k colours is:
1 n gcd(n, i )
(n, k) = ∑k
n i=1
Special case is when n = a prime number p
then
1
(p, k) = {(p − 1)k +  k p }
Fig. 9 Fig. 10 p
Example 1:  Number of necklace that can
be formed by 3 beads of 3 colours is
1 1 1 3 1
(3, 3) = (3 +  3 +  3 ) = (3 + 3 + 27) =  11
3 3
Example 2:  Number of necklace that can
be formed by 6 beads of 4 colours is

Fig. 11 Fig. 12 1 1 2 3 2 1 6
(6, 4 )  (4   4   4   4   4   4 )
6
1
 (4 1  6   64 1
  6   4   4096)
6
1
 (4200)   700
6
As in case (iii) both rotations and reflections are
equivalent (We call it as bracelet): number of bracelet
Fig. 13 Fig. 14 when n = p a prime number with k colour is
p+ 1
For case (iii), since both rotations and 1  p 2

reflections are equivalent hence Fig number b(p, k ) =  k +  p × k +  (p − 1)k 
2p  
140  Theory of Counting

Number of Necklace
Colour (k) → 1 2 3 4 5 6 7 8 9 10
Beads (n) ↓
1 1 2 3 4 5 6 7 8 9 10
2 1 3 6 10 15 21 28 36 45 55
3 1 4 11 24 45 76 119 176 249 340
4 1 6 24 70 165 336 616 1044 1665 2530
5 1 8 51 208 629 1560 3367 6560 11817 20008
6 1 14 130 700 7826 19684 19684 43800 88725
7 1 20 315 2344 11165 39996
8 1 36 834 8230 48915
9 1 60 2195 29144
10 1 108 5934
Larger values are omitted from the table

Number of Bracelets

Colour (k) → 1 2 3 4 5 6 7 8 9 10
Beads (n) ↓
1 1 2 3 4 5 6 7 8 9 10
2 1 3 6 10 15 21 28 36 45 55
3 1 4 10 20 35 56 84 120 165 220
4 1 6 21 55 120 231 406 666 1035 1540
5 1 8 39 136 377 888 1855 3536 6273 10504
6 1 13 92 430 1505 4291 10528 23052
7 1 18 198 1300 5895 20646
8 1 30 498 4435 25395
9 1 46 1219 15084
10 1 78 3210
Larger values are omitted from the table
Circular Arrangement  141

1
Concept Deviator (CD)
Ideal Time Revise your concept (IIT advance or
5 Min. tougher questions)
Per Question

1. How many ways we can pick 4 non-adja- 4. A round table conference is to be held
cent balls from 20 different balls if they between 20 delegates of 20 countries. In
are arranged in a circle. how many ways can they be seated if two
(a) 17C 16C
3 (b) 3 particulars delegates are always to sit
(c) 17C
3–
15C
2 (d) None of these together? [IIT JEE 1970]
2. How many necklace of 5 beads can be (a) 3(18!) (b) 2(18!)
formed from the infinite number of iden- (c) 18! (d) None of these
tical red and infinite number of identical
5. In how many ways vertices of a square
green beads?
can be coloured with 4 distinct colour if
(a) 12 (b) 8
rotations are considered to be equivalent,
(c) 15 (d) None of these but reflections are distinct?
3. Eighteen guests have to be seated half
(a) 65 (b) 70
on each side of a long table. Four particu-
lar guests desire to sit on one particular (c) 71 (d) None of these
side and three other on the other side. 6. In how many ways vertices of a square
Determine the number of ways in which can be coloured with 4 distinct colour if
the sitting arrangements can be made? both rotations and reflections are equiv-
[IIT JEE 1991] alent?
(a) (11!)(9P 4 )(9P 3) (b) (9!)(9P4)(9P3) (a) 65 (b) 70
(c) (11!)(9C 4 )(9C 3) (d) None of these (c) 71 (d) None of these

Response Grid
1. a b c d 2. a b c d 3. a b c d 4. a b c d 5. a b c d
6. a b c d
142  Theory of Counting

2
Concept Eliminator (CE)
Ideal Time Revise your concept (Maths Olympiad
Just Solve It or tougher questions)

1. If ‘K’ balls are arranged in circular order, of the same colour are assigned a num-
then what is the number ofways of se- ber 0. What is the greatest possible sum
lecting four of the balls such that no two of all the numbers assigned to the arcs?
of which are consecutive? (a) 100 (b) 120
k(k − 7)(k − 6)(k − 5) (c) 150 (d) None of these
(a)
4! 3. In how many rotationally distinct ways
k(k − 7)(k − 6)(k − 5)(k − 4) can the vertices of a cube be coloured
(b)
4! with black or white colour?
k(k − 7)(k − 6)
(c) (a) 18 (b) 20
4!
(d) None of these (c) 23 (d) None of these
2. This question is based on Maths Olympiad: 4. In how many ways can a strip of cloth
with 5 stripes on it be colored with
Sixty points, of which thirty are coloured
3 different colours? Assume that the strip
red, twenty are coloured blue, and ten
of cloth can be flipped over i.e do not count
are coloured green, are marked on a
as different patterns that are equivalent
circle. These points divide the circle into
if the cloth is turned around. As example
sixty arcs. Each of these arcs is assigned
following two strips are same
a number according to the colours of its
endpoints: an arc between a red and a R W G R W
green point is assigned a number 1, an
arc between a red and a blue point is as- W R G W R
signed a number 2, and an arc between (a) 135 (b) 270
a blue and a green point is assigned a
(c) 105 (d) None of these
number 3. The arcs between two points

Response Grid
1. a b c d 2. a b c d 3. a b c d 4. a b c d

Solutions
Concept Deviator (CD)
1. (d) 2. (b) 3. (a) 4. (b) 5. (b) 6. (d)
Circular Arrangement  143
1. (d) 1st ball (say A) can be selected in We can also solve this question by
20 ways, then remaining three (say applying Burnside Lemma.
B, C, and D) are selected such that 3. (a) As per the given condition 9 seats
number of balls between them is a,
are on each side, 4 persons can be
b, c and d, from the given condition
arranged on one side in (9P4) ways,
a, b, c, d > 0
and 3 persons on other side in (9P3)
And a + b + c + d = 20 – 4 = 16, but ways remaining 11 can be arranged
since all of them are more than 0 hence
in (11!) ways, So total number of
number of ways is (From partitioning
ways is (11!)(9P4)(9P3).
theory) is (16 – 1)C4 – 1 = 15C3.
But 1st one in 20 ways and there is a 4. (b) If we tie two delegates together then
repetition of 4 (since these four balls this can be done in (2!) ways, now we
can be selected when we start with have 19 distinct identities and that
B or C or D so we have to divide the can be arranged around a circle in
final answer by 4) hence final answer (18!) ways.
is (20 × 15C3)/4 = 20 × 13 × 14 × 15/4 So total number of ways is 2(18!).
× (3!) = 2275.
5. (b) Here in this case condition is
2. (b) We have following cases similar to formation of necklace i.e.,
Cases Red Green No. of 1 n gcd(n , i )
Beads Beads ways (n, k) = ∑k
n i=1
Case (i) 5 0 1
We can use this formula or from the
Case (ii) 4 1 1 table (you shouldn’t memorize it)
Case (iii) 3 2 2 required number of ways is 70.
Case (iv) 2 3 2 6. (d) This case is similar to the formation
Case (v) 1 4 1 of bracelet so from the formula or
Case (vi) 0 5 1 table b(4,4) = 55.
So total number of ways is 8

Concept Eliminator (CE)


1. (a) 2. (a) 3. (c) 4. (a)

1. (a) 1st ball (say A) can be selected in But 1st one in K ways and there is a
K ways, then remaining three (say repetition of 4 (since these four balls
B, C, and D) are selected such that can be selected when we start with B
number of balls between them is a, or C or D so we have to divide the final
b, c and d, from the given condition answer by 4) hence final answer is
a, b, c, d > 0. k(k − 7)(k − 6)(k − 5)
(k/4)[(k – 5)C3] =
And a + b + c + d = K – 4, but since 4!
all of them are more than 0, hence 2. (a) Number of red coloured card–30
number of ways is (from partitioning Number of blue coloured card-20
theory) is (k – 5)C4 – 1 = (k – 5)C3. Number of green coloured card-10
144  Theory of Counting
Point between red and green is 1 Hence the number of orbits is
Point between red and blue is 2 1
(1 × 256 +  6 × 4 +  8 × 16 +  3 × 16 +  6 × 16)
Point between blue and green is 3 24
Point between same colour is 0 = 23.
For maximum point, we will keep all 4. (a) If we did not consider strips to be the
the 30 red coloured points alternately
same when turned around or flipped
and fill up with blue and green colour
over, then each of the 5 stripes can
in the vacant place, so total points is
be filled with any of 3 colors, or total
2 × 20 × 2 + 2 × 10 × 1 = 100.
number of ways is 53 making a grand
3. (c) Each vertex can be coloured in 2 ways,
total of kn possible strips.
so there are 28 = 256 ways (when
But the answer must be lesser than
vertices are fixed under the action of
the trivial symmetry) now consider 125, because when turn the strip
symmetry under different rotations. around, it matches with one that has
There are 22 = 4 (out of 256) that the opposite coloring as explained in
are fixed under a 90 degree rotation the question itself.
about the axis joining the mid-points The answer is given by adding the
of two opposite faces. number of symmetric cases to the
There are 24 = 16 (out of 256) that number of non-symmetric cases
are fixed under a 120o rotation about divided by 2, so we have to find
the axis joining two diametrically the number of symmetric and non-
opposite corners. (In this case symmetric cases:
rotation splits up the 8 vertices as If we look out carefully then a
1 + 3 + 3 + 1 = 8). symmetric strip has the similar
There are 24 = 16 (Out of 256) that pattern on the right as on the left, so
are fixed under a 180o rotation about once we know what’s on the left, the
the axis joining the mid-points of pattern on the right is determined.
There’s a different case with odd and
two opposite faces. (In this case
even sized strips. For an even number
rotation splits up the 8 vertices as
of stripes, say when n = 2k, there are
2 + 2 + 2 + 2 = 8).
km different symmetric possibilities
There are 24 = 16 (out of 256) that
and if n is odd, n = 2k + 1, there are
are fixed under a 180o rotation about
km+1 symmetric possibilities.
the axis joining the mid-points of
In this case n = 5, so number of
two opposite edges. (In this case symmetric possibilities are 33 = 27
rotation splits up the 8 vertices as So required number of ways is
2 + 2 + 2 + 2 = 8).   (35 + 33)/2 = 135.

The number of ways in which we can make a strip with n strip and k colour is given by:
 n+ 1 
 
n
k + k 2 
2
here {x} is the greatest integer function.
6
Word/Number Formation  145

Word/Number Formation

Topics Covered
 Formation of word

 Formation of number

 Rank of a word in Dictionary

 Sum of all the numbers formed

 Number of palindrome numbers

Part A: Topic Number of Questions


Solved Example 10
Concept Applicator 15
Concept Builder 20
Concept Cracker 28
Part B: Topic Number of Questions
Concept Deviator 17
Concept Eliminator 11
Total 101
146  Theory of Counting
Part A

Concept 1 Formation of Word

n!
Word/number formation question is one of the or x =
{( p !)(q !)(r !)}
most important type of questions. To solve
these type of questions we have to understand
the arrangement of identical articles. Example 1:  How many words can be
formed from the letters of the word
The number of arrangements of n articles HINDUSTAN?
taken all at a time when p of them are
Solution:  Number of letters in the word
identical and are of one type, q of them are
HINDUSTAN is 9, out of these 9 letters N
identical and of second type, r of them are
is repeated twice, hence number of distinct
identical and of third type, and remaining are words that can be formed (9!)/2.
n!
all of different is . Example 2:  How many words can be
{( p !)(q !)(r !)}
formed from the letters of the word
PERMUTATION?
To understand this let us assume that
required number of permutations is x. Since Solution:  Number of letters in the word
p different things can be arranged among PERMUTATION is 11, out of these 11
themselves in (p!), hence if we replace p letters T is repeated twice, hence number of
identical articles by p distinct articles then distinct words that can be formed (11!)/2.
the number of permutations will become (p!)
Example 3:  How many words that
(x). Similarly, if we replace q identical article
starts with T and end with T can be
by q distinct articles then the number of
formed from the letters of the word
permutations will become (p!)(q!)(x) and if we
PERMUTATION?
replace r identical article by r distinct articles
then the number of permutations will become Solution:  Number of letters in the word
(p!)(q!)(r!)(x) which is equal to (n!) PERMUTATION is 11, out of these 11
letters T is repeated twice, since 1st and last
Hence, (p!)(q!)(r!)(x) = n!

QR Code / Video Link

For Smart phone/ Tablet users

Video Link for Desktop/Laptops users http://dishapublication.com/video-resources


Word/Number Formation  147
place is fixed for letter ‘T’ hence number of Solution:  Number of letters in the word
distinct words that can be formed (9!). PERMUTATION is 11, out of these 11
Example 4:  How many words that letters T is repeated twice, since 1st and
starts with E and end with N can be last place is fixed for letters ‘E’ and ‘N’
formed from the letters of the word respectively hence number of distinct words
PERMUTATION? that can be formed (9!)/2.

Concept 2 Formation of number

Number formation is very much similar to  5 – All numbers that end with 0 or 5 are
word formation and the only difference is that divisible by 5.
at 1st place we can not put ‘0’ as example if it  6 – A number that is divisible by 2 and 3
is given that we have to form a 4 digit number both are divisible by 6.
‘abcd’ then the digit ‘a’ can not be 0. Variation
 8 – A number divisible by 8 if number
in the question may occur if some property
formed by last three digits of the number
of number is given as example divisible by
is divisible by 8 or last three digits are
2 or 5 etc. To understand this just revise the
zeroes.
divisibility rule-
 9 – A number is divisible by 9 if sum of its
 2 – All even number is divisible by 2, or digits is divisible by 9.
the numbers end with an even number or
 11 – A number is divisible by 11 if the
zero is divisible by 2,
difference between the sum of the digits
 3- A number is divisible by 3 if sum of its in the odd and the sum of the digits in the
digits is divisible by 3. even places is either 0 or multiple of 11.
 4 – A number is divisible by if 4 if the  Ex: Is 245718 divisible by 11?
number formed by the last two digits of
 Sol: Here sum of the digits at odd places
the number is divisible by 4, or last two
is 2 + 5 + 1 = 8, and sum of the digits
digits are 0’s.

QR Code / Video Link

For Smart phone/ Tablet users

Video Link for Desktop/Laptops users http://dishapublication.com/video-resources


148  Theory of Counting
at even places is 4 + 7 + 8 = 19 and the 1st place can be filled up in 5 ways i.e., by
difference between these two is 19 – 8 = 11 1, 2, 3, 4 or 5 then for remaining 3 places
that is divisible by 11 hence the given we can select any 3 from remaining 5 digits
in 5C3 ways and can arrange them in 3! =
number is divisible by 11.
6 ways.
 12 – A number is divisible by 12 when it
So total number of numbers formed is
is divisible by 3 and 4 both. 5(5C3)(3!) = 5 × 10 × 6 = 300
 16 – A number is divisible by 16 when Example 6:  Find the number of 4 digit
number formed by last 4 digits of the numbers.
given number is divisible by 16 or last 4
Solution:  Since 1st place can be filled up
digits are zeroes. in 9 ways and remaining 3 places can be
 7 and 13 – A number is divisible by 7 or 13 filled up in 10 ways each so total number of
if and only if the difference of the number numbers is 3 × 10 × 10 × 10 = 3000
of its thousands and the remainder of its
Example 7:  Find the number of 5 digit
division by 1000 is divisible by 7 or 13.
numbers that can be formed from the
Ex: Is 13153 divisible by 7. digits 2, 2, 3, 3, 3.
Sol: Here number of thousands = 13 and Solution:  Total number of given digits is 5
the remainder when it is divided by 1000 out of which there are 2 two’s and 3 three’s
is 153 and difference 153 – 13 = 140 is hence number of ways we can arrange it is
divisible by 7 hence the number 13153 is (5!)/(2!)(3!) = 10
divisible by 7.
Example 8:  Find the number of 5 digit
Example 5:  How many four digit numbers
numbers that can be formed from the
can be formed from the digits 0, 1, 2,
digits 2, 2, 3, 3, 3 such that the number
3, 4 and 5 if repetition of digits is not
is divisible by 4 .
allowed?
Solution:  Total number of given digits is 5
Solution:  Number of digits is 4 so 1stdigit
out of which there are 2 two’s and 3 three’s.
(i.e., thousandth digit) can not be zero Since number is divisible by 4 hence last
two digits must be 32, then remaining 2,
2, 3 can be arranged in (3!)/(2!)(1!) ways
= 3 ways.
Word/Number Formation  149

Rank of a word in Dictionary


Concept 3
Rank of a word is the position of that word, Solution:  We will solve this question stepwise
when we arrange the words formed by
(i) Arrange all the alphabets of the word
alphabets of that given word in dictionary
MOTHER in alphabetical order like
order. Lets see an example.
(E, H, M, O, R, T).
To understand this type of problem just
(ii) Now in dictionary words will appear in
assume that we have only 3 letters A, B and C
alphabetical order, so first words will
in the English alphabets then the dictionary
appear starting alphabet “E”. If E is fixed
will have total meaning full or meaning less
at first position, rest 5 alphabets can be
3! = 6 words and their order is
arranged in 5! = 120 ways so there are
ABC → 1st word
120 words in that dictionary that starts
ACB→ 2nd Word with ‘E’
BAC → 3rd Word (iii) Next alphabet is “H” and similar to ‘E’
BCA → 4th Word there will be 5! = 120 words starting with
CAB → 5th Word “H”.
CBA → 6th Word (iv) Now starting with “M”, just hold on,
given word MOTHER also starts with ‘M’
So, if I ask you what is the rank of word CAB
so now we will consider the 2nd letter.
the answer is 5th.
(v) Next alphabet as “E” we will have 4! = 24
Example 9:  If all the letters of the word words that starts with ‘ME’.
MOTHER is arranged in dictionary (vi) Similarly starting with “M”, and next
format then find the rank/position of alphabet as “H” we will have 4! = 24
the word MOTHER. words that start with ‘MH’.

QR Code / Video Link

For Smart phone/ Tablet users

Video Link for Desktop/Laptops users http://dishapublication.com/video-resources


150  Theory of Counting
(vii) Next will be starting with “M”, and next coefficient of 5!
alphabet as “O” and that combination is Then strike off the next letter
in the given word MOTHER. O (E, H, M, O, R, T)
(viii) Next as “E” we’ll have 3! = 6 words that Then count the number of letters before
starts with MOE. O and it is equal to 2 which is coefficient
(ix) Similarly starting with “MOH”, and of 4!
“MOR” we’ll have 3! = 6 words. Then strike off the next letter
(x) Next will be starting with “MOTE”, we’ll T (E, H, M, O, R, T)
have 2! = 2 words. Then count the number of letters before
(xi) Next will be starting with “M”, and next T and it is equal to 3 which is coefficient
alphabet as “O” and next as “T” and next of 3!
as “H” will have 2! = 2 words but the first Then strike off the next letter
word will be M > O > T > H > E > R which H (E, H, M, O, R, T)
is the desired word. Then count the number of letters before
So the rank of word MOTHER in H and it is equal to 1 which is coefficient
dictionary will be 5! + 5! + 4! +4! + 3! + 3! of 2!
+ 3! + 2! +1 which equals 309. Then strike off the next letter
E (E, H, M, O, R, T)
Alternately Then count the number of letters before
(i) Arrange all the alphabets of the word E and it is equal to 0 which is coefficient
MOTHER in alphabetical order like of 1!
(E, H, M, O, R, T). Then the next word is MOTHER.
Now strike off the first letter M from So rank of the word MOTHER is 2(5!) +
(E, H, M, O, R, T). 2(4!) +3(3!) + 1(2!)+ 0(1!) + 1 = 240 + 48 +
Then count the number of letters before 18 + 2 + 1 = 309.
M, and it is equal to 2,which is the Hence rank of word MOTHER is 309.

Sum of All The


Concept 4 Numbers Formed
Consider three digits 1, 2 and 3, from these numbers and these numbers are : 123, 132,
three digits we can form 3! = 6 three digit 213, 231, 312, 321. If we need to find the sum
of all these 6 numbers then
Word/Number Formation  151
123 = 1 × 100 + 2 × 10 + 3 × 1 Then the sum of numbers formed
132 = 1 × 100 + 3 × 10 + 2 × 1 = sum of the digits at units place × 1
+ sum of the digits at tens place × 10
213 = 2 × 100 + 1 × 10 + 3 × 1
+ sum of the digits at hundreds place × 100
231 = 2 × 100 + 3 × 10 + 1 × 1
+………………………......……………………….
312 = 3 × 100 + 1 × 10 + 2 × 1 +……………………………………………………
321 = 3 × 100 + 2 × 10 + 1 × 1 or to summarize this sum of the numbers of
n digits formed with n different digits
Sum = 12 × 100 + 12 × 10 + 12 × 1
(Number of numbers formed ) ×
= 1332
=
(Sum of the digits)(1111…11 n times)
n
Alternately
Here sum of the digits at units place = 12 Example 10:  Find the sum of all the four
digit numbers formed by the digits 1,
Sum of the digits at tens place = 12 2, 3 and 4
Sum of the digits at hundreds place = 12 Solution:  Required summation is given by
(Number of numbers formed ) ×
Sum of the numbers formed
=
(Sum of the digits)(1111…11 n times)
= 12 × 1 + 12 × 10 + 12 × 100 = 1332 n
Thus in order to find the sum of the numbers (4!)(1 + 2 + 3 + 4 )(1111) = 24 × 1 0  × 1 111 =  66660
=
formed we must find the sum of the digits at 4 4
units, tens, hundreds places.

1
Concept Applicator (CA)
Ideal Time Apply your concepts with easy and
15 Min. conceptual questions

1. How many distinct words can be formed 3. How many distinct words can be
from all the letters of the word SOCIETY? formed from the letters of the word
ARRANGEMENT?
(a) 2(7!) (b) (7!)
(a) (11!)/(2!)(2!) (b) (11!)/(2!)
(c) 3(7!) (d) None of these
(c) (11!)/(8) (d) None of these
2. How many distinct words can be 4. How many distinct words can be formed
formed from the letters of the word from the letters of the word CREATIVITY
CREATIVITY? that starts with C?
(a) (10!)/(2!)(2!) (b) (10!)/(2!) (a) (10!)/(2!)(2!) (b) (9!)/(2!)(2!)
(c) (10!)/(4!) (d) None of these (c) (9!)/(2!)(2!)(2!) (d) None of these
152  Theory of Counting
5. How many distinct words can be formed 11. How many three digit numbers more
from the letters of the word CREATIVITY than 300 can be formed from the digits 1,
that starts with C and ends with Y? 2, 3, 4, 5 and 0?
[Repetition of digit is not allowed]
(a) (9!)/(2!)(2!)
(b) (8!)/(2!)(2!) (a) 60 (b) 120
(c) (8!)/(2!)(2!)(2!) (c) 80 (d) None of these
(d) None of these 12. How many numbers between 500 and
6. How many distinct words can be formed 5000 can be formed from the digits 0, 1, 3,
from the letters of the word CREATIVITY 5, 7, 9? [Repetition of digit is not allowed]
if all the vowels are together? (a) 600 (b) 180
(a) 12(7!) (b) 6(7!) (c) 160 (d) None of these
(c) (4!)(7!) (d) None of these
13. How many 4 digit even number can
7. How many distinct words can be formed
from the letters of the word CREATIVITY be formed from 0, 2, 3, 4, 5, 6 and 7?
if all the vowels are not together? [Repetition of digit is not allowed]
(a) 12(7!) (b) 6(7!) (a) 600 (b) 240
(c) (4!)(7!) (d) None of these (c) 440 (d) None of these
8. How many other words can be formed
from the letters of the word PATNA if 14. How many 4 digit odd number can
letter T is always at the middle? be formed from 0, 2, 3, 4, 5, 6 and 7?
(a) 12 (b) 11 [Repetition of digit is not allowed]
(c) 24 (d) None of these
(a) 300 (b) 220
9. How many 5 digit numbers can be
formed from the digits, 1, 2, 3, 4 and 5 if (c) 400 (d) None of these
repetition of digit is not allowed? 15. How many 4 digit numbers more than
(a) 112 (b) 120 2500 can be formed from 0, 1, 2, 3, 4, 5,
(c) 240 (d) None of these
and 6?
10. How many 4 digit numbers can be
formed from the digits, 1, 2, 3, 4 and 5 if [Repetition of digit is not allowed]
repetition of digit is not allowed?
(a) 600 (b) 520
(a) 120 (b) 121
(c) 240 (d) None of these (c) 400 (d) None of these

Response Grid
1. a b c d 2. a b c d 3. a b c d 4. a b c d 5. a b c d
6. a b c d 7. a b c d 8. a b c d 9. a b c d 10. a b c d
11. a b c d 12. a b c d 13. a b c d 14. a b c d 15. a b c d
154  Theory of Counting
12. How many numbers greater than 1000 17. How many numbers more than 4000 can
but not greater than 4000 can be formed be formed from the digits 0, 2, 4, and 6 if
with the digits 0, 1, 2, 3 and 4 if repetition repetition of digits is not allowed?
of digits being allowed? (a) 18 (b) 12
(a) 375 (b) 376 (c) 15 (d) None of these
(c) 360 (d) None of these 18. How many numbers less than 1000
13. How many 5 digit even numbers can be can be formed if repetition of digits is
formed from the digits 1, 2, 3, 4, and 5? not allowed and the number formed is
(a) 48 (b) 60 divisible by 5?
(c) 72 (d) None of these (Number should not be single digit)
14. How many 4 digit numbers are not (a) 168 (b) 162
divisible by 5? (c) 144 (d) None of these
(a) 4000 (b) 7200 19. How many 5 digit even number divisible
(c) 1800 (d) None of these by 5 can be formed from the digits 1, 2, 3,
15. How many 5 digit numbers greater than 4, 5, 6, 7, and 8
12345 can be formed from the digits 1, 2, (a) 1684 (b) 162
3, 4 and 5? (c) 1152 (d) None of these
(a) 59 (b) 71 20. Consider a set S = {0, 1, 2, 3, 4, 5},
(c) 95 (d) None of these positive integers are formed by taking
16. What is the number of 5 digit numbers not any number of digits from elements of
multiple of 5 formed only by the odd digits? set S. Total how many positive integers
[Repetition of digit is not allowed] can be formed?

(a) 59 (b) 96 (a) 1684 (b) 1612


(c) 95 (d) None of these (c) 1630 (d) None of these

Response Grid
1. a b c d 2. a b c d 3. a b c d 4. a b c d 5. a b c d
6. a b c d 7. a b c d 8. a b c d 9. a b c d 10. a b c d
11. a b c d 12. a b c d 13. a b c d 14. a b c d 15. a b c d
16. a b c d 17. a b c d 18. a b c d 19. a b c d 20. a b c d
154  Theory of Counting
10. How many 5 digit number divisible by (a) 59 (b) 71
125 can be formed from the digits 0, 1, 2, (c) 95 (d) None of these
3, 4, 5, 6 and 7? 16. What is the number of 5 digit numbers not
[Repetition of digit is not allowed] multiple of 5 formed only by the odd digits?
[Repetition of digit is not allowed]
(a) 72 (b) 84 (a) 59 (b) 96
(c) 88 (d) None of these (c) 95 (d) None of these
11. How many 5 digit number divisible by 25 17. How many numbers more than 4000 can
but not by 125 can be formed from the be formed from the digits 0, 2, 4, and 6 if
digits 0, 1, 2, 3, 4, 5, 6 and 7? repetition of digits is not allowed?
[Repetition of digit is not allowed] (a) 18 (b) 12
(c) 15 (d) None of these
(a) 232 (b) 244 18. How many numbers less than 1000
(c) 248 (d) None of these can be formed if repetition of digits is
12. How many numbers greater than 1000 not allowed and the number formed is
but not greater than 4000 can be formed divisible by 5?
with the digits 0, 1, 2, 3 and 4 if repetition (Number should not be single digit)
of digits being allowed? (a) 168 (b) 162
(a) 375 (b) 376 (c) 144 (d) None of these
(c) 360 (d) None of these 19. How many 5 digit even number divisible
by 5 can be formed from the digits 1, 2, 3,
13. How many 5 digit even numbers can be 4, 5, 6, 7, and 8
formed from the digits 1, 2, 3, 4, and 5? (a) 1684 (b) 162
(a) 48 (b) 60 (c) 1152 (d) None of these
(c) 72 (d) None of these 20. Consider a set S = {0, 1, 2, 3, 4, 5},
14. How many 4 digit numbers are not positive integers are formed by taking
divisible by 5? any number of digits from elements of
(a) 4000 (b) 7200 set S. Total how many positive integers
(c) 1800 (d) None of these can be formed?
15. How many 5 digit numbers greater than (a) 1684 (b) 1612
12345 can be formed from the digits 1, 2, (c) 1630 (d) None of these
3, 4 and 5?
Word/Number Formation  155

3
Concept Cracker (CC)
Ideal Time Boost up your confidence with good
60 Min. questions

1. Letters of the word MATHEMATICS are (c) (11!)/(2!)(2!)(2!)(3!)


arranged in all the possible ways, in how (d) None of these
many words letter C is between S and 5. Letters of the word MATHEMATICS are
H (these three letters are not necessary arranged in all the possible ways, in how
together)?
many words both the 1st and the last
(a) (11!)/(2!)(2!)(2!) letter is a vowel?
(b) (11!)/(3!)(2!)(2!) (a) (9!)/(2!)(2!)(3!)(3!)
(c) (11!)/(3!)(3!)(2!) (b) (9!)/(2!)(2!)(2!)
(d) None of these
(c) (9!)/(2!)(2!)(2!)(3!)
2. Letters of the word MATHEMATICS are (d) None of these
arranged in all the possible ways, in how 6. How many 4 letters word containing
many words letter C is exactly between S 2 vowels and 2 consonants can be
and H ? formed from the letters of the word
(a) (9!)/(3!)(2!)(2!) DEVASTATION?
(b) (9!)/(2!)(2!)(2!) (a) 1638 (b) 1635
(c) (9!)/(3!)(3!)(2!) (c) 1644 (d) None of these
(d) None of these 7. How many words can be formed from the
3. Letters of the word MATHEMATICS are letters of the word ACCIDENT without
arranged in all the possible ways, in how changing the relative order of vowels?
many words letter C is exactly between (a) 1680 (b) 3360
two M’s? (c) 2440 (d) None of these
(a) (9!)/(2!)(2!) 8. How many words can be formed from the
(b) (9!)/(3!)(2!)(2!) letters of the word ACCIDENT keeping
(c) (9!)/(3!)(3!)(2!) the position of each vowel fixed?
(d) None of these (a) 180 (b) 60
4. Letters of the word MATHEMATICS are (c) 120 (d) None of these
arranged in all the possible ways, in how 9. How many words can be formed from the
many words both the A’s are before both letters of the word ACCIDENT without
the M’s? changing the relative order of vowels/
(a) (11!)/(2!)(2!)(3!)(3!) consonants?
(b) (11!)/(2!)(2!)(2!) [A vowel can occupies only vowel place]
156  Theory of Counting
(a) 180 (b) 160 the 1st slot which can not be 0. One fine
(c) 360 (d) None of these day Rajesh forgot his answer key and
10. How many words can be formed from the only he remember is that no more than
letters of the word CINEMA such that it 2 distinct digits are used to lock the key.
neither starts with C nor ends with A? How many attempts he has to make to
(a) 504 (b) 696 certainly open the lock?
(c) 360 (d) None of these (a) 444 (b) 216
11. How many words can be formed from the (c) 576 (d) None of these
letters of the word CINEMA such that it 17. What is the number of 3 digit numbers
neither starts with C nor end with A and with distinct digits in which digits are in
E is not at the 3rd position? A.P?
(a) 504 (b) 426 (a) 112 (b) 121
(c) 480 (d) None of these (c) 111 (d) None of these
12. How many words can be formed from the 18. Find the sum of all the four digit numbers
letter of the word PROGRAMMING such formed by digits 1, 2, 3 and 4.
that it neither starts with M nor end with (a) 111600 (b) 666600
M? (c) 66660 (d) None of these
(a) (10)(9!)/2 (b) 8(9!) 19. Find the number of 5 digit numbers
(c) 9(9!) (d) None of these formed with 1, 2, 3, 4, and 5 such that at
13 If letters of the word CIRCLE are least 2 digits are identical.
arranged as in dictionary format, then (a) 3025 (b) 3005
what is the rank of the word CIRCLE? (c) 3115 (d) None of these
(a) 67 (b) 68 20. If P represents the number of 7 digit
(c) 66 (d) None of these numbers in which no two consecutive
14. What is the total number of nine digit digits are identical and Q represents the
numbers that can be formed if repetition number of 5 digit numbers in which no
of digits is not allowed? two consecutive digits are identical then
(a) 17(9!) (b) 9(9!) find the difference between P and Q.
(c) 8(9!) (d) None of these (a) 80(95) (b) 81(95)
(c) 82(95) (d) None of these
15. All the possible 5 digit numbers are
formed from the digits 0, 1, 2, 3, 4, 7 and 8 21. What is the sum of all the four digit
what percentage of this number is divisible number that can be formed with the
by 3 if repetition of digits is not allowed? digits 0, 1, 2, and 3.
(a) 41.45 (b) 34.44 (a) 38664 (b) 38564
(c) 33.333 (d) None of these (c) 38554 (d) None of these
16. Rajesh has a numeric lock that has 4 slots 22. What is the sum of all the four digit
and each slots can take any digit except number that can be formed with the
digits 1, 1, 2, and 3.
Word/Number Formation  157
(a) 23341 (b) 23331 (c) 32.77 (d) None of these
(c) 233331 (d) None of these 26. How many 5 digit numbers exist such
23. Find the number of +ve even numbers that its 3rd digit is the highest digit and
that can be formed from the digits 0, 3, no digit is repeated?
6, 8 and 9 if repetition of digits is not (a) (4!)(10C5) – (3!)(9C4)
allowed? (b) (5!)(10C5) – (4!)(9C4)
(a) 164 (b) 182 (c) (5!)(10C5) – (3!)(9C4)
(c) 162 (d) None of these (d) None of these
24. How many 7 digit numbers divisible by 9 27. What is the number of natural numbers
can be formed such that no digit is 0 and which are less than 200000000 and can
no digit is repeated? be formed by using digits 1 and 2?
(a) 4(9!) (b) 4(7!) (a) 225 (b) 510
(c) 9! – 7! (d) None of these (c) 766 (d) None of these
25. Susmit is arranging all the letters of 28. How many four digit number ‘abcd’ can
the word MATHEMATICS. In what be formed such that the digits are in
percentage of the total words he will get decreasing order?
either both A’s or both T’s together?
(a) 150 (b) 210
(a) 31.77 (b) 31.83
(c) 240 (d) None of these

Response Grid
1. a b c d 2. a b c d 3. a b c d 4. a b c d 5. a b c d
6. a b c d 7. a b c d 8. a b c d 9. a b c d 10. a b c d
11. a b c d 12. a b c d 13. a b c d 14. a b c d 15. a b c d
16. a b c d 17. a b c d 18. a b c d 19. a b c d 20. a b c d
21. a b c d 22. a b c d 23. a b c d 24. a b c d 25. a b c d
26. a b c d 27. a b c d 28. a b c d
158  Theory of Counting

Solutions
Concept Applicator (CA)
1. (b) 2. (a) 3. (d) 4. (b) 5. (b) 6. (b)
7. (d) 8. (b) 9. (b) 10. (a) 11. (a) 12. (b)
13. (d) 14. (a) 15. (b)

1. (b) The given word is SOCIETY, the 6. (b) The given word is CREATIVITY,
number of letters is 7 and these 7 the number of letters is 10 out of
letters can be arranged in 7! ways. these there are 2T’s, and 2I’s and
other vowels are E, and A. Now tie
2. (a) The given word is CREATIVITY, the
all the vowels together then it can
number of letters is 10. Out of these
be arranged in (4!)/(2!) ways. If we
there are 2T’s, and 2I’s so these 10 treat vowels as 1 unit then we will
letters can be arranged in (10!)/(2!) have 7 units out of them there are 2
(2!) ways. T’s which can be arranged in (7!)/(2)
3. (d) The given word is ARRANGEMENT, ways, so total number of ways is (4!)
the number of letters is 11. Out of (7!)/(2!)(2!) = 6(7!)
these there are 2A’s, 2R’s 2 N’s, 7. (d) Total number of words that can
and 2E’s and these 11 letters can be formed without restriction is
be arranged in (11!)/(2!)(2!)(2!)(2!) (10!)/(2!)(2!) from question number 4.
ways. Now if all the vowels are together
then number of words formed is
4. (b) The given word is CREATIVITY, the 6(7!).
number of letters is 10 out of these
So number of ways that all the
there are 2T’s, and 2I’s since 1st vowels are not together is (10!)/(2!)
letter is C then remaining 9 letters (2!) – 6(7!) = 180(7!) – 6(7!) = 174(7!)
can be arranged in (9!)/(2!)(2!)
8. (b) Since letter T is at middle so
ways. remaining 4 letters can be arranged
5. (b) The given word is CREATIVITY, the at 4 places in (4!)/(2!) ways = 12 ways
number of letters is 10 out of these out of these 1 word is PATNA itself
there are 2T’s, and 2I’s since 1st so 11 other words can be formed.
letter is C and last letter is Y then 9. (b) Given number of digits is 5, we
remaining 8 letters can be arranged can arrange them in 5! ways = 120
in (8!)/(2!)(2!) ways. ways.
Word/Number Formation  159
10. (a) From 5 digits we can select 4 in 5C4 can be filled up in 6, 5 and 4 ways
ways, and then we can arrange them respectively, so number of numbers
in (4!) hence total number of such in this case is 6 × 5 × 4 = 120.
number is (5C4)(4!) = 5 × 4 × 3 × 2
Case (ii) When unit’s digit is not 0,
= 120
then it can be filled up in 3 ways
11. (a) Since number is more than 300 (either 2, 4 or 6) and thousand’s,
hence Hundred’s digit can be filled hundred’s and ten’s place can be filled
up from 3, 4, and 5 i.e, in 3 ways. up in 5, 5 and 4 ways respectively,
Then remaining 2 places can be so number of numbers in this case is
filled up in 5 and 4 ways hence total 3 × 5 × 5 × 4 = 300
number numbers is 3 × 4 × 5 = 60.
Hence total number of such numbers
12. (b) Here we have two cases- is 120 + 300 = 420
Case (i) :- When 3 digit number is 14. (a) For an odd number unit digit must
formed- Hundred’s place can be be either 3, 5, or 7,
filled up in 3 ways(5, 7 and 9) then
Unit digit can be filled up in 3 ways
remaining 2 places can be filled up
(3, 5 or 7), then thousand’s, hundred’s
in 5 and 4 ways respectively, so
and ten’s place can be filled up in 5, 5
number of ways is 3 × 5 × 4 = 60.
and 4 ways respectively, so number
Number of number in this case
of numbers in this case is
is 60.
  3 × 5 × 5 × 4 = 300.
Case (ii) :- When 4 digit number is
formed- Thousand’s place can be 15. (b) Here we have two cases-
filled up in 2 ways (either 1 or 3) Case (i) If 2 is at thousand place then
then remaining 3 places can be filled hundred’s place can have either 5 or
up in 5, 4 and 3 ways respectively, 6 (2 ways) and remaining 2 places
so number of ways is 2 × 5 × 4 × 3 can be filled up in 5 and 4 ways, so
= 120. Number of number in this total number of numbers in this case
case is 120. is 2 × 5 × 4 = 40
Total number of such numbers is Case (ii) If 3, 4, 5 or 6 (in 4 ways) is
60 + 120 = 180 at thousand place then remaining 3
13. (d) Case (i) For an even number places can be filled up in 6, 5 and 4
unit digit must be either 0, 2, ways, so total number of numbers in
4, or 6, Now we have two cases- this case is 4 × 6 × 5 × 4 = 480
Case (i) When unit digit is 0, then So total number of numbers is
thousand’s, hundred’s and ten’s place 480 + 40 = 520
160  Theory of Counting

Concept Builder (CB)


1. (a) 2. (d) 3. (a) 4. (d) 5. (c) 6. (b)
7. (d) 8. (c) 9. (b) 10. (c) 11. (a) 12. (b)
13. (a) 14. (b) 15. (d) 16. (b) 17. (b) 18. (c)
19. (d) 20. (c)

1. (a) Given word is PERMUTATION it arranged in (4!)/(3!) = 4 ways.


has 11 letters out of them 5 vowels Hence total number of ways is
and 6 consonants out of which there 4 × 5 × 4 = 80 ways.
are 2T’s. 5. (c) The word NINETEEN has 3N’s, 3 E’s,
Since Vowels can take only vowels 1 I and 1T. There are 4 consonants
place. Hence 5 vowels can be arranged and four vowels.
in (5!) ways, and 6 consonants We have two cases-
can be arranged in (6!)/(2!) ways. Case (i) – When word starts with
Total number of such words is vowels then vowels will take odd
(5!)(6!)/2. places and consonants will take even
2. (d) The given word is ALGEBRA in this places and they can be arranged in
we have 7 letters comprising 3 vowels (4!)/(3!) × (4!)/(3!) = 4 × 4 = 16 ways.
that includes 2 A’s and 4 distinct Case (ii) – When word starts with
consonants. Vowels can occupy only consonants then consonants will take
even place means only at 2nd, 4th odd places and vowels will take even
and 6th place this can be done in places and they can be arranged in
(3!)/(2!) = 3 ways and 4 consonants (4!)/(3!) × (4!)/(3!) = 4 × 4 = 16 ways.
can be arranged in (4!) ways.
So total number of ways is
So total number of such words is
3(4!) = 72. 16 +16 = 32
3. (a) The word NINETEEN has 3N’s, 6. (b) Without restriction total number of
3 E’s, 1 I and 1T. words formed is (8!)/(3!)(3!)
We can arrange 3 E’s, 1 I and 1T in = (4 × 5 × 6 × 7 × 8)/(6) = 1120
(5!)/(3!) = 20 ways that will create 6 Number of words when all the vowels
spaces out of which 3 can be selected and consonants are together is
in 6C3 = 20 ways. (2!)(4!)(4!)/(3!)(3!) = 32
So total number of words is 20 × 20 = 400. So required number of ways is
4. (d) The word NINETEEN has 3N’s, 3 1120 – 32 = 1088
E’s, 1 I and 1T. 7. (d) The word NINETEEN has 3N’s,
There are 4 consonants and four 3 E’s, 1 I and 1T
vowels. 4 consonants can be Without restriction total number of
arranged in (4!)/(3!) = 4ways. This words formed is
will create 5 places in which we (8!)/(3!)(3!) = (4 × 5 × 6 × 7 × 8)/(6) = 1120
have to place 4 vowels. 4 places Number of words when all the vowels
can be selected out of 5 in 5C4 = 5 are together is {(4!)/(3!)}{(5!)/(3!)}
ways, and then 4 vowels can be = 4 × 20 = 80
Word/Number Formation  161
Number of words when all the Case (i) – If last three digits are
consonants are together is {(4!)/(3!)} 125 or 375, 625 then remaining two
{(5!)/(3!)} = 4 × 20 = 80 places can be filled up in 4 and 4
Number of words when all the vowels ways so number of ways is 3 × 4 × 4
and consonants are together is (2!) = 48
(4!)(4!)/(3!)(3!) = 32 Case (ii) – If last three digits are 250
So total number of words such that or 650 then remaining two places
either all the consonants or all the can be filled up in 5 and 4 ways so
vowels or both all the consonants number of ways is 2 × 5 × 4 = 40.
and all the vowels are together is So total number of numbers is
80 + 80 – 32 = 128 48 + 40 = 88
So required number of ways is 11. (a) From solution of last two questions
1120 – 128 = 992 is 320 – 88 = 232
8. (c) The word INTERMEDIATE has 12. (b) Here thousandth place can be filled
2 I’s, 3 E’s 2 T’s and remaining are up in 3 ways (1 or 2 or 3) then
single letters. remaining 3 places can be filled up
As per the given condition all the in 5 ways each, so total number of
consonants are together, We can numbers is 3 × 5 × 5 × 5 = 375
arrange all the consonants in Here number 4000 is also included
(6!)/(2!) ways, now we will treat so total number of numbers is 375 +
these consonants as one letter then 1 = 376
the number of words is [(6!)/(2!)][(7!)/ 13. (a) Unit digit can be either 2 or 4, then
(2!)(3!)] = (6!)(7!)/(2!)(2!)(3!) = (5!)(7!)/ remaining 4 digits can be arranged
(2!)(2!) in (4!) = 24 ways, so total number of
9. (b) A number is divisible by 25 if last number is 2 × 24 = 48
two digit is divisible by 25, hence 14. (b) Total number of 4 digit number is
last two digit must be either 25, 50, 9000, now we have to remove the
or 75 we will divide it in two cases: numbers that are multiple of 5.
Case (i) – If last two digits are 25 Since number of 4 digit number
or 75 then remaining three places multiple of 5 is 1800
can be filled up in 5, 5 and 4 ways
So number of 4 digit numbers not
so number of ways is 2 × 5 × 5 × 4
multiple of 5 is 9000 – 1800 = 7200
= 200.
15. (d) Total number of numbers formed
Case (ii) – If last two digits are 50
is (5!) = 120 out of 120 smallest
then remaining three places can
number is 12345, hence remaining
be filled up in 6, 5 and 4 ways so
119 numbers are more than 12345.
number of ways is 6 × 5 × 4 = 120.
16. (b) Number of such numbers is
So total number of numbers is
200 + 120 = 320 4 × 4 × 3 × 2 = 96
st
17. (b) 1 place can be filled up in 2 ways
10. (c) A number is divisible by 125 if last
three digit is divisible by 125, hence then remaining 3 places can be filled
last three digit must be either 125, up in 3! = 6 ways.
250, 375, 625, or 750, we will divide Hence required number of ways
it in two cases- is 12.
162  Theory of Counting
18. (c) We have two cases Case (ii) if number formed is 2
Case (i) If unit digit is 0 then digit number then number of such
remaining two places can be filled numbers is 5 × 5 = 25
up in 9 × 8 = 72 ways. Case (iii) if number formed is 3
Case (i) If unit digit is 5 then digit number then number of such
remaining two places can be filled numbers is 5 × 5 × 4 = 100
up in 9 × 8 = 72 ways. Case (iv) if number formed is 4
So total number of numbers is digit number then number of such
72 + 72 = 144 numbers is 5 × 5 × 4 × 3 = 300
19. (d) If a number is even and also divisible Case (v) if number formed is 5
by 5 then it must be divisible by 10 digit number then number of such
or unit digit must be 0, so from the
numbers is 5 × 5 × 4 × 3 × 2 = 600.
given digits we can not form any
such number. Case (vi) Number of 6 digit number
20. (c) We have 5 cases here- 5 × 5 × 4 × 3 × 2 = 600.
Case (i) if number formed is 1 digit Hence total number of numbers is
number then number of such number 600 + 600 + 300 + 100 + 25 + 5 = 1630
is 5.

Concept Cracker (CC)


1. (b) 2. (d) 3. (a) 4. (c) 5. (d) 6. (a)
7. (b) 8. (b) 9. (c) 10. (a) 11. (b) 12. (c)
13. (b) 14. (b) 15. (b) 16. (c) 17. (a) 18. (c)
19. (b) 20. (a) 21. (a) 22. (b) 23. (c) 24. (b)
25. (c) 26. (a) 27. (c) 28. (b)

1.
(b) Letters of the word MATHEMATICS 3. (a) Lets the letters C, and 2 M’s together,
are 2 M’s, 2 Ts, 2 A’s and one each H, then they can be arranged in only 1
C, S, E, & I total number of words way. Now we will treat C, and 2M’s
that can be formed is as one unit, now total number of
(11!)/(2!)(2!)(2!) arrangements is
Now consider the letters C, S and H [(9!)/(2!)(2!)] = [(9!)/(2!)(2!)]
they can be arranged in (3!) = 6 ways
out of these 6 only in two ways C is 4. (c) Letters of the word MATHEMATICS
between S and H. are 2 M’s, 2 Ts, 2 A’s and one each H,
C, S, E, & I total number of words that
So required number of ways is [(11!)/
(2!)(2!)(2!)][(2!)/(3!)] = (11!)/(3!)(2!)(2!) can be formed is (11!)/(2!)(2!)(2!)
Now consider 2 M’s and 2 A’s these
2. (d) Lets the letters C, S and H together,
four letters can be arranged in
then out of (3!) = 6 ways only in two
(4!)/(2!)(2!) ways = 6 ways, out of these
ways C is between S and H. then we 6 ways there is only one way where
will treat C, S and H as one unit, both the A’s are before both the M’s, so
now total number of arrangements required number of ways is [(11!)/(2!)
is [(9!)/(2!)(2!)(2!)][(2!)] = [(9!)/(2!)(2!)] (2!)(2!)][1/6] = (11!)/(2!)(2!)(2!)(3!)
Word/Number Formation  163
5. (d) Case (i)- Letters of the word 7. (b) Given word ACCIDENT has 3
MATHEMATICS are 2 M’s, 2 Ts, distinct vowels, and 5 consonant
2 A’s and one each H, C, S, E, & I, where only C is twice.
out of these vowels are 2 A’s, one Total number of arrangements of
E and one I. Here we have 3 cases- these 8 letters is (8!)/(2!) Now consider
Case (i) – when both 1st and last vowels they can be arranged in
letter is A then remaining 9 letters 3! = 6 ways and out of these 6 ways
can be arranged in (9!)/(2!)(2!) ways. there is only one way where they
Case (ii)- When one of the letter is A will be in the given order hence total
and one letter is either E or I, then number of such words is
they can be arranged in (2!) ways and = 1/6 × (8!)/(2!) = 3360
remaining 9 letters can be arranged
in (9!)/(2!)(2!) ways. 8. (b) If position of vowels is fixed then
5 consonants can be arranged in
Hence total number of ways is (9!)/(2!).
(5!)/(2!) ways = 60 ways.
Case (iii) When neither 1st nor last
9. (c) Since Consonants can be arranged
letter is A then 1st or last letter can
in (5!)/2 = 60 ways while vowels
be E or I and they can be arrange in
can be arranged in 3! = 6 ways, so
(2!) ways, then remaining 9 letters
total number of ways is 6 × 60 = 360
can be arranged in (9!)/(2!)(2!)(2!)
ways.
ways hence total number of ways is
(9!)/(2!)(2!) 10. (a) Number of letters in the word
Total number of ways CINEMA is 6, without any restriction
we can form (6!) = 720 number of words.
(9!)/(2!)(2!) + (9!)/(2!) + (9!)/(2!)(2!) Number of words that starts with
= (9!)/(2!)(2!) [ 1 + 2+1] = (9!) C is (5!) = 120 Number of words
6. (a) The word DEVASTATION has 2 T’s, that ends with A is (5!) = 120
2 A’s and remaining 1 letters, total Number of words that starts with C
number of letters is 11. We have as well as end with A is (4!) = 24
following cases- Number of words that either starts
with C or end with A is
Case 2 T’s (1 way) 2 A’s (1 (4!)/(2!)(2!)
(i) way) =6 120 + 120 – 24 = 216
Case 2 T’s ( 1 way) 2 distinct 6 × (4!)/(2!) Number of words that neither starts
(ii) vowels = 72 with C nor end with A is 720 – 216
(4 C 2 = 6 = 504
ways) 11. (b) Number of letters in the word
Case 2 distinct 2 A’s (1 10 × (4!)/(2!) CINEMA is 6, without any restriction
(iii) consonants way) = 120 we can form (6!) = 720 number of
(5C2 = 10 words. Number of words that starts
ways) with C is (5!) = 120. Number of words
that ends with A is (5!) = 120
Case 2 distinct 2 distinct (10×6)(4!)
Number of words that has E
(iv) consonants vowels = 1440
at the 3rd position is (5!) = 120
(5C2 = 10 (4 C 2 = 6
Number of words that starts with C
ways) ways)
as well as end with A is (4!) = 24
Total 1638
Number of words that starts with
number
C as well as E at the 3rd position is
of ways
(4!) = 24
164  Theory of Counting
Number of words that ends with A 14. (b) At 1st place we can not place 0 hence
as well as E at the 3rd position is it can be filled up in 9 ways then for
(4!) = 24 remaining 8 places we have to select
Number of words that starts with C, 8 digits from available 9 digits that
E at the 3rd position and end with A can be done in 9C8 ways or in 9 ways
is (3!) = 6 , now these 8 digits can be arranged
in (8!) ways, hence total number of
Number of words that either starts such numbers is (9)(9)(8!) = (9)(9!)
with C or E at the middle or end
with A is 15. (b) Number of 5 digit numbers formed
by the given digits is
120+120 + 120 – 24 – 24 – 24 + 6 = 294
6 × 6 × 5 × 4 × 3 = 2160
Number of words that neither starts
with C nor end with A is Now we have to find the number of
720 – 294 = 426 5 digit number divisible by 3 and
for that 1st we have to select the
12. (c) Given word is PROGRAMMING it required digits.
has 2 R’s, 2G’s 2 M’s and remaining
distinct letters, total number of Case (i) if digit 0 is included then we
letters is 11. have 4 such combinations and these
are (0, 1, 2, 7, 8), (0, 1, 2, 4, 8), (0, 2,
Total number of words formed is 4, 7,8) and (0, 1, 3, 7, 8) and in each
(11!)/(2!)(2!)(2!) case we will get 4 × 4 × 3 × 2 = 96
Number of words that starts with M numbers so total number of numbers
is (10!)/(2!)(2!) in this case is 4 × 96 = 384.
Number of words that ends with M Case (ii) if digit 0 is not included then
is (10!)/(2!)(2!) we have 3 such combinations and these
Number of words that starts with M are (2, 3, 4, 7, 8), (1, 2, 3, 7, 8), (1, 2, 3, 4, 8)
and end with M is (9!)/(2!)(2!) and in each case we will get 5 × 4 × 3
So number of words that neither × 2 = 120 numbers so total number
starts with M nor end with M is of numbers in this case is
(11!)/(2!)(2!)(2!) – [(10!)/(2!)(2!) + 3 × 120 = 360.
(10!)/(2!)(2!) – (9!)/(2!)(2!)] Total number of such numbers is
= (9!)/(2!)(2!) [10 × 11/2 – 10 – 10 + 1] 360 + 384= 744
= (36)(9!)/(2!)(2!) = 9(9!) Required percentage is
13. (b) In dictionary the 1st word will start 744 × 100/2160 = 34.44 %
with CC, and the number of words 16. (c) Here we will take two cases-
that starts with CC is 4! = 24, then
number of words that starts with CE Case (i) If 0 is one of the digit and
is 4! = 24. used only once then possible numbers
in the form of (aaa0, aa0a, a0aa) and
Then we will get CIC and number of a can take 9 values hence number of
words is 3! = 6, similarly number of numbers in this case is 3 × 9 = 27.
words that starts with CIE is 3! = 6,
number of words that starts with Case(ii) If 0 is one of the digit and
CIL is 3! = 6. used twice then possible numbers in
the form of (aa00, a00a, a0a0) and a
Then next word will be CIRCEL and can take 9 values hence number of
finally we will get CIRCLE.
numbers in this case is 3 × 9 = 27.
Hence rank of CIRCLE is 68.
Word/Number Formation  165
Case(iii) If 0 is one of the digit and 18. (c) Number of numbers formed is
used thrice then possible numbers (4!) = 24
in the form of (a000) and a can take Sum of digits = 1+2+3+4 = 10
9 values hence number of numbers
So required summation is
in this case is 9.
(24 × 10)(1111)/4 = 66660
Case(iv) if only one digit is used then
number of numbers is 9. 19. (b) Total number of 5 digit numbers
from the given condition is 55 = 3125,
Case (v) if two digits a and b are
out of these the numbers number of
used then possible numbers are
5 digit numbers in which no digit is
(aaab, aaba, abaa, baaa) number of
similar is 5!, hence required number
numbers in this case is
of number is 3125 – 120 = 3005
(9C2)(2!)(4) = 36 × 2 × 4 = 288
20. (a) Consider P it is the number of 7 digit
Case (vi) if two digits a and b are numbers in which no two consecutive
used then possible numbers are digits are identical hence 1st place
(aabb, abba, abab, bbaa, baab, baba) can be filled up in 9 ways (as 0 can
number of numbers in this case is not be the 1st digit) then next place
(9C2)(6) = 36 × 6 = 216 can filled up in 9 ways and so on
So total number of numbers is hence P = 97
27 + 27 + 9 + 9 + 288 + 216 = 576
Similarly Q = 95 and
17. (a) Let d be the common difference of the
7 5 5 5
A.P then we have following cases- P – Q =  9 – 9 =  (81 − 1)9 =  80(9 )
Case (i) if d = 1 then possible pair of 21. (a) Number of numbers that can be
digits are (1, 2,3), (2, 3, 4), (3, 4, 5) and formed with the digits is (4!) = 24
so on till (7, 8, 9) total 7 such sets. [ here 0 can be at 1st place]
Case (ii) if d = 2 then possible set of Then
digits are (1, 3, 5), (2, 4, 6) (3, 5, 7),
(4, 6, 8), (5, 7, 9) total 5 such sets Sum of numbers =
Case (iii) if d = 3 then possible set of (Number of numbers formed) ×
digits are (1, 4, 7). (2, 5, 8), (3, 6, 9) (Sum of digits)(1111..n times)

total 3 such sets. Number of digits
Case (iv) if d = 4 the only possible set = (24 × 6 × 1111)/4
is (1, 5, 9)
Now sum of all the numbers that can
So total possible sets is 7+5+3+1 be formed by the digits 1, 2 and 3 is
= 16 and in each set we have
(6)(6 × 111)
(3!) = 6 numbers hence total number
of numbers is 16 × 6 = 96. 3
Required summation is
Now consider if 0 is one of the
24 × 6 × 1111 (6)(6 × 111)
number then possible sets are – = 38664
(0, 1, 2), (0, 2, 4), (0, 3, 6), (0, 4, 8) and 4 3
from each set we can form 4 numbers 22. (b) Number of numbers that can be
hence number of numbers is 4 × 4 = 16
formed with the digits is (4!)/2 = 12
So total number of 3 digit numbers
Then
is 96 + 16 = 112
166  Theory of Counting
Sum of numbers = So total number of words is
(Number of Numbers formed) × 11!
.
(Sum of digits) × (1111..n times) ( )(2!)(2!)
2!
Number of digits Now, we have to find the number of
12 × 7 × 1111 words in which either both A or both
= T are together,
4
Number of words in which both the
= 23331
10!
23. (c) We have 5 cases here- A’s are together is
Case (i) if number formed is 1 digit (2!)(2!)
number then number of such number Number of words in which both the
is 2. 10!
T’s are together is
Case (ii) if number formed is 2 (2!)(2!)
digit number then number of such Number of words in which both the
numbers is 4 + 6 = 10. A’s are together and both the T’s are
Case (iii) if number formed is 3 9!
together is
digit number then number of such 2!
numbers is 12 + 18 = 30. Hence Number of words in which
Case (iii) if number formed is 4 either both A’s or both T’s are together
digit number then number of such 10! 10! 9! 9 (9!)
+ – =
numbers is 24 + 36 = 60. (2!)(2!) (2!)(2!) 2! 2
Case (iii) if number formed is 5
Required percentage is
digit number then number of such
numbers is 24 + 36 = 60.  9 (9!)
 
Hence total number of numbers is  2 
100 = 360/11= 32.77%
2 + 10 + 30 + 60 + 60 = 162.  11! 
24. (b) Since number is divisible by 9, hence  
 (2!)(2!)(2!)
sum of digits should be divisible
by 9 26. (a) Number of ways of selecting 5 digits
Sum of 1 + 2 + 3 +…8 + 9 = 45 so we is 10C5
have to remove two digits from these We have to form a 5 digit number
9 digits whose summation is 9. The whose 3rd digit is the highest, hence
pair of numbers that are left is (1, 8), out of 5 selected digits the place of
(2, 7), (3, 6) and (4, 5) highest digit is fixed so remaining
And remaining 7 digits we can form 4 can be arranged in (4!) ways so
(7!) numbers hence total number of number of ways is (4!)(10C5).
numbers is 4(7!) But in this case we have to subtract
25. (c) Total number of letters in the word the number of numbers when 1st
MATHEMATICS is 1 1 and out of digit is 0 (as it will be a 4 digit
these 11 – number).
M – 2 , T – 2, A – 2, H – 1, I – 1, C – 1 So number of ways of forming a
and S – 1 number with its 1st digit as 0 and
Word/Number Formation  167
with the given condition is (3!)(9C4) Case (viii) :- Consider a eight digit
So required number of numbers is number:- Each place can be filled up
(4!)(10C5) – (3!)(9C4) with two choices (either 1 or 2) so
number of numbers formed is 28.
27. (c) We have following cases-
Case (ix):- Consider a nine digit
Case (i) :- Consider a single digit
number:- except the 1st place (which
number:- 1, and 2 so only 2 numbers
is occupied with 2) remaining each
in this case.
place can be filled up with two
Case (ii) :- Consider a two digit choices (either 1 or 2) so number of
number:- 11, 12, 21 and 22 so only 4 numbers formed is 28.
= 22 numbers in this case.
So total number of numbers formed
Case (iii) :- Consider a three digit is 2 + 22 + 23 +…+ 28 + 28 = 766
number:- Each place can be filled up
28. (b) Out of 10 digits 4 digits can be
with two choices (either 1 or 2) so
selected in 10C4 = 210 ways.
number of numbers formed is 23.
Similarly
168  Theory of Counting
Part B

Concept 1 Formation of Word

Word/number formation question is one of the (p!) hence if we replace p identical articles
most important type of questions. To solve by p distinct articles then the number of
these type of questions we have to understand permutations will become (p!)(x). Similarly
the arrangement of identical articles. if we replace q identical article by q distinct
The number of arrangements of n articles articles then the number of permutations will
taken all at a time when p of them are become (p!)(q!)(x) and if we replace r identical
identical and are of one type, q of them are
article by r distinct articles then the number
identical and of second type, r of them are
identical and of third type, and remaining are of permutations will become (p!)(q!)(r!)(x)
n! which is equal to (n!)
all of different is .
{ }
(p!)(q!)(r!) Hence (p!)(q!)(r!)(x) = n! or x =
n!
To understand this let us assume that required { }
(p!)(q!)(r!)
number of permutations is x, Since p different
things can be arranged among themselves in Number formation is very much similar to

Concept 2 Formation of Number

word formation the only difference is that at 2- All even number is divisible by 2, or the
1st place we can not put ‘0’ as example if it is numbers end with and even number or zero is
given that we have to form a 4 digit number divisible by 2,
3- A number is divisible by 3 if sum of its digits
‘abcd’ then the digit ‘a’ can not be 0. Variation
is divisible by 3.
in the question may occur if some property
4- A number is divisible by if 4 if the number
of number is given as example divisible by formed by the last two digits of the number is
2 or 5 etc. To understand this just revise the divisible by 4, or last two digits are 0’s.
divisibility rule- 5- All numbers that end with 0 or 5 are
divisible by 5.
Word/Number Formation  169

6- A number that is divisible by 2 and 3 both 12- A number is divisible by 12 when it is


are divisible by 6. divisible by 3 and 4 both.
8- A number divisible by 8 if number formed
16- A number is divisible by 16 when number
by three digits of the number is divisible by 8
or last three digits are zeroes. formed by last 4 digits of the given number is
9- A number is divisible by 9 if sum of its digits divisible by 16 or last 4 digits are zeroes.
is divisible by 9.
7 and 13- A number is divisible by 7 or 13 if
11- A number is divisible by 11 if the difference
and only if the difference of the number of its
between the sum of the digits in the odd and
the sum of the digits in the even places is thousands and the remainder of its division
either 0 or multiple of 11. by 1000 is divisible by 7 or 13.

Concept 3 Rank of a Word in Dictionary

Rank of a word is the position of that word, ABC → 1ST word


when we arrange the words formed by ACB→ 2nd Word
alphabets of that given word in dictionary
BAC → 3rd Word
order. Lets see an example.
BCA → 4th Word
To understand this type of problem just
assume that we have only 3 letters A, B and C CAB → 5th Word
in the English alphabets then the dictionary CBA → 6th Word
will have total meaning full or meaning less So if I ask you what is the rank of word CAB
3! = 6 words and their order is- the answer is 5th.

Concept 4 Sum of all the Numbers Formed

Sum of the numbers of n digits formed with n ( Number  of  numbers  formed ) ×
different digits (Sum  of  the  digits )(1111…11 ntimes
  )
=
n
170  Theory of Counting

Concept 5 Number of Palindrome Number

A palindromic number is a number that is the The numbers of palindromic numbers less
same when written forwards or backwards, i.e., than 1000 = 103 is 108
The numbers of palindromic numbers less
of the form aba etc. The first few palindromic
than 10000 = 104 is 198
numbers are as follows- 0, 1, 2, 3, 4, 5, 6, 7, 8,
9, 11, 22, 33, 44, 55, 66, 77, 88, 99, 101, 111, Number of palindromic numbers less than
121, 131, ... . 10n is
The numbers of palindromic numbers less  n 
2  10 2 – 1 if n is even
than 10- is 9  
The numbers of palindromic numbers less n −1
than 100 = 102 is 18 11 × 10 2 – 2 if n is odd

1
Concept Deviator (CD)
Ideal Time Revise your concept (IIT advance or
5 Min. tougher questions)
Per Question

Direction (Qs. 1 and 2): 3. Consider two n digit numbers P and Q


All the 7 digit numbers that are not divisible such that P > Q, If ‘K’ represents the
by 5 are formed from the digits 1, 2, 3, 4, 5, number of pairs of P and Q such that
6, and 7, then these numbers are arranged in Q can be subtracted from P without
the increasing order. borrowing. Then how many of the
following statements are correct?
1. Total how many digits are written in this
process? (i) Highest power of 5 in K is n–1
(a) 8(7!) (b) 7(7!) (ii) Highest power of 3 in K is 2n
(c) 6(7!) (d) None of these (iii) Unit digit of K is 5
2. What will be the 14000th digit? (a) Only (i) & (ii) (b) Only (ii) & (iii)
(a) 3 (b) 6 (c) All three (d) None of these
(c) 7 (d) None of these
Word/Number Formation  171
4. How many 9 digit number can be formed (a) 16 (b) 36
from the digits 1 to 9 without repetition (c) 60 (d) 180
of digits such that pattern ‘123’, ‘456’ or 11. The number of arrangements of the letters
‘789’ do not appear ? of the word BANANA in which the two
(a) 348120 (b) 348114 N’s do not appear adjacently is:
(c) 348234 (d) None of these [IIT JEE 2002]
5. How many words of six letters can be (a) 40 (b) 60
formed from the three letters of the word (c) 80 (d) 100
HINDUSTAN if each letter is used at
12. The letters of the word COCHIN are
least once?
permuted and all the permutations are
(a) 15120 (b) 30296 arranged in an alphabetical order as
(c) 30240 (d) None of these in an English dictionary. The number
of words that appear before the word
6. How many 5 digit numbers can be formed
COCHIN is [IIT JEE 2007]
with repetition of digits is allowed and
digits are in non increasing order from (a) 360 (b) 192
left to right it is given that non of the (c) 96 (d) 48
digits is 0? 13. Ten different letters of an alphabet are
(a) 59 (b) 13C given. Words with five letters are formed
5
from the given letters. Then the number
(c) 59 – 49 (d) None of these of words which have at least one letter
7. Find the sum of all the three digit repeated as: [IIT JEE 1980]
numbers that has only odd digits. (a) 69760 (b) 30240
(a) 29875 (b) 489375 (c) 99748 (d) None of these
(c) 69375 (d) None of these 14. A five digit number divisible by 3 is to be
8. Find the sum of all the three digit formed using the numbers 0,1,2,3, 4 and
5, without repetition. The total number of
numbers that has only even digits (0, 2,
ways this can be done is: [IIT JEE 1989]
4, 6 and 8).
(a) 216 (b) 240
(a) 54440 (b) 54000
(c) 600 (d) 3125
(c) 54400 (d) None of these
15. Consider all possible permutations of the
9. An n- digit number is a positive with letters of the word ENDEANOEL
exactly n digits. Nine hundred distinct n- Match the following:
digit numbers are to be formed using only
the three digits 2,5 and 7. The smallest A- The number of (p) 5!
value of n for which this is possible, is permutations con-
equal to- [IIT JEE 1998] taining word EN-
DEA is
(a) 6 (b) 7 B- The number of (q) 2 × 5!
(c) 8 (d) 9 permutations in
10. How many different nine digits numbers which the letter E
can be formed the number 223355888 by occurs in which the
letter E occurs in the
rearranging its digits so that the odd digits
1st and the last posi-
occupy even positions?   [IIT JEE 2000]
tion is
172  Theory of Counting

C- The number of (r)7 × 5! 16. The number of seven digit integers, with
permutations in sum of the digits equal to 10 and formed
which D, L and N by using the digits 1, 2 and 3 only is
occurs in the last 5
positions is [IIT JEE 2009]
(a) 55 (b) 66
D- The number of (s) 21× 5!
permutations in (c) 77 (d) 88
which the letters A, 17. How many numbers greater than 1000 but
E, O occur only in
not greater than 4000 can be formed with
odd position
the digits 0, 1, 2, 3 and 4 if repetition of
[IIT JEE 2008] digits being allowed?   [IIT JEE 1965]
(a) A-p; B- q; C-r; D-s
(b) A-p; B- r; C-q; D-s (a) 375 (b) 400
(c) A-p; B- q; C-s; D-r (c) 425 (d) None of these
(d) None of these

Response Grid
1. a b c d 2. a b c d 3. a b c d 4. a b c d 5. a b c d
6. a b c d 7. a b c d 8. a b c d 9. a b c d 10. a b c d
11. a b c d 12. a b c d 13. a b c d 14. a b c d 15. a b c d
16. a b c d 17. a b c d

2
Concept Eliminator (CE)
Ideal Time Revise your concept (Maths Olympiad
Just solve it or tougher questions)

1. This question is based on Chinese Maths How many 7 digit numbers can be
Olympiad: formed by using only the digits 5 and 7
Consider a 101 digit number N = such that number formed is divisible by
pa1a2a3…a99a100 where p is a prime digit both 5 and 7?
and ai is any digit. How many numbers (a) 24 (b) 9
N can be formed such that digit 9 is used (c) 48 (d) None of these
odd number of times? 3. This question is based on Maths
(a) 2(10100 – 8100) (b) 1/2(10100–8100) Olympiad:
(c) (10100 – 8100) (d) None of these All possible 6 digit numbers, in each of
2. This question is based on Maths which the digits occur in non increasing
Olympiad: order (From left to right e.g 877550)
Word/Number Formation  173
are written as a sequence in increasing the nine places get different colours.
order. Find the 2005th number in this Suppose the ticket bearing the number
sequence. 1222222222 is red and that bearing the
(a) 864110 (b) 864000 number 222222222 is green. Determine,
the colour of of the ticket bearing the
(c) 864100 (d) None of these
number 123123123.
4. This question is based on Regional Maths
Olympiad: (a) Green (b) Red
Find the number of all 5-digit numbers
(c) Blue (d) None of these
each of which contains the block 15 and
is divisible by 15? (Example of such 8. This question is based on Regional Maths
number is 31545, 34515 etc) Olympiad:-
(a) 479 (b) 487 Find the number of 4 digit numbers
(c) 480 (d) None of these having non – zero digits and which are
5. This question is based on Regional Maths divisible by 4 but not by 8.
Olympiad:-
Find the number of all 6-digit numbers (a) 729 (b) 209
that satisfy the following conditions- (c) 734 (d) None of these
The digits of each number are all from
9. Find the number of 6 digit palindrome
the set { 1, 2, 3, 4, 5}
number.
Any digit that appears in the number
appears at least twice (eg- 225252 is (a) 10998 (b) 1998
acceptable but 222133 is not) (c) 9898 (d) None of these
(a) 1405 (b) 1400
(c) 1200 (d) None of these 10. Fifteen coupons are numbered 1, 2, 3,. .
. . 15. In how many ways seven coupons
6. This question is based on Regional Maths
are selected such that the largest number
Olympiad:-
appearing on the selected coupon is 9?
Find the sum of all the 3 digit natural
numbers which contain at least one odd (a) 97 – 87 (b) 107 – 87
digit and one even digit. (c) 87 – 77 (d) None of these
(a) 494550 (b) 370775 11. This question is based on Regional Maths
(c) 69375 (d) None of these Olympiad:-
7. This question is based on Maths Find the number of 6 digit natural
Olympiad:- numbers such that sum of their digits is
In a lottery, tickets are given nine – 10 and digits 0, 1, 2 and 3 occur at least
digit numbers using only the digits once in them.
1, 2, 3. They are also coloured red, (a) 190 (b) 490
blue or green in such a way that two
(c) 240 (d) None of these
tickets whose numbers differ in all

Response Grid
1. a b c d 2. a b c d 3. a b c d 4. a b c d 5. a b c d
6. a b c d 7. a b c d 8. a b c d 9. a b c d 10. a b c d
11. a b c d
174  Theory of Counting

Solutions
Concept Deviator (CD)
1. (c) 2. (d) 3. (d) 4. (b) 5. (c) 6. (b)
7. (c) 8. (c) 9. (b) 10. (c) 11. (a) 12. (c)
13. (a) 14. (a) 15. (d) 16. (c) 17. (a)

1. (c) Total number of numbers formed is 4315267, 4315276, 4315627, 4315672


6(6!) and in each number we have 7 Hence 14000th digit is 2
digits hence total number of digits is 3. (d) As per the given condition each
6(7!) digit of P is more than or equal to
2. (d) Since one number has 7 digits so we corresponding digit of Q.
have to find out 14000/7 = 2000th Consider the 1st digit, let 1st digit of
number P is p1 and that of Q is q1 as per the
Number of numbers that starts with given condition p1 ≥ q1
1 is 5(5!) = 600 If p1 = 9 then q1 can take any value
from 1 to 9 so 9 ways
Number of numbers that starts with
If p1 = 8 then q1 can take any value
2 is 5(5!) = 600
from 1 to 8 so 8 ways and so on-
Number of numbers that starts with If p1 = 2 then q1 can take any value
3 is 5(5!) = 600 from 1 to 2 so 2 ways
Till now we have formed 600 If p1 = 1 then q1 is 1 so 1 way.
+ 600 + 600 = 1800 numbers Total number of ways is 1+2+…
Number of numbers that starts with +8+9 = 45 [or else 10C2 = 45 ways]
41 is 4(4!) = 96 Similarly from 2nd to nth digit for
Number of numbers that starts with each place we have 1+2+3+…+9+10
42 is 4(4!) = 96 = 55 ways (as 0 is included)
Till now we have formed 600 + 600 [11C2 = 55 ways].
+ 600 + 96 + 96 = 1992 numbers, So total number of ways is K = (45)
now we need only 8 numbers. (55)n–1
Number of numbers that starts with Highest power of 5 in K is n and that
4312 is 4. Till now we have formed of 3 is 2.
1992 + 4 = 1996 numbers, now we 4. (b) Total number of numbers that can
need only 4 numbers and these be formed without restriction is 9!
numbers starts with 4315 hence Number of numbers that contain
these numbers are- pattern ‘123’ is 7!
Word/Number Formation  175
Number of numbers that contain of digits is allowed and digits are in
pattern ‘456’ is 7! non increasing order is n+r–1Cr
Number of numbers that contain Number of 5 digit numbers that can
pattern ‘789 is 7! be formed is 5+9–1C5 = 13C5
Number of numbers that contain 7. (c) The digits are from the set {1, 3, 5, 7, 9}.
pattern ‘123’ and ‘456’ is 5! Suppose the digit in unit’s place
Number of numbers that contain is 1. Then we have 5 ways for
pattern ‘123’ and ‘789, is 5! each of ten’s and hundred place.
Number of numbers that contain So there are 25 numbers that has
pattern ‘456’ and ‘789’ is 5! unit digit 1. Similarly, there are
Number of numbers that contain 25 numbers whose unit digit 3, 5,
pattern ‘123’ ‘456’ and ‘789’ is 3! 7 and 9 respectively. Thus the sum
of the digits in unit’s place of all the
So required number of numbers is 9!
numbers is
– 3(7!) + 3(5!) – 3! = 362880 – 15120
+ 360 – 6 = 348114 25(1 + 3 + 5 + 7 + 9) = 25 × 25 = 625.
5. (c) The given word is HINDUSTAN similarly the sum of digits in ten’s
which has H – 1, I – 1, N – 2, D – 1, place of all the numbers is 625 and
U – 1, S – 1, T – 1, A – 1, that in hundred’s place is also 625.
Thus the sum of all the numbers is
This word has 8 distinct letters so
three letters can be selected in 8C3 625(100 + 10 + 1) = 625 × 111 = 69375.
ways let three selected letters is H, I Alternately :
and N We can use the formula directly-
 
Total number of words that can be
formed without restriction is 36 (Number of numbers formed )×
Sum of all
Total number of words when H is
the numbers =
(Sum of digits)(111..1ntimes)
not present is 26 similarly number Number of digits
of words when I and N is not present
is 26 for both. Required summation is
Number of words when H & I is (125)(1 + 3 + 5 + 7 + 9)(111) = (125)(25)(111)
absent, when H & N is absent and 5 5
when I & N is absent is 16 = 1 for
= 69375
all.
Number of words when H I and N is 8. (c) In this case the digit in hundred’s
absent is 0. place is never 0. Suppose the digit
Number of words when H I and N is in unit’s place is 0. There are 4 ×
used at least once is 5 = 20 such numbers. Similarly,
20 numbers each having digits 2, 4,
36 – 3(26) + 3(16) – 0 = 540
6, 8 in their unit’s place. Thus the
Total number of ways is sum of the digits in unit’s place of all
(8C3)(540) = 30240 the numbers is
6. (b) Number of ways of writing r digit 20(0 + 2 + 4 + 6 + 8) = 20 × 20 = 400.
number from n digits with repetition
176  Theory of Counting
Similarly the sum of the digits in Number of words starts with CN is
ten’s place is 400, but the sum of the 4! = 24
digits in hundred’s place of all the Next word is the given word COCHIN
numbers is 25 × 20 = 500. Thus the
So required number is 24 × 4 = 96
sum of all the numbers is
13. (a) Without restriction number of 5
500 × 100 + 400 × 10 + 400 = 54400.
letter words is 105 = 100000
9. (b) Number of ‘n’ digit numbers formed
Number of 5 letters word with no
by digits 2, 5 and 7 is 3n as per the
repetition is (10C5)(5!) = (10P5)
given condition 3n > 900 so least
value of n is 7 = 10 × 9 × 8 × 7 × 6 = 30240

10. (c) The given digits are 223355888, So required number of numbers is
odd digits are 3, 5, 5 and 5, as per 100000 – 30240 = 69760
the given condition these odd digits 14. (a) Here we have two cases-
should occupy even positions i.e 2nd, Case (i) If digits are 1, 2, 3, 4 and 5 so
4th, 6th and 8th position, number of 5 digit numbers formed is
So odd digits can arranged in 5! = 120
4! Case (ii) if digits are 0, 1, 2, 4 and 5 so
=  6
(2!)(2!) ways number of 5 digit numbers formed is
4(4!) = 96
And even digits can arranged in
5! So total number of numbers is
=  10 ways 120+96 = 216
( )(2!)
3!
Total number of ways is 6 × 10 = 60 15. (d) The given word is ENDEANOEL
now consider each case separately-
11. (a) Total numbers of words formed is
(6!)/(3!)(2!) = 60 The number of permutations
containing word ENDEA is 5! Since
Number of words in which both the
the word ENDEA is treated as
N’s are together is (5!)/(3!) = 20
one character and then remaining
So required number is 60-20 =40 4 letters along with this can be
Alternately the number must be less arranged in 5! Ways.
than 60 and only one option is less The number of permutations in
than 60 i.e 40 is the answer. which the letter E occurs in the 1st
12. (c) The given word is COCHIN, if we and the last position is- 21 × 5! Since
arrange them in alphabetical order the required value is 2 × 5!
then C, C, H, I N, O The number of permutations in
Number of words starts with CC is which D, L and N occurs in the last
4! = 24 5 positions is {(6!)/(2!)}(2!)}
Number of words starts with CH is The number of permutations in
4! = 24 which the letters A, E, O occur only
Number of words starts with CI is 4!  5!   4! 
in odd position is     =  2 ×  5 !
= 24  3!   2! 
Word/Number Formation  177
16. (c) Let number of 1’s 2’s and 3’s are used 7 digit number is (7!)/(4!)(3!) = 35
is a, b and c then a + 2b + 3c = 10 So total number of numbers is 42+35
And a + b + c = 7 = 77
We have following cases- 17. (a) 1st place can be filled up by either 1,
Case (i) (a,b,c) = (5,1,1) then number of 2, or 3 i.e, in 3 ways and remaining
7 digit number is (7!)/(5!) = 42 places can be filled up in 5 ways
hence total number of numbers is
Case (ii) (a,b,c) = (4,3,0) then number of
3 × 5 × 5 × 5 = 375

Concept Eliminator (CE)

1. (a) 2. (b) 3. (a) 4. (a) 5. (a) 6. (b)


7. (b) 8. (a) 9. (d) 10. (a) 11. (b)

1. (a) P can be selected in 4 ways we have to replace 7 by 5 and then


Number of numbers when 9 is used new number should be divisible by 7.
once is 4(100C1)(999) When 5 divided by 7 remainder is 5
When 50 divided by 7 remainder is 1
Number of numbers when 9 is used
When 500 divided by 7 remainder is 3
thrice is 4(100C3)(997)
When 5000 divided by 7 remainder
Similarly number of numbers when is 2
9 is used 99 times is 4(100C99)(9)
When 500000 divided by 7 remainder
So total number of numbers formed is 6
is 4[ (100C1)(999) + (100C3)(997)+…+ When 5000000 divided by 7
(100C99)(9)] remainder is 4
We know that 10100 = (9+1)100 Number is divisible by 7 if sum of 2
=(100C0)(9100)+ (100C1)(999) + (100C3) or more of the remainders above is
divisible by 7.
(997)+…+ (100C99)(9) + (100C100)(1)]
Since 5+2 = 7, so we can replace
And 8100 = (9-1)100 =(100C0)(9100) 7 and 7000 by 5 and 5000 and the
- (100C1)(999) + (100C3)(997)+…- new number is 7775775, applying
( C99)(9) + (100C100)(1)]
100 the similar logic we have following
numbers that satisfy above
Hence 10100 – 8100 = 2[(100C1)(999) +
condition-
(100C3)(997)+…+ (100C99)(9)]
7775775, 7757575, 5577775,
So required number of numbers is 7575575, 5777555, 7755755,
4[1/2(10100 – 8100) = 2(10100 – 8100) 5755575, 5557755, 7555555
2.
(b) A number is divisible by 5 if its unit So total 9 such numbers exist.
digit is 0 or 5, in this case unit digit 3. (a) Since number of ways of writing
is 5. r digit number from n digits with
repetition of digits and digits are in
Now consider a 7 digit number
non increasing order is n+r-1Cr
7777777 it is divisible by 7, in this
178  Theory of Counting

Number Number of
Digits that can be
starting n r numbers is Total
used n+r–1C
with r
1 1, 0 2 5 6C =6 6
5
2 2, 1, 0 3 5 7C = 21 27
5
3 3,2,1,0 4 5 8C = 56 83
5
4 4,3,2,1,0 5 5 9C = 126 209
5
5 5,4,3,2,1,0 6 5 10C = 252 461
5
6 6,5,4,3,2,1,0 7 5 11C = 462 923
5
7 7, 6,5,4,3,2,1,0 8 12C
5 = 792 1715
From 800000 10C
5,4,3,2,1,0 6 5 = 252 1967
to 855555 5

From 860000 7C
3210 4 4 = 35 2002
to 863333 4

Next 2003rd number is 864000, a, b such that a+b gives remainder 1


2004th number is 864100, when divided by 3 or else number of
2 digit number ‘ab’ gives remainder
2005th number is 864110
1 when divided by 3 which is 30, but
4. (a)
If a number is divisible by 15 then out of these 30 numbers consider
it must be divisible by 3 and 5 so we numbers 31515, 61515 and 91515
have following cases- we have already counted in case (i)
Case (i) Number is in the form of abc15, so actual number of numbers in this
here a ≠ 0, so we have to find number case is 30 – 3 = 27.
of 3 digit number ‘abc’ divisible by 3 Case (vi) Number is in the form of 15ab0
which is 300. so we have to find pair a , b such that
Case (ii) Number is in the form of ab150 a+b is divisible by 3 which is 34 but
here a ≠ 0, so we have to find number out of these 34 numbers consider
of 2 digit number ‘ab’ divisible by 3 numbers 15015, 15315, 15615 and
which is 30 15915 we have already counted in
Case (iii) Number is in the form of ab155 case (i) so actual number of numbers
here a ≠ 0, so we have to find number in this case is 34 – 4 = 30
of 2 digit number ‘ab’ divisible by 3 Case (vii) Number is in the form of
which is 30 15ab5, so we have to find pair a , b
Case (iv) Number is in the form of a15b0 such that a+b is gives remainder 1
here a ≠ 0, so we have to find pair a , when divided by 3 which is 33, but
b such that a+b is divisible by 3 or in out of these 34 numbers consider
other words number of 2 digit number numbers 15155 we have already
‘ab’ divisible by 3 which is 30 counted in case (iii) so actual number
of numbers in this case is 33 – 1 = 32
Case (v) Number is in the form of a15b5
here a ≠ 0, so we have to find pair So total number of numbers is 300 +
30 + 30 + 30 + 27 + 30 + 32 = 479
Word/Number Formation  179
5. (a) We have following cases- interchange and these numbers can
be arranged in (6!)/(4!)(2!) = 15 ways,
Case (i) If number is made up with three
so number of numbers in this case is
digits occurring twice. Three digits
2×15×10 = 300.
can be selected in 5C3 = 10 ways and
these numbers can be arranged in Case (iv) When all the digits are same:-
(6!)/(2!)(2!)(2!) = 90 ways, so number we will get 5 such numbers.
of numbers in this case is Total number of numbers is
90 × 10 = 900. 900 + 200 + 300 + 5 = 1405
6. (b) To find the summation of the given
Case (ii) If number is made up with two situation refer to question number 7
digits occurring thrice. Two digits and 8 of Concept Deviator (CD)
can be selected in 5C2 = 10 ways and
these numbers can be arranged in Required summation is the sum of
(6!)/(3!)(3!) = 20 ways, so number of all the 3 digit numbers minus the
numbers in this case is answers of question number 7 and 8
of Concept Deviator (CD)
20 × 10 = 200. Sum of all the three digit numbers is
Case (iii) If number is made up with two 999 × 1000 99 × 100
– =  494550
digits, one digit occurs 4 times while 2 2
other occurs 2 times. Two digits So required summation is
can be selected in 5C2 = 10 ways 494550 – 69375 – 54400 = 370775
then there are two ways they can

7. (b) The following sequence can be inferred –

Number Ticket No Colour Reason

1 122222222 Red Given

2 222222222 Green Given

3 313113113 blue Each digit is different from 1st & 2nd

4 231331331 Green Each digit is different from 1st & 3rd

5 331331331 Blue Each digit is different from 1st & 2nd

6 123123123 Red Each digit is different from 4th & 5th

Hence required colour is Red.


180  Theory of Counting
8.
(a) We have following 4 cases – 9. (d) Number of palindromic numbers
Case (i): Assume that number ends less than 10n is
with 8, then number must be in  n 
the form of ab28, ab48, ab68, ab88 2  10 2 – 1 if n is even
for numbers ab28 and ab48, b must  
be one of the following (2, 4, 6 & 8) n −1
while for numbers ab68 and ab88 b 11 × 10 2 – 2 if n is odd
must be one of the following (1, 3, 5,
7, 9) hence the number of numbers Number of palindromic number less
in this case is (9 × 4 × 2)+(9 × 5 × 2) than 107 is 11 × 1000 – 2 = 10998
= 72+90 = 162
Number of palindromic number less
Case (ii): Assume that number ends than 106 is 2 × 999 = 1998
with 6, then number must be in the
form of ab16, ab36, ab56, ab76, and So required number is 10998 – 1998
ab96 for numbers ab36 and ab76, b = 9000
must be one of the following (2, 4, 6 & 10. (a) Since 7 coupons numbered from 1 to
8) while for numbers ab16, ab56 and 9 so that ‘9’ is selected at least once.
ab76, b must be one of the following Thus total number of favourable
(1, 3, 5, 7, 9) hence the number of ways are, 97– 87
numbers in this case is (9 × 4 × 2)+(9
× 5 × 3) = 72+135 = 207 ⇒ Required probability = 97 – 87
Case (iii): Assume that number ends 11. (b) Since sum of the digits is 10 and
with 4, then number must be in the 0+1+2+3 = 6 so sum of the remaining
form of ab24, ab44, ab64, and ab84, two digits must be 4 and possible
for numbers ab24 and ab64, b must combination for this is (0, 4), (1, 3)
be one of the following (1, 3, 5, 7, 9) and (2,2)
while for numbers ab44, and ab84, b
must be one of the following (2, 4, 6 So we have following three cases-
& 8) hence the number of numbers Case (i): If digits {0, 0, 1,2,3,4} so number
in this case is (9 × 5 × 2)+(9 × 4 × 2) of numbers in this case is 4×(5!)/2 = 240
= 90+72 = 162
Case (iv): Assume that number ends Case (ii): If digits {0, 1, 1, 2, 3, 3} so
with 2, then number must be in the number of numbers in this case is
form of ab12, ab32, ab52, ab72, and (5!)/(2!)(2!) + 2(5!)/2! = 150
ab92 for numbers ab32 and ab72,
b must be one of the following (1, When 1st digit is 2 then number of
3, 5, 7, 9) while for numbers ab12, numbers is (5!)/(2!)(2!) and when 1st digit
ab52 and ab72, b must be one of the is either 1 or 3 then number of numbers
following (2, 4, 6 & 8) hence the is 2(5!)/2!
number of numbers in this case is (9 Case (iii): If digits {0, 1,2, 2, 2,3} so
× 5 × 2)+(9 × 4 × 3) = 90+108 = 207 number of numbers in this case is
So total number of numbers 2(5!)/(3!) +(5!)/2 = 40+60= 100
formed is 162+207+162+198 = 729
Option (a) So total number of numbers is
240 + 150 + 100= 490
7
Distribution  181

Distribution

Topics Covered
 Partitioning Theory.

 Distribution of Distinct Articles.

 Distribution in Groups.

 Stanly’s 12 fold of distribution.

Part A: Topic Number of Questions


Solved Example 26
Concept Applicator 10
Concept Builder 10
Concept Cracker 30
Part B: Topic Number of Questions
Concept Deviator 4
Concept Eliminator 5
Total 85
182  Theory of Counting
Part A

Concept 1 Partitioning Theory

Distribution of ‘n’ articles into ‘r’ groups: In this case 1st student gets 0, 2nd gets 7 and
In this type of questions we have different 3rd student gets 7 chocolates.
cases that depends on whether ‘n’ articles are C C C C P P C C C C C C
identical or distinct, similarly ‘r’ groups are In this case 1st student gets 4, 2nd gets 0 and
identical or distinct. We will discuss each and 3rd student gets 6 chocolates.
every case separately:-
In this way we have many more cases but
Case (i) [Partitioning Theory]: Number of the question is how many cases? If we look at
ways of dividing ‘n’ identical things among carefully then actually we are arranging 10
‘r’ distinct groups or ‘r’ persons when each identical C’s and 2 identical P’s in a straight line
group can get any number of things ≤ n is
(n+r-1)C
and that can be done in (10 + 2)! = 10+3 –1C3 –1
r-1 (10!)(2!)
Proof: Let us assume that we have to Now lets generalize it when we have
distribute 10 identical chocolates among ‘n’ identical chocolates and we have to
three students, to understand this condition distribute it to ‘r’ students.
just assume that these 10 Chocolates are kept Again arrange ‘n’ chocolates in a straight line-
on a table
C C C C C C C C ……………C C
C C C C C C C C C C
In order to divide it to ‘r’ students we have
Here ‘C’ denotes the chocolates, to divide to put (r – 1) Ps so the situation is similar to
it in three parts we have to do only two arrange ‘n’ identical Cs and (r – 1) identical
partition, Let us assume position of (n + r − 1)!
partition is as shown below (for one case) Ps, this can be done in (r − 1)!(n)! = n+r–1Cr–1
C C C P C C C P C C C C
Example 1:  In how many ways 10 identical
Here ‘P’ denotes the point from where we have
chocolates can be distributed among 4
done partitioning. In this case 1st student gets
students?
3, 2nd gets 3 and 3rd student gets 4 chocolates.
In order to understand this lets see few more Solution:  Here we have n = 10 and r = 4 hence
cases- required number of ways is
(10+4–1)C 13C ways.
P C C C C C C C P C C C 4–1 = 3

QR Code / Video Link

For Smart phone/ Tablet users

Video Link for Desktop/Laptops users http://dishapublication.com/video-resources


Distribution  183
Example 2:  In how many ways 10 identical 1st we will give 1 chocolate to each student
chocolates can be distributed among 4 then we will have 6 remaining chocolates
students if each student should get at now we have n = 6 and r = 4.
least 1 chocolate? Hence required number of ways is
Solution:  As per the given condition each (6+4–1)C4–1 = 9C3 ways.
student should get at least 1 chocolate so

Concept 2 Distribution of Distinct Article

Distribution of ‘n’ distinct articles to then next is what we are sending? It is rings
‘r’ distinct groups: The number of ways of and the number of rings is 10 hence from
distributing ‘n’ distinct articles to ‘r’ distinct (Where)(What) formula required number of
group is rn. ways is 510
1st article can be dealt in r ways, 2nd article
can be dealt in r ways and so on each article Arrangements in groups
can be dealt in r ways. • The number of ways in which n different
Hence number of ways is rn things can be arranged into r different
group (when a particular group may be
Example 3:  In how many ways 10 distinct
empty) is r(r+1)(r+2)…(r+n–1)
rings can be send to 5 distinct boxes?
(r + n –1)!
Solution:  1string can be dealt in 5 ways, =
(r -1)!
2ndring again in 5 ways and so on hence
total number of ways is 5×5×5…×5 = 510 • The number of ways in which n different
things can be arranged into r different
In general students used to get confuse that
whether 510or 105 is the correct answer, to group (when no group is empty) is n – 1Cr – 1
get rid of this confusion we can remember 1 • The number of ways in which n different
simple formula (where)(What). things can be distributed into r different
In this case ‘Where’ we are sending the groups (when no group is empty) is
articles? It is boxes and number of boxes is 5,
rn – rC1(r–1)n +….. + (–1)r – 1(rCr – 1)

QR Code / Video Link

For Smart phone/ Tablet users

Video Link for Desktop/Laptops users http://dishapublication.com/video-resources


184  Theory of Counting
Example 4:  In how many ways 10 distinct Similarly 3rd ring has 7 ways and so on-
rings can be worn on 5 fingers?
So, the total number of way of distribution
Solution:  Consider 1string it can go in any of rings is = 5 × 6 × 7 × ... × 14 = (14!)/(4!)
of the five fingers, so it has 5 ways to deal
Alternately:
with. 2nd ring can go in any of the five
fingers but it has six ways. Since arrangements of ‘n’ distinct articles to
‘r; distinct groups when order in a group is
Let us assume that 1st ring is worn on 4th (n+r – 1)!
finger then for 2nd ring there are two places important:
(r – 1)!
on 4th finger one before 1st ring and one
In this case n = 10 and r = 5.
after 1st ring.
So required number of ways is (14!)/(4!)

Concept 3 Distribution in Groups

Type 1: Number of ways of dividing (m+n) Solution:  From the above formula the
distinct articles in two groups containing m 12!
and n articles respectively (m ≠ n) is (m+nCn) required number of ways is
(6!)(6!)(2)
{(m + n)!} Type 3: Number of ways of dividing 2m
(mCm) = (m+nCm)(nCn) = {(m !)(n !)}
distinct articles between two persons such
Example 5:  In how many ways 12 distinct {(2m)!}
that each of them get m articles is .
cricket balls can be distributed in two {(m !)(m !)}
groups having 5 and 7 cricket balls? Example 7:  In how many ways 12 distinct
Solution:  From the above
formula the cricket balls can be distributed
12! between two friends Rishita and
required number of ways is Sinjini such that each of them getting
(5!)(7!)
6 cricket balls?

Type 2: Number of ways of dividing 2m Solution:  From the above


formula the
distinct articles in two groups containing m 12!
{(2m)!} required number of ways is
(6!)(6!)
articles in each group
{(m !)(m !)(2!)} Type 4: Number of ways of dividing 3m
Example 6:  In how many ways 12 distinct distinct articles in three groups containing m
cricket balls can be distributed in two {(3m)!}
articles in each group
groups each having 6 cricket balls? {(m !)(m !)(m !)(3!)}
Distribution  185
Example 8:  In how many ways 12 distinct Example 11:  In how many ways 20 distinct
cricket balls can be distributed in three cricket balls can be distributed in 5
groups each having 4 cricket balls? groups such that the groups has 2, 3, 4,
5 and 6 cricket balls?
Solution:  From the above
formula the Solution:  From the above formula the required
12! 20!
required number of ways is number of ways is
(4!)(4!)(4!)(3!) (2!)(3!)(4!)(5!)(6!)
Type 5: Number of ways of dividing 3m distinct Type 8: Number of ways in which n distinct
articles among three persons such that each objects can be distributed among r persons
{(3m)!} such that some person get a1 objects, another
of them get m articles is . person get a2 objects ......... and similarly
{(m !)(m !)(m !)} someone gets ar objects
Example 9:  In how many ways 12 distinct n n − a1
C a2 ) × ( n − a1 − a2 C aa ) × ...…
= ( C a1 ) × (
cricket balls can be distributed among 3

three friends Rishita, Trina and (n!)(r!)


=
Sinjini such that each of them getting (a1!)(a2!)(a3!)…(ar!)
4 cricket balls? Here a1 + a2 + a3 + ...... + ar = n.
Solution:  From the above
formula the Example 12:  In how many ways 20 distinct
12! cricket balls can be distributed in
required number of ways is among 5 students such that they get 2,
(4!)(4!)(4!)
3, 4, 5 and 6 cricket balls?
Type 6: Number of ways of dividing (m+n+p) Solution:  From the above formula the required
distinct articles in three groups containing m, (20!)(5!)
{(m + n + p)!} number of ways is
n, p articles in three groups is (2!)(3!)(4!)(5!)(6!)
{(m !)(n !)( p !)} Type 9: Number of ways in which m × n
distinct objects can be divided equally into n
Example 10:  In how many ways 12 distinct
(mn)!
cricket balls can be distributed in groups (unmarked) = .
three groups such that the groups has (m !)n (n !)
3, 4 and 5 cricket balls? Example 13:  In how many ways 20 distinct
Solution:  From the above formula the required cricket balls can be distributed in 4
12! groups such that each group has 5
number of ways is cricket balls?
(3!)(4!)(5!)
Solution:  From the above formula the
Type 7: Number of ways, in which n distinct required number of ways is
objects can be divided into r unequal groups
(20!) (20!)
containing a1, a2, a3, ......,ar things (ai ≠ aj) =
n − a1
(5!)(5!)(5!)(5!)(4!) (5!)4 (4!)
n
= ( C a1 ) × ( C a2 ) × ( n − a1 − a2 C aa ) × ...
3 Type 10: Number of ways in which m × n
n! different object can be distributed equally
=
a1! a2! a3! … ar! among n persons (or numbered groups)  =
Here a1 + a2 + a3 + ...... + ar = n. (number of ways of dividing) × (number of
groups)! = (mn)!n!/(m!)n n! = (mn)!/(m!)n
186  Theory of Counting
Stanley’s “12-fold way” for Distribution
Consider a question: How many ways can r pieces of Chocolates be distributed among nboxes?
Answer of this question depends on following factors-
1. Chocolates are Identical or Distinct.
2. Boxes are identical or distinct.
3. Is there any restriction on number of chocolates in a box?
Answer of all these can be given by following tables-

Number of Number of
Chocolates
Boxes (n) No restriction Chocolates in a Chocolates in a
(r)
bag ≤ 1 bag ≥ 1

nr n! (n!) S(r, n)
Distinct Distinct (n – r)!
Case (i) Case (ii) Case (iii)
nC r–1C
n+r–1C r n–1
Identical r–1
Distinct
Case (iv) Case (v) Case (vi)
n 1 if r ≤ n
S(r, n)
∑ S(r, k) 0 if r >n
Distinct Identical k =1
Case (vii) Case (viii) Case (ix)
n 1 if r ≤ n
Pn(r)
Identical Identical
∑ Pk(x) 0 if r >n
k =1
Case (x) Case (xi) Case (xii)
Here S(r, n) is sterling number and is defined as-
1 r
S(n, r) = [ C0 (rn) – rC1 (r – 1)n + rC2(r – 2)n – …. + (–1)r–1(rCr–1)(1n)]
r!
And S(n, 1) = 1,
S(n, 2) = 2n–1 – 1 , S(n, n–1) = nC2, S(n, n) = 1
Pn(r) represents the number of ways r identical chocolates can be placed into 4
identical bags such that no bag is empty.
Since there is only 1 way to express the number n as the sum of 1 number
(namely, n = n), so P1(n) = 1.
Similarly, there is only 1 way to express the number n as the sum of n numbers (namely,
n = 1 + 1 + 1 + … + 1), so Pn(n) = 1.
Consider, Pn – 1(n) = 1, since there will be 1 bag with 2 Chocolates and the rest of the bags will
have 1 chocolates each.
n – 1
In general, when n is even, P2(n) = n/2, and when n is odd, P2(n) = .
2
P2(8) = 4, and P2(11) = 5.
Distribution  187
Example 14:  In how many ways 20 distinct Example 18:  In how many ways 5 distinct
cricket balls can be distributed among chocolates can be distributed to 10
4 students such that each student get distinct bags such that each bag gets
5 cricket balls? at least 1 chocolates ?
Solution:  From the above formula the required Solution:  This case is similar to distributing
(20!) (20!) r distinct articles to n distinct groups
number of ways is = such that each bag gets at most chocolates,
(5!)(5!)(5!)(5!) (5!)4 this is explain in column 5 and row 2 (Case
Example 15:  In how many ways a pack iii) of ‘12-fold way” table.
of 52 cards be divided into 4 sets such
Since number of chocolates are less than
that each set has 13 cards?
the number of bags hence number of ways
Solution:  Required number of ways is is zero.
52!
Example 19:  In how many ways 5 identical
(13!)4 (4!) chocolates can be distributed to 10
Example 16:  In how many ways 5 distinct distinct bags if there is no restriction?
chocolates can be distributed to 10 Solution:  This case is similar to distributing
distinct bags with no restrictions? r identicalarticles to n distinct groups this
Solution:  This case is similar to distributing r is explain in column 3 and row 3 (Case iv)
distinct articles to n distinct groups without of ‘12-fold way” table, that can be done in
n+r–1C 10+5–1C
any restriction, this is explain in column 3 r–1 = 10–1
and row 2 (Case i) of ‘12-fold way” table. 14 14! 14 × 13 × 12 × 11 × 10
= C9 = =
So total number of ways is nr = 105 = 100000 (9!)(5!) 1×2×3×4×5
ways. = 2002 ways.
Example 17:  In how many ways 5 distinct
Example 20:  In how many ways 5 identical
chocolates can be distributed to 10 distinct
chocolates can be distributed to 10
bags such that each bag gets at most 1
distinct bags if a bag can not have
chocolates ? (Refer Stanley’s Table)
more than 1 chocolate?
Solution:  This case is similar to distributing
Solution:  This case is similar to distributing
r distinct articles to n distinct groups
such that each bag gets less than or equal r identical articles to n distinct groups
to 1 chocolates, this is explain in column 4 such that if a bag can not have more than
1 chocolate this is explain in column 3 and
and row 2 (Case ii) of ‘12-fold way” table.
row 4 (Case v) of ‘12-fold way” table,
10! 10!
So total number of ways is = Number of ways is nCr = 10C5
(10 − 5)! 5!
10! 10 × 9 × 8 × 7 × 6
= 10 × 9 × 8 × 7 × 6 = 30240 ways. = = = 252 ways
(5!)(5!) 1 × 2 × 3 × 4 × 5
Another explanation of this can be given as
since one bag can have maximum 1 chocolates Alternate way to solve is that select 5 bag out
hence we have to select just 5 bags out of 10 of 10 and that can be done in 10C5 ways and
bags and then we will put 1 chocolate in those since chocolates are identical hence we don’t
5 bags, this can be done in (10C5)(5!) have to arrange it, so number of ways is 10C5
10! 10! 10! 10 × 9 × 8 × 7 × 6
= (10P5) = = = = = 252 ways
(10 − 5)! 5! (5!)(5!) 1 × 2 × 3 × 4 × 5
= 10 × 9 × 8 × 7 × 6 = 30240 ways.
188  Theory of Counting
Example 21:  In how many ways 5 identical this is explain in column 4 and row 4 (Case
chocolates can be distributed to 10 viii) of ‘12-fold way” table,
distinct bags if a bag must have at Since number of chocolates is less than the
least 1 chocolate?
number of bags hence number of ways is 1.
Solution:  This case is similar to distributing
Example 24:  In how many ways 5 distinct
r identical articles to n distinct groups
chocolates can be distributed to 10
such that if a bag can not have more than
1 chocolate this is explain in column 4 and identical bags if each bag gets at least
row 4 (Case vi) of ‘12-fold way” table, 1 chocolate?
Number of ways is 0 since number of Solution:  This case is similar to distributing
chocolates is less than number of bags. r distinct articles to n identical groups this
is explain in column 4 and row 4 (Case ix) of
Example 22:  In how many ways 5 distinct ‘12-fold way” table,
chocolates can be distributed to 10 From the table required number of ways is
identical bags? S(5, 10) = 0
Solution:  This case is similar to distributing
Since number of chocolates is less than the
r distinct articles to n identical groups this
number of bags hence number of ways is zero.
is explain in column 4 and row 4 (Case vii)
of ‘12-fold way” table, Example 25:  In how many ways 5 identical
Number of ways is S(5,1) + S(5, 2) + S(5, 3) chocolates can be distributed to 10
+ … + S(5, 9) + S(5,10) identical bags?
1 r Solution:  This case is similar to distributing
Since S(n, r) = [ C0 (rn) – rC1(r – 1)n + r distinct articles to n identical groups this
r! is explain in column 4 and row 4 (Case x) of
rC (r – 2)n – …. + (–1)r–1(rC )(1n)]
2 r–1 ‘12-fold way” table,
And S(n, 1) = 1, S(n, 2) = 2n – 1 – 1 , S(n, n – 1) Since Pn(r) represents the number of ways
= nC2, S(n, n) = 1 r identical chocolates can be placed into
4 identical bags such that no bags is empty.
Here in this case required number of ways
P1(5) = 1, P2(5) = 2,
is S(5,1) + S(5,2) + S(5,3) + S(5,4) + S(5,5)
P3(5) = 2 ways as can be seen (1, 1, 3) (1, 2, 2)
Since S(5,1) = 1, S(5,2) = 24 – 1 = 15, S(5, 4) P4(5) = 1 ways as can be seen (1, 1, 1,2)
= 5C2 = 10 and S(5,5) = 1 so only we need to P5(5) = 1 ways as can be seen (1, 1, 1, 1, 1)
find is S(5,3) Total number of ways is 1+2+2+1+1 = 7
From the formula S(5,3)=1/3! Example 26:  In how many ways 5 identical
243 −96 +  3 chocolates can be distributed to 10
[3C0(35) – 3C1(25)+3C2(15)] = = 25
6 identical bags if each bag gets at least
So required number of ways is 1+15 + 25 + 1 chocolate?
10 + 1 = 52 Solution:  This case is similar to distributing
Example 23:  In how many ways 5 distinct r distinct articles to n identical groups this
is explain in column 4 and row 4 (Case xii)
chocolates can be distributed to 10
of ‘12-fold way” table,
identical bags if each bag gets at most
From the table required number of ways is
1 chocolate?
P10(5) = 0
Solution:  This case is similar to distributing Since number of chocolates is less than the
r distinct articles to n identical groups number of bags hence number of ways is zero.
Distribution  189

1
Concept Applicator (CA)
Ideal Time Apply your concepts with easy and
10 Min. conceptual questions

1. In how many ways 20 identical chocolates 6. In how many ways 20 distinct chocolates
be distributed among 5 students? be distributed among 5 students?
(a) 24C5 (b) 24C4 (a) 520 (b) 24C
4
25
(c) C4 (d) None of these
(c) 205 (d) None of these
2. In how many ways 20 identical chocolates
7. In how many ways 20 distinct chocolates
be distributed among 5 students such
be distributed among 5 students such
that a particular student Rohan receive
that a particular student Rohan receive
at least 1 chocolate?
at least 1 chocolate?
(a) 24C5 (b) 24C4
(a) 5(2019) (b) 20(519)
(c) 25C
4 (d) None of these 23C
(c) 4 (d) None of these
3. In how many ways 20 identical chocolates
be distributed among 5 students such that 8. In how many ways 20 students can be
each student receive at least 1 chocolate? send to 4 class rooms?
24C 19C (a) 420 (b) 23C3
(a) 5 (b) 4
25C (c) 204 (d) None of these
(c) 4 (d) None of these
9. Ricky has 10 friends to invite, in how
4. In how many ways 20 identical chocolates many ways can he send the invitation
be distributed among 5 students such cards to them if there are three different
that each student receive at least 2 courier services are available?
chocolates?
(a) 103 (b) 13C3
(a) 14C4 (b) 19C4 10
(c) 3 (d) None of these
(c) 19C
5 (d) None of these 10. In how many ways 15 different articles
5. In how many ways 20 identical chocolates can be distributed equally among 3
be distributed among 5 students such persons?
that each student receive at least 3 15! 15!
chocolates? (a) (3)(5!)3 (b) (5!)3
(a) 19C 19C
5 (b) 4
15!
(c) 9C
4 (d) None of these (c) (3!)(5!)3 (d) None of these

Response Grid
1. a b c d 2. a b c d 3. a b c d 4. a b c d 5. a b c d
6. a b c d 7. a b c d 8. a b c d 9. a b c d 10. a b c d
190  Theory of Counting

2
Concept Builder (CB)
Ideal Time Revise your concepts with questions
15 Min. medium difficulty level questions
1. In how many ways 50 grapes can be 52!
distributed to 5 students? (c) (3!)(17!)3 (d) None of these
(a) 54C5 (b) 54C4
24
(c) 5 (d) None of these
7. In how many ways 225 distinct choco-
2. In how many ways 50 grapes can be lates can be distributed equally to 15
distributed to 5 students such that each
students?
of them receives at least 5 grapes?
(a) 49C5 (b) 29C4 225! 225!
29
(a) (b)
(c) C5 (d) None of these (15!)2! (15!)15
3. In how many ways 50 grapes can be 225!
distributed to 5 students such that 1st (c) (d) None of these
student get at least 1, 2nd student get at (15!)14!
least 2 and so on 5th student get at least 8. In how many ways 50 distinct chocolates
5 grapes? can be distributed equally among 10
(a) 39C4 (b) 29C4 identical boxes?
(c) 29C5 (d) None of these 50! 50!
(a) (b)
4. In how many ways 50 grapes can be 5
{(10!) }(5!) {(10!)5 }
distributed to 6 students such that each
of them receives at least 8 grapes? 50!
(c) (d) None of these
(a) 7C6 (b) 9C4 {10!}(5!)
(c) 7C5 (d) None of these 9. In how many ways 50 distinct chocolates

5. In how many ways a pack of 52 cards can can be distributed equally among 10
students?
be distributed equally to 4 students?
50! 50!
52! 52! (a) (b)
5
(a) (13!)
4 (b) (4)(13!)4 {(10!) }(5!) {(10!)5 }
50!
4(52!) (c) (d) None of these
(c) (13!)4 (d) None of these {10!}(5!)

10. What is the number of ways in which
6. In how many ways a pack of 52 cards
a mixed double game can be arranged
can be distributed to 4 sets such that
amongst 10 married couples if no husband
3 of them having 17 cards each and
and wife play in the same game?
remaining 1 with 1 card?
(a) (10C2)(8C2)(2) (b) (10C2)(8C2)/(2)
52! 51!
(a) (b) (c) (10C2)(8C2) (d) None of these
(17!)3 (3!)(17!)3
Response Grid
1. a b c d 2. a b c d 3. a b c d 4. a b c d 5. a b c d
6. a b c d 7. a b c d 8. a b c d 9. a b c d 10. a b c d
Distribution  191

3
Concept Cracker (CC)
Ideal Time Boost up your confidence with good
60 Min. questions

1. How many cricket matches can be friend receive at least 1 fruit of each
conducted from 22 players if a team has type?
11 players? (a) 44656 (b) 46616
(a) (22C11) (b) (22C11)/2 (c) 46656 (d) None of these
22
(c) ( C11)(2) (d) None of these
7. If 5 dice are rolled then what is the
2. In how many ways m distinct books can number of possible outcome in which at
be distributed to n students? least 1 die will show a 6?
(a) mn (b) (n!)(m!) (a) 4651 (b) 4620
(c) mPn (d) None of these (c) 4629 (d) None of these
3. In how many ways 2n persons can be
8. What is the number of ways in which 5
divided into n pairs?
balls of distinct colors can be distributed
(2n)! to three persons if each person should
(a) 2nC
n (b) get at least 1 ball?
2n
(c) (2n)(2nC
n) (d) None of these (a) 120 (b) 150
4. In how many ways 15 different articles (c) 180 (d) None of these
can be distributed equally in 3 sets? 9. Find the number of ways in which we
15! 15! can distribute 10 distinct balls to 8 boxes
(a) (b)
(3)(5!)3 (5!)3 such that at most 6 of them are empty?
(a) 810 – 8 (b) 108 – 8
15!
(c) (d) None of these (c) 108 – 10 (d) None of these
(3!)(5!)3
5. Total number of fruits with Ram is 10 10. In how many ways 26 distinct fruits can
Mangoes, 5 Oranges and 5 Apples, in be distributed among 4 persons, A, B, C
how many ways he can distribute it and D such that the number of chocolates
among his three friends such that any received by them (in the given order) is
friend may receive any number of fruit in AP with common difference 1?
Assume that fruits of the same species 26!
are all alike? (a)
( )( )(7!)(8!)
5! 6!
(a) 29106 (b) 1386 26!
(c) 87318 (d) None of these (b)
(5!)(6!)(7!)(8!)(4!)
6. Total number of fruits with Gunjan is 26!
(c)
10 Mangoes, 10 Oranges and 10 Apples,
in how many ways he can distribute it
(5!)(6!)(7!)(8!)(4)
among his three friends such that each (d) None of these
192  Theory of Counting
11. In how many ways six faces of a cube can 19. In how many ways 10 distinct chocolates
be painted with 1 colour ? can be distributed to 5 identical bags if
(a) 66 (b) 6! each bag gets at most 1 chocolate ?
6
(c) C6 (d) None of these (a) 510 (b) 14C
4
12. In how many ways 10 distinct chocolates (c) 0 (d) None of these
can be distributed to 5 distinct bags with
20. In how many ways 10 distinct chocolates
no restrictions?
can be distributed to 5 identical bags if
(a) 510 (b) 105 each bag gets at least 1 chocolate?
(c) 10P5 (d) None of these (a) 42525 (b) 86472
13. In how many ways 10 distinct chocolates
(c) 86582 (d) None of these
can be distributed to 5 distinct bags such
that each bag gets at most 1 chocolates ? 21. In how many ways 10 identical
(a) 510 (b) 105 chocolates can be distributed to 5
identical bags?
(c) 10P5 (d) None of these
(a) 7 (b) 30
14. In how many ways 10 distinct chocolates
can be distributed to 5 distinct bags such (c) 25 (d) None of these
that each bag gets at least 1 chocolates ? 22. In how many ways 10 identical
(a) 510 (b) 105 chocolates can be distributed to 5
10
(c) P5 (d) None of these identical bags if each bag gets at most 1
chocolate?
15. In how many ways 10 identical
chocolates can be distributed to 5 (a) 7 (b) 30
distinct bags if there is no restriction? (c) 0 (d) None of these
(a) 510 (b) 14C4
23. In how many ways 10 identical
14
(c) C9 (d) None of these chocolates can be distributed to 5
16. In how many ways 10 identical identical bags if each bag gets at least 1
chocolates can be distributed to 5 chocolate?
distinct bags if a bag can not have more (a) 7 (b) 30
than 1 chocolate?
(c) 25 (d) None of these
(a) 510 (b) 14C4
(c) 0 (d) None of these 24. What is the number of ways in which 8
distinct chocolates can be packed in two
17. In how many ways 10 identical
boxes such that no box remains empty?
chocolates can be distributed to 5
distinct bags if a bag must have at least (a) 256 (b) 128
1 chocolate? (c) 510 (d) None of these
(a) 510 (b) 9C4 25. There are 10 lamps in a room, if room
(c) 14C9 (d) None of these can be illuminated by any lamp then the
18. In how many ways 10 distinct chocolates total number of ways in which room can
can be distributed to 5 identical bags? be illuminated is
(a) 42525 (b) 86472 (a) 1024 (b) 1023
(c) 86582 (d) None of these (c) 511 (d) None of these
Distribution  193
26. In how many ways n identical chocolates Direction Qs. 29 and 30: At a famous ice
can be distributed to 3n students such cream parlor “The cold way” total 15 flavors
that no student receive more than one are available.
chocolate? 29. Niyati went to purchase an ice-cream
(a) 3nCn (b) 2nCn cone that can have three scoops how
(c) 3nCn – 2nCn (d) None of these many different ways she can order if
flavors can not be repeated ? (It is known
27. In a class of 10 students if two prizes (1st
that order of flavor is not important.
and 2nd ) has to be given in three subjects
Physics, Chemistry & Mathematics then (a) 17C3 (b) 17C2
in how many ways this can be done? (c) 15C3 (d) None of these
(a) 34300 (b) 72900 30. Niyati went to purchase an ice-cream
(c) 24300 (d) None of these cone that can have at most three scoops
how many different ways she can order if
28. In how many ways 7 people can be
flavors can not be repeated ? (It is known
divided into 3 identical groups?
that order of flavor is not important).
(a) 34323 (b) 520
(a) 17C3 (b) 17C2
(c) 61642 (d) None of these
(c) 15C3 (d) None of these

Response Grid
1. a b c d 2. a b c d 3. a b c d 4. a b c d 5. a b c d
6. a b c d 7. a b c d 8. a b c d 9. a b c d 10. a b c d
11. a b c d 12. a b c d 13. a b c d 14. a b c d 15. a b c d
16. a b c d 17. a b c d 18. a b c d 19. a b c d 20. a b c d
21. a b c d 22. a b c d 23. a b c d 24. a b c d 25. a b c d
26. a b c d 27. a b c d 28. a b c d 29. a b c d 30. a b c d

Solutions
Concept Applicator (CA)
1. (b) 2. (d) 3. (b) 4. (a) 5. (c) 6. (a)
7. (b) 8. (a) 9. (c) 10. (b)
194  Theory of Counting
1. (b) Here we are distributing identical question number 6. Since chocolates
articles so we have to use partitioning are distinct, so 1st chocolate can
theory, in this case n = 20 and r = 5 be distributed in 5 ways, 2nd in 5
hence required number of ways is ways and so on, so total number of
20 + 5 – 1C 24C ways is 5 × 5 × 5 × 5 × ... × 5 = 520.
5–1= 4
2. (d) Here we are distributing identical We can also use the formula.
articles so we have to use partitioning (Where)(What), where we are sending
theory, in this case n = 20 and r = 5 the articles answer is to students
but restriction is a particular student and number of students is 5, what
Rohan receive at least 1 chocolate, we are sending answer is chocolate
so to remove this restriction 1st give and number is 20 hence required
him 1 chocolate now we left with 19 number of ways is 520.
chocolates or n = 19. Hence required 7. (b) Here also the articles are distinct, so
number of ways is 19+5–1C5–1 = 23C4 1st we will give a chocolate to Rohan
3. (b) Here we are distributing identical and that can be done in 20 ways then
articles so we have to use partitioning remaining 19 distinct chocolates can
theory, in this case n = 20 and r = 5 be distributed among 5 students in
but restriction is that each student 519 ways. Hence total number of
receive at least 1 chocolate, so to ways is 20(519).
remove this restriction 1st give them 8. (a) Here students must be distinct.
1 chocolate each now we left with 15 Hence required number of ways is
chocolates or n =15. Hence required 420
number of ways is 15+5–1C5–1 = 19C4
9. (c) 1st invitation card can be dealt in 3
4. (a) Here we are distributing identical ways similarly 2nd and 4rd and so on
articles so we have to use partitioning so total number of ways is 310.
theory, in this case n = 20 and r = 5
We can also use the formula
but restriction is that each student
(Where) Where we are sending the
receive at least 2 chocolates, so to
invitation card answer is to courier
remove this restriction 1st give them
company and how many of them the
2 chocolates each now we left with 10
answer is 3 hence base is 3, next
chocolates or n =10 hence required
what we are sending the answer is
number of ways is 10+5–1C5–1 = 14C4
invitation cards, and how many of
5. (c) Similar to the above question n = 5, them the answer is 10 so exponent
and r = 5. Hence required number of must be 10, hence total number of
ways is 5+5–1C5–1 = 9C4 ways is 310.
6. (a) The main difference between 10. (b) In this case each person will get 5
question number 1 and 6 is that the articles, so required number of ways
things we are going to distribute 15! 15!
(Chocolate in this case) is identical is =
in question number 1 but distinct in
(5!)(5!)(5!) (5!)3
Concept Builder (CB)
1. (b) 2. (b) 3. (a) 4. (c) 5. (a) 6. (c)
7. (b) 8. (a) 9. (b) 10. (a)
Distribution  195
1. (b) Here although it is not given that Hence required number of ways is
grapes are identical or distinct, but 52!
if nothing is given then we will apply
our logic, can we distinguish these
(13!)4
50 grapes the answer is no so we will 6.
(c) We know that number of ways of
assume that they are identical, then dividing 3m distinct articles in three
n = 50 and r = 5. Hence required groups containing m articles in each
number of ways is 50+5–1C5–1 = 54C4
{(3m)!}
2. (b) Here we are distributing identical group
{(m !)(m !)(m !)(3!)}
articles so we have to use partitioning
th
Here 4 student can be given 1 card
theory, in this case n = 50 and r = 5
but restriction is that each student in 52 ways, then remaining 51 cards
receive at least 5 grapes, so to remove now we have to distribute equally
this restriction 1st give them 5 grapes among 3 students and that can be
each now we left with 25 grapes or 51!
n = 25. Hence required number of done in
(3!)(17!)(17!)(17!)
ways is 25 + 5 – 1C5 – 1 = 29C4 Hence required number of ways
3. (a) Here we are distributing identical (52)(51!) 52!
articles so we have to use partitioning =
(3!)(17!)(17!)(17!) (3!)(17!)3
theory, in this case n = 50 and r = 5
but restriction is that 1st student get 7. (b) Since we know from Type 10:-
at least 1, 2nd student get at least 2 Number of ways in which m × n
and so on 5th student get at least 5 different object can be distributed
grapes, so to remove this restriction equally among n persons (or
1st give them required number of numbered groups) = (number of ways
grapes i.e 1+2+3+4+5 =15 grapes of dividing) × (number of groups)! =
now we left with 35 grapes or n = 35. (mn)!n!/(m!)n n! = (mn)!/(m!)n
Hence required number of ways is Hence required number of ways is
35+5–1C 39C
5–1 = 4
225!
4. (c) Here we are distributing identical
articles so we have to use partitioning (15!)15
theory, in this case n = 50 and r = 6 8. (a) From the condition given in
but restriction is that each student Type 9:-Number of ways in which
receive at least 8 grapes, so to m × n distinct objects can be divided
remove this restriction 1st give them equally into n groups (unmarked) =
8 grapes each now we left with 2 (mn)!/(m!)n n!.
grapes or n = 2. Hence required number of ways is
Hence required number of ways is 50!
2+6–1C 7
6–1 = C5
{(10!)5 }(5!)
5. (a) We know that : Number of ways in
which m × n different object can be 9. (b) From solution of previous question
distributed equally among n persons the required number of ways is
(or numbered groups)  = (number of 50! 50!
ways of dividing) × (number of groups)! 5
 ×  (5!) =
{(10!) } (5!) (10!) 5
= (mn)!n!/(m!)n n! = (mn)!/(m!)n
196  Theory of Counting
10. (a) Two males can be selected out of selected women are W3 and W4 then
10 in 10C2 ways, then two females number of games can be (M1W3) with
can be selected out of 8 remaining (M2W4) and (M1W4) with (M2W3)
couples in 8C2 ways, Let us assume hence in two ways,
that selected males are M1, M2 and So total number of ways is (10C2)
(8C2)(2)
Concept Cracker (CC)
1. (b) 2. (d) 3. (b) 4. (c) 5. (a) 6. (c)
7. (a) 8. (b) 9. (a) 10. (a) 11. (d) 12. (a)
13. (d) 14. (d) 15. (c) 16. (c) 17. (b) 18. (d)
19. (c) 20. (a) 21. (b) 22. (c) 23. (a) 24. (d)
25. (b) 26. (a) 27. (b) 28. (d) 29. (c) 30. (d)

1. (b) Number of ways of distributing 22 And 5 identical Apples can be


players in two teams such that they distributed among 3 students in
play against each other is (22C11)/2 5+3–1C 7
3–1 = C2 = 21 ways.

2. (d) Consider 1st book it can be dealt in So total number of ways is 66 × 21 ×


n ways, next book again can be dealt 21 = 29106
in n ways and so on hence number of
6. (c) 10 identical mangoes can be
ways is nm
distributed among 3 students such
Alternately: that each student receive at least 1
mango is 7+3–1C3–1 = 9C2 = 36 ways.
Since we have to distribute distinct
articles to distinct groups hence we Similarly oranges and apples can be
can use the formula (Where)What distributed in 36 ways so total number
hence number of ways is nm of ways is 36 × 36 × 36 = 46656
3. (b) Total number of persons is 2n and 7. (a) Total number of possible outcome is
we need n pairs so required number 65. Now consider the outcomes when
(2n)! 6 doesn’t appear which is 55 hence
of pairs is required number of outcomes is
2n
65 – 55 = 7776 – 3125 = 4651
4. (c) In this case each person will get 5
articles, so required number of ways 8. (b) Here we have two cases-
15! 15!
is = Case (i) 2+2+1 = 5, total number of
(3!) (5!)(5!)(5!) (3!) (5!)3 ways in this case is (5C2)(3C2)(1C1)
5. (a) 10 identical mangoes can be (3) = 90
distributed among 3 students in
10+3–1C 12C = 66 ways. Case (ii) 3+1+1 = 5, total number of
3–1 = 2
ways in this case is (5C1)(4C1)(3C3)
Similarly 5 identical Oranges can (3) = 60
be distributed among 3 students in
5+3–1C 7
3–1 = C2 = 21 ways.
198  Theory of Counting
1 r From the table required number of
Since S(n, r) = [ C0 (rn) – rC1 (r – 1)n
r! ways is S(10, 5) = 42525 (we have
+ rC2(r – 2)n - … + (–1)r–1(rCr–1)(1n)] already calculated that in previous
And S(n, 1) = 1, S(n, 2) = 2n–1 – 1 , questions)
S(n, n–1) = nC2, S(n, n) = 1 21. (b) This case is similar to distributing
r identical articles to n identical
Here in this case required number of
groups this is explain in column 4 and
ways is S(10,1) + S(10, 2) + S(10, 3)
row 4 (Case x) of ‘12-fold way” table,
+ S(10, 4) + S(10,5)
Required number of ways is P1(10) +
Since S(10, 1) = 1, S(10, 2) = 29 – 1 = 511,
P2(10) + P3(10) + P4(10) + P5(10)
From the formula S(10, 3) = 1/3!
Since Pn(r) represents the number
[3C0 (310) – 3C1(210) + 3C2(110)]
of ways r identical chocolates can
= 59049 −3072 +  3 = 9330 be placed into n identical bags such
6 that no bags is empty.
From the formula S(10, 4) = 1/4! [4C0
(410) – 4C1(310) + 4C2(210) – 4C3(110)] P1(10) = 1, P2(10) = 5,
P3(10) = 8 ways as can be seen
1048576 − 236196 + 6144 − 4
= (1, 1, 8) (1, 2,7), (1, 3, 6), (1, 4, 5), (2,
24 2, 6), (2, 3, 5), (2, 4, 4), (3, 3, 4)
= 34105
P4(10) = 9 ways as can be seen (1,
Hence S(10, 5) = 1/(5!) [5C0(510) – 5C1
1, 1,7), (1, 1, 2, 6), (1, 1, 3,5), (1, 1,
410 + 5C2(3)10 – 5C3 210 + 5C4(1)10]
4,4), (1, 2, 2,5), (1, 2, 3, 4), (1, 3,
= 1/(5!) [9765625 – 5242880 + 590490
3,3), (2, 2, 2, 4), (2, 2, 3, 3),
– 10240 + 5] = 1/(5!) [5103000]
P5(10) = 7 ways as can be seen
= 42525
(1, 1, 1, 1, 6)(1,1,1,2,5), (1,1,1,3,4)
Required Number of ways is S(10,1) (1,1,2,2,4), (1,1,2,3,3)(1,2, 2, 2,3),
+ S(10, 2) + S(10, 3) + S(10, 4) + (2,2,2,2,2)
S(10,5) = 1+511+9330+34105+42525 Total number of ways is
= 86472
  1+5+8+9+7 = 30
19. (c) This case is similar to distributing r
distinct articles to n identical groups 22. (c) This case is similar to distributing r
this is explain in column 4 and row distinct articles to n identical groups
4 (Case viii) of ‘12-fold way” table, this is explain in column 4 and row 4
(Case xi) of ‘12-fold way” table,
Since number of chocolates is more
than the number of bags hence Since number of chocolates is more
number of ways is 0. than number of bags hence number
20. (a) This case is similar to distributing r of ways is 0
identical articles to n identical groups 23. (a) This case is similar to distributing r
this is explain in column 4 and row 4 distinct articles to n identical groups
(Case ix) of ‘12-fold way” table, and it this is explain in column 5 and row 4
is definition of sterling number (Case xii) of ‘12-fold way” table,
198  Theory of Counting
1 r From the table required number of
Since S(n, r) = [ C0 (rn) – rC1 (r – 1)
r! ways is S(10, 5) = 42525 (we have
n + rC (r – 2)n - … + (–1)r–1(rC )(1n)]
2 r-1 already calculated that in previous
And S(n, 1) = 1, S(n, 2) = 2n-1 – 1 , questions)
S(n, n–1) = nC2, S(n, n) = 1 21. (b) This case is similar to distributing r
identical articles to n identical groups
Here in this case required number of
this is explain in column 4 and row 4
ways is S(10,1) + S(10, 2) + S(10, 3)
(Case x) of ‘12-fold way” table,
+ S(10, 4) + S(10,5)
Required number of ways is P1(10) +
Since S(10, 1) = 1, S(10, 2) = 29 – 1 = 511,
P2(10) + P3(10) + P4(10) + P5(10)
From the formula S(10, 3) = 1/3!
Since Pn(r) represents the number
[3C0 (310) – 3C1(210) + 3C2(110)] =
of ways r identical chocolates can
59049 −3072 +  3 be placed into n identical bags such
= 9330
6 that no bags is empty.
From the formula S(10, 4) = 1/4! [4C0
(410) – 4C1(310) + 4C2(210) – 4C3(110)] P1(10) = 1, P2(10) = 5,
P3(10) = 8 ways as can be seen
1048576 − 236196 + 6144 − 4
= (1, 1, 8) (1, 2,7), (1, 3, 6), (1, 4, 5), (2,
24 2, 6), (2, 3, 5), (2, 4, 4), (3, 3, 4)
= 34105
P4(10) = 9 ways as can be seen (1,
Hence S(10, 5) = 1/(5!) [5C0(510) – 5C1
1, 1,7), (1, 1, 2, 6), (1, 1, 3,5), (1, 1,
410 + 5C2(3)10 – 5C3 210 + 5C4(1)10] =
4,4), (1, 2, 2,5), (1, 2, 3, 4), (1, 3,
1/(5!) [9765625 – 5242880 + 590490
3,3), (2, 2, 2, 4), (2, 2, 3, 3),
– 10240 + 5] = 1/(5!) [5103000] =
P5(10) = 7 ways as can be seen
42525
(1, 1, 1, 1, 6)(1,1,1,2,5), (1,1,1,3,4)
Required Number of ways is S(10,1) (1,1,2,2,4), (1,1,2,3,3)(1,2, 2, 2,3),
+ S(10, 2) + S(10, 3) + S(10, 4) + (2,2,2,2,2)
S(10,5) = 1+511+9330+34105+42525 Total number of ways is
= 86472
  1+5+8+9+7 = 30
19. (c) This case is similar to distributing r
distinct articles to n identical groups 22. (c) This case is similar to distributing r
this is explain in column 4 and row 4 distinct articles to n identical groups
(Case viii) of ‘12-fold way” table, this is explain in column 4 and row 4
(Case xi) of ‘12-fold way” table,
Since number of chocolates is more
than the number of bags hence Since number of chocolates is more
number of ways is 0. than number of bags hence number
20. (a) This case is similar to distributing r of ways is 0
identical articles to n identical groups 23. (a) This case is similar to distributing r
this is explain in column 4 and row 4 distinct articles to n identical groups
(Case ix) of ‘12-fold way” table, and it this is explain in column 5 and row 4
is definition of sterling number (Case xii) of ‘12-fold way” table,
Distribution  199
From the table required number of 28. (d) From the ’12 fold table’ required
ways is P5(10) = 7 number of ways is S(7, 1) + S(7, 2) +
S(7, 3)
24. (d) Each chocolate has 2 options hence
total number of ways is 2 × 2 × … Since S(n, 1) = 1, and S(n, 2) = 2n–1 – 1,
2 = 28 = 256, but it is given that no
Hence S(7, 1) = 1 and S(7, 2) = 26 – 1 =
box remains empty hence required
63, now we have to calculate S(7, 3)
number of ways is 256 – 2 = 254
25. (b) Total number of ways is 210–1 = 1023 From the formula S(7, 3)

26. (a) Here we just have to select n = 1/3! [3C0 (37) – 3C1(27) + 3C2(17)]
students from 3n students and that 2187 −384 +  3
can be done in 3nCn, since chocolates = = 301
6
are identical so we can arrange them Required Number of ways is
in only 1 way.
1 + 63 + 301 = 365 ways.
27. (b) In Physics 1st and 2nd prize can be
given in 10 × 9 = 90 ways and similarly 29. (c) Total number of ways is 15C3
prizes in other 2 subjects can be given
30. (d) Total number of ways is
in 90 ways, Hence total number of
ways is 90 × 90 × 90 = 729000   15C + 15C2 +15C3
1
200  Theory of Counting
Part B

Concept 1 Partitioning Theory

Distribution of ‘n’ articles into ‘r’ groups:- Case (i) [Partitioning Theory] :- Number of
In this type of questions we have different ways of dividing ‘n’ identical things among
cases that depends on whether ‘n’ articles are ‘r’ distinct groups or ‘r’ persons when each
identical or distinct, similarly ‘r’ groups are group can get any number of things ≤ n is
identical or distinct. We will discuss each and
(n+r-1)C
every case separately: r-1

Concept 2 Distribution of Distinct Article

Distribution of ‘n’ distinct articles to ‘r; Arrangements in groups:


distinct groups:- The number of ways of
• The number of ways in which n different
distributing ‘n’ distinct articles to ‘r’ distinct
things can be arranged into r different
group is rn.
group (when a particular group may be
1st article can be dealt in r ways, 2nd article empty) is r(r+1)(r+2)…(r+n–1)
can be dealt in r ways and so on each article
can be dealt in r ways hence number of ways (r + n − 1)!
=
is rn (r − 1)!
In general students used to get confuse that • The number of ways in which n different
whether 510or 105 is the correct answer, to things can be arranged into r different
get rid of this confusion we can remember 1 group (when no group is empty)
simple formula (where)(What).
is n – 1Cr – 1
In this case ‘Where’ we are sending the
articles? It is boxes and number of boxes is 5, • The number of ways in which n different
then next is what we are sending? It is rings things can be distributed into r different
and the number of rings is 10 hence from groups (when no group is empty) is
(Where)(What) formula required number of rn – rC1(r–1)n +….. + (–1)r–1(rCr–1)
ways is 510
Distribution  201

Concept 3 Distribution in Groups

Type 1: Number of ways of dividing (m+n) Type 7: Number of ways, in which n distinct
distinct articles in two groups containing m objects can be divided into r unequal groups
and n articles respectively (m ≠ n) is (m+nCn) containing a1, a2, a3, ......,ar things (ai ≠ aj)
{(m + n)!}
(mCm) = (m+nCm)(nCn) = n n–a1 n–a1 –a2 n!
{(m !)(n !)} = Ca . Ca . Ca =
1 2 r a1! a 2! a 3!… ar!
Type 2: Number of ways of dividing 2m
Here a1 + a2 + a3 + ... + ar = n.
distinct articles in two groups containing m
{(2 m)!} Type 8: Number of ways in which n distinct
articles in each group
{(m !)(m !)(2!)} objects can be distributed among r persons
Type 3: Number of ways of dividing 2m such that some person get a1 objects, another
distinct articles between two persons such person get a2 objects ......... and similarly
{(2 m)!} someone gets ar objects
that each of them get m articles is
{(m !))(m !)} n n–a1 n–a1 –a2
= Ca . Ca . Ca
1 2 r
Type 4: Number of ways of dividing 3m
(n!)(r!)
distinct articles in three groups containing m =
a1! a 2! a 3!… ar!
{(3m)!}
articles in each group Here a1 + a2 + a3 + ... + ar = n.
{(m !)(m !)(m !)(3!)}

Type 5: Number of ways of dividing 3m distinct Type 9: Number of ways in which m × n


articles among three persons such that each distinct objects can be divided equally into n
{(3m)!} groups (unmarked) = (mn)!/(m!)n n!.
of them get m articles is .
{(m !)(m !)(m !)}
Type 10: Number of ways in which m × n
Type 6: Number of ways of dividing (m+n+p) different object can be distributed equally
distinct articles in three groups containing m, among n persons (or numbered groups)  =
{(m + n + p )!} (number of ways of dividing) × (number of
n, p articles in three groups is
{(m !)(n !)( p !)} groups)! = (mn)!n!/(m!)n n! = (mn)!/(m!)n.
202  Theory of Counting
Stanley’s “12-fold way” for Distribution
Consider a question: How many ways can r pieces of Chocolates be distributed among nboxes?
Answer of this question depends on following factors-
1. Chocolates are Identical or Distinct.
2. Boxes are identical or distinct.
3. Is there any restriction on number of chocolates in a box?
Answer of all these can be given by following tables-

Number of Number of
Chocolates
Boxes (n) No restriction Chocolates in a Chocolates in a
(r)
bag ≤ 1 bag ≥ 1

nr n! (n!) S(r, n)
Distinct Distinct (n – r)!
Case (i) Case (ii) Case (iii)
n+r–1C nC r–1C
r
Identical r–1 n–1
Distinct
Case (iv) Case (v) Case (vi)
n
1 if r ≤ n
∑ S(r, k) 0 if r >n
S(r, n)
k =1
Distinct Identical

Case (vii) Case (viii) Case (ix)


n 1 if r ≤ n
Pn(r)
Identical Identical
∑ Pk(x) 0 if r >n
k =1
Case (x) Case (xi) Case (xii)
Here S(r, n) is sterling number and is defined as-
1 r
S(n, r) = [ C0 (rn) – rC1 (r – 1)n + rC2(r – 2)n – ... + (–1)r–1(rCr–1)(1n)]
r!
And S(n, 1) = 1,
S(n, 2) = 2n–1 – 1 , S(n, n–1) = nC2, S(n, n) = 1
Pn(r) represents the number of ways r identical chocolates can be placed into 4
identical bags such that no bag is empty.
Since there is only 1 way to express the number n as the sum of 1 number
(namely, n = n), so P1(n) = 1.
Similarly, there is only 1 way to express the number n as the sum of n numbers (namely, n =
1 + 1 + 1 + … + 1),so Pn(n) = 1.
Consider, Pn–1(n) = 1, since there will be 1 bag with 2 Chocolates and the rest of the bags will
have 1 chocolates each.
n – 1
In general, when n is even, P2(n) = n/2, and when n is odd, P2(n) = .
2
For e.g. P2(8) = 4, and P2(11) = 5.
Distribution  203

1
Concept Deviator (CD)
Ideal Time Revise your concept (IIT advance or
5 Min. tougher questions)
Per Question

1. An ice cream parlor has 20 different of the balls. The number of ways in which
flavors of ice cream available. If they sell the balls, one each in a box, could be placed
ice cream in a cone in triples (3 flavors in such that a ball does not go to a box of its
a cone), how many different set of tastes own colour is [IIT JEE 1992]
it can offer to its customer? (a) 9 (b) 11
(a) 1140 (b) 1250 (c) 13 (d) None of these
(c) 1540 (d) None of these
4. Five balls of different colours are to be
2. The total number of ways in which 5 balls placed in three boxes of different sizes.
of different colours can be distributed Each box can hold all five. In how many
among 3 persons so that each person gets different ways can we place the balls so
at least one ball is:     [IIT JEE 2012] that no box remains empty?
(a) 75 (b) 150 [IIT JEE 1981]
(c) 210 (d) 243 (a) 420 (b) 240
3. There are four balls of different colours
(c) 300 (d) None of these
and four boxes of colours, same as those

Response Grid
1. a b c d 2. a b c d 3. a b c d 4. a b c d

2
Concept Eliminator (CE)
Ideal Time Revise your concept (Maths Olympiad
Just solve it or tougher questions)

1. If P represents the number of ways of n+2 persons such that exactly 3 persons
distributing n distinct objects to n+2 gets nothing then find the ratio of P:Q
persons such that exactly 2 persons gets
(a) n(n–1)/6 (b) 4/n(n–1)
nothing and Q represents the number of
ways of distributing n distinct objects to (c) n(n–1)/4 (d) None of these
204  Theory of Counting
2. Ramesh has 2n number of fruits out 4. What is the total number of permutations
of which n of them are identical and of n distinct things taken not more than
remaining n are distinct, In how many r at a time if each thing may be repeated
ways he can distribute these fruits to
any number of times?
his two children Bhavesh and Sanjesh
(a) n+r–1C n–r+1C
such that both of them will receive equal r–1 (b) r–1
number of fruits?
n(n r – 1)
(a) 2n–1 (b) 2n (c) (d) None of these
n–1
(c) 2n+1 (d) None of these
5. Amrita has 5 distinct rings, in how many
3. In how many ways 5 chocolates of
ways can she wear these rings on her 10
different brands can be distributed into
three students such that each student fingers?
gets at least one chocolate? (a) 14C 14C
4. (b) 5
(a) 120 (b) 150 14P
(c) 5 (d) None of these
(c) 180 (d) None of these

Solutions
Concept Deviator (CD)

1. (c) 2. (b) 3. (a) 4. (c)

1. (c) Here in this question repetition of Alternate way:


flavor is not allowed and order in
Required number of ways is 20+3–1C3
which the three flavors are given is
not important. Required number of = 22C3 = 1540
ways (20C3) + 2(20C2) +(20C1) = 1140 2. (b) Total number of ways is given by
+ 380 + 20 = 1540 ways. 35 – 3C1 × (3 – 1)5 + 3C2(3 – 2)5
= 243 – 3 × 32 + 3 = 150
Distribution  205
Alternately:- 3. (a) This is question of De-arrangement
so required number of ways is
Case (i) If distribution is done such that
1 1 1 1
5 = (1, 2, 2) then number of ways is (4!)1 – + – + =9
 1! 2! 3! 4! 
 5!   3! 
    =  90
 (2!)(2!)  2!  4. (c) There are two cases-

Case (ii) If distribution is done such that Case (i) if it is (2, 2, 1) and number of
5 = (1, 1, 3) then number of ways is ways (3!)(5C2)(3C2)(1C1) = 120
 5!   3!  Case (ii) if it is (3, 1, 1) and number of
    =  60
 (3!)  2!  ways (3!)(5C3)(2C1)(1C1) = 180
So total number of ways is 90 + 60 Total number of ways is 120 + 180 = 300
= 150

Concept Eliminator (CE)

1. (d) 2. (c) 3. (b) 4. (c) 5. (c)

1. (d) Since we have to distribute n distinct 2. (c) Ramesh has to 1st select n fruits out
objects to n+2 persons just that of 2n fruits , for this following cases
exactly 2 persons gets nothing that may arise-
means remaining n persons gets 1
Case (i) Number of distinct fruits 0 and
objects, so number of ways is (n+2C2)
number of identical fruits n then
(n!) = P
number of ways is nC0
Similarly for 2nd case 1st we will
Case (ii) Number of distinct fruits 1 and
select 3 persons out of n+2 and that
number of identical fruits n–1 then
can be done in n+2C3. Now we have
number of ways is nC1
to distribute n distinct objects to
n-1 persons so exactly one person Case (iii) Number of distinct fruits 2
will get 2 objects that can be done and number of identical fruits n–2
in (n–1C1)(nC2) then remaining n-2 then number of ways is nC2
distinct objects can be distributed And so on
to n–2 persons in (n–2)! Ways hence
Last case Number of distinct fruits
total number of ways is
n and number of identical fruits 0
Q=(n+2C3)(n–1C1)(nC2)(n–2)! then number of ways is nCn
Required ratio is P:Q = 6/n(n–1) So total number of ways is nC0 + nC1
+ nC2 +… + nCn = 2n ways.
206  Theory of Counting
Since there are two sons so it can 4. (c) The number of ways if 1 item is
de distributed in (2!) ways. Hence taken is n
required number of ways is (2)(2n)
The number of ways if 2 items is
= 2n+1
taken is n2
3. (b) We know that the number of ways
The number of ways if 3 items is
in which n different things can be
taken is n3
distributed into r different groups
(when no group is empty) is Similarly …

rn – rC1(r–1)n +….. + (–1)r–1(rCr–1) The number of ways if r items is


taken is nr
Hence required number of ways is
Hence total number of ways is n + n2
35 – 3C1(3–1)5 + 3C2(3–2)5 – 3C3(3–3)5
n(n r – 1)
= 243 – 96 + 6 – 0 = 150 +n3+…+nr =
n–1
Alternately: 5. (c) Since all the 10 fingers are distinct
and distinguishable. Consider 1st
Here we have two cases-
ring there are 10 places for it, now
Case (i) 2+2+1 = 5, total number of ways consider the 2nd ring it has total 11
in this case is (5C2)(3C2)(1C1)(3) = 90 places (since 1st ring created two
Case (ii) 3+1+1 = 5, total number of separated places for 2nd ring on the
ways in this case is (5C1)(4C1)(3C3) finger), similarly 3rd ring has 12, 4th
(3) = 60 has 13 and 5th has 14 places.

Hence total number of ways is 90 + So total number of ways is 10 × 11 ×


60 = 150 12 × 13 × 14 = (14!)/(9!) = 14P5.
8
Number of Integral Solution  207

Number of Integral Solution

Topics Covered
 Number of Integral Solution

 Multinomial Theorem

 Integral Solution from Multinomial Theorem

 Application of Multinomial Theorem

 De-arrangement

Part A: Topic Number of Questions


Solved Example 10
Concept Applicator 10
Concept Builder 10
Concept Cracker 15
Part B: Topic Number of Questions
Concept Deviator 10
Concept Eliminator 5
Total 60
208  Theory of Counting
Part A

Concept 1 Number of Integral Solution

In last chapter we have studied portioning the number of ways from the partitioning
theory, as per that theory- Number of ways theory is 10+3–1C3–1 = 12C2 ways.
of dividing ‘n’ identical things among ‘r’
distinct groups or ‘r’ persons when each Example 2:  Find the number of non
group can get any number of things ≤ n is negative integral solution of the
(n+r-1)C equation p + q + r + s = 10
r-1
Now consider an equation with non negative Solution:  Required number of ways from the
integral solution- a + b + c = 20 partitioning theory is 10+4–1C4–1 =13C3 ways.
We need to find the non negative integral Example 3:  Find the number of positive
solution of the equation. As per the given integral solution of the equation
condition a ≥ 0, b ≥ 0, and c ≥ 0. If we x + y + z = 10
observe carefully then situation is similar to
Solution:  Question is similar to, in how
distributing 20 identical chocolates among
three students say A, B and C such that many ways we can distribute 10 chocolates
number of chocolates received by A, B and C among three students such that each
is a, b and c respectively then a + b + c = 20 student receive at least 1 chocolates and
and a ≥ 0, b ≥ 0, and c ≥ 0. the number of ways from the partitioning
theory is 10–1C3–1 = 9C2 ways.
Hence any such question can be dealt by
formula/theory of partitioning theory. Hence
Alternately:-
required number of solution of the given
equation is 20+3–1C3–1 = 22C2. Let as assume that x = 1 + x1 such that x1 ≥ 0
Similarly y = 1 + y1, z = 1 + z1 such that y1 ≥
Example 1:  Find the number of non
0 and z1 ≥ 0,
negative integral solution of the
equation x + y + z = 10 Then the given equation x + y + z = 10 can be
reduced to (1 + x1) + (1 + y1 ) + (1 + z1 ) = 10
Solution:  Question is similar to, in how many
ways we can distribute 10 chocolates among Or the new equation is x1 + y1 + z1 = 7 such
that x1 ≥ 0, y1 ≥ 0 and z1 ≥ 0,
three students without any condition and
QR Code / Video Link

For Smart phone/ Tablet users

Video Link for Desktop/Laptops users http://dishapublication.com/video-resources


210  Theory of Counting
Let us assumethat the different things = Co-efficientof xr in { ( x + x2 + x3 +
be denoted by the letters a, b, c, … ……… + xp) ( x + x2 + x3 + ……. + xp)
Then consider the term ( x + x2 + x3 + ……….. + xr)}
(1 + ax + a2x2 + . . . + apxp) (1 + bx + b2x2 = Co-efficientof xr in [ ( 1 + x + x2 +
+ . . . + bqxq) (1 + cx + c2x2 + . . . + csxs). . . ……. + xp – 1 ) ( 1 + x + x2 + …… + xq–1 )
( 1 + x + x2 + ….. + xr – 1 )]
In this expansion the exponent of the
letters a, b, c is same as that in x. Now
consider the coefficient of xr in the (iv) Number of terms in the expansion of
expansion it is same as the number of (a1 + a2 + a3 + …. + an)r
ways of taking r of the letters a, b, c, . . . In any term let exponents of a1, a2… an
. . . with the restriction that maximum are x1, x2, … and xn then x1 + x2+… xn
number of a’ s is p, maximum number of =r
b’ s is q, maximum number of c’ s is and
= number of ways of distributing r
so on.
identical things among n persons when
Since the coefficient of xr in the expansion each person can get zero or more things
will not change if we take a = b = c = . . . = n + r – 1 Cr–1
. . . = 1.
Hence the number of combinations/ Example 5:  Four students A, B, C and D
selections of r things out of n things of plucked some flowers if it is known
which p are alike and are of one kind, q that each of them plucked at least
are alike and are of second, s are alike 3 flower and all together they have
and are of third kind and so on. plucked 16 flowers then in how many
= co-efficient of xr in [( 1 + x + x2 + . . . + ways this can be done?
xp) (1 + x + x2 + . . . + xq) ( 1 + x + x2 + . . Solution:  Let us assume that the number of
. + xs)] flowers plucked by A, B, C and D is a, b,
(ii) Number of combinations/selections c, and d respectively then from the given
of r things out of n things of which condition we have to find the integral
p are alike and are of one kind, q solution of the equation a + b + c + d = 16,
are alike and are of second kind and
where a, b, c, d ≥ 3
remaining ( n – p – q ) things are all
different kind is given by There are different methods to solve a
= co-efficient of xr in [( 1 + x + x2 + …….. question, we will solve this question by four
+ xp)( 1 + x + x2 : …………………. + xq ) different methods-
{( 1+ x) (1 + x) ……….. to (n – p – q )
factors} Method 1: (partitioning theory)
= co-efficient of xr in { (1 + x + x2 … xp) This question is similar to the question - In
( 1 + x + . . . + xq ) ( 1 + x )n – p – q} how many ways 16 identical Chocolates can
(iii) Restricted Combination :-Number of be distributed among 4 persons if each person
selections of r things out of n things of gets at least 3 Chocolates.
which p are alike and are of one kind, q We have solved these type of questions in
are alike and are of second kind, r are
last chapter, 1st we will give 3 chocolates to
alike and are of third kind such that n =
each of 4 persons then remaining chocolates is
p+q+r and each things is taken at least
once 16 – 12 = 4, now we have to distribute these 4
210  Theory of Counting
Let us assumethat the different things = Co-efficientof xr in { ( x + x2 + x3 +
be denoted by the letters a, b, c, … ……… + xp) ( x + x2 + x3 + ……. + xp) ( x
Then consider the term + x2 + x3 + ……….. + xr)}
(1 + ax + a2x2 + . . . + apxp) (1 + bx + b2x2 = Co-efficientof xr in [ ( 1 + x + x2 +
+ . . . + bqxq) (1 + cx + c2x2 + . . . + csxs). . . ……. + xp – 1 ) ( 1 + x + x2 + …… + xq -1 )
( 1 + x + x2 + ….. + xr – 1 )]
In this expansion the exponent of the
letters a, b, c is same as that in x. Now
consider the coefficient of xr in the (iv) Number of terms in the expansion of
expansion it is same as the number of (a1 + a2 + a3 + …. + an)r
ways of taking r of the letters a, b, c, . . . In any term let exponents of a1, a2… an
. . . with the restriction that maximum are x1, x2, … and xn then x1 + x2+… xn
number of a’ s is p, maximum number of =r
b’ s is q, maximum number of c’ s is and
= number of ways of distributing r
so on.
identical things among n persons when
Since the coefficient of xr in the expansion each person can get zero or more things
will not change if we take a = b = c = . . . = n + r – 1 Cr–1
. . . = 1.
Hence the number of combinations/ Example 5:  Four students A, B, C and D
selections of r things out of n things of plucked some flowers if it is known
which p are alike and are of one kind, q that each of them plucked at least
are alike and are of second, s are alike 3 flower and all together they have
and are of third kind and so on. plucked 16 flowers then in how many
= co-efficient of xr in [( 1 + x + x2 + . . . + ways this can be done?
xp) (1 + x + x2 + . . . + xq) ( 1 + x + x2 + . . Solution:  Let us assume that the number of
. + xs)] flowers plucked by A, B, C and D is a, b,
(ii) Number of combinations/selections c, and d respectively then from the given
of r things out of n things of which condition we have to find the integral
p are alike and are of one kind, q solution of the equation a + b + c + d = 16,
are alike and are of second kind and where a, b, c, d ≥ 3
remaining ( n – p – q ) things are all
different kind is given by There are different methods to solve a
question, we will solve this question by four
= co-efficient of xr in [( 1 + x + x2 + ……..
+ xp)( 1 + x + x2 : …………………. + xq ) different methods-
{( 1+ x) (1 + x) ……….. to (n – p – q )
factors} Method 1: (partitioning theory)
= co-efficient of xr in { (1 + x + x2 … xp) This question is similar to the question - In
( 1 + x + . . . + xq ) ( 1 + x )n – p – q} how many ways 16 identical Chocolates can
(iii) Restricted Combination :-Number of be distributed among 4 persons if each person
selections of r things out of n things of gets at least 3 Chocolates.
which p are alike and are of one kind, q We have solved these type of questions in
are alike and are of second kind, r are last chapter, 1st we will give 3 chocolates to
alike and are of third kind such that n =
each of 4 persons then remaining chocolates is
p+q+r and each things is taken at least
16 – 12 = 4, now we have to distribute these 4
once
Number of Integral Solution  211
chocolates among 4 students and that can be Then p + q + r + s = 16 – 12 = 4
done in 4 + 4 – 1C4 – 1 = 7C3 ways. and p, q, r, s ≥ 0
Required number of solution is =7C3
Method 2: (Multinomial Theorem)
Required number = coefficient of x16 in ( x3 + Method 4: (Listing down method)
x4 + … + x7 )4 = coefficient of x16 in x12 ( 1 + x
+ … + x4)4 Here we have to make sets of four positive
= coefficient of x4 in (1 + x + … + x4)4 integers each greater than or equal to 3 whose
4 sum is 16 are
1 – x 5 
= coefficient of x4 in =   { 3, 3, 3, 7}, {3, 3, 4, 6}, { 3, 3, 5, 5}, {3, 4, 4, 5},
 1 – x  { 4, 4, 4, 4}

= coefficient of x4 in (1 – x 5 )4 (1 – x )−4  Now elements of 1st set i.e{ 3, 3, 3, 7} can be
  arranged in 4!/3! = 4 ways
= coefficient of x4 in Elements of 2nd set i.e. {3, 3, 4, 6} can be
[( 1 – 4x5 + 6x10 – ……. ) ( 1 – x)–4 ] arranged in 4!/2! = 12 ways
= coefficient of x4 in ( 1 – x) –4[ leaving terms Elements of 3rd set i.e. { 3, 3, 5, 5) can be
containing power 5 of x greater than 4] arranged in 4!/ 2! 2! = 6 ways
= 7C 4 = 7C 3 Elements of 4th set i.e. {3, 4, 4, 5} can be
arranged in 4!/2! = 12 ways
Method 3: (Non negative integral solution
method ) Elements of 5th set i.e. {4, 4, 4, 4} can be
arranged in 4!/4! = 1 way
Given equation is a + b + c + d = 16,
where a, b, c, d ≥ 3 Required number = 4 + 12 + 6 + 12 + 1 = 35
Let p = a – 3, q = b – 3, r = c – 3, s = d – 3 = 7C 3

Integral Solution from


Concept 3 Multinomial Theorems

(i) Number of non-negative integral (ii) Number of positive integral solution


Solution Equation of equation
a + 2b + 3c + 4d = n a + 2b + 3c + 4d = n
= Co- efficient of xn in = Co- efficient of xn – ( 1 + 2 + 3 + 4 )
[(1 – x )–1( 1 – x2) – 1( 1 – x3 ) – 1( 1 – x4 )–1] In [( 1 – x)– 1( 1 – x2 )– 1( 1 – x3 ) – 1
(1 – x4) –1]
212  Theory of Counting
(iii) Number of non- negative integral (iv) Number of positive integral solution
solution of equation of equation
x1 + 2x2 + 3x3 = … + nxn = m x1 + 2x2 + 3x3 + … + nxn = m
= coefficient of xm in Coefficient of xm in
[(1 + x + x2 + x3 + …) ( 1 + x2 + x4 + x6 + [( x + x2 + x3 + … ) ( x2 + x4 + x6 + … )
…) ( 1 + x3 + x6 + x9 + …) (1 + xn + x2n + (x3 + x6 + x9 …) … ( xn + x2n + x3n + …)]
x3n + ....)] = coefficient of xm – ( 1 + 2 + 3 + … + n ) in
= coefficient of xm in [ 1 + x + x2 + … ) ( 1 + x2 + x4 + …) ( 1 +
[( 1 – x) – 1( 1 – x2 ) – 1( 1 – x3 )–1 … x3 + x6 + …) … ( 1 + xn + x2n + …)]
( 1 – xn )–1] = coeff. of xm – n( n + 1)2 in [(1 – x )– 1
( a – x2 )– 1 ( 1 – x3)- 1 … ( 1 – xn) – 1]

Concept 4 Application of Multinomial Theorem

Permutation with  x x2 xp  x x2 xq  


 1
   + +    
+…+  1
   + +    
+…+  … 
Repetition  1! 2! p !   1! 2! q!  

The number of permutation of r things out
of n things of which p are of one kind, q are Example 6:  In how many ways 4 letters
second kind and so on can be selected from the letters of the
word EXAMINATION.
= r! .co-efficient of xr in
Solution:  In the given word EXAMINATION
 x x2 xp  x x2 xq   there are 11 letters and these are A,A; I,I;
 1
   + +    
+…+  1
   
+ +  
+…+   …  N,N; E,X, M, T, O.
 1! 2! p !   1! 2! q!  
 Method 1: (Listing down cases)
Proof:- Let the different things be denoted by Case (i) When all 4 letters re distinct then 4
a, b, c, ……… consider the product letters can be selected from 8 different letters
i.e, A, I, N, E, X, M, T, & O, this can be done
 2x 2 
 ax a a p x p   bx b2 x 2 bq x q 
1   + + +  ..  +  1 +  +  +…+
     8!
 1! 2! p !   1! 2! q !  in 8C4 ways = = 70
 ( )(4!)
4
...(1)
Case (ii) When 2 letters are identical and
Required number of arrangements = sum of remaining two are distinct, Two identical
all possible terms of the form letters can be selected in 3 ways (either A, or I or
r! /p! /q! apbq …. , where p + q … = r N) and 2 distinct letters can be selected from
= r! .co-efficient of xr in remaining 7 letters in 7C2 = 21 ways so total
number of ways in this case is 3 × 21 = 63.
Number of Integral Solution  213
Case (ii) When 2 letters are identical and So total number of words formed
remaining two are also identical this can be = 1680 + 756 +18 = 2454.
done in 3 ways. Method 2: (Multinomial Theorem)
So total number of ways = 70 + 63 + 3 =136 Required number of permutation or required
Method 2: (Multinomial Theorem) number of words is given by
3
Required number of combinations = coefficient  x x2   x
5
of x4 in ( 1 + x + x2)3 ( 1 + x)5 coefficient of x4 in 4!   1
   + +   1
   
+ 
 1! 2!   1! 
3
 3
= Coefficient of x4 in 1 − x  (1 + x )5  x2 
3

 1 − x 
= coefficient of x4 in 4!  1
   +  x  + 
2
(1  +  x )5

= Coefficient of x4 in (1 − x 3 )3 (1 − x )−3 (1 + x )5 = coefficient of x4 in
Coefficient of x4 in  3 x
6
3 2  5
−3 5 4! (1    +  x ) + + (1 +  x ) x 2 +  ¾ x 4 (1 + x ) (1 +  x )
(1 − x 9 + 3x 6 − 3x 3 ) (1 − x ) (1 + x ) = 136  8 2 

Example 7:  In how many ways 4 letter = 4! {8C4 + 0 + 3/2. 7C2 + 3/4}
word can be formed from the letters of = 1680 + 765 + 18 = 2454
the word EXAMINATION.
Solution:  In the given word EXAMINATION Example 8:  Consider a word ‘PARALLEL’
in how many ways 4 letters of this word
there are 11 letters and these are A,A; I,I;
can be selected?
N,N; E,X, M, T, O.
Solution:  The given word ‘PARALLEL’ has
Method 1:- (Listing down cases)
L-3, A-2, P-1, R-1, E-1 and total number of
Case (i) When all 4 letters re distinct then 4 letters is 8.
letters can be selected from 8 different letters
Method 1: (Listing down method)
i.e, A, I, N, E, X, M, T, & O, this can be done
Case (i) When 3 letters are identical and
8!
in8C4 ways = = 70, so number of words remaining one different, Number of ways is 4.
(4 )(4!)
Case (ii) When 2 letters are identical and
formed in this case is 70(4!) = 70 × 24 = 1680 remaining 2 are identical, means 2 Ls and 2
Case (ii) when 2 letters are identical and As are selected so number of ways is 1
remaining two are distinct, Two identical Case (iii) When 2 letters are identical and
letters can be selected in 3 ways (either A, or
remaining 2 are different, Two identical can
I or N) and 2 distinct letters can be selected
be selected in 2 ways (either L or A) and
from remaining 7 letters in 7C2 = 21 ways so
total number of ways in this case is 3 × 21 = remaining 2 are selected from remaining 4
63.so number of words formed in this case is letters so total number of ways is 2(4C2) = 12
63(4!/2!) = 63 × 12 = 756 ways.
Case (iii) when 2 letters are identical and Case (iv) When all the 4 letters are different
remaining two are also identical this can be then 4 letters can be selected from 5 letters in
done in 3 ways, so number of words formed in 5C ways or in 5 ways.
4
 4!  Total number of ways is 4 + 1 + 12 + 5 = 22
this case is 3  = 3 × 6 = 18
 2!2! 
214  Theory of Counting
Method 2: (Multinomial Theorem) As are selected so number of ways is 1, And
The given word ‘PARALLEL’ has L-3, A-2,  4! 
number of words formed is 1  =6
P-1, R-1, E-1 and total number of letters is 8.  2!2! 
Required number: Case (iii) When 2 letters are identical and
remaining 2 are different, Two identical can
= Coefficient of x4 in
be selected in 2 ways (either L or A) and


(
 )(
 1 +  x  +  x 2 +  x 3 1
)
   +  x  +  x 2 (1
3
   +  x ) 

remaining 2 are selected from remaining 4
letters so total number of ways is 2(4C2) = 12
 4! 
= Coefficient of x4 in (1 – x) – 5(1 – x2)3 ( 1 – x3) ways, And number of words formed is 12  
 2! 
(1 – x4) =12 × 12 = 144
= Coefficient of x4 in (1 – x) – 5(1 – x3 – x4 + x7) Case (iv) When all the 4 letters are different
(1 – 3x2 + 3x4 + x6) then 4 letters can be selected from 5 letters in
5C ways or in 5 ways, And number of words
= Coefficient of x4 in (1 – x)–5(1 – x3 + 2x4 – 3x2) 4
formed is 5(4!) =120
= Coefficient of x4 in ( 1 – x)–5 – Coefficient of
x in ( 1 – x) – 5 + 2 Coefficient of x6 in (1 – x) – 5 So total number of words formed is 16 + 6 +
– 3 Coefficient of x2 in ( 1 – x) – 5 144 + 120 = 286

= (8C4) –( 5) + (2) – 3(6C2) = 22 Method 2:- (Multinomial Theorem)

Example 9:  Consider a word ‘PARALLEL’ The given word ‘PARALLEL’ has L-3, A-2,
in how many ways 4 letter word can P-1, R-1, E-1 and total number of letters is 8.
be formed from the letters of the word Since the number of permutation of r things
PARALLEL out of n things of which p are of one kind, q
Solution:  The given word ‘PARALLEL’ has are second kind and so on
L-3, A-2, P-1, R-1, E-1 and total number of = r! .co-efficient of xr in
letters is 8.  x x2 xp  x x2 xq  
 1
   + +    
+…+  1
   + +    
+…+  … 
Method 1:- (Listing down method)  1! 2! p !   1! 2! q!  

Case (i) When 3 letters are identical and Required number of words is given by-
remaining one different, Number of ways is 4.
= Coefficient of x4 in
 4! 
And number of words formed is 4   = 16 3
 3!   x x2   x x2 x3   x
4!  1
   + ! +   1
   + + ! +   1   + 
Case (ii) When 2 letters are identical and  1 2!   1! 2 3!   1! 
remaining 2 are identical, means 2 Ls and 2
= 24 ( 2 + 6 + 7/2 + 5/12) = 2863
Number of Integral Solution  215

Concept 5 De- arrangement

What is De-arrangement? To understand


 1 1 1 
this let us assume a situation where three = (4!)1 − 1 + − +
friends A, B and C stay in a hostel, on a  2 6 24 

particular night none of them slept in their 12 − 4 + 1 
own room in how many ways this is possible. = (4!)  = 12 − 4 + 1 = 9
 24 
To understand this situation we will list down
all the possibilities- 1 1 1 1 1 1
D(5) = (5!) – + – + − 
Let us assume that room of A is ‘a’, that of B  0! 1! 2! 3! 4! 5! 
is ‘b’ and that of C is ‘c’ then
 1 1 1 1 
Own Person Case (i) Case (ii) = (5!)1 − 1 + − + −
 2 6 24 120 
Rooms
a A b c  60 − 20 + 5 − 1 
= (120 )  = 44
b B c a  120 

c C a b Partial De-arrangement - Consider n articles
In this case we have seen that 3 articles can be only r articles are de arranged remaining
de arranged in two ways. Now lets generalize (n – r) articles are at their own place. Then
it- this can be done in (nCr)[D(r)]
If  n distinct items are arranged in a row, then
the number of ways they can be rearranged such Example 10:  Suvrojyoti writes letters to
that none of them occupies its original position is his 5 students but while putting letters in
called De-arrangement, and it can be calculated the addressed envelopes he was not able
with the help of formula- to put a single letter in its correct envelope
in how many ways he can do so?
1 1 1 1
D(n) = n !  – + – +     +
(−1)1  Solution:  As per the given condition we need
 0! 1! 2! 3! n !  to find the value of D(5)
Note: De-arrangement of 1 object is not 1 1 1 1 1 1
possible. D(5) = (5!) – + – + − 
 0! 1! 2! 3! 4! 5! 
D(1) = 0
 1 1 1 1 
D(2) = 1 = (5!)1 − 1 + − + −
 2 6 24 120 
D(3) = 2
 60 − 20 + 5 − 1 
1 1 1 1 1 = (120 )  = 44
D(4) = (4!) – + – +   120 
 0! 1! 2! 3! 4! 
216  Theory of Counting

1
Concept Applicator (CA)
Ideal Time Apply your concepts with easy and
10 Min. conceptual questions

1. Find the number of non-negative integral 6. Find the number of positive integral
solution of the equation x + y = 20. solution of the equation
(a) 21 (b) 20 w + x + y + z = 20.
(a) 22C (b) 20C3
(c) 19 (d) None of these 3
(c) 23C
3 (d) None of these
2. Find the number of positive integral
7. Find the number of non negative integral
solution of the equation x + y = 20. solution of the equation a+b+c+d+e = 15
(a) 21 (b) 20 (a) 19C (b) 19C16
5
(c) 14C
(c) 19 (d) None of these 4 (d) None of these
3. Find the number of non-negative integral 8. Find the number of positive integral
solution of the equation x + y + z = 20. solution of the equation
22C a + b + c + d + e = 15
(a) 3 (b) 22C2
(a) 14C (b) 14C11
(c) 23C 10
2 (d) None of these 14C
(c) 6 (d) None of these
4. Find the number of positive integral
9. Find the number of non negative integral
solution of the equation x + y + z = 20. solution of the equation
(a) 22C (b) 22C2
3 a + b + c +…+ z = 5
(c) 23C
2 (d) None of these (a) 30C (b) 30C5
9
(c) 30C
5. Find the number of non-negative integral 26 (d) None of these
solution of the equation 10 Find the number of positive integral
solution of the equation
w + x + y + z = 20.
a + b + c +…+ z = 5
(a) 22C (b) 21C3
3 30C
23C
(a) 9 (b) 30C5
(c) 3 (d) None of these 30C
(c) 26 (d) None of these

Response Grid
1. a b c d 2. a b c d 3. a b c d 4. a b c d 5. a b c d
6. a b c d 7. a b c d 8. a b c d 9. a b c d 10. a b c d
218  Theory of Counting

3
Concept Cracker (CC)
Ideal Time Boost up your confidence with good
30 Min. questions

1. 100 students are standing in a straight (a) 105 (b) 510


line. In how many ways, two students can 10
(c) C5 (d) None of these
be selected such that there are at most 7. What is the number of outcomes when 20
10 students between the two selected identical cubical dice (numbered from 1
students. to 6) are thrown?
(a) 90C2 (b) 100C2 – 90C2 (a) 620 (b) 206
(c) 100C2 – 88C2 (d) None of these 25
(c) C5 (d) None of these
2. Find the number of positive integral 8. Find the number of integral solutions of
solution for a + b + c < 10. the equation ‘a + b + c + d = 16’, where a,
(a) 74 (b) 77 b, c, d ≥ 3
(c) 79 (d) None of these (a) 16C3 (b) 16C14
7
(c) C3 (d) None of these
3. Metro train route in Kolkata is from
Garia to Dumdum, if there are 20 stations 9. Hasan, a book lover, went to his favorite
between Garia to Dumdum, In how many book store from where he purchased
ways can a train be made to stop at 5 of 30 books of 8 different languages such
these 20 intermediate stations if no two that he has at least 2 books from each
of which are consecutive? language, in how many ways he can do
(a) 16C4 (b) 16C5 so if the book store has unlimited supply
17 of books?
(c) C5 (d) None of these
(a) 21C7 (b) 29C7
4. What is the number of ways of selecting
(c) 27C7 (d) None of these
15 flowers out of unlimited number of
red, green, blue, yellow and white colored 10. What is the number of non – negative
flowers? integral solutions of the equation
  a + b + c + 4d = 20?
(a) 16C4 (b) 19C5
(c) 19C4 (d) None of these (a) 21C20 (b) 22C20
18
(c) C16 (d) None of these
5. Find the number of integral solution of
the equation a + b + c + d = 0 if a≥ –10, 11. What is the number of non – negative
b≥ – 10, c ≥ –10, and d ≥ –10. integral solutions of the equation
(a) 39C3 (b) 43C3 x + y + z = 10 and x ≤ 6, y ≤ 7 and z ≤ 8
49
(c) C3 (d) None of these OR
6. At an ice cream parlor, ice creams of 10 In how many ways a bill of ` 10 can be
different brands are available. Trina paid by three students Hasan, Susmit
went to that parlor in order to buy only and Madhur, if they have ` 6, 7 and 8
5 ice cream, in how many different ways respectively.
she can place an order of 5 ice cream?
218  Theory of Counting

3
Concept Cracker (CC)
Ideal Time Boost up your confidence with good
30 Min. questions

1. 100 students are standing in a straight (a) 105 (b) 510


line. In how many ways, two students can 10
(c) C5 (d) None of these
be selected such that there are at most 7. What is the number of outcomes when 20
10 students between the two selected identical cubical dice (numbered from 1
students. to 6) are thrown?
(a) 90C (b) 100C2 – 90C2
2 (a) 620 (b) 206
(c) 100C2 – 88C2 (d) None of these 25
(c) C5 (d) None of these
2. Find the number of positive integral 8. Find the number of integral solutions of
solution for a + b + c < 10. the equation ‘a + b + c + d = 16’, where a,
(a) 74 (b) 77 b, c, d ≥ 3
(c) 79 (d) None of these (a) 16C3 (b) 16C14
7
(c) C3 (d) None of these
3. Metro train route in Kolkata is from
Garia to Dumdum, if there are 20 stations 9. Hasan, a book lover, went to his favorite
between Garia to Dumdum, In how many book store from where he purchased
ways can a train be made to stop at 5 of 30 books of 8 different languages such
these 20 intermediate stations if no two that he has at least 2 books from each
of which are consecutive? language, in how many ways he can do
(a) 16C4 (b) 16C5 so if the book store has unlimited supply
17 of books?
(c) C5 (d) None of these
(a) 21C7 (b) 29C7
4. What is the number of ways of selecting
(c) 27C7 (d) None of these
15 flowers out of unlimited number of
red, green, blue, yellow and white colored 10. What is the number of non – negative
flowers? integral solutions of the equation
  a + b + c + 4d = 20?
(a) 16C4 (b) 19C5
(c) 19C4 (d) None of these (a) 21C20 (b) 22C20
18
(c) C16 (d) None of these
5. Find the number of integral solution of
the equation a + b + c + d = 0 if a≥ -10, 11. What is the number of non – negative
b≥ – 10, c≥ –10, and d ≥ –10. integral solutions of the equation
(a) 39C3 (b) 43C3 x + y + z = 10 and x ≤ 6, y ≤ 7 and z ≤8
49
(c) C3 (d) None of these OR
6. At an ice cream parlor, ice creams of 10 In how many ways a bill of ` 10 can be
different brands are available. Trina paid by three students Hasan, Susmit
went to that parlor in order to buy only and Madhur, if they have ` 6, 7 and 8
5 ice cream, in how many different ways respectively.
she can place an order of 5 ice cream?
Number of Integral Solution  219
OR the word ‘INDEPENDENT’ and Q is the total
Anshuman went to a park where he saw number of ways of forming five letter word
3 different varieties of flowers namely from the letters of the word ‘ INDEPENDENT’
Rose, Calendula, and Lavender. He then
plucked total 10 flowers, if it is given that 13. What is remainder when P divided by 9?
in the number of flowers are 6 (Rose), 7 (a) 8 (b) 3
(Calendula) and 8 (Lavender), in how (c) 7 (d) None of these
many ways he can do so?
14. What is remainder when Q divided by
(a) 47 (b) 49 9?
(c) 43 (d) None of these
(a) 8 (b) 3
12. How many positive integers less than (c) 7 (d) None of these
1000000 have the sum of the digits 18 ?
15. In how many ways six friends Hasan,
(a) 7722 (b) 33649 Madhur, Susmit, Anshuman, Abhishek
(c) 25927 (d) None of these and Ricky can pay a bill of less than $18
Direction for question number 13 if none of them have more than $6?
and 14:- Consider the letters of the word (a) 17C11 – 6. 11C5 (b) 17C11– 11C5
‘INDEPENDENT’ if P is the total number of 17 11
(c) C11 + 6. C5 (d) None of these
ways of selecting five letters from the letters of

Response Grid
1. a b c d 2. a b c d 3. a b c d 4. a b c d 5. a b c d
6. a b c d 7. a b c d 8. a b c d 9. a b c d 10. a b c d
11. a b c d 12. a b c d 13. a b c d 14. a b c d 15. a b c d
220  Theory of Counting

Solutions
Concept Applicator (CA)

1. (a) 2. (c) 3. (b) 4. (d) 5. (c) 6. (d)


7. (d) 8. (a) 9. (b) 10. (d)

1. (a) Since we have to find the number of 7. (d) The question is similar to the
non negative integral solution hence condition that we have to distribute
required number is 15 chocolates to 5 students and that
20+2–1C 21C = 21 can be done in
2–1 = 1
2. (c) Since we have to find the number of   15+5–1C5-1 = 19C4 = 19C15
positive integral solution hence required 8. (a) The question is similar to the
number is 20–1C2–1 = 19C1= 19 condition that we have to distribute
3. (b) Since we have to find the number of 15 chocolates to 5 students such that
non negative integral solution hence each student gets at least 1 chocolate
required number is 20+3–1C3–1 = 22C2. and that can be done in
15–1C =14C4 = 14C10 ways
4. (d) Since we have to find the number 5–1
of positive integral solution hence 9. (b) The question is similar to the
required number is condition that we have to distribute
20 – 1C 19C . 5 chocolates to 26 students and that
3–1 = 2
5. (c) Since we have to find the number of can be done in 5+26–1C26–1 =30C25 = 30C5
non negative integral solution hence 10. (d) The question is similar to the
required number is condition that we have to distribute
20 + 4 – 1C
4–1 =
23C .
3
5 chocolates to 26 students such
6. (d) Since we have to find the number that each of them receive at least 1
of positive integral solution hence chocolate but 5 < 26 hence number
required number is 20 – 1C4 – 1 = 19C3. of ways is 0.

Concept Builder (CB)


1. (b) 2. (a) 3. (c) 4. (c) 5. (d) 6. (a)
7. (b) 8. (d) 9. (c) 10. (b)

1.
(b) Since as per the give condition so z = 2+c or (2 + a) + (2+b) + (2+c) =
x > 1, so assume that x = 2 + a here 20 or a+b+c = 14 and we need to find
a is non negative integer, the number of non negative integral
y > 1 so y = 2 + b and similarly z > 1 solution of the equation a+b+c = 14
which is 14+3–1C3–1 = 16C2
Number of Integral Solution  221
2. (a) Since as per the give condition or  x + b + c = 23
x > 1, so assume that x = 2 + a here and we need to find the number of
a is non negative integer, non negative integral solution of
y > 2 so y = 3 + b and similarly z > 3 the equation x + b + c = 23 which is
23 + 3 –1C 25C = 25C
so z = 4+c or (2 + a) + (3+b) + (4+c) = 3–1 = 2 23
20 or a+b+c = 11 and we need to find 7. (b) Let the number of Roses, , Calendula,
the number of non negative integral Jasmine and Lavender that Hasan
solution of the equation a+b+c= 11 plucked is a, b, c and d respectively
which is 11+3–1C3–1 = 13C2 then from the given information
3. (c) Just assume that you have 100 a + b + c + d = 20 and a ≥0, b ≥ 0,
chocolates and you have to distribute c ≥ 0, d ≥ 0.
it among 5 students such that they Hence required number of ways is
should get more than 5, 10, 15, 20 20+4–1C
4–1=
23C
3
and 25 chocolates respectively.
8. (d) Let the number of Roses, , Calendula,
1st give them 6, 11, 16, 21 and 26 Jasmine and Lavender that Hasan
chocolates then remaining chocolates plucked is a, b, c and d respectively
is 100 – (6+11+16+21+26) = 20 which
then from the given information a +
can be distributed among 5 students
b + c + d = 20 and a ≥1, b ≥ 1, c ≥ 1, d ≥ 1.
in 20+5–1C5–1 = 24C4
4. (c) As per the given condition x1 + x2 Hence required number of ways is
20–1C 19C
+…+x10 = 50 such that each variable 4–1 = 3
is more than or equal to 2. 9. (c) From the given equations we can
Assume x1 = y1 +2 such that y1≥ 0 deduce b + c + e = 10 and a + d = 10
similarly x2 = y2 +2 such that y2≥ 0 Number of non negative integral
hence we will have a new equation solutions of the equation
that is y1 + y2 + …. + y10 = 50 – 20
= 30, so required number of ways is b + c + e =10 is 10+3 – 1C3 – 1 = 12C2 = 66
30+10–1C
10–1 =
39C
9
Number of non negative integral
5. (d) Since as per the give condition solutions of the equation a + d=10 is
10+2–1C 11C = 11
x > –1, so x is non negative integer, 2–1 = 1
this question is same as question Hence required number of solution
number 3 of Concept Applicator (CA) is 66 × 11 = 726
Since we have to find the number of 10. (b) Maths faculty can distribute 10
non negative integral solution hence
identical chocolates among 3 students
required number is 20+3–1C3–1 = 22C2.
in 10+3–1C3–1 = 12C2 ways, similarly
6. (a) Since as per the give condition
x > – 1, so x is non negative integer, reasoning and English faculty can
distribute in 22C2 and 32C2 ways.
y > – 2 so y = –1 + b and similarly
Total number of ways is (12C2)(22C2)
z > 3 so z = – 2 + c
(32C2)
or (x) + (–1 + b) + (– 2 + c) = 23
222  Theory of Counting

Concept Cracker (CC)

1. (b) 2. (d) 3. (b) 4. (c) 5. (b) 6. (d)


7. (c) 8. (c) 9. (a) 10. (d) 11. (a) 12. (c)
13. (d) 14. (a) 15. (a)

1. (b) Let 100 students are standing in a There are ‘b’ stations between 1st
straight line, stoppage and 2nd stoppage then b > 0
Let after ‘a’ students 1st student is There are ‘c’ stations between 2nd
selected, and there are ‘b’ students stoppage and 3rd stoppage then c > 0
between the two students and after 2nd There are ‘d’ stations between 3rd
student there are ‘c’ students then as stoppage and 4th stoppage then d > 0
per the given condition a + b + c = 98
There are ‘e’ stations between 4th
As per the given condition maximum stoppage and 5th stoppage then e > 0
value of b is 10.
And there are ‘f’ stations between 5th
Now consider the reverse condition stoppage and the final destination
that is b > 10 and a + b + c = 98 then Dumdum then f ≥ 0
number of integral solution is
Since total number of stations is 20
98 – 10 + 3 – 1C 90C .
3–1 = 2 out of which 5 stations has stoppage
Without restriction we can select hence-
2 students out of 100 students in The equation is a + b + c + d + e + f =
100C .
2 15 and a ≥ 0, b > 0, c > 0, d > 0, e > 0,
So required number of ways is and f ≥ 0.
100C – 90C
2 2 So required number of positive
2. (d) Since a, b and c are positive integers integral solution is 16C5
hence we have following cases- Alternately :
a + b + c = 3 then number of positive Total number of stations is 20 out of
integral solution is 3 – 1C3 – 1 = 2C2 = 1. which train stopped at 5 stations and
a + b + c = 4 then number of positive doesn’t stop at 15 stations, consider
integral solution is 4 – 1C3 – 1 = 3C2 = 3. these 15 stations a1, a2, and a15 this
a + b + c = 5 then number of positive will create 16 places, so we just have
integral solution is 5 – 1C3 – 1 = 4C2 = to select 5 out of 16 and that can be
6 and so on done in 16C5 ways.
a + b + c = 9 then number of positive 4. (c) From the given condition a + b + c
integral solution is 9–1C3–1 = 8C2 = + d + e = 15, here, a, b, c, d and e
28 is the number of red, green, blue,
yellow and white colored flowers
Hence required summation is 1 + 3+
respectively.
6 +10 + 15 +21+ 28 = 84
The condition is similar to number
3. (b) Let 1st stoppage is after ‘a’ stations
of non negative integral solution of
then a ≥ 0,
Number of Integral Solution  223
the equation a+b+c+d+e = 15, which Method 2: (Multinomial
is 15 + 5 – 1C5 – 1 = 19C4 Theorem)
5. (b) Given equation is a + b + c + d = 0 Required number = coefficient of x16
and a ≥ –10, b ≥ –10, c ≥ –10, and d ≥ –10. in ( x3 + x4 + … + x7 )4 = coefficient of
Let us assume that a = –10 + a1, x16 in x12 ( 1 + x + … + x4)4
b = –10 + b1, c = –10 + c1 and d = –10 + d1 = coefficient of x4 in ( 1 + x + … + x4)4
Then we will have to find the number 4
1 – x 5 
of non negative integral roots of the = coefficient of x4 in =  
equation a1 + b1+ c1+ d1 = 40, = coefficient of x4 in  1 – x 
So required number of integral solution (1 – x 5 )4 (1
   – x )−4 
is given by 40 + 4 – 1C4 – 1 = 43C3.  
6. (d) Let the number of ice creams of 1st = coefficient of x4 in
brand she ordered is x1, that of 2nd [( 1 – 4x5 + 6x10 – ...) ( 1 – x)–4 ]
brand is x2 and so on for 10th brand = coefficient of x4 in ( 1 – x)–4
x10, then from the given condition = 7C 4 = 7C 3
x1+ x2 + … + x10 = 5 and xi ≥ 0,
[leaving terms containing power 5
Required number of solution is of x greater than 4]
10 + 5 – 1C 14C .
5–1 = 4
Method 3: (Non negative integral
7. (c) Let the number of times that ‘1’ solution method )
appeared is x1, that of 2 is x2 and so
on for 6 is x6, then from the given Given equation is
condition x1+ x2+…+ x6 = 20 and a + b + c + d = 16, where a, b, c, d ≥ 3
xi ≥ 0 Let p = a – 3, q = b – 3, r = c – 3,
Required number of solution is s=d–3
20 + 6 – 1C 25C .
6–1 = 5 Then p+q+r+s = 16 – 12 =4 and p, q,
8. (c) We will solve this question by four r, s ≥ 0
different methods- Required number of solution is
Method 1: (partitioning theory) =7C3
This question is similar to the Method 4: (Listing down method)
question - In how many ways Here we have to make sets of four
16 identical Chocolates can be positive integers each greater than
distributed among 4 persons if each or equal to 3 whose sum is 16 are
person gets at least 3 Chocolates. { 3, 3, 3, 7}, {3, 3, 4, 6}, { 3, 3, 5, 5},
We have solved these type of questions {3, 4, 4, 5}, { 4, 4, 4, 4}
in last chapter, 1st we will give 3 Now elements of 1st seti.e{ 3, 3, 3, 7}
chocolates to each of 4 persons then can be arranged in 4!/3! = 4 ways
remaining chocolates is 16 – 12 = 4,
now we have to distribute these 4 Elements of 2nd set i.e. {3, 3, 4, 6}
chocolates among 4 students and can be arranged in 4!/2! = 12 ways
that can be done in 4 + 4 – 1C4 – 1 = Elements of 3rd set i.e. {3, 3, 5, 5} can
7C ways. be arranged in 4!/ 2! 2! = 6 ways
3
Elements of 4th set i.e. {3, 4, 4, 5} can
be arranged in 4!/2! = 12 ways
224  Theory of Counting
Elements of 5th set i.e {4, 4, 4, 4} can = 1 + 6C4 + 10C8 + 14C12 + 18C16 + 22C20
be arranged in 4!/4! = 1 way =1 + 15 + 45 + 91 + 153 + 231 = 536.
Required number = 4 + 12 + 6 + 12 + 1 Method 2: (Listing down method)
= 35 = 7C3 Here maximum value of d is 5 hence
9. (a) Method 1: we have 6 different cases-
Let the number of books of first, Case I: When d = 0, In this case
second, … , eighth languages that a + b + c = 20.
Hasan has bought is x1, x2, … , x8 Number of non-negative integral
respectively, then solutions
x1 + x2 + … + x8 = 30, where x1, x2, …, = n + r – 1Cr = 3 + 20 – 1C20 = 22C20
x8 ≥ 2 Case II: When d = 1, number of non-
Let yi = xi – 2 such that yi ≥ 0 negative integral solutions
Then = 3 + 16 – 1C16 = 18C16
y1 + y2 + … + y8 = ( x1 + x2 + … + x8) – 16 Case III: When d = 2, number of non-
Or, y1 + y2 + … + y8 = 14, negative integral solutions = 14C12
where y1, y2, …, y8 ≥ 0 Case IV. When d = 3, number of non-
negative integral solutions = 10C8
Required number of ways
CaseV. When d = 4, number of non-
  = 8 + 14 – 1C14 = 21C14 = 21C7 negative integral solutions = 6C4
Method 2: (by Multinomial Case VI. When d = 5, number of
Theorem) non-negative integral solutions = 2C0
Required number of ways is Required number = 22C20 + 18C16 +
= coefficient of x30 in (x2 + x3 +...+ x16)8 14C + 10C + 8C + 2C .
12 8 4 0
= coefficient of x30 in x16 (1 + x +...+ 11. (a) In this question lower as well as
x14)8 upper limits of variables are given
= coefficient of x14 in (1 + x +... + x14)8 hence we have to use multinomial
theorem.
8
 15 
The required number of ways
= coefficient of x14 in 1 – x 
 1 – x  = coefficient of x10 in (1 + x + x2 + ... + x6)
(1 + x + x2 + ... x7) (1 + x + x2 + ... + x8)
= coefficient of x14 in ( 1 – x)– 8
= coefficient of x10 in
= 21C14 = 21C7.
1 – x 7  1 – x 8  1 – x 9 
10. (d) Method 1: (Multinomial Theorem)   x    x   
Required number of solution is  1 − x   1 − x   1 – x 
= coefficient of x20 in ( 1 – x) – 1 = coefficient of x10 in (1 – x7) (1 – x8)
( 1 – x) – 1 ( 1 – x) – 1 ( 1 – x4 ) – 1 (1 – x9) (1 – x)–3
= coefficient of x20 in = coefficient of x10 in (1 – x7 – x8 – x9)
(1 + 3C1x + 4C2x2 + 5C3x3 + ... 12C10x10)
[( 1 + x) – 3 ( 1 – x4 ) – 1 ]
= 12C2 – 5C3 – 4C2 – 3C1
= coefficient of x20 in (1 + 3C1x +
4C x2 + 5C x3 + … + 22C x20 + …) = 66 – 10 – 6 – 3 = 47.
2 3 20
(Leaving the powers greater than 10)
(1 + x4 + x8 + x12 + x16 + x20)
Number of Integral Solution  225
12. (c) Since the number is between 1 and (iii) Two D’s and three out of I, E,
1000000 so number must be in the P, N, T, then number of ways is
form of a1, a2, a3, a4, a5, a6 1 × 5C3 = 10.
here 0 ≤ ai ≤ 9, i = 1, 2, 3, …, 6 So total number of ways in this case
As per the question sum of the digits is 10 + 10 + 10 = 30.
= 18 or a1 + a2 + a3 + a4 + a5 + a6 = 18, Case (iii) When out of 5 selected
0 ≤ ai ≤ 9, i = 1, 2, 3, …,6 letters two letters are identical and
Required number = coefficient of x18 in are of one type, two are identical
( 1 + x + x2 + ……. + x9)6 and are of second type and rest one
is different that means 2 out of E, N
6
1 – x10  and D is selected (in 3 ways) and 1
= coefficient of x18 in   more letter from remaining 4 letters
 1 – x  are selected hence required number
= coefficient of x18 in of ways is 3 × 4 = 12.
Case (iv) When three letters are
(1 – x10 )6 (1
   – x )−  6 
  identical and remaining two are
different, that means 2 out of E,
= coefficient of x18 in
and N is selected (in 2 ways) and 2
[(1 – 6C1 x10) (1 – x)–6] more letter from remaining 5 letters
[Ignoring terms having powers of x are selected (5C2 = 10 ways) hence
greater than 18] required number of ways is
= [coefficient of x18 in (1 – x)–6 ] –    = 2 × 10 = 20.
[6C1 . coefficient of x8 in (1 – x)–6] Case (v) When three letters are
= 6 + 18 – 1C18 – 6.6 + 8 – 1 C8 identical and remaining two are
= 23C5 – 6.13C5 = 33649 – 7722 identical. For identical 3 letters we
can make selection in 2 ways (Either
= 25927.
E or N) and for remaining two
13. (d) Total number of letters = 11 identical in 2 ways so total number
(E–3, N–3, D–2, , I–1, P–1 and T–1) of ways in this case is 2x2 = 4 ways.
Method 1: (Listing down with Required number = 6 + 30 + 12 + 20
different cases) + 4 = 72.
Case (i) When all the 5 letters are So P = 72 and required remainder is 0
different then number of ways to
Method 2: (Multinomial Theorem)
select 5 letters out of 6 is 6C5 = 6
Total number of letters = 11(E – 3, N
Case (ii) When the two letters are
– 3, D – 2, , I – 1, P – 1 and T – 1).
identical and remaining three are
different then we will have following Required number of selections of 5
sub cases- letters = coefficient of x5 in [(1 + x +
x2 + x3 )2 (1 + x + x2)(1 + x)3]
(i) Two E’s and three out of I, N,
P, D, T then number of ways is = coefficient of x5 in [( 1 + x2 + x4 +
1 × 5C3 = 10. x6 + 2x + 2x2 + 2x3 + 2x3 + 2x4 + 2x5)
(1 + x + x2 )(1 + x)3].
(ii) Two N’s and three out of I, E,
P, D, T, then number of ways is = coefficient of x5 in [(1 + 2x + 3x2 +
1 × 5C3 = 10. 4x3 + 3x4 + 2x5) {(1 + x + x2) (1 + 3x
+ 3x2 + x3)}].
226  Theory of Counting
[Ignoring terms containing powers one is different that means 2 out of
of x greater than 5] E, N and D is selected (in 3 ways)
= coefficient of x5 in [( 1 + 2x + 3x2 and 1 more letter from remaining
+ 4x3 + 3x4 + 2x5){( 1 + x + x2) + 4 letters are selected hence
(3x + 3x2 + 3x3) +( 3x2 + 3x3 +3x4 ) + required number of ways is 3x4 =
12, and number of words formed is
(x3 + x4 + x5)}]
5!
= coefficient of x5 in [( 1 + 2x + 3x2 + (12) 2!2!  =12 × 30 = 360
4x3 + 3x4 + 2x5)( 1 + 4x + 7x2 + 7x3 +  
4x4 + x5)] Case (iv) When three letters are
= 1 + 8 + 21 + 28 + 12 + 2 = 72 identical and remaining two are
different, that means 2 out of E,
So P = 72 and required remainder is 0
and N is selected (in 2 ways) and 2
14. (a) Total number of letters = 11
more letter from remaining 5 letters
(E–3, N–3, D – 2, , I – 1, P–1 and T – 1) are selected (5C2 = 10 ways) hence
Method 1: (Listing down with required number of ways is = 2 × 10
different cases) = 20, and number of words formed is
Case (i) When all the 5 letters are 5!
different then number of ways to
(20) 3!  = 20 × 20 = 400
 
select 5 letters out of 6 is 6C5 = 6 and
Case (v) When three letters are
number of words formed is 6(5!) = 720.
identical and remaining two are
Case (ii) When the two letters are
identical. For identical 3 letters we
identical and remaining three are
different then we will have following can make selection in 2 ways (Either
sub cases- E or N) and for remaining two
(i) Two E’s and three out of I, N, identical in 2 ways so total number
P, D, T then number of ways is of ways in this case is 2 × 2 = 4 ways
1 × 5C3 = 10 and number of words formed is
(ii) Two N’s and three out of I, E, 5!
P, D, T, then number of ways is
(4 ) 3!2! 
= 4 × 10 40
 
1 × 5C3 = 10 Required number
(iii) Two D’s and three out of I, E, = 720 + 1800 + 360 + 400 + 40
P, N, T, then number of ways is
1 × 5C3 = 10 = 3320
So total number of ways in this case So Q = 3320 and required remainder
is 10 + 10 + 10 = 30, and number of is 8
 5!  Method 2: (Multinomial
words formed is (30)   = 30 × 60
= 1800.  2!  Theorem)
Case (iii) When out of 5 selected Since the number of permutation of
letters two letters are identical and r things out of n things of which p
are of one type, two are identical are of one kind, q are second kind
and are of second type and rest and so on
Number of Integral Solution  227

 x x2 xp  x x2 xq  
= r! .co-efficient of xr in  1
   + +    
+…+  1
   + +    
+…+  … 
 1! 2! p !   1! 2! q!  

Required number of 5 letter words formed
 2 3
x x2 x3   x x2   x
= Coefficient of x5 
in (5!)   1    + +  +   1   + +      +  
1
 1! 2! 3!   1! 2!   1!  
 
= Coefficient of x5 in
 x4 x3 x4 x5   x2  
(5!) 1  +  x 2 + 4 +  2x  +  x 2 + 3 +  x 3  + 3 + 6   1  +  x  + 2  (1 +  3x  +  3x 2 +  x 3 )
   

= Coefficient of x5 in
 4 x 3 7x 4 x 5   13 11 5 x5  
(5!) 1  +  2x  +  2x 2  + 3 + 12  + 6   1  +  4x  + 2 x 2 + 2 x 3 + 2 x 4 + 2  
   

1
= (5!) ( + 5 + 11 + 26/3 + 7/3 + 1/6)
2
= (5!) (3 + 30 + 66 + 525 + 14 + 1/6) = 20 × 166 = 3320
So Q = 3320 and required remainder is 8
15. (a) Let x1, x2, ... , x6 be the amount with Hasan, Madhur, Susmit, Anshuman, Abhishek
and Ricky respectively.
From the given condition x1 + x2 + x3 + ... + x6 ≤ 17.
Let us assume a variable x7 ≥ 0, such that x1 + x2 + . . . + x6 + x7 = 17 here 1 ≤ xi≤ 6
where i = 1, 2, . . . , 6 and 0 ≤ x7
Required number of solution = coefficient of x17 in ( x + x2 + . . . + x6 )6 (1 + x + x2 + x3 + ...)
= coefficient of x11 in (1 – x6)6 (1 – x)–7 = coefficient of x11 in (1 – 6x6) (1 – x)–7
= coefficient of x11 in ( 1 – 6x6) ( 1 + 7C1x + . . . + 10C4x4 + 11C5x5 + . . . 17C11x11 +. . . )
= 17C11 – 6. 11C5
228  Theory of Counting
Part B
Concept 1 Integral Solution

In last chapter we have studied portioning groups or ‘r’ persons when each group can get
theory, as per that theory. Number of ways of any number of things ≤ n is (n + r – 1)Cr – 1.
dividing ‘n’ identical things among ‘r’ distinct

Concept 2 Multinomial Theorem

As per the multinomial theorem: −1


(iii) (1 − x ) = 1 − x + x 2 − x 3 + x 4 −………….
nx n (n − 1)x
2
(i) (1 + x )n = 1 + + +… 2
1! 2!  1  2 x  3x 2  4 x 3  5x 4 
(iv) 1  x
( −n)( − x ) ( −n) (−n − 1)( − x )2
(ii) (1 − x )−n = 1 − + +…
1! 2!

Integral Solution from


Concept 3 Multinomial Theorem
(i)
Number of non-negative integral (ii) Number of positive integral solution of
Solution Equation equation
a + 2b + 3c + 4d = n a + 2b + 3c + 4d = n
= Co- efficient of xn in = Co-efficient of xn – (1 + 2 + 3 + 4)
[( 1 – x )–1 ( 1 – x2) – 1 (1 – x3 )– 1( 1 – x4 )–1] in [(1 – x)–1( 1 – x2 )–1 ( 1 – x3) – 1(1 – x4)–1]
Number of Integral Solution  229
(iii) Number of non- negative integral solution Coefficient of xm in
of equation
[( x + x2 + x3 + …)(x2 + x4 + x6 + …)
x1 + 2x2 + 3x3 + … + nxn = m
= coefficient of xm in ( x3 + x6 + x9 …)…(xn + x2n + x3n + …)]
[( 1 + x + x2 + x3 + …)( 1 + x2 + x4 + x6 + …)
= coefficient of xm – ( 1 + 2 + 3 + … + n ) in
( 1 + x3 + x6 + x9 + …)(1 + xn + x2n + x3n + ...)]
= coefficient of xm in [1 + x + x2 + …)(1 + x2 + x4 + …)
[(1 – x)–1(1 – x2)–1(1 – x3)–1 … (1 – xn)– 1] (1 + x3 + x6 + …)…(1 + xn + x2n + …)]
(iv) Number of positive integral solution of n(n + 1)
m–
equation = coeff. of x 2 [(1 – x)–1(1 – x2 )–1
in
x1 + 2x2 + 3x3 + … + nxn = m (1 – x3)–1 … ( 1 – xn)–1]

Application of
Concept 4 Multinomial theorem
Permutation with Repetition
The number of permutation of r things out of n things of which p are of one kind, q are second
kind and so on
 x x2 xp   x x2 xq  
= r! .co-efficient of xr in   1 + +    
+…+    1 + +    
+…+  … 
1! 2! p!   1! 2! q!  
 
Proof: Let the different things be denoted by a, b, c, ………
consider the product

 2
 ax a2 x a p x p   bx b 2 x 2 bq x q 
  
1  
+ + +  ..  
+  1 
+  
+    
+…+    ...(1)
 1! 2! p !   1! 2! q ! 

Required number of arrangements = sum of all possible terms of the form
R! /p! /q! apbq …. , where p + q … = r

 x x2 xp   x x2 xq  
= r! .co-efficient of xr in   1 + +    
+…+   
1  
+ +  
+…+   … 
1! 2! p!   1! 2! q!  
 
230  Theory of Counting

Concept 5 De- arrangement

1 1 1 1 1 1
1 1 1 1 (−1)1  D(5)  (5!)  –  –   
D(n) = n !  – + – +     +   0! 1! 2! 3! 4! 5! 
 0! 1! 2! 3! n ! 
 1 1 1 1 
Note: De-arrangement of 1 object is not  (5!) 1  1     
 2 6 24 120 
possible.  60  20  5  1 
 (120)    44
D(1) = 0  120 

D(2) = 1 Partial De-arrangement


D(3) = 2 Consider n articles only r articles are de
1 1 1 1 1 arranged remaining (n-r) articles are at their
D(4)  (4!)  –  –  
 0! 1! 2! 3! 4!  own place. Then this can be done in (nCr)
 1 1 1 [D(r)].
 (4!) 1  1    
 2 6 24 
 12  4  1 
 (4!)    12  4  1  9
 24 
Number of Integral Solution  231

1
Concept Deviator (CD)
Ideal Time Revise your concept (IIT advance or
5 Min. tougher questions)
Per Question

1. If n articles are arranged in a straight a + b = 10, a + b + c + d = 21, a + b + c + d + e


line in how many ways 4 articles can + f = 33, a + b + c + d + e + f + g + h = 46, and
be selected so that no two of them are so on till a + b + c + d + … + x + y + z = 208.
consecutive? (a) 22P10 (b) 22P11
(a) n – 4C4 (b) n – 5C4 (c) 22P12 (d) None of these
(c) n – 3C3 (d) None of these 7. In a science conference total 100 students
2. What is the number of distinct throws that from different schools participated.
is obtained when ‘n’ indistinguishable Participants are standing in a straight
dice are thrown? [viz. if n = 3 then result line in 5 different rooms. Find the total
5,2,3 and 3,2,5 is considered as same as number of different arrangements that
throw] are possible.
(a) n+5C5 (b) n6 (a) 104P100 (b) 104P4
(c) 6n (d) None of these (c) 105P100 (d) None of these
3. In how many ways 5 boys and 50 girls 8. Let n and k be positive integers such that
can be arranged in a straight line such k (k  +1)
that between any two boys there must be n  ≥ . The number of solutions (x1,
at least 5 girls? 2
x2, …, xk), x1 ≥ 2,… , xk ≥ k all integers
(a) (5!)(50!) 41C5 (b) (5!)(50!) 35C5 satisfying x1 + x2 + … + xk = n is given by
35
(c) (5!)(50!) C4 (d) None of these which one of the following assume that
4. Find the number of non negative integral 2 n = 2 p + k 2 + k   ?
solution of the equation x1+ x2+…+xk ≤ n (a) k + p +1Cp (b) k + p – 1 Cp
(a) n + kCn – 1 (b) n + kCn (c) k + p – 1 Ck (d) None of these
(c) n + kCn + 1 (d) None of these 9. What is the number of positive integral
5. What is the number of ways of selecting 4 solutions of the in equality a+b+3c ≤ 30?
persons from a group of n persons sitting (a) 1212 (b) 1215
around a circle such that no two of them (c) 1218 (d) None of these
are consecutive?
10. Find the number of integral of the
(a) (n/4)(n – 5C3) (b) (n)(n – 5C3) equation a + b + c + c + d = 20 if it is given
(c) (4n)( n – 5C3) (d) None of these that a > b > c > d > 0.
6. Find the number of non negative (a) 22 (b) 24
solutions of the system of equations: (c) 23 (d) None of these
Response Grid
1. a b c d 2. a b c d 3. a b c d 4. a b c d 5. a b c d
6. a b c d 7. a b c d 8. a b c d 9. a b c d 10. a b c d
232  Theory of Counting

2
Concept Eliminator (CE)
Ideal Time Revise your concept (Maths Olympiad
Just Solve It or tougher questions)

1. If n articles are arranged in a straight 4. Mr. Anshuman has thrown a dice 6 times
line in how many ways ‘r’ articles can in how many ways we can get a sum
be selected so that no two of them are greater than 17?
consecutive?
(a) 31518 (b) 31508
(a) n – r + 1C n – r + 1C
r – 1 (b) r
n – r – 1C
(c) 31528 (d) None of these
(c) r – 1 (d) None of these
5. In an Ice cream parlor at South City
2. In how many ways k boys and ‘nk’ girls
Mall kolkata, 4 different varieties of
can be arranged in a straight line such
that between any two boys there must be ice creams namely Vanila, Strawberry,
at least p girls? Chocolate and Butter Scotch were
(a) nk + kp + p + k – 2C available. On a particular day it was
k–1
(b) nk – kp + p + k – 2C noticed that each customer buys at least
k–2
nk + kp + k – 2C one ice cream and at max 10 ice creams,
(c) k–2
on further investigation it was noticed
(d) None of these
that no two customer buys same set
3. Three friends went to a shopping mall
of ice creams then find the number of
with $ 6, 7 and 8 with them in how many
customers visited the ice cream shop on
ways they can pay a bill of $10 if they
have note of only one denomination. that particular day.

(a) 47 (b) 94 (a) 800 (b) 1000


(c) 49 (d) None of these (c) 1009 (d) 1004

Response Grid
1. a b c d 2. a b c d 3. a b c d 4. a b c d 5. a b c d
Number of Integral Solution  233

Solutions
Concept Deviator (CD)

1. (d) 2. (a) 3. (b) 4. (b) 5. (a) 6. (c)


7. (a) 8. (b) 9. (b) 10. (c)

1. (d) Let after ‘a’ articles we have selected 3. (b) 1st arrange 50 girls that can be done
in 50! ways,
1st article here a ≥ 0
There are ‘b’ articles between 1st and Let 1st boy is after ‘a’ girls so that
2nd article here b ≥ 1 a ≥ 0, there are ‘b’ number of girls
between 1st boy and 2nd boy such
Then there are ‘c’ articles between
that b ≥ 5, similarly there are ‘c’
2nd and 3rd article here c ≥ 1
number of girls between 2nd and 3rd
Similarly there are ‘d’ articles boy, ‘d’ number of girls between 3rd
and 4th boy, ‘e’ number of girls between
between 3rd and 4th article here d ≥ 1.
4th and 5th boy and finally there are f
And there are ‘e’ articles after 4th number of girls after 5th boy.
article here e ≥ 0. So from the given condition a + b + c
+ d + e + f = 50.
Then from the given condition a + b
And as per the given condition a ≥ 0,
+ c + d + e = n – 4 with the conditions
b ≥ 5, c ≥ 5, d ≥ 5, e ≥ 5 and f ≥ 0.
a ≥ 0, b ≥ 1, c ≥ 1, d ≥ 1, e ≥ 0.
Let us assume b = 5 + b1 then b1≥0
Required number of positive integral similarly c = 5 + c1, d = 5 +d1 and
solution n–3C4. e = 5 + e1 where c1 ≥ 0, d1 ≥ 0 and e1 ≥ 0.
2. (a) Let after ‘n’ throws number of times
that ‘1’ appeared is a, number of Then the new equation is a + b1 + c1
times that ‘2’ appears is b and so + d1 + e1 + f = 30 where each variable
on number of times that 6 appears is non negative positive integer,
is f then a + b + c + d + e + f = n Hence required number of ways is
30+6–1C 35C .
and we have to find the number of 6–1 = 5
non negative integral solution of the Then girls can be arranged in (50!)
above equation. ways and boys can be arranged in (5!).
From partitioning theory the number
of non negative integral solution is So total number of ways is (5!)(50!)
n+6–1C n + 5C . (35C5).
6–1 = 5
234  Theory of Counting
4. (b) We have following cases: Next equation is a + b + c + d = 21

If x1 + x2 + … + xk = 1 then number hence c + d = 11 and number of

of integral solution is k + 1 – 1C solutions of this equation is 12.


k – 1
= kCk – 1 = kC1. Similarly for third equation a + b +

If x1 + x2 + … + xk = 2 then number c + d + e + f = 33 or e + f = 12 or

of integral solution is k + 2 – 1C number of solutions is 13.


k–1
= k + 1C = k + 1C
k–1 2. Similarly for last equation a + b + c

If x1+ x2 +… + xk = 3 then number of + d + … + x + y + z = 208, or

integral solution is y + z = 22 or number of solution is 23.

k + 3 – 1C = k + 2C = k + 2C
k–1 k–1 3. Required number of ways is 11 × 12 ×

And so on… 13 × ... × 21 × 22 = (22!)/(10!) = 22P12.

If x1 + x2 + … + xk = n then number of 7. (a) Let a, b, c, d, e, be the number of

integral solution is k + n – 1Ck – 1. students in 5 different rooms say A,


B, C, D, and E respectively.
So total number of ways is kCk – 1 +
k + 1C k + 2C k + n–1C Then from the given condition
k–1 + k – 1 + ... + k–1

= kC + k + 1C2 + k + 2C + … + a + b + c + d + e = 100.
1 3
k + n – 1C = n + kC .
n n Number of ways of dividing 100
5. (a) Given that n persons are sitting in a persons in 5 distinct groups.
circle, let number of persons between = n + r – 1 Cr
1st and 2nd is a, that between 2nd and
= 5 + 100 – 1C100 = 104C100.
3rd is b, between 3rd and 4th is c and
But number of ways of dividing 100
between 4th and 1st is d then a + b +
students in 5 sets containing a, b, c,
c + d = n – 4, here a, b c and d each
d, e, students respectively is
of them is more than 1, so required
number of ways is 100!

(n – 8) + 4 – 1C = n – 5C ways, but (a !)(b !)( c !)(d !)(e !)


4–1 3
1st one can be selected in n/4 ways, Now students in first set can be
so required number of ways is (n/4) arranged among themselves in the
(n – 5C3). list in a! ways.
6. (c) Consider the equation a + b = 10 Similarly, b, c, d, e, persons can be
number of solutions of this equation arranged among themselves in b!, c!,
is 10 + 2 – 1C = 11.
2–1 d!, e!, ways respectively.
Number of Integral Solution  235
Hence required number of ways Case (i) c1 = 0 then a1 + b1 + d = 25

= 206C6 and number of non negative integral

100! solution is 25 + 3 – 1C3–1 = 27C2 = 351.


 × (a !)(b !)( c !)(d !)(e !)
(a. !)(b !)( c !)(d !)(e !) Case (ii) c1 =1 then a1 + b1 + d = 22
and number of non negative integral
= 104C100 . 100! = 104P100 solution is 22 + 3 – 1C3 – 1 = 24C2 = 276.
Case (iii) c1 = 2 then a1 + b1 + d = 19
8. (b) As per the given condition (x1, x2, and number of non negative integral
…, xk), x1 ≥ 2,… , xk ≥ k so we can solution is 19 + 3 – 1C3 – 1 = 21C2 = 210.
assume y1 = x1 – 1, y2 = x2 – 2.. and Case (iv) c1 = 3 then a1 + b1 + d = 16
so on yk = xk – k then from the given and number of non negative integral
solution is 16 + 3 – 1C3 – 1 = 18C2 = 153.
condition y1, y2, …,yk ≥ 0.
Case (v) c1 = 4 then a1 + b1 + d = 13
Since x1+ x2 + … + xk = n and number of non negative integral
solution is 13 + 3 – 1C3 – 1 = 15C2 = 105.
Thus y1 + y2 + … + yk = x1 + x2 + ….
Case (vi) c1 = 5 then a1 + b1 + d = 10
+ xk – ( 1 + 2 + … + k) and number of non negative integral
k ( k  +  1) 2 n − k 2 – k solution is 10 + 3 – 1C3 – 1 = 12C2 = 66.
= n  – = =p
2 2 Case (vii) c1 = 6 then a1 + b1 + d = 7
2n − k 2 – k and number of non negative integral
Thus y1 + y2 + … + yk = = p, solution is 7 + 3 – 1C3 – 1 = 9C2 = 36.
2
where yi ≥ 0 for i = 1, 2, …, k Case (viii) c1 = 7 then a1 + b1 + d = 4
k+p–1C
and number of non negative integral
Required number = p. solution is 4 + 3 – 1C3 – 1 = 6C2 = 15.
9. (b) Since it is given that a + b + 3c ≤ 30, Case (ix) c1 = 8 then a1 + b1 + d = 1
so assume a number d ≥ 0 such that and number of non negative integral
solution is 1 + 3 – 1C3 – 1 = 3C2 = 3.
a + b + 3c + d = 30, a, b and c are
So total number of integral solution
the positive integers so a = a1+ 1, b
is 51 + 276 + 210 + 153 + 105 + 66 +
= b1+1, and c = c1+1 then the given
36 + 15 + 3 = 1215.
equation we can write as (a1+ 1) +
(b1 + 1) + 3(c1 + 1) + d = 30. Method 2:

Or a1 + b1 + 3c1 + d = 25. Instead of listing of all the possible


solutions we can find a general term
Since coefficient of c1 is 3 hence it
and then solve-
can not take a value more than 8 if
we form cases then we will have 9 Since a1 + b1 + 3c1 + d = 25 hence
cases: a1 + b1 + d = 25 – 3c1.
236  Theory of Counting
Let c1 = n where n varies from 1 to 8 then the new equation is
then a1 + b1 + d = 25 – 3n. (d + x + y + z) + (d + x + y) + (d + x) +
Number of non-negative integral d = 20 or 4d + 3x + 2y + z = 20
solution of this equation is given by here d, x, y, z > 0.
= 3 + 25 – 3n – 1 C = 27 – 3nC2.
3–1
Minimum value of 4d + 3x + 2y + z is
( 27 3n)(26 3n) 4 + 3 + 2 + 1 = 10.

2
3 Required number =
 (3n2 – 53n  – 234)
2 coefficient of x20 – 10
here n varies from 1 to 8
in ( 1 – x4) – 1 (1 – x3) – 1 (1 – x2)–1 ( 1 – x) –1.
Required number is
= coefficient of x10 in [(1 + x4 + x8 )
8 (1 + x3 + x6 + x9)( 1 + x2 + x4 + x6 + x8 +
3/2 ∑ (3n2 − 53n − 234) x10)(1 + x + x2 + x3 + …….. + x10)].
n= 1
[Ignoring terms containing powers
3  17   9 
= 3  8 ×  9  ×   – 53  8 ×  +  234 ×  9   of x greater than 10]
2   6  2 
= coefficient of x10 in [( 1 + x3 + x4 +
= 1215.
x6 + x7 + x8 + x9 + x10)( 1 + x + 2x2 +
10. (c) Given equation is a + b + c + d = 20,
2x3 + 3x4 + 3x5 + 4x6 + 4x7 + 5x8 +
a > b > c > d > 0.
4x9 + 6x10)]
c = d + x, b = d + x + y and
= 6 + 4 + 4 + 3 + 2 + 2 + 1 + 1 = 23
a=d+x+y+z

Concept Eliminator (CE)

1. (b) 2. (b) 3. (a) 4. (b) 5. (b)

1. (b) Let after ‘a1’ articles we have selected And there are ‘ar + 1’ articles after
1st article here a1 ≥ 0. rtharticle here ar + 1 ≥ 0.
There are ‘a2’ articles between 1st
Then from the given condition
and 2nd article here a2 ≥ 1.
a1 + a2 + a3 + …. + ar + ar+1 = n – r and
Then there are ‘a3’ articles between
a1 ≥ 0, a2 ≥ 1, a3 ≥ 1, …. ar ≥ 1, ar + 1 ≥ 0.
2ndand 3rd article here a3 ≥ 1.
And so on Required number of positive integral

There are ‘ar’ articles between (r – 1)th solution n – r + 1Cr.


and rtharticle here ar ≥ 1.
Number of Integral Solution  237
2. (b) 1st arrange ‘nk’ girls that can be So the numbers of ways in which
done in (nk)! Ways. they can denote Rs. 10 is the same

Let 1st boy is after ‘a1’ girls so that as the number of solution to the

a1 ≥ 0, there are ‘a2’ number of girls question

between 1st boy and 2nd boy such x1 + x2 + x3 = 10


that a2 ≥ p, similarly there are ‘a3’
Subject to conditions
number of girls between 2nd and 3rd
boy, ‘a4’ number of girls between 3rd 0 ≤ x1≤ 6, 0 ≤ x2≤ 7, 0 ≤ x3≤ 8

and 4th boy, and so on and finally Hence the required number of ways
there are ak+1 number of girls after = coefficient of x10 in (1+ x + x2 + ...+ x6)
kth boys. (1 + x + x2 + ... x7) (1 + x + x2 + ... + x8)
So from the given condition a1 + a2 + = coefficient of x10 in (1 – x7) (1 – x8)
a3 + … + ak + ak + 1 = nk. (1 – x9) (1 – x)–3.
And as per the given condition a1 ≥ 0,
= coefficient of x10 in
a2 ≥ p, a3 ≥ p and so on ,ak ≥ p, ak + 1 ≥ 0.
( 1 – x 7 – x 8 – x 9) ( 1 + 3C 1x + 4C 2x2 +
Let us assume a2 = p + a2’ then 5C 3 12C x10).
3x + ... 10
a2’ ≥ 0 similarly a3 = p + a3’ and so
= 12C2–5C3 – 4C2 – 3C1
on, ak = p + ak’ and ak+1 ≥ 0.
(ignoring powers higher than 10)
Then the new equation is a1 + a2 +
= 66 – 10 – 6 – 3 = 47.
a3+ …. + ak + ak + 1 = nk – (k – 1)p
where each variable is non negative 4. (b) Let x1, x2, ... , x6 be the number that
positive integer, Hence required appears on the six dies.
number of ways is
As per the given condition x1 + x2 +
nk – kp + p + k – 1 – 1C
k–1–1 x3 + ... + x6 ≤ 17.
= nk – kp + p + k – 2Ck – 2. Introducing a dummy variable x7
3. (a) They have note of same denomination (0 ≤ x7) the inequality becomes an
hence that denomination must be equation
HCF of 6, 7 and 8 which is 1, so each x1 + x2 + ... + x6 + x7 = 17.
of them have notes of $1.
Here 1 ≤ xi ≤ 6 where i = 1, 2, ... , 6
Now let us assume that the number
and 0 ≤ x7.
of notes that they have is x1, x2, and
x3.
238  Theory of Counting
So that No. of solution 5. (b) Let a particular customer buys

= coefficient of x17 in ( x + x2 + ... + x6 )6 ice creams Vanila, Strawberry,


Chocolate and Butter Scotch of a, b ,
(1 + x + x2 + x3 + ...)
c and d numbers then from the given
= coefficient of x11 in (1 – x6)6 (1 – x)–7 condition 0 < a + b + c + d ≤ 10.

= coefficient of x11 in (1 – 6x6) (1 – x)–7 Let us assume a dummy variable


k such that a + b + c + d + k = 10
= coefficient of x11 in ( 1 – 6x6) then number of integral solution of
(1 + 7C1x + ... + 10C4x4 + 11C5x5 + ...
this equation is 10 + 5 – 1C5 – 1 = 14C4
17C x11 + ...) = 17C – 6. 11C
11 11 5 14! 14  13  12  11
=  = 1001.
(4!)(10!) 24
Since total number of outcome is 66
But there is one case when k =10
Required number of ways to get a sum then a + b + c + d = 10, this case is
greater than 17 = 66 – (17C11 – 611C5) not applicable case hence required
= 46656 – 12376 – 6 × 462 = 31508. answer is 1001 – 1 = 1000.
9
Probability Introduction  239

Probability Introduction

Topics Covered
 Introduction and properties of sets

 Probability Introduction

 Properties of Probability

 Odds in favor & against

Part A: Topic Number of Questions


Solved Example 10
Concept Applicator 10
Concept Builder 15
Concept Cracker 15
Part B: Topic Number of Questions
Concept Deviator 20
Concept Eliminator 7
Total 77
240  Theory of Counting
Part A

Concept 1 Introduction
Partitioning
and Properties
Theory of Sets

In our day to day life we use statements like are 1, 2, 3, 4, 5 and 6) and its result can
“probability of catching a train” or “Probability of not be predicted before hand.
passing an exam” etc. If we talk in mathematical Example (ii) “Tossing of a fair coin”
language then we can say the probability may is a random experiment because it has
vary between 0 to 1. If probability is 0 then it more than 1 possibility (possibilities are
is called impossible event e.g. probability of Head and Tail ) and its result can not be
getting a red ball from a basket that has only predicted before hand
10 black balls. And if probability is 1 then it is 2. Sample space (or Possible out comes):-
called sure event like getting head or tail when While defining Random experiment we
a fair coin is tossed. have seen that a random experiment
The theory of probability has its origin in must have more than 1 possible out comes
gambling when people went to casinos where and the set of all possible outcomes of a
they used to get certain money if they won the random experiment is called the sample
game. Mathematicians wanted to find out as space. It is usually denoted by S.
to how much a person will earn if the game is Example (i): “Throwing an unbiased
played a large number of times or a certain die” is a random experiment and it has
number of times. more than 1 possibility, possibilities are
1, 2, 3, 4, 5 and 6 and a set that has all
Some basic concepts/ terms in these elements are the sample space of
Probability: this random experiment.
1. Experiment & Random Experiment: For this experiment Sample space
Any operation which may result in S = {1, 2, 3, 4, 5, 6}
some well defined outcomes is called Number of elements in this set is 6 and
an experiment and any Experiment that can be denoted as n(S) or |S| = 6
whose outcome cannot be predicted with A sample space S is called a discrete
certainly is called a random experiment. sample space if S is a finite set.
In other words any experiment which Example (ii): “Tossing of a fair coin” is
has more than 1 possible outcome and a random experiment and it has more
it is not possible to predict the result of than 1 possibility and they are Head (H)
experiment before hand is called random and Tail (T) hence set Sample space for
experiment. Consider few examples: this random experiment is S = { H, T}
Example (i): “Throwing an unbiased In this case number of elements in this
die” is a random experiment because it set is 2 and that can be denoted as n(S)
has more than 1 possibility (possibilities or |S| = 2
Probability Introduction  241
Example (iii): “Tossing of two fair coins the number of elements in the sample
is a random experiment and it has more space.
than 1 possibility and set Sample space Solution:  Let the two apples be A1 and A2
for this random experiment is S = {HH, and three bananas as B1, B2, B3 then the
HT, TH, TT}
outcome will be as follows S = {HA1, HA2,
In this case number of elements in this HB1, HB2, HB3, T}
set is 4 and that can be denoted as n(S)
So number of elements in sample space
or |S| = 4.
n(S) = 6.
In other words we can say that we have
4 cases or 4 outcomes when we toss two Example 3:  Trina now decided to toss a
unbiased coins and these are: coin until she get a Tail write down all
the sample spaces and ?
1st 2nd
Cases Solution:  As per the given condition she will
coin coin
continue doing toss until she get a Tail
Case (i) H H HH So sample space hence sample space here would be
for this random
Case (ii) H T HT experiment is- n(S) = {T, HT, HHT, HHHT, …}
S = {HH, HT, Event: A subset of the sample space S is called
Case (iii) T H TH
TH, TT} an event and is generally denoted by E.
Case (iv) T T TT Or n(S) = 4 (i) Consider Tossing of a fair coin it is a
random experiment and Sample space
Example 1:  List down all the elements of for this random experiment is S = {H, T},
sample space if a coin is tossed thrice? Events of this experiment is {φ}, {H}, {T}
Solution:  Required sample space is {H or T}
S = {HHH, HHT, HTH, HTT, THH, THT, (ii) Impossible event: Ф (null set) is also a
TTH, TTT) subset of S (Sample space) and is called
And n(S) = 8. an impossible event.
Alternately : From fundamental principle (iii) Sure Event: S is also a subset of S which
of multiplication the total number of is called a sure event or a certain event.
elements is 23 = 8 ways. If we observe then we can find that
Sample space S plays the same role as
Example 2:  Trina has a fruit basket that
the universal set
has 2 Apples and 3 bananas, She tossed
a coin if she gets head then she will Each element of the sample space is called a
take out one fruit from the basket and sample point or an even point.
if she get Tail then she wont pick any Simple Event elementary Event: a
fruits, then list down all the possible singleton subset of the sample space S is
outcomes (sample space) and also find called a simple event.

QR Code / Video Link

For Smart phone/ Tablet users

Video Link for Desktop/Laptops users http://dishapublication.com/video-resources


242  Theory of Counting
Examples: “Throwing an unbiased die” is a “E” getting an Even number which is {2, 4,6}
random experiment and it has more than 1 and Event “O” getting an odd number which
possibility, possibilities are 1, 2, 3, 4, 5 and 6 is {1, 3, 5}. Number of elements in event E is 3
and a set that has all these elements are the and that in event O is also three hence we can
sample space of this random experiment. say that events E and O are equally likely.
When a die thrown, sample space S = {1, 2, 3,
Operations of Sets
4, 5, 6}
Let A = {5} = the event of occurrence of 5. Union of Sets: Union of two or more sets is
the set of all elements that belong to any one
B = {2} = the event of occurrence of 2.
of the sets or present in both of them. The
Here A and B are simple events. symbol used for union of sets is ‘∪’ i.e.
Example: When a coin is tossed, sample
A∪B = Union of set A and set B
space S = {H,T}
Example: A = {1, 2, 3, 4} and B = {2, 4, 5, 6}
{H} = event of occurrence of head and {T} =
then A∪B = {1, 2, 3, 4, 5, 6}
the event of occurrence of tail and these are
simple events Properties of Union:
Mixed Event or compound Event or
A∪B=B∪A
composite Event: a subset of the sample
space S which contains more than one element A∪A=A
is called a mixed event or in other words any A ∪ U = U, where U is the universal set
event which is not a simple or elementary A ∪ Φ =A
event is a mixed or compound event.
A ⊂ A∪ B and B ⊂ A ∪ B
Examples:
If B ⊆ A, then A ∪ B = A
(i) When a coin is tossed, sample space
A ∪ (B ∪ C) = (A ∪ B) ∪ C, this is known
S = {H,T}.
as associative law
Now consider an event P = {H,T} = the
event of occurrence of a head or tail. Here P Representation of union of sets:
has 2 elements hence it is a mixed event.
(ii) When a die is thrown, sample space A B
S = {1,2,3,4,5,6}
Now consider an event E ={2, 3, 5} = the event
of occurrence of a prime number and it has 3
elements so it is mixed events.
Equally Likely Events: If outcomes are
equally likely or when number of elements in
two events set is equal then these two events
are called Equally Likely Events. Intersection of Sets:
If a fair coin is tossed then event of getting
It is the set of all the elements, which are
Head is {H} and that of tell is {T} in both the
common to all the sets. The symbol used for
cases number of elements is 1 hence these two intersection of sets is ‘∩’ i.e.
events are equally likely.
A ∩ B = {x| x ∈A and x ∈B}
If a fair die is thrown then sample space is {1, Example: If A = {1, 2, 3, 4} and B = {2, 4, 5, 6}
2, 3, 4, 5, 6} now consider two events, Event then A ∩ B = {2, 4}
Probability Introduction  243
Properties of Intersection: A–Φ=A
A∩B=B∩A (A – B) ∪ B = A ∪ B and (A – B) ∩ B = Φ
A∩A=A
If A and B are disjoint sets then A ∩ B = Φ Complement of a Set:
A∩Φ=Φ The complement of a set A is the set of
(A ∩ B) ⊆ A and (A ∩ B) ⊆ B elements which do not belong to A, in other
A ∩ (B ∩ C) = (A ∩ B) ∩ C, i.e., associative law words, U

A B
– A. Symbolically, denoted as, Ac or A′
A′ = {x|x∈ U, x ∉ A} = U – A
For every event A of an experiment there
exist another event A′ or Ac such that there
is no common element in events A and Ac and
collectively they form sample space.
When a die is thrown, sample space
Difference of Sets: The difference of set A to
B denoted as A – B is the set of those elements S = {1,2,3,4,5,6}
that are in the set A but not in the set B i.e. Now consider an event E ={2, 3, 5} = the event
A – B = {x| x ∈ A and x ∉ B}.
of occurrence of a prime number and it has 3
Similarly B – A = {x: x ∈ B and x ∉ A}
elements so its complementary event of this
Example: If A = {1, 2, 3, 4} and B = {3, 4, 5, 6,
element is E′ = { 1, 4, 6}
7} then A–B = {1, 2} and B–A = {5, 6, 7}.

A–B
A B

Here shaded area represents A′

Exhaustive Events:
A B B–A
For a random experiment A, set of events
is said to be exhaustive if one of them must
necessarily happen every time the experiment
is performed. If a fair die is thrown then
sample space is {1, 2, 3, 4, 5, 6} now consider
two events, Event “E” getting an Even number
which is {2, 4,6} and Event “O” getting an odd
number which is {1, 3, 5}.
Properties of Difference of Sets:
If we consider event E and O together then
A–B≠B–A no element is left hence they are Exhaustive
A – B ⊆ A and B – A ⊆ B Events.
244  Theory of Counting

Concept 21 Partitioning
Probability
Theory

The probability of occurrence of an event E is Example 5:  Consider an experiment


denoted by P(E) and is defined as - of throwing a fair die, what is the
n (E ) probability of getting an even
P  (E ) =
n (S ) number?
Here n(E) represents number of elements in E Solution:  Sample space S = All possible cases
= number of cases favorable to event E = {1,2,3,4,5,6} and number of elements in
and n(S) represents number of elements in sample space S is n(S) = 6
Sample space S = total number of cases Let E = the event of occurrence of an Even
Example 4:  Consider an experiment number = {2,4,6} and n(E) = 3
of tossing a fair coin, what is the n (E ) 3 1
Then P (E ) = = =
probability of getting a head? n (S ) 6 2
Solution:  Sample space S = All possible cases Example 6:  When two coins are tossed,
= {H,T} and then what is probability of getting one
number of elements in sample space S is head and one tail?
n(S) =2
Solution:  Sample space S= {HH,HT,TH,TT}
Let E = the event of occurrence of a head. so n(S) = 4
Then E = {H} and number of elements in Let E = the event of occurrence of one head
favorable case n(E) = 1 and one tail, then E = {HT,TH} so n(E) = 2
Thus P(E) = Probability of occurrence of n (E ) 2 1
Now P (E ) =
n (E ) 1 n (S )
= =
4 2
head = =
n (S ) 2

QR Code / Video Link

For Smart phone/ Tablet users

Video Link for Desktop/Laptops users http://dishapublication.com/video-resources


Probability Introduction  245

Concept 31 Properties
Partitioning
of Probability
Theory

Property 1: Probability of occurrence of an Complement of an Event: Let S be the


event is a number lying between 0 and 1 sample space for a random experiment and E
If probability is 0 then it is called impossible be an event.
event e.g probability of getting a red ball from Complement of event E is denoted by E′ or Ec.
a basket that has only 10 black balls. By definition of complement of an event, E′
If probability is 1 then it is called sure event like means non- occurrence of event E, hence E'
getting head or tail when a fair coin is tossed. occurs if and only if E does not occur.
Proof: Let S be the sample space and E be Proof: Let S be the sample space, then
an event. n(E) + n(E') = n(S)
Then we know that φ ⊆ E ⊆ S n (E ) n (E ′ )
\ +    or  P (E ) +  P (E ′ ) = 1
=1
\ n(φ) ≤ n(E) ≤ n (S) n (S ) n (S )

Or  0 ≤ n (E) ≤ n (S) since n(φ) = 0
0 n (E ) n (S ) Example 7:  If probability that Ramesh
Or  ≤ ≤ [Dividing by n (S)] will pass the exam is 3/5 then find the
n (S ) n (S ) n (S )
probability that he fail in the exam.
Or 0 ≤ P (E) ≤ 1 Solution:  Since event of passing and failing in
Property 2: If E is any event and E′ be the the same exam is complement to each other
complement of event E, then P(E) + P(E′) = 1 hence required probability is 1 – 3/5 = 2/5

Concept 41 Odds
Partitioning
in Favor &Theory
Against

Consider a random experiment, S be the its Odds in favor of event E is defined as


sample space and E be an event. Let E' denote  n (E )
the complement of event E, then we know that  
n (E )  n (S ) P (E )
n(E) + n(E′) = n(S) and P(E) + P(E') = 1 = = =
n (E ')  n (E ') P (E ')
• Odds in favor of event E  
 n (S ) 
246  Theory of Counting
Odds against an even E In other words here sample space
Odds against an even E = n(E′)/n(E) n(S) = 16, favorable event n(E) = 5 and
n(E′) = 11 hence odd against that event is
 n (E ')
  n(E′)/n(E) = 11/5
 n (S )  P (E ') Example 9:  If the probability of occurr-
= =
 n (E ) P (E ) ence of an event is 5/16, find the odds
  in favor of the event.
 n (S )
Solution:  In this case sample space n(S) = 16,
If odds in favor of an event is ‘F’ and against favorable event n(E) = 5 and n(E′) = 11 hence
an event is ‘A’ then F × A = 1 odds against that event is n(E)/n(E') = 5/11
Example 8:  If the probability of occurr- Example 10:  If odd against an event is
ence of an event is 5/16, find the odds 4/11 then what is the probability of
against the event. occurrence of that event?
Solution:  Out of every 16 events 5 are Solution:  From the definition of odds against an
favorable and events not favorable is 11 event E is n(E')/n(E) = 4/11 so n(E) = 11 and
hence odd against the event is 11/5 n(S) = 4+11 = 15, so P(E) = n(E)/n(S) = 11/15

1
Concept Applicator (CA)
Ideal Time Apply your concepts with easy and
10 Min. conceptual questions

1. If a fair coin is tossed, what is the 4. What is the probability that Mr. John
probability that it shows a head? was born on a Sunday?
1 1 1 1
(a) 2 (b) 4 (a) 2 (b) 4
1 1
(c) 3 (d) None of these (c) 3 (d) None of these
2. If a fair coin is tossed twice, what is the 5. What is the probability that Mr. John
probability that it shows a tail and a head? was born on a Saturday or on a Sunday?
1 1 1 1
(a) 2 (b) 4 (a) 2 (b) 4
1 2
(c) (d) None of these
(c) 3 (d) None of these 7
3. If a fair coin is tossed twice, what is the 6. If a fair die with 6 faces number 1 to 6
probability that it shows a tail in 1st toss is rolled, what is the probability that it
and a head in 2nd ? shows an odd number?
1 1 1 1
(a) 2 (b) 4 (a) 2 (b) 4
1 1
(c) 3 (d) None of these (c) 3 (d) None of these
Probability Introduction  247
7. If a fair die with 6 faces number 1 to 6 9. If a fair die with 6 faces number 1 to 6
is rolled, what is the probability that it is rolled, what is the probability that it
shows a composite number? shows a perfect square number?
1 1 1 1
(a) 2 (b) 4 (a) 2 (b) 4
1 1
(c) 3 (d) None of these (c) 3 (d) None of these
8. If a fair die with 6 faces number 1 to 6 10. If a fair die with 6 faces number 1 to 6 is
is rolled, what is the probability that it rolled, what is the probability that It shows
shows either a prime or an even number? either a perfect square or an even number?
1 1 1 1
(a) 2 (b) 4 (a) 2 (b) 4
1
1
(c) 3 (d) None of these (c) (d) None of these
3
Response Grid
1. a b c d 2. a b c d 3. a b c d 4. a b c d 5. a b c d
6. a b c d 7. a b c d 8. a b c d 9. a b c d 10. a b c d

2
Concept Builder (CB)
Ideal Time Revise your concepts with questions
25 Min. medium difficulty level questions

1. If two fair dies with 6 faces numbers that both of them will show either prime
1 to 6 is rolled together, what is the or odd?
probability that one of them will show 1
even and other odd? (a) 4/9 (b)
6
1 1
1 (c) 4 (d) None of these
(a) 3 (b)
6 4. If two fair dies with 6 faces numbers
1 1 to 6 is rolled together, what is the
(c) 4 (d) None of these probability that at least one of them will
2. If two fair dies with 6 faces numbers 1 to 6 show a prime number?
is rolled together, what is the probability 1 1
that both of them will show prime number? (a) 3 (b)
6
1 1 1
(a) 3 (b) (c) 4 (d) None of these
6
1 5. N is a two digit number what is probability
(c) 4 (d) None of these that N is a prime number?
3. If two fair dies with 6 faces numbers 1 to 6 (a) 1/25 (b) 7/10
is rolled together, what is the probability (c) 7/30 (d) None of these
248  Theory of Counting
6. N is a two digit number what is probability
381
that N is a prime or a perfect square number? (c) (d) None of these
2530
(a) 1/25 (b) 3/10
11. If the probability of occurrence of an event is
(c) 29/90 (d) None of these
3/16, find the odds against the event.
Directions Qs. 7 to 10: In each case there is
a class with 10 boys and 15 girls then- (a) 3/13 (b) 13/3
7. What is the probability that a randomly (c) 13/16 (d) None of these
chosen student is a boy? 12. If the probability of occurrence of an event is
3/16, find the odds in favor of the event.
(a) 2/5 (b) 3/5
(a) 3/13 (b) 13/3
(c) 1/4 (d) None of these
(c) 13/16 (d) None of these
8. What is the probability that a random
13. If odd against an event is 5/11 then what is
selection of 2 students will comprise one the probability of occurrence of that event?
boy and one girl? (a) 5/16 (b) 13/16
(a) 2/5 (b) 3/5 (c) 12/16 (d) None of these
(c) 1/4 (d) None of these 14. If odd in favor of an event is 5/11 then
9. What is the probability that a random what is the probability of occurrence of
selection of 5 students will comprise 3 that event?
boys and 2 girls? (a) 5/16 (b) 13/16
(a) 2/5 (b) 60/121 (c) 12/16 (d) None of these
(c) 60/253 (d) None of these 15. A bag contains 30 balls of which 11 are
10. What is the probability that a random white, 7 are yellow, 7 are black and 5 are
selection of 5 students will comprise at green. What is the probability that a ball
least 3 boys? drawn at random is either white or black
or green?
762 381
(a) (b) (a) 7/10 (b) 3/10
1265 1265 (c) 23/30 (d) None of these
Response Grid
1. a b c d 2. a b c d 3. a b c d 4. a b c d 5. a b c d
6. a b c d 7. a b c d 8. a b c d 9. a b c d 10. a b c d
11. a b c d 12. a b c d 13. a b c d 14. a b c d 15. a b c d

3
Concept Cracker (CC)
Ideal Time Boost up your confidence with good
30 Min. questions

1. Find the probability of getting the sum 2. Find the probability of getting the sum
as a prime number when two dice are as a perfect square number when two
thrown together. dice are thrown together.
(a) 5/12 (b) 7/18 (a) 5/12 (b) 7/18
(c) 11/36 (d) None of these (c) 7/36 (d) None of these
Probability Introduction  249
3. What is the probability that equal 10. A bag contains 4 red & 6 balls white. Two
numbers will come up on them if two balls are drawn at random. What is the
dice are thrown together? probability that both are red?
(a) 5/12 (b) 1/6 (a) 2/7 (b) 1/7
(c) 7/36 (d) None of these (c) 7/52 (d) None of these
4. A number between 1 to 25 (both excluding) 11. A bag contains 4 red & 6 white balls. Two
is selected, what is probability that it is balls are drawn at random. What is the
neither multiple of 5 nor that of 7? probability that one of them is red and
(a) 16/23 (b) 7/23 the other is white?
(c) 16/25 (d) None of these (a) 7/15 (b) 8/15
5. Find the probability of getting a sum greater (c) 4/15 (d) None of these
than 9 if two dice are thrown at a time. 12. A bag contains 6 apples, 4 bananas and
(a) 5/12 (b) 1/6 8 mangoes. If three fruits are drawn at
random, then what is the probability
(c) 7/36 (d) None of these
that all the three are apples?
6. What is the probability that a leap year
(a) 5/204 (b) 7/204
selected at random will have 53 Fridays?
(c) 11/204 (d) None of these
(a) 2/7 (b) 1/7
13. Five coins are tossed together. What is
(c) 7/52 (d) None of these
the probability that at least one tail will
7. What is the probability that a non leap year appear?
selected at random will have 53 Fridays?
(a) 31/32 (b) 1/32
(a) 2/7 (b) 1/7
(c) 15/32 (d) None of these
(c) 7/52 (d) None of these
14. A team of 5 is to be constituted out of 6
8. If three coins are tossed simultaneously, what girls and 6 boys, then what is probability
is the probability of getting three heads? that the team has 3 girls and 2 boys?
(a) 1/4 (b) 1/8 (a) 75/198 (b) 79/198
(c) 3/8 (d) None of these (c) 75/199 (d) None of these
9. If three coins are tossed simultaneously, 15. If ratio of odds against an event to odds
what is the probability of getting two in favor of an event is 9/25 then find the
heads and one tail? probability of occurrence of the event.
(a) 1/4 (b) 1/8 (a) 3/8 (b) 5/8
(c) 3/8 (d) None of these (c) 1/8 (d) None of these

Response Grid
1. a b c d 2. a b c d 3. a b c d 4. a b c d 5. a b c d
6. a b c d 7. a b c d 8. a b c d 9. a b c d 10. a b c d
11. a b c d 12. a b c d 13. a b c d 14. a b c d 15. a b c d
250  Theory of Counting

Solutions
Concept Applicator (CA)
1. (a) 2. (a) 3. (b) 4. (d) 5. (c) 6. (a)
7. (c) 8. (d) 9. (c) 10. (d)

1. (a) When a fair coin is tossed the sample 5. (c) Since in a week we have 7 days.
space is {H, T}. Number of elements Hence number of elements in the
in sample space is 2. Hence required sample space is 7 and favorable cases
1
probability is . are two. Hence required probability
2
2. (a) If a fair coin is tossed twice then is 2/7
sample space is {HH, HT, TH, TT}. 6. (a) Required probability is 3/6 = 1/2
Number of elements in sample
7. (c) From 1 to 6 composite numbers are 4
space is 4 and favorable cases are
{HT, TH} and number of elements in and 6 i.e 2 numbers. Hence required
this is 2. Hence required probability probability is 2/6 = 1/3
is 2/4 = 1/2. 8. (d) Favorable numbers in this case are 2,
3. (b) If a fair coin is tossed twice then 3, 4, 5, and 6 so required probability
sample space is {HH, HT, TH, TT}. is 5/6
Number of elements in sample space
is 4 and favorable case is {TH} and 9. (c) From 1 to 6 only 2 perfect square
number of elements in this is 1. numbers exist (1 and 4) so required
Hence required probability is 1/4. probability is 2/6 = 1/3
4. (d) Since in a week we have 7 days hence 10. (d) Number of favorable cases are
number of elements in the sample {1, 2, 4, 6} so required probability is
space is 7 and favorable case is
4/6 = 2/3
only one. Hence required probability
is 1/7

Concept Builder (CB)


1. (d) 2. (c) 3. (a) 4. (d) 5. (c) 6. (b)
7. (a) 8. (d) 9. (c) 10. (b) 11. (b) 12. (a)
13. (d) 14. (a) 15. (c)
Probability Introduction  251
1. (d) Probability of being even in 1st 1
1 1 1 required probability is 150/300 = 2
and odd in 2nd is 2 × 2 = 4 and .
probability of being odd in 1st and Alternately
1 1 1
even in 2nd is 2 × 2 = 4 Here we have two cases:
1 1 1 Case (i) 1st selected students is boy and
So required probability is + = 2nd student is a girl then required
4 4 2
10 15 10 × 15
2. (c) Probability that a die will show prime probability is × =
1 25 24 25 × 24
is 3/6 = 2 and required probability Case (ii) 1st selected students is girl and
1 1 1 2nd student is a boy then required
is 2 × 2 = 4 15 10 10 × 15
probability is × =
3. (a) Probability that a die will show 25 24 25 × 24
prime or odd is 4/6 =2/3 and required So required probability is
probability is 2/3 × 2/3 = 4/9
10 × 15 10 × 15  10 × 15 
4. (d) Probability that a die will show +  = 2×
1 25 × 24 25 × 24  25 × 24 
prime is 3/6 = . Required probability
2 300 1
= =
1 1 1 3 600 2
= + − =
2 2 4 4
5. (c) Number of two digit numbers is 9.
(c) Out of 10 boys and 15 girls, 3 boys and
90. Hence sample space has 90 2 girls can be selected in 10C3 × 15C2 =
elements and number of two digit 10 × 9 × 8  15 × 14 
prime number is 21. Hence required  ×  = 120 × 105
 6   2 
probability is 21/90 = 7/30 ways and 5 students can be selected
6. (b) Number of two digit numbers is out of 25 students in 25C5 ways
90. Hence sample space has 90 25 × 24 × 23 × 22 × 21
elements and number of two digit =
120
prime number is 21 and number = 5 × 23 × 22 × 21 ways
of two digit perfect square number
is 6 hence number of elements in so required probability is
favorable case is 21+6 = 27 hence 120 × 105 60 60
= =
required probability is 27/90 = 3/10 5 × 23 × 22 × 21 23 × 11 253
7. (a) Number of boys is 10. Hence 1 boy can 10. (b) Out of 10 boys and 15 girls, we have
be selected in 10C1 ways = 10 ways. to select at least 3 boys hence we
Sample space is equal to selection of have the following cases-
1 student out of 25 students and that Case (i) Number of boys is 3 and number
can be done in 25C1 = 25 ways. Hence of girls is 2 then number of ways is
required probability is 10/25 = 2/5 10C × 15C = 120 × 105
3 2
8. (d) Out of 10 boys and 15 girls, 1 boy and Case (ii) Number of boys is 4 and
1 girl can be selected in 10C1 × 15C1 = number of girl is 1 then number of
10 × 15 = 150 ways and two students ways is 10C4 × 15C1 = 210 × 15 =
can be selected out of 25 students in
25C ways = 25 × 24/2 = 300 ways so Case (iii) Number of boys is 5 then
2
number of ways is 10C5 = 36 × 7
252  Theory of Counting
So total number of ways of selecting In other words here sample space
at least 3 boys is 120 × 105 + 210 × n(S) = 16, favorable event n(E) = 3
15 + 36 × 7 = (1800 + 450 + 36) × 7 = and n(E′) = 13. Hence odd against that
2286 × 7 event is n(E′)/n(E) = 13/3
5 students can be selected out of 25 12. (a) In this case sample space n(S) = 16,
students in 25C5 ways favorable event n(E) = 3 and n(E’) = 13
hence odds against that event is
25 × 24 × 23 × 22 × 21
=   n(E)/n(E’) = 3/13
120
13. (d) From the definition of odds against
= 5 × 23 × 22 × 21 an event E is n(E’)/n(E) = 5/11 so
= so required probability is n(E) = 11 and n(S) = 5+11 = 16, so
  P(E) = n(E)/n(S) = 11/16
2286 × 7 762
= 14. (a) From the definition of odds in favor
5 × 23 × 22 × 21 5 × 23 × 22 of an event E is n(E)/n(E’) = 5/11 so
381 381 n(E) = 5 and n(S) = 5+11 = 16, so
= =
5 × 23 × 11 1265   P(E) = n(E)/n(S) = 5/16
15. (c) Total number of balls is 30, and
11. (b) Out of every 16 events 3 are favorable number of balls that are either white
and events not favorable is 13. Hence or black or green is 11+7+5 = 23, so
odd against the event is 13/3 required probability is 23/30
Concept Cracker (CC)
1. (a) 2. (c) 3. (b) 4. (a) 5. (b) 6. (a)
7. (b) 8. (b) 9. (c) 10. (d) 11. (b) 12. (a)
13. (a) 14. (a) 15. (a)

1. (a) Sample space in this case is 6 × 6 = 36 2. (c) Sample space in this case is
Sum of two result is a prime number   6 × 6 = 36
we have following cases- Sum of two results is a perfect square
Case (i) if sum is 2 then we have only number we have following cases-
one option (1, 1) Case (i) If sum is 4 then we have following
Case (ii) If sum is 3 then we have 3 cases (1, 3), (2, 2) and (3,1)
following 2 cases (1, 2) and (2,1) Case (ii) If sum is 9 then we have
Case (iii) If sum is 5 then we have following 4 cases (3, 6), (4, 5), (5, 4)
following 4 cases (1, 4), (2, 3), (3,2) and (6,3)
and (4,1) So total number of ways is 3 + 4 = 7
Case (iv) If sum is 7 then we have So required probability is 7/36
following 6 cases (1, 6), (2, 5), (3,4), 3. (b) Sample space in this case is
(4,3), (5,2) and (6,1)   6 × 6 = 36
Case (v) If sum is 11 then we have Sum of two results is equal numbers
following 2 cases (5, 6), and (6,5) we have following 6 cases- (1, 1),
So total number of ways is (2, 2), (3, 3), (4, 4), (5, 5), (6, 6) so
1+2+4+6+2 = 15 total number of ways is 6/36 = 1/6
So required probability is 4. (a) Here in this case sample space has
15/36= 5/12 23 elements.
Probability Introduction  253
In the given range numbers that are selected in 4C1 × 6C1 ways such that
either multiple of 5 or 7 is 5, 7, 10, one of them is red and the other is
14, 15, 20 and 21 i.e total 7 numbers white hence required probability is
so total 23 – 7 = 16 numbers neither
multiple of 5 nor that of 7. Hence 4×6×2 8
=
required number of ways is 16/23.   10 × 9 15
5. (b) Sample space in this case is 12. (a) Total number of fruits is 6 + 4 + 8
  6 × 6 = 36 = 18, so three fruits can be selected
Sum of two results is a perfect square in 18C3 ways and that represents the
number we have following cases- sample space.
Case (i) If sum is 10 then we have 3 apples can be selected out of 6 in
6C ways so required probability is
following 3 cases (4, 6), (5, 5) and 3
(6,4) 6
C3 6×5×4 5
= =
Case (ii) If sum is 11 then we have 18
C3 18 ×17 × 16 204
following 2 cases (5, 6), and (6,5)  
Case (iii) If sum is 12 then we have Alternately:
following 1 case (6, 6) Probability that 1st one is apple is
So total number of ways is 6/18, 2nd one apple is 5/17 and 3rd one
apple is 4/16 so required probability
  3 + 2 + 1 = 6
6×5×4 5
So required probability is 6/36 = 1/6 is =
18 × 17 × 16 204
6. (a) Since a leap year has 52 weeks and
13. (a) Probability that none of them will
2 days hence required probability is
appear tail is (1/2)5 = 1/32
2/7
So required probability that at least
7. (b) Since a non leap year has 52 weeks
one tail will appear is 1 – 1/32 = 31/32
and 1 day hence required probability
is 1/7 14. (a) Out of 12 students 5 can be selected
in 12C5 = (12 × 11 × 10 × 9 × 8)/120 =
8. (b) Probability of getting a head in a
792 ways and the number of ways of
single throw is 1/2 hence required selecting 3 girls and 2 boys is 6C3 ×
probability is (1/2)(1/2)(1/2) = 1/8 6C = 20 × 15 = 300.
2
9. (c) Probability of getting a head in a So required probability is 300/792 =
single throw is 1/2 hence required 75/198
probability is 3(1/2)(1/2)(1/2) = 3/8 15. (a) From the definition odds in favor
of an event is n(E)/n(E′) and odds
10. (d) Here sample space is selecting 2 out against an event is n(E′)/n(E) and
of 10 i.e10C2and 2 red balls can be from the given information is
selected in 4C2 ways hence required  n(E) 
 
4
C 4 ×3 2  n(E′ )  = 9   or n(E) = 3
probability is 10 2 = =  n(E′ )  25 n(E′ ) 5
C2 10 × 9 15  
 n(E) 
11. (b) Here sample space is selecting 2
out of 10 i.e 10C2 and 2 balls can be hence required probability is 3/8
254  Theory of Counting
Part B

Concept 1 Introduction and Properties of Sets

Operations of Sets Properties of Intersection:


Union of Sets: Union of two or more sets is A∩B=B∩A
the set of all elements that belong to any one A∩A=A
of the sets or present in both of them. The
If A and B are disjoint sets then A ∩ B = Φ
symbol used for union of sets is ‘∪’ i.e.
A∩Φ=Φ
A ∪ B = Union of set A and set.
(A ∩ B) ⊆ A and (A ∩ B) ⊆ B
Example: A = {1, 2, 3, 4} and B = {2, 4, 5, 6}
then A ∪ B = {1, 2, 3, 4, 5, 6} A ∩ (B ∩ C) = (A ∩ B) ∩ C, i.e., associative law
Properties of Difference of Sets:
Properties of Union:
A–B≠B–A
A∪B=B∪A
A – B ⊆ A and B – A ⊆ B
A∪A=A
A–Φ=A
A ∪ U = U, where U is the universal set
(A – B) ∪ B = A ∪ B and (A – B) ∩ B = Φ
A ∪ Φ =A
A ⊂ A∪ B and B ⊂ A ∪ B Complement of a Set:
If B ⊆ A, then A ∪ B = A The complement of a set A is the set of elements
A ∪ (B ∪ C) = (A ∪ B) ∪ C, this is known as which do not belong to A, in other words, U – A.
associative law Symbolically, denoted as, Ac or A′
A′ = {x|x∈ U, x ∉ A} = U – A

Concept 2 Probability

The probability of occurrence of an event E is Here n(E) represents number of elements in


denoted by P(E) and is defined as - E = number of cases favorable to event E and
n (E) n(S) represents number of elements in Sample
P  (E) = space S = total number of cases
n (S )

Probability Introduction  255

Concept 3 Properties of Probability

Property 1: Probability of occurrence of an Property 2: If E is any event and E' be the


event is a number lying between 0 and 1 complement of event E, then P(E) + P(E') = 1

Concept 4 Odds in Favor & Against

Consider a random experiment, S be its sample Odds against an Event E


space and E be an event . Let E′ denote the
Odds against an event E = n(E′)/n(E)
complement of event E, then we know that
n(E) + n(E′) = n(S) and P(E) + P(E′) = 1  n (E ')
• Odds in favor of event E  
 n (S )  = P (E ')
Odds in favor of event E is defined as =
 n (E) P (E)
 n (E)  
   n (S )
n (E)  n (S ) P (E)
= = = If odds in favor of an event is ‘F’ and against
n (E’)  n (E’) P (E’)
  an event is ‘A’ then F × A = 1
 n (S ) 
256  Theory of Counting

1
Concept Deviator (CD)
Ideal Time Revise your concept (IIT advance or
5 Min. tougher questions)
Per Question

1. Four fair dice D1, D2, D3 and D4, each 5. Following question contains
having six faces numbered 1, 2, 3, 4, 5 STATEMENT − 1 (Assertion) and
and 6, are rolled simultaneously. The
STATEMENT – 2 (Reason).
probability that D4 shows a number
appearing on one of D1, D2 and D3 is: It has 4 choices (a), (b), (c) and (d) out of
which ONLY ONE is correct. Select the
[IIT JEE 2012]
correct one- [IIT JEE 2007]
(a) 91/216 (b) 108/216
Let H1, H2, …, Hn be mutually exclusive
(c) 125/216 (d) 127/216
and exhaustive events with P(Hi) > 0,
Direction (Qs. 2 and 3): i = 1, 2, …, n. Let E be any other event
Let U1 and U2 be two urns such that U1 with 0 < P(E) < 1.
contains 3 white and 2 red balls , and U2
STATEMENT -1: P(Hi|E) > P(E | Hi).
contains only 1 white ball. A fair coin is
P(Hi) for i = 1, 2, …, n
tossed. If head appears then 1 ball is drawn at
random from U1 and put into U2. However if because n
tail appears then 2 balls are drawn at random STATEMENT -2: P (Hi )=1

from U1 and put into U2. Now 1 ball is drawn i=1
at random from U2. (a) Statement–1 is True, Statement
2. The probability of the drawn ball from –2 is true; Statement-2 is a correct
U2 being white is (IIT JEE 2011) explanation for Statement-1
(a) 13/30 (b) 23/30 (b) Statement–1 is True, Statement–2 is
(c) 19/30 (d) 11/30 True; Statement-2 is NOT a correct
3. Given that the drawn ball from U2 is explanation for Statement-1
white, the probability that head appeared
(c) Statement–1 is True, Statement–2
on the coin is (IIT JEE 2011)
is False
(a) 17/23 (b) 11/23
(d) Statement–1 is False, Statement–2
(c) 15/23 (d) 12/23
is True
4. Let ω be a complex cube root of unity with
ω ≠ 1. A fair die is thrown three times. If Direction (Qs. 6 to 8):
r1, r2, and r3 are the numbers obtained There are n urns each containing n + 1 balls
on the die , then the probability that ω r1 such that the ith urn contains i white balls
+ ω r2 + ω r3 = 0 is [IIT JEE 2010]
and (n + 1 − i) red balls. Let ui be the event of
(a) 1/18 (b) 1/9
selecting ith urn, i = 1, 2, 3 …, n and w denotes
(c) 2/9 (d) 1/36 the event of getting a white ball.
Probability Introduction  257
6. If P(ui) α I, where I = 1, 2,3 … n then each having numbered seats, 3 in front
lim P( w) is equal to   [IIT JEE 2006] and 4 at the back such that 3 girls sit
n→∞
together at the back row of a van?
(a) 1 (b) 2/3
[IIT JEE 1996]
(c) 3/4 (d) 1/4
(a) 9/91 (b) 4/91
7. If P(ui) = c, where c is a constant ∀i then
(c) 1/91 (d) None of these
P(ui/w) is equal to [IIT JEE 2006]
13. Two fair dice are thrown what is the
(a) 2/(n+1) (b) 1/(n+1)
probability that 1st die will show 4 and
(c) n/(n+1) (d) 1/2 2nd one will not show 4?
8. If n is even and E denotes the event of (a) 1/6 (b) 5/12
choosing even numbered urn ( P(ui) =
(c) 5/36 (d) None of these
1/n) , then the value of P(ui / E) is
14. Two fair dice are thrown. What is the
[IIT JEE 2006]
probability that summation 12 will
(a) (n+2)/(2n+1) (b) (n+2)/2(n+1) appear on 12th throw?
(c) n/(n+1) (d) 1/(n+1)
3511 3511
9. If P(B) = 3/4, P(A∩B∩C′) = 1/3 and (a) (b)
3612 3611
P(A′∩B∩C′) = 1/3 then P(B∩C) is, where
A, B, C are in 3 events. [IIT JEE 2003] 3510
(c) (d) None of these
(a) 1/12 (b) 1/6 3612
(c) 1/15 (d) 1/9 15. 3 people are select at random from a
large population. What is the probability
10. Two numbers are selected randomly
that they all have the same birthday (for
from the set S = {1, 2, 3, 4, 5, 6} without
non leap year)?
replacement one by one. The probability
that minimum of the two numbers is less 1 1
(a) 2
(b)
than 4 is [IIT JEE 2003] 365 3653
(a) 1/15 (b) 14/15 2
(c) (d) None of these
(c) 1/5 (d) 4/5 365
11. A person has to go through three 16. A box contains 2 black, 4 white and 3
successive tests. Probability of his passing red balls. One ball is drawn at random
first exam is P. Probability of passing from the box and kept aside. From the
successive tests is P or P/2 according remaining balls in the box, an- other ball
as he passed the last test or not. He is is drawn at random and kept beside the
selected if he passes at least two tests. first. The process is repeated till all the
Find the probability of his selection. balls are drawn from the box. Find the
[IIT JEE 2003] probability that the balls drawn are in
(a) P2(1 – P) (b) P2(2 – P) the sequence 2 black, 4 white and 3 red.
(c) P2(3 – P) (d) None of these [IIT JEE 1978]
12. What is the probability that three girls (a) 1/630 (b) 1/1260
and nine boys can be seated in two vans
(c) 1/2520 (d) None of these
258  Theory of Counting
17. Two events A and B have probabilities of 19. A determinant is chosen at random from
0.25 and 0.50 respectively. The probability the set of all determinants of order 2 with
that both A and B occurs simultaneously elements 0 or 1 only. The probability
is 0.14. Then the probability that neither that the value of the determinant chosen
A nor B occurs is    [IIT JEE 1980] is positive is. [IIT JEE 1982]
(a) 0.39 (b) 0.25 (a) 0.3024 (b) 0.2349
(c) 0.11 (d) none of these (c) 0.6976 (d) None of these
18. An anti-aircraft gun can take a maximum
20. A and B are two candidates seeking
of four shots at an enemy plane moving
admission in IIT. The probability that A
away from it. The probabilities of hitting
is selected is 0.5 and the probability that
the plane at the first, second, third and
both A and B are selected is at most 0.3.
fourth shot are 0.4, 0.3, 0.2 and 0.1
What is the maximum value of probability
respectively. What is the probability that
of B getting selected?    [IIT JEE 1982]
the gun hits the plane?  [IIT JEE 1981]
(a) 0.7 (b) 0.8
(a) 0.3024 (b) 0.2349
(c) 0.6976 (d) None of these (c) 0.6 (d) None of these

1. a b c d 2. a b c d 3. a b c d 4. a b c d 5. a b c d
6. a b c d 7. a b c d 8. a b c d 9. a b c d 10. a b c d
11. a b c d 12. a b c d 13. a b c d 14. a b c d 15. a b c d
16. a b c d 17. a b c d 18. a b c d 19. a b c d 20. a b c d

2
Concept Eliminator (CE)
Ideal Time Revise your concept (Maths Olympiad
Just solve it or tougher questions)

1. This question is Based on Maths Olympiad 2. This question is Based on Maths Olympiad
A bag contains 8 white balls and 2 red Nine numbered chits, numbered 1, 2,,,,9,
were put into a packet. Now A draws a ball
balls. Each time one ball is drawn and
from the packet, noted that it is of number
replaced by a white one. a ,and puts back it. Then B also draws a
Then the probability of drawing out all of ball from the packet and noted that it is
the red balls just in the fourth draw is- of number b. Then the probability for the
inequality a – 2b + 10 > 0 to hold is:
(a) 434/10000 (b) 234/10000 (a) 53/81 (b) 61/81
(c) 334/10000 (d) None of these (c) 62/81 (d) 65/81
Probability Introduction  259
3. If A, B, C are events such that P(A) = 0.3, 5. Three identical dice are rolled. The
P(B) = 0.4, P(C)= 0.8 probability that the same number will
P(A ∩ B) = 0.08, P(A ∩ C) = 0.28 appear on each of them is
P(A ∩ B ∩ C) = 0.09 (a) 1/6 (b) 1/36
If P(A ∪ B ∪ C) ≥ 0.75, then find the range (c) 1/18 (d) 3/28
of x = P(B ∩ C) lies in the interval 6. If M and N are any two events, the
(a) 0.23 ≤ x ≤ 0.48 probability that exactly one of them
(b) 0.23 ≤ x ≤ 0.47 occurs is

(c) 0.22 ≤ x ≤ 0.48 (a) P(M) + P(N) – 2P( M ∩ N)

(d) None of these (b) P(M) + P(N) – P(M ∩ N)′

4. Fifteen coupons are numbered 1, 2, …, (c) P(M′) + P(N′) – 2P (M′ ∩ N′)


15 respectively. Sevens coupons are (d) P(M ∩ N′) + P (M′ ∩ N)
selected at random one at a time with 7. A box contains 100 tickets numbered 1, 2,
replacement. The probability that the … 100. Two tickets are chosen at random.
largest number appearing on a selected It is given that the maximum number on
coupon as 9, is the two chosen tickets is not more than
6 7 10. The minimum number on them is 5
 9  8
(a)   (b)   with probability is
 19   15 
7 (a) 7/15 (b) 3/5
 3
(c)   (d) None of these
 5 (c) 13/15 (d) None of these

Response Grid
1. a b c d 2. a b c d 3. a b c d 4. a b c d 5. a b c d
6. a b c d 7. a b c d
260  Theory of Counting

Solutions
Concept Deviator (CD)
1. (a) 2. (b) 3. (d) 4. (c) 5. (d) 6. (b)
7. (a) 8. (b) 9. (a) 10. (d) 11. (b) 12. (c)
13. (c) 14. (a) 15. (a) 16. (b) 17. (a) 18. (c)
19. (d) 20. (b)

1. (a) In this case sample space is n(S) × 2/3) = 23/30

= 6 × 6 × 6 × 6 = 64 3. (d) From conditional probability

Let E be the event that D4 shows a P(H/W) = (P(W/H) × P(H)) / P(W/T) .


number appearing on one of D1, D2 P(T) + P(W/H). P(H) = 12/23
and D3 4. (c) Required probability :
Then n(E′) = 6 × 5 × 5 × 5
3! × 2 C 1 × 2 C 1 × 2 C 1
= 2/9
So P(E′) = (6 × 5 × 5 × 5)/(6 × 6 × 6 × 6) 6×6×6
= 125/216 5. (d) From statement 1, P(H∩E) = 0 then
P(Hi/E) = P(E/Hi) = 0
Hence P(E) = 1–P(E′) = 1–125/216
And if P(H∩E) ≠ 0 then
= 91/216
P(Hi/E) = P(Hi∩E)/P(Hi) × P(Hi)/P(E)
2. (b) As per the given condition if outcome
> P(E/Hi) × P(Hi)
is H – 1 ball from U1 to U2
Since 0 < P(E) < 1
And if outcome is T – 2 ball from U1
to U2 So statement 1 is not always true,
while 2 is true
E : 1 ball drawn from U2
6. (b) Since P(ui) α i or P(ui) α ki and
P(W from U2 ) = 1/2 × ( 3/5 × 1 ) + 1/2
∑P (ui )=1
× (2/5 × 1/2) +1/2 × ( 3C2/5C2 ) + 1/2 ×
hence k = 2/n(n+1)
(2C2/5C2 × 1/3) + 1/2 × ( 3C1 . 2C1 / 5C2
Probability Introduction  261
n So total probability
2i 2
And lim P( w) = lim ∑
n→∞ n→∞ i = 1 n (n + 1)2 = P 3 +  2 P 2 (1 − P ) =  P 2 (2 − P )

2 n(n + 1)(2 n + 1) 2 12. (c) 3 girls can be seated at the back row
= lim =
in 2 × 2(3!) ways, and remaining 9
n→∞ 6n(n + 1)2 3
boys can be arranged on remaining
 n  11 seats in (11P9) ways
c 
 n + 1 = 2
7. (a) Since P(ui/w) =
( )
 ∑i  n + 1
c
so total number of ways is 2 × 2(3!)
 (11P9) = 12!
n + 1
 
8. (b) Here we need to find P(W/E) Total number of ways is (14!)/2

2 + 4 + 6 + .. + n n+2 So probability is (12!)/{14!)/(2!)} = 1/91


= =
 n (n + 1) 2 (n + 1) 13. (c) Sample space is 6 × 6 = 36
 
 2 
And favorable case is (3, 1) (3, 2), (3, 4),
9. (a) We know that P(B∩C) (3, 5) and (3, 6)

= P(B) – P(A∩B∩C′) – P(A′∩B∩C′) So required probability is 5/36

= (3/4) – (1/3) – (1/3) = 1/12 Alternately:

10. (d) Sample space is (2!)(6C2) = 30, Probability for 1st dice is 1/6 and that
Favorable cases are 24 so required for 2nd dice is 5/6 since these two
probability is 24/30 = 4/5 are independent events so required
probability is (1/6)(5/6) = 5/36
11. (b) As per the given condition Person is
selected if either he passes all the 14. (a) Probability of getting summation as
three tests or he passes exactly two 12 is 1/36 and not getting sum as 12
of the tests. is 35/36.

Probability that he passed all the Since summation 12 appeared on


three tests is P×P×P = P3 12th throw so till 11th throw each


throw result in a summation which
Now We need to find the probability of
is not 12 so required probability
passing the two tests
11 11
 35   1  35
 P  P =      = 12
P 2 (1 − P )+  P   (1 − P )+  (1 − P )  P    36  36 36
 2  2
=  2 P 2 (1 − P )
262  Theory of Counting
15. (a) Let 1st guy has any birth day Hence required probability is
Say 20th Dec- then for 2nd guy P(A∪B)′ = 1 – P(A∪B) = 1–0.61 = 0.39
probability is 1/365 and for 3rd guy
18. (c) Probability that none of them hit the
it is 1/365 and required probability
plane is (1–0.4)(1–0.3)(1–0.2)(1–0.1)
 1  1  1
is     = = 0.6 × 0.7 × 0.8 × 0.9 = 0.3024
 365   365  3652
So required probability is 1 – 0.3024
16. (b) Probability that 1st two balls are
= 0.6976
black is (2/9)(1/8)
19. (d) In this case sample space is 24 = 16
Probability that next 4 balls are
4 3 2 1 Number of determinant with positive
white is        
 7   6  5  4 values is 3
Remaining must be red, hence Required probability is 3/16
required probability is
20. (b) As per the given information P(A) = 0.5
 2   1  4   3  2   1  1
            = Probability that both A and B are
9 8 7 6 5 4 1260
selected is P(A ∩ B) ≤ 0.3
17. (a) As per the given condition P(A) + P(B) – P(A ∪ B) ≤ 0.3
P(A) = 0.25, P(B) = 0.50 and P(A∩B) = 0.14 P(B) ≤ 0.3 + P(A∪B) – P(A)

And P(A∪B) = P(A) + P(B) – P(A∩B) ≤ P(A∪B) – 0.2


= 0.25 + 0.50 – 0.14 = 0.61 So P(B) ≤ 0.8

Concept Eliminator (CE)


1. (a) 2. (b) 3. (a) 4. (d) 5. (b) 6. (a,c,d)
7. (c)
Probability Introduction  263
1. (a) In this case we have three cases-

1st 2nd 3rd 4th


Probability
draw draw draw draw

Case 1 Red White White Red  2  9  9  1  162
        =
10 10 10 10 10000

Case 2 White Red White Red  8  2  9  1  144
        =
10 10 10 10 10000

Case 3 White White Red Red  8  8  2  1  128
        =
10 10 10 10 10000
Total
434/10000
Probability

2. (b) Total number of outcome in two Total number of favorable cases


draw is 9 × 9 = 81, = 45 + 7 + 5 + 3 + 1= 61
Given inequality is a – 2b + 10> 0 or So required probability = 61/81
2b < 10 + a
3. (a) Since P(A∪B∪C) = P(A) + P(B) +P(C)
Now we have following cases- – P(A ∩ B) – P(B∩C) – P(C ∩ A) +
Case (i) if b = 1, 2, 3, 4 or 5 then b can P(A ∩ B ∩ C)
take any value from 1 to 9 and it Or P(A∪B∪C) = 0.3+0.4+0.8 – (0.08
will satisfy the given inequality, so +0.28 + P(B ∩ C)) + 0.09
number of favorable cases is 9 × 5
= 1.23 – P(B ∩ C)
= 45
Or P(B ∩ C) = 1.23 – P(A∪B∪C)
Case (ii) If b = 6, then a can take any
But we know that 0 ≤ P(A∪B∪C) ≤ 1
value from 3 to 9 i.e 7 values.
Hence 0.23 ≤ P(B ∩ C) ≤ 0.48
Case (iii) If b = 7, then a can take any
value from 5 to 9 i.e 5 values. 4. (d) Here sample space is 157

Case (iv) If b = 8, then a can take any Favorable cases are (97 – 87)
value from 7 to 9 i.e 3 values. 97 – 87
So required probability is
Case (v) If b = 9, then a can take only 157
one value i.e 9.
264  Theory of Counting
5. (b) Sample space is 66 Now P(E/D) = P(E ∩ D) / P(E)
Number of favorable case is 6C1 = 6 = n(E ∩ D) / n(E)
So required probability
Now the number of ways of getting
 6  1 a number ‘a’ on the two tickets is
=  =
 216  36
coefficient of xa in the expansion of
6. (a,c & d) As per the given condition exactly (x1 + x2 +…+ x100)2 = x2 (1 + … + x99)2
one of M and N occur so = x2 [(1 – x100)/(1 – x)]2 = x2 (1 – 2x100
Option (a) P(M) + P(N) – 2P(M∩N) is + x200) (1 – x)-2
correct Now, x2(1 – 2x100 + x200)(1 + 2x +3x2
And Option (c) is correct P(M′) + + 4x3 + …)
P(N′) – 2P (M′ ∩ N′)
Thus , co eff. of x2 = 1, of x3 = 2, of x4 = 3
7. (c) let assume that E be the event that of x10 = 9
has maximum number on the two
Hence ,
chosen tickets is not more than 10,
that is the no. on them ≤ 10 and D n(E ∩ D) = 4 + 5 + 6 + 7 + 8 + 9 = 39
the event that the minimum no. on N(E) = 1 + 2 + 3 + 4 + … + 9 = 45
them is 5 that the minimum no. on
Therefore , required probability
them is 5 that is the no. on them is
≥ 5. We have to find P (E/D). = P(D/E) = 39/45 = 13/15
10
Dependent and Independent Events  265

Dependent and
Independent Events

Topics Covered

 Dependent & Independent Events

 Addition Theorems of Probability

Part A: Topic Number of Questions


Solved Example 10
Concept Applicator 20
Concept Builder 16
Concept Cracker 20
Part B: Topic Number of Questions
Concept Deviator 15
Concept Eliminator 5
Total 86
266  Theory of Counting
Part A
Dependent and
Concept 1 Independent Events
Two or more events of an experiment are Properties of Independent events
said to be independent if occurrence or Consider two independent events A and B:
non- occurrence of any one of them does not
• A and B′ are independent events
affect the probability of occurrence or non-
• A′ and B′ are independent events
occurrence of other event.
• A′ and B are independent events
Consider A and B as two independent
• For two independent events
events then probability of event B i.e P(B) is
P(A ∩ B) = P(A) × P(B)
completely independent from event A i.e P(A)
• P (A ∪ B) = 1 – P(A′) P(B′)
Let us take some examples to understand it—
Example 1:  Consider two independent
Examples : events A and B such that P(A) = 1/4
and P(B) = 1/3 then find the value of
(i) When a fair coin is tossed twice, the event P(A ∩ B).
of occurrence of ‘Tail’ in the first throw Solution:  We know that for two independent
and the event of occurrence of ‘Head’ in events P( A ∩ B) = P(A) × P(B) = (1/4)(1/3) = 1/12.
the second throw are independent events,
Example 2:  Consider two independent
in other word result of 1st coin has no
events A and B such that P(A) = 1/4 and
relation with the result of 2nd coin. P(B) = 1/3 then find the value of P(A ∪ B).
(ii) When two dice are thrown then Solution:  We know that for two independent
probability of getting an odd number events P( A ∩ B) = P(A) × P(B) = (1/4)(1/3) = 1/12.
from 1st die and even number from 2nd Then P(A ∪ B) = P(A) + P(B) – P( A ∩ B) =
die is independent from each other (1/4) +(1/3) – (1/4)(1/3) = 1/2.
(iii) Let a bag contain 10 red and 5 black Example 3:  Consider two independent
balls. Two balls are drawn one by one events A and B such that P(A) = 1/4 and
with replacement, then probability of P(B) = 1/3 then find the value of P(A′ ∩ B′)
getting a red ball in 2nd draw depends Solution:  We know that for two independent
on the result of 1st draw hence these two events P(A ∪ B) = 1 – P(A′) P(B′) or P(A′)
results are dependent events. P(B′) = 1 – (1/2) = 1/2

QR Code / Video Link

For Smart phone/ Tablet users

Video Link for Desktop/Laptops users http://dishapublication.com/video-resources


Dependent and Independent Events  267
Example 4:  If two die are thrown Let P(A) = Probability of getting a prime number
simultaneously then what is the = (3/6) = (1/2)
probability that 1st one will show
Let P(B) = Probability of getting a composite
a prime while 2nd one will show a
number = (2/6) = (1/3)
composite number?
Solution:  Result from 1st dice and that from So required probability
2nd are independent from each other. P(A ∩ B) = P(A) × P(B)= (1/2)(1/3) = 1/6.
Hence we can use P( A ∩ B) = P(A) × P(B).

Addition Theorems
Concept 2 of Probability
If A and B be any two events in a sample Intersection of Sets
space S of an experiment, then the probability It is the set of all the elements, which are
of occurrence of at least one of the events A common to all the sets. The symbol used for
and B is given by intersection of sets is ‘∩’ i.e.
P ( A ∪ B ) = P (A) + P (B) – P (A ∩ B) A ∩ B = {x| x ∈A and x ∈ B}.
Representation of Union of Sets Example: If A = {1, 2, 3, 4} and B = {2, 4, 5, 6}
then A ∩ B = {2, 4}
A B Properties of Intersection:
A∩B=B∩A
A∩A=A
If A and B are disjoint sets then A ∩ B = Φ
A∩Φ=Φ
(A ∩ B) ⊆ A and (A ∩ B) ⊆ B
A ∩ (B ∩ C) = (A ∩ B) ∩ C,
i.e., associative law
From the set theory we know that: A B
n( A ∪ B ) = n(A) + n (B) – n ( A ∩ B )
Dividing both sides by n (S), we get

n(A  B) n(A) n(B) n(A  B)


  –
n(S) n(S) n(S) n(S)

Or P(A ∪ B) = P(A) + P(B) – P (A ∩ B)


268  Theory of Counting
Some more properties If A, B, C are mutually exclusive events, then
(i) If A and B are mutually exclusive events, A ∩ B = φ, B ∩ C = φ, A ∩ C = φ, A ∩ B ∩ C = φ
then A ∩ B = φ and hence \ P(A ∪ B ∪ C) = P(A) + P(B) + P(C)
P( A ∩ B ) = 0
\ P(A ∪ B) = P (A) + P(B) Example 5:  If probability that A can solve
a question is 2/3 and that B can solve
(ii) Two events A and B are mutually exclusive
the same problem is 4/5, if both of them
if and only if
attempted this question then what is
P(A ∪ B) = P(A) + P(B) the probability that none of them is
(iii) Two events A and B are mutually able to solve that question?
exclusive and collectively exhaustive if Solution:  Since probability that A can solve
and only if
that question is P(A) = 2/3
P(A ∪ B) = P(A) + P(B) = 1
Hence P(A′) = 1/3 and similarly P(B′) = 1/5.
(iv) P(A′) = 1 – P(A)
So probability that none of them can solve the
Addition theorem for three events question is 1/3 × 1/5 = 1/15.
Consider three events A, B and C in a sample
Example 6:  If probability that A can
space S then from the addition theorem-
solve a question is 2/3 and that B can
P( A ∪ B ∪ C) = P(A) + P(B) + P(C) solve the same problem is 4/5, if both
– P(A ∩ B) – P(B ∩ C) of them attempted this question then
– P(A ∩ C) + P(A ∩ B ∩ C) what is the probability that question
is solved?
Proof: From set theory, we know that
n(S) = n(A) + n(B) + n(C) – n(A ∩ B) Solution:  This case is complement of the
previous case, so required probability
– n( B ∩ C) – n(A ∩C) + n ( A ∩ B ∩ C)
= 1 – 1/15 = 14/15
Dividing both sides by n (S), we get
n( A  ∪  B ∪  C ) n( A ) n( B ) n(C ) Example 7:  Probability that A will pass
  
n(S ) n(S ) n(S ) n(S ) the exam is 1/2, B will pass the exam
n( A  ∩  B ) n( B ∩  C ) is 2/3 and C will pass the exam is 3/4,
– – what is the probability that exactly
n(S ) n(S )
one of them will pass the exam?
n( A ∩ C ) n( A  ∩ B ∩ C )
–  Solution:  Since P(A) = 1/2. So, P(A′) = 1/2,
n(S ) n(S )
P(B) = 2/3. So, P(B′) = 1/3 and P(C) = 3/4
\ P(A ∪ B ∪ C) = P(A) + P(B) + P(C) – P( A ∩ B)
so P(C′) = 1/4
– P(B ∩ C) – P(A ∩ C) + P( A ∩ B ∩ C).

QR Code / Video Link

For Smart phone/ Tablet users

Video Link for Desktop/Laptops users http://dishapublication.com/video-resources


Dependent and Independent Events  269
Exactly one of them will pass the exam so Solution:  Probability of getting a ‘head’ is
required probability is P(A)P(B′)P(C′) + P(A′) P(H) = 1/2
P(B)P(C′) + P(A′)P(B′)P(C) = (1/2)(1/3)(1/4) +
It is given that Kushal started the game and
(1/2)(2/3)(1/4) + (1/2)(1/3)(3/4) = 6/24= 1/4
he wins the game in the following number of
tosses-
Example 8:  Gargy speaks the truth in 60%
cases while Ashmeet speaks the truth Case (i) Number of toss is 1- the probability
in 90% of the cases. In what percentage is 1/2.
of cases they are likely to contradict
Case (ii) Number of toss is 3- This is possible
each other is stating the same fact?
when Kushal didn’t get ‘head’ in 1st throw,
Solution:  There are two cases when they will Karina didn’t get ‘head’ in 2nd throw and
contradict each other-
then finally Kushal get ‘head’ in 3rd throw so
Case (i) When Gargy is speaking truth and probability in this case
Ashmeet is lying then probability is (6/10)
(1/10) = 6/100. = (1/2)(1/2)(1/2) = 1/8.

Case (ii) When Gargy is lying and Ashmeet is Case (iii) Number of toss is 5- similar to above
speaking truth then probability is case probability in this case is 1/32.
(4/10)(9/10) = 36/100. So total probability that Kushal wins the
1
Required probability
 1  1  1  2
game =    +    +    + …∞  = 2 =
= 6/100 + 36/100 = 42/100 = 42%  2   8   32  1 3
1−
4
Example 9:  Kushal and Karina is playing
Example 10:  In the above question what
a game “Pahle Tum, Pahle Tum” in
is the probability that Karina will win
which they throw a coin alternately till
the match.
one of them gets a ‘head’ and wins the
game. Find the probability that Kushal Solution:  Probability that Karina will win is
wins the game if he started the game? 1 – 2/3 = 1/3.

1
Concept Applicator (CA)
Ideal Time Apply your concepts with easy and
20 Min. conceptual questions

1. If P(A) = 2/11, P(B) = 2/5 and P(A ∩ B) 3. If P(A) = 1/8, P(B) = 2/5 and A and B are
= 1/5 then find the value of P(A ∪ B) mutually exclusive then find P(A ∪ B).
(a) 23/55 (b) 19/55 (a) 23/40 (b) 19/40
(c) 21/55 (d) None of these (c) 21/40 (d) None of these
2. If P(A) = 2/11, P(B) = 2/5 and P(A∩B) 4. If P(A) = 1/8, P(B) = 2/5 and A and B are
= 1/5 then find the value of P(A′∩B′) mutually exclusive then find P(A′ ∪ B′).
(a) 34/55 (b) 37/55 (a) 23/40 (b) 19/40
(c) 21/55 (d) None of these (c) 21/40 (d) None of these
270  Theory of Counting
5. For two independent events A and B, it 12. If P(A) = 1/3 and P(B) = 1/4 and it is given
is known that P(A) = 0.4, P(A ∪ B) = 0.6 that events A and B are independent
then find the value of P(B). events then find P(A ∩ B).
(a) 1/2 (b) 1/3 (a) 5/12 (b) 7/12
(c) 2/3 (d) None of these (c) 1/12 (d) None of these
6. For two independent events A and B, it Direction for question number 13 to 15:
is known that P(A) = 0.4, and P(B) = 0.6 A bag contains 5 red and 6 black balls, while
the find the value of P(A ∪ B). another bag contains 4 red and 5 black balls.
One ball is drawn from each bag then:
(a) 0.47 (b) 0.56
13. Find the probability that both the balls
(c) 0.76 (d) None of these
are of red color?
7. For two independent events A and B, it
(a) 20/99 (b) 23/99
is known that P(A′) = 0.4, and P(B′) = 0.6
the find the value of P(A ∪ B). (c) 35/99 (d) None of these
(a) 0.47 (b) 0.56 14. Find the probability that both the balls
are of black color?
(c) 0.76 (d) None of these
(a) 20/99 (b) 23/99
8. Amar and Bimal are two contestants for
(c) 35/99 (d) None of these
an election, probability that Amar will
win is 1/4 and that Bimal will win the 15. Find the probability that one of the ball
election is 1/5, then what is probability is of red color while other of black color?
that either Amar or Bimal will win the (a) 57/99 (b) 25/99
election. (c) 49/99 (d) None of these
(a) 13/20 (b) 9/20 16. A bag contains 4 red, 5 green and 6 black
(c) 11/20 (d) None of these balls. Two balls are drawn one after
9. The probability that a police inspector the other with replacement what is the
Ravi will catch a thief in a day is 1/4 and probability that both the balls are of red
colored?
the probability he will catch a robber in
that day is 1/5 and the probability that (a) 20/225 (b) 16/225
he will catch both a thief and a robber in (c) 32/225 (d) None of these
a day is 1/15 then what is the probability 17. A bag contains 4 red, 5 green and 6 black
that Ravi will catch at least 1 mischief? balls. Two balls are drawn one after the
(a) 23/60 (b) 19/60 other without replacement what is the
(c) 7/20 (d) None of these probability that both the balls are of red
colored?
10. A bag contains 6 green and 5 black
balls if two balls are drawn what is the (a) 2/35 (b) 3/35
probability that both of them are of same (c) 4/35 (d) None of these
color? 18. Gunjan has two bags 1st bag contains 5
(a) 41/110 (b) 9/22 black and 6 red balls while another bag
(c) 2/11 (d) None of these has 6 black and 5 red balls, Gunjan has
drawn two balls one from each bag, what
11. A bag contains 6 green and 5 black if two is the probability that one ball is of black
balls are drawn what is the probability and one is red color?
that both of them are of different color?
(a) 30/121 (b) 45/121
(a) 41/110 (b) 9/22
(c) 60/121 (d) None of these
(c) 2/11 (d) None of these
Dependent and Independent Events  271
19. If probability that A can solve a question is 20. If probability that A can solve a question
3/4 and that B can solve the same problem is 3/4 and that B can solve the same
is 4/5, if both of them attempted this problem is 4/5, if both of them attempted
question then what is the probability that this question then what is the probability
none of them able to solve that question? that question is solved?
(a) 1/20 (b) 3/20 (a) 19/20 (b) 13/20
(c) 7/40 (d) None of these (c) 17/40 (d) None of these

Response Grid
1. a b c d 2. a b c d 3. a b c d 4. a b c d 5. a b c d
6. a b c d 7. a b c d 8. a b c d 9. a b c d 10. a b c d
11. a b c d 12. a b c d 13. a b c d 14. a b c d 15. a b c d
16. a b c d 17. a b c d 18. a b c d 19. a b c d 20. a b c d

2
Concept Builder (CB)
Ideal Time Revise your concepts with questions
25 Min. medium difficulty level questions

1. If P(A′) = 9/11, P(B′) = 3/5 and P(A∩B) = Direction for question number 5 to 7:
1/5 then find the value of P(A ∪ B) Rajesh appeared in IIT JEE exam and the
(a) 23/55 (b) 19/55 probability that he will pass in Paper I is 4/5
and the probability that he will pass in Paper
(c) 21/55 (d) None of these
II is 1/3. If the probability that he will pass in
2. If P(A′) = 9/11, P(B′) = 3/5 and P(A∩B) both the papers is 1/2 then:
= 1/5 then find the value of P(A′ ∩ B′) 5. What is the probability that he will pass
(a) 34/55 (b) 37/55 in at least one subject?
(c) 21/55 (d) None of these (a) 19/30 (b) 23/30
(c) 17/30 (d) None of these
3. If P(A′) = 7/8, P(B′) = 3/5 and A and B are
mutually exclusive then find P(A ∪ B). 6. What is the probability that he will fail
in at least one subject?
(a) 23/40 (b) 19/40
(a) 19/30 (b) 23/30
(c) 21/40 (d) None of these (c) 17/30 (d) None of these
4. If P(A′) = 7/8, P(B′) = 3/5 and A and B are 7. What is the probability that he will fail
mutually exclusive then find P(A′ ∪ B′). in both the subjects?
(a) 23/40 (b) 19/40 (a) 19/30 (b) 23/30
(c) 21/40 (d) None of these (c) 17/30 (d) None of these
272  Theory of Counting
8. Amar and Bimal are two contestants for (c) 41/99 (d) None of these
an election, probability that Amar will 13. If A and B are two independent events
lose is 3/4 and that Bimal will lose the and P(A' ∩ B') = 3/8 while P(A ∩ B) = 1/8
election is 4/5, then what is probability then find the value of P(A) – P(B) if
that either Amar or Bimal will win the P(A) > P(B).
election.
(a) 1/3 (b) 1/4
(a) 13/20 (b) 9/20
(c) 3/4 (d) None of these
(c) 11/20 (d) None of these st
14. Suman has 3 bags, 1 bag has 3 green
9. Amar and Bimal are two contestants for and 1 blue balls, 2nd bag has 2 green and
an election, odds against Amar will win 2 blue balls, while 3rd bag has 1 green
is 4 : 1 and odds against Bimal will win is and 3 blue balls. One ball is selected from
5 : 1 then what is probability that either each bag then what is the probability
Amar or Bimal will win the election? that the three selected balls will have 2
(a) 13/20 (b) 9/20 blue and 1 green ball.
(c) 11/20 (d) None of these (a) 19/32 (b) 17/32
10. Sumpa throws a die twice find the (c) 13/32 (d) None of these
probability of getting an even number in 15. Consider 2 independent events A and B,
1st throw and a prime number in 2nd throw.
if P(A) = 3P(B), P(A ∪ B) = 13/16 then
(a) 1/3 (b) 1/4
find the value of P(A)
(c) 3/4 (d) None of these
(a) 1/4 (b) 3/4
11. Sumpa throws a die twice find the
probability of getting at least 3 in 1st (c) 2/5 (d) None of these
throw and at most 3 in 2nd throw. 16. Two friends A and B appeared in IIT
(a) 1/3 (b) 1/4 JEE exam, Probability that A will crack
(c) 3/4 (d) None of these the exam is 2/5 and probability that B
12. A bag contains 4 red and 5 green balls will crack the exam is 3/4, what is the
while another bag contains 5 red and 6 probability only 1 from these two cracked
green balls, two balls are drawn one from the exam?
each bag what is probability that both of
them are of same color? (a) 11/20 (b) 9/20
(a) 49/99 (b) 71/99 (c) 13/20 (d) None of these

Response Grid
1. a b c d 2. a b c d 3. a b c d 4. a b c d 5. a b c d
6. a b c d 7. a b c d 8. a b c d 9. a b c d 10. a b c d
11. a b c d 12. a b c d 13. a b c d 14. a b c d 15. a b c d
16. a b c d
Dependent and Independent Events  273

3
Concept Cracker (CC)
Ideal Time Boost up your confidence with good
30 Min. questions

1. If P(A) = 1/4, P(B) = 2/5 then find the Bimal will win is 5 : 1 and odds in favor
range of P(A ∪ B) of Chetan will win 2 : 3 then what is
probability that either Amar or Bimal or
(a) (1/5, 13/20) (b) (1/4, 13/20)
Chetan will win the election.
(c) (2/5, 13/20) (d) None of these (a) 23/20 (b) 11/30
2. If P(A) = 1/4, P(B) = 2/5 then find the (c) 7/10 (d) None of these
range of P(A′ ∩ B′) 8. Ashmit and Bishmit are two students
who appeared in Class X exam,
(a) (1/4, 13/20) (b) (7/20, 3/5)
Probability that Ashmit will pass is 0.5
(c) (7/20, 13/20) (d) None of these while probability that both of them will
pass is at least 0.1 and at most 0.3, and
Direction for question number 3 to 5: probability that Bishmit will pass is P(B)
If A, B, C are mutually exclusive and then find the range of P(B)
collectively exhaustive events associated with (a) (0.2, 0.8) (b) (0.15, 0.25)
a random experiment then- (c) (0.1, 0.8) (d) None of these
3. Find P(A) if P(A) : P(B) : P(C) = 1 : 2 : 3 9. Gargy speaks truth in 80% cases and
(a) 1/6 (b) 1/3 Ashmeet speaks truth in 75% cases,
(c) 1/2 (d) None of these Hasan asked them a fact what is the
4. Find P(A′) if P(A′) : P(B′):P(C′) = 1 : 2 : 3 probability that contradict?
(a) 1/6 (b) 1/3 (a) 12/25 (b) 9/20
(c) 1/2 (d) None of these (c) 7/20 (d) None of these
5. Find P(A) if P(A′) : P(B) : P(C′) = 1 : 2 : 3 10. Three students A, B and C appeared in
IT JEE exam and probability that they
(a) 1/6 (b) 1/3
will pass the exam is 1/2, 1/3 and 1/4
(c) 1/2 (d) None of these
what is the probability that exactly two
6. If A1, A2, … An are mutually exclusive and of them will pass the exam?
collectively exhaustive events associated
with a random experiment then find the (a) 1/3 (b) 2/3
summation of P(A1′) + P(A2′) + P(A3′) + … (c) 1/4 (d) None of these
P(An′) 11. Aman and Binay are playing dart game
(a) n – 1 and it is known that Aman can hit the
(b) n target 4 out of 5 shots while Binay can
(c) n + 1 hit the target 3 out of 4 shots, what is the
(d) Can not be determine probability that target will be hit if both
of them try?
7. Amar, Bimal and Chetan are three
contestants for an election, odds against (a) 1/30 (b) 1/20
Amar will win is 4 : 1 and odds against (c) 1/40 (d) None of these
274  Theory of Counting
12. Kushal and Karina are playing with 17. Gargy speaks the truth in 60% cases
a dice wherein they throw a dice while Ashmeet is lying in 30% of the
alternately. Kushal wins if he throws a cases. In what percentage of cases they
prime number and Karina wins if she are likely to agree to each other is stating
throws a composite number. Kushal the same fact?
starts the game and game continues till (a) 54% (b) 27%
one of them win. What is the probability
(c) 49% (d) None of these
that Kushal will win the game?
18. At Pioneer Career Kolkata, Teacher
(a) 1/4 (b) 1/2
asked a question to his three students A,
(c) 3/4 (d) None of these B and C and the probabilities that they
13. In the above question what is the can solve that question is 1/3, 2/7 and 3/8
probability that Karina wins the game? respectively. If all the three students try
(a) 1/4 (b) 1/2 to solve the question simultaneously then
(c) 3/4 (d) None of these what is the probability that the exactly
one of them will solve the question?
14. Raj and Sanchita are playing game in
which they throw a die alternately till (a) 23/56 (b) 25/56
one of them gets a six. Which one of the (c) 85/156 (d) None of these
following could be the probability that 19. Amar can solve 80% of the questions of
Sanchita win the game?
Maths while Bimal can solve only 70%
(a) 7/15 (b) 6/11 of the questions. What percentage of
(c) 4/9 (d) None of these questions can be solved by them from a
15. Raj and Sanchita are playing game in maths book.
which they throw two dice alternately
till one of them gets 9. Which one of the (a) 89% (b) 91%
following could be the probability that (c) 93% (d) None of these
Sanchita win the game?
20. Probability that a man who is 40 year old,
(a) 7/15 or 8/15 (b) 6/11 or 5/11
living till 75 years is 5/16, and another
(c) 8/17 or 9/17 (d) None of these
man who is 35 years old living till 70
16. Gargy speaks the truth in 75% cases years is 3/7 then what is the probability
while Ashmeet in 80% of the cases. In
that at least one of them will be alive till
what percentage of cases they are likely
35 years hence?
to contradict each other is stating the
same fact? (a) 11/28 (b) 19/28
(a) 7/20 (b) 9/20 (c) 17/28 (d) None of these
(c) 11/20 (d) None of these

Response Grid
1. a b c d 2. a b c d 3. a b c d 4. a b c d 5. a b c d
6. a b c d 7. a b c d 8. a b c d 9. a b c d 10. a b c d
11. a b c d 12. a b c d 13. a b c d 14. a b c d 15. a b c d
16. a b c d 17. a b c d 18. a b c d 19. a b c d 20. a b c d
Dependent and Independent Events  275

Solutions
Concept Applicator (CA)
1. (c) 2. (a) 3. (c) 4. (b) 5. (b) 6. (c)
7. (c) 8. (b) 9. (a) 10. (c) 11. (d) 12. (c)
13. (a) 14. (d) 15. (c) 16. (b) 17. (a) 18. (c)
19. (a) 20. (a)

1. (c) From the formula P(A ∪ B) 7. (c) Here P(A) = 1 – 0.4 = 0.6 and P(B) =
= P(A) + P(B) – P(A ∩ B) 1 – 0.6 = 0.4.
2 2 1 10 +  22 –1   1 21 Since P(A ∪ B) = P(A) + P(B) – P(A ∩ B)
= + – = =
11 5 5 55 55 = 0.6 + 0.4 – (0.6) × (0.4) = 1 – 0.24
2. (a) Since we know that = 0.76.
P(A′ ∩ B′) = P(A ∪ B)′ 8. (b) From the given information
probability that Amar will win the
= 1 – P(A ∪ B)
tournament is P(A) = 1/4 and Bimal
= 1 – 21/55 = 34/55 will win is P(B) = 1/5. Now since
3. (c) Since A and B are mutually exclusive both of them can not win together so
so P(A ∩ B) = 0 P(A ∩ B) = 0 so P(A∪B) = P(A) + P(B)
Now P(A ∪ B) = P(A) + P(B) – P(A ∩ B) = 1/4 + 1/5 = (5+4)/20 = 9/20
1 2 5 + 16 21 9. (a) Let P(A) = probability that Ravi will
= + = =
8 5 40 40 catch a thief 1/4
4. (b) Since P(A′ ∪ B′) = 1 – P(A ∪ B) P(B) = The probability that Ravi will
21 19 catch a robber = 1/5
=1   – =
40 40 Then from the given information
5. (b) We know that P(A ∩ B) = 1/15
P(A ∪ B) = P(A) + P(B) – P(A ∩ B) Then the probability that Ravi will
catch at least 1 mischief = P(A ∪ B)
For two independent events = P(A) + P(B) – P(A ∩ B)
P(A ∩ B) = P(A) × P(B)
1 1 1 15 +1  2 −4 23
So P(A ∪ B) = P(A) + P(B) – P(A) × P(B) = + – = =
4 5 15 60 60
Or 0.6 = 0.4 + P(B) – (0.4) × P(B) or
10. (c) In this case we have two cases-
0.2 = 0.6 P(B) or P(B) = 2/6 = 1/3.
Case (i) if both of them are green
6. (c) Since P(A ∪ B) = P(A) + P(B) – P(A ∩ B)
= 0.6 + 0.4 – (0.6) × (0.4) 65
then probability is
11  10
= 1 – 0.24 = 0.76.
276  Theory of Counting
Case (ii) if both of them are black 15. (c) Here we have two cases:
54 Case (i) Red ball from 1st bag
then probability is
11  10 and black ball from 2nd bag then
So required probability is probability of this event
6 5 54 20 2 = (5 × 5)/99 = 25/99.
  
11 10 11 10 110 11 Case (ii) Black ball from 1st bag
Alternately: and Red ball from 2nd bag then
Required probability is also given by probability of this event
= (4 × 6)/99 = 24/99.
{( 6 C2 )   (5 C2 )} 6 5 54
  So required probability is 25/99 +
11 11 10 11 10
C2 24/99 = 49/99.
20 2 Alternately:
 
110 11 From the solution of question
11. (d) Having different colour is complement number 10 and 11 probability that
of having same colour. both of them are of same color
Hence required probability is = 20/99 + 30/99 = 50/99
  1 – 2/11 = 9/11 So required probability is
12. (c) Since A and B are independent   1 – 50/99 = 49/99.
events so P(A ∩ B) = P(A) × P(B) =
1/3 × 1/4 = 1/12. 16. (b) Here 1st of all we have to understand
that these two events (drawing
13. (a) Here we have two events and
two balls one after the other) is
probability of getting red from 1st
independent events.
bag and that from the 2nd bag is
independent from each other. Let P(A) = Probability of getting red
Probability of getting red from 1st ball in 1st draw = 4/15
bag = P(A) = 5/11 Let P(B) = Probability of getting red
Probability of getting red from 2nd ball in 2nd draw = 4/15
bag = P(B) = 4/9 Since these two are independent
The probability that both the balls events hence P(A ∩ B) = P(A) × P(B)
are of red color = P(A ∩ B) = P(A) × = 4/15 × 4/15 = 16/225.
P(B) = 5/11 × 4/9 = 20/99. 17. (a) Here 1st of all we have to understand
14. (d) Here we have two events and that these two events (drawing
probability of getting black from two balls one after the other) is
1st bag and that from the 2nd bag is independent events.
independent from each other. Let P(A) = Probability of getting red
Probability of getting black from 1st ball in 1st draw = 4/15
bag = P(A) = 6/11 Let P(B) = Probability of getting red
Probability of getting black from ball in 2nd draw = 3/14
2ndbag = P(B) = 5/9
Since these two are independent
The probability that both the balls events hence P(A ∩ B) = P(A) × P(B)
are of red color = P(A ∩ B) = P(A) ×
= 4/15 × 3/14 = 4/70 = 2/35.
P(B) = 6/11 × 5/9 = 30/99.
Dependent and Independent Events  277
18. (c) Here we have two cases: Required probability is
  30/121 + 30/121 = 60/121
Case (i) black ball is from 1st bag and
19. (a) Since probability that A can solve
red ball from 2nd bag then probability
that question is P(A) = 3/4 hence
is P(A) = 5/11 × 6/11 = 30/121. P(A') = 1/4 and similarly P(B’) = 1/5
Case (ii) red ball is from 1st bag So probability that none of them can
and black ball from 2nd bag then solve the question is 1/4 × 1/5 = 1/20.
probability is 20. (a) This case is complement of the
previous case, so required probability
  P(B) = 6/11 × 5/11 = 30/121   = 1 – 1/20 = 19/20.

Concept Builder (CB)


1. (c) 2. (a) 3. (c) 4. (b) 5. (a) 6. (d)
7. (d) 8. (b) 9. (d) 10. (b) 11. (a) 12. (d)
13. (b) 14. (c) 15. (a) 16. (a)

1. (c) Since P(A′) = 9/11 then P(A) = 2/11 Since


similarly P(B) = 2/5 21 19
From the formula P(A′ ∪ B′) = 1 – P(A ∪ B) =1
   – =
40 40
P(A ∪ B) = P(A) + P(B) – P(A ∩ B)
5.
(a) Let probability that Rajesh will pass
2 2 1 in paper I is = P(A) = 4/5
  –
11 5 5 And probability that he will pass in
10   22 –1
  1 21 paper II is = P(B) = 2/3
 
55 55 And from the given information
2. (a) Since P(A') = 9/11 then P(A) = 2/11 P(A∩B) = 1/2
similarly P(B) = 2/5 We have to find the value of
Since we know that P(A ∪ B) = P(A) + P(B) – P(A∩B)
P(A′ ∩ B′) = P(A ∪ B)′ 4 1 1
  –  
= 1 – P(A ∪ B) 5 3 2
= 1 – 21/55 = 34/55 24 10 –1
  5 19
 
3. (c) P(A') = 7/8 then P(A) = 1/8 similarly 30 30

P(B) = 2/5
6. (d) Rajesh will fail in at least one will be
Since A and B are mutually exclusive represented by
so P(A ∩ B) = 0
1 1
Now = 1 – P(A ∩ B) = 1
   –=
2 2
P(A ∪ B) = P(A) + P(B) – P(A ∩ B) 7. (d) Rajesh will fail in both will be
1 2 5 + 16 21 represented by P(A′ ∪ B′) then
= + = =
8 5 40 40 19 11
4. (b) P(A′) = 7/8 then P(A) = 1/8 similarly P(A′ ∪ B′) = 1 – P(A ∪ B) =1
   – =
P(B) = 2/5 30 30
278  Theory of Counting
8. (b) From the given information So required probability is
probability that Amar will win the 20 30 50
tournament is P(A) = 1 – 3/4 = 1/4 + =
99 99 99
and Bimal will win is P(B) = 1 – 4/5 = 1/5.
13. (b) Given that
Now since both of them can not win
together so P(A∩B) = 0. P(A′ ∩ B′) = 3/8 
So P(A ∪ B) = P(A) + P(B) = 1/4 + 1/5 = or P(A ∪ B) = 1 – 3/8 = 5/8
(5 + 4)/20 = 9/20. So P(A ∪ B) = P(A) + P(B) – P( A ∩ B)
9. (d) From the given information or  5/8 = P(A ) + P(B) – 1/8 or P(A) +
probability that Amar will win the P(B) = 5/8 + 1/8 = 6/8 = 3/4
tournament is P(A) = 1/5 and Bimal So now we have two equations–
will win is P(B) = 1/6. Now since P(A) × P(B) = 1/8
both of them can not win together so
and P(A) + P(B) = 3/4
P(A ∩ B) = 0.
So P(A∪B) = P(A) + P(B) = 1/5 + 1/6 on solving these two equations we
= 11/30 will get P(A) = 1/2 and P(B) = 1/4
hence P(A) – P(B)= 1/4
10. (b) Probability of getting an even
number in 1st throw = P(A) = 3/6 14. (c) In this question we have three
= 1/2 , and probability of getting a cases—
prime number in 2nd throw is Case (i) 1 green ball is from 1st bag
  P(B) = 3/6 = 1/2. then probability is
Since P(A) and P(B) are independent  3  1  3 9
=     =
hence  4   2   4  32
P(A ∩ B) = P(A) × P(B) Case (ii) 1 green ball is from 2nd bag
= (1/2) × (1/2) = 1/4 then probability is
11. (a) Probability of getting at least 3 in  1   1  3 3
=        =
1st throw = P(A) = 4/6 = 2/3 , and  4   2   4  32

probability of getting at most 3 in
Case (iii) 1 green ball is from 3rd bag
2nd throw is P(B) = 3/6 = 1/2.
then probability is
Since P(A) and P(B) are independent
hence  1  1  1 1
=        =
P(A ∩ B) = P(A) × P(B)  4   2   4  32

= (2/3) × (1/2) = 1/3 So required probability is
12. (d) We have two cases here- 9 3 1 13
+ + =
Case (i) if both the balls are of red 32 32 32 32
colored then probability is 15. (a) Let P(A) = x then the equation we
4 5 20 will get is
 
9 11 99 13
=  x  +  3x  – 3x 2 =  4 x  – 3x 2
Case (ii) if both the balls are of green 16
colored then probability is
or 13 =  64 x  – 48 x 2
5 6 30 or 48 x 2 – 64 x  +1
  3 =  0
 
9 11 99 On solving this quadratic equation
we will get
Dependent and Independent Events  279
Now we have two cases:
64 ±   642 – 4 × 48 × 13 64 ±  40
x  = = Case (i) A cracked the exam and B
2 × 48 96 failed to crack then probability is
Or x = 24/96 = 1/4 P(A) × P(B') = 2/5 × 1/4 = 1/10
Case (ii) B cracked the exam and A
(ignoring the value more than 1)
failed to crack then probability is
16. (a) From the given information P(A′) × P(B) = 3/5 × 3/4 = 9/20
P(A) = 2/5 and P(A′) = 3/5 Required probability is
and P(B) = 3/4, P(B′) = 1/4. 1/10 + 9/20 = 11/20

Concept Cracker (CC)


1. (c) 2. (b) 3. (a) 4. (b) 5. (c) 6. (a)
7. (d) 8. (c) 9. (c) 10. (a) 11. (d) 12. (c)
13. (a) 14. (b) 15. (c) 16. (a) 17. (a) 18. (b)
19. (d) 20. (c)

1. (c) From the formula 3. (a) Since A, B, C are mutually exclusive


and collectively exhaustive events
P(A ∪ B) = P(A) + P(B) – P(A ∩ B)
hence P(A)+P(B)+P(C) = 1.
 1  2 – P(A ∩ B) Hence hence the value of P(A) = 1/6
4 5
5   8 4. (b) Since A, B, C are mutually exclusive
   P(A ∩ B) and collectively exhaustive events
20
hence P(A) + P(B) + P(C) = 1
13
   P(A ∩ B) Let P(A′) = x then P(B′) = 2x and
20
P(C′) = 3x hence P(A) = 1 – x,
So maximum value of P(A ∪ B) = 13/20
P(B) = 1 – 2x and P(C) = 1 – 3x.
And minimum value of P(A ∪ B) is
max{ P(A), P(B)} = 2/5 Hence P(A) + P(B) + P(C)
Hence required range is (2/5, 13/20). = (1 – x) + (1 – 2x) + (1 – 3x)
2. (b) Since we know that = 3 – 6x = 1
or x = 2/6 or 1/3
P(A′ ∩ B′) = P(A ∪ B)′ = 1 – P(A ∪ B)
So P(A') = 1/3
So minimum value of 5. (c) Since A, B, C are mutually exclusive
P(A′ ∩ B′) = 1 – 13/20 = 7/20 and collectively exhaustive events
hence P(A) + P(B) + P(C) = 1
And maximum value of
Let P(A′) = x then P(B) = 2x and
P(A′ ∩ B′) = 1 – 2/5 = 3/5
P(C′) = 3x hence P(A) = 1 – x, P(B) = 2x
So required range is (7/20, 3/5). and P(C) = 1 – 3x.
280  Theory of Counting
Hence P(A) + P(B) + P(C) = (1 – x) + Hence P(B) = 0.5 + 0.1 – 0.5 = 0.1.
(2x) + (1 – 3x) = 2 – 2x = 1 or x = 1/2 So required range is (0.1, 0.8).
So P(A) = 1/2 9. (c) There are two cases when they will
6. (a) Since A1, A2, ... An are mutually contradict:
exclusive and collectively exhaustive Case (i) Gargy speak truth and
events associated with a random Ashmeet speak lie then probability
experiment so is 4/5 × 1/4 = 1/5
P(A1) + P(A2) + P(A3) + ... P(An) = 1 Case (ii) Gargy speak lie and Ashmeet
Or [1 – P(A1′) ] + [1 – P(A2′) ] speak truth then probability is 1/5 ×
+ [1 – P(A3′) ] + ... [1 – P(An′)] = 1 3/4 = 3/20.
Or P(A1′) + P(A2′) + P(A3′) + ... P(An′) Hence required probability is 1/5 +
3/20 = 7/20.
=n–1
10. (a) Let probability that A, B and C will
7. (d) From the given information
pass the exam is P(A), P(B) and P(C)
probability that Amar will win the
respectively then from the given
tournament is P(A) = 1/5 and Bimal
information P(A) = 1/2 , P(B) = 1/3 and
will win is P(B) = 1/6 and same for
P(C) = 1/4 and also P(A′) = 1 – 1/2 =
Chetan is P(C) = 2/5.
1/2 , P(B′) = 1 – 1/3 = 2/3 and P(C′) =
Since these events are mutually 1 – 1/4 = 3/4.
exclusive.
Now we have three cases:
Hence P(A) + P(B) + P(C) = 1/5 + 1/6 +
Case (i) A and B will pass the
2/5= 23/30.
exam and C fails that exam then
8. (c) Let P(A) = probability that Ashmit probability is P(A) × P(B) × P(C′) =
will pass, and P(B) = probability 1/2 × 1/3 × 3/4 = 1/8.
that Bishmit will pass, then from
Case (ii) A and C will pass the
the given information
exam and B fails that exam then
P(A) = 0.5, 0.1 ≤ P(A ∩ B) ≤ 0.3. probability is P(A) × P(B′) × P(C) =
We have to find the value of P(B). 1/2 × 2/3 × 1/4 = 1/12.
Since Case (iii) B and C will pass the
P(A ∪ B) = P(A) + P(B) – P(A ∩ B) exam and A fails that exam then
probability is P(A′) × P(B) × P(C) =
Or P(B) = P(A ∪ B) + P(A ∩ B) – P(A)
1/2 × 1/3 × 3/4 = 1/8.
For maximum value of P(B) both
Total probability is
P(A ∪ B) and P(A ∩ B) should be
maximum, maximum value of 1/8 +1/12 +1/8 = 8/24 = 1/3
P(A ∪ B) is 1 and maximum value of 11. (d) Probability that Aman will hit the
P(A ∩ B) = is 0.3. target is P(A) = 4/5 and probability
Hence P(B) = 1 + 0.3 – 0.5 = 0.8. that Aman will not hit the target is
Similarly For minimum value of P(B) P(A′) = 1 – 4/5 = 1/5.
both P(A ∪ B) and P(A ∩ B) should Probability that Binay will hit the
be minimum, minimum value of target is P(A) = 3/4 and probability
P(A∪B) = P(A) = 0.5 and minimum that Binay will not hit the target is
value of P(A ∩ B) = is 0.1. P(A′) = 1 – 3/4 = 1/4.
Dependent and Independent Events  281
Probability that none of them will And if Raj starts the game then
hit the target is probability that Sanchita wins the
P(A′ ∩ B′) = 1/5 × 1/4 = 1/20 game is 1 – 6/11 = 5/11.
So probability that at least one of 15. (c) When two dice is thrown then sample
them will hit the target is space has 6 × 6 = 36 elements so
n(S) = 36
1 – 1/20 = 19/20
Now consider the event of getting 9
12. (c) Let P(A) probability of getting a prime is (3, 6), (4, 5), (5, 4) and (6, 3).
number = 3/6 = 1/2 then P(A′) = 1/2.
So probability of getting 9 when two
And P(B) = probability of getting dice is thrown is 4/36 = 1/9.
a composite number 2/6 = 1/3 then
If Sanchita starts the game then the
P(B′) = 2/3.
probability that she wins is
Now consider Kushal wins the game 1   8   8  1 
it is possible in following cases-            ...    
9  9  9  9 
Case (i) in one throw—if Kushal
1
gets a prime = P(A) = 1/2.
1 81 9
Case (ii) in three throws—If  9     
64 9 17 17
Kushal fails to get a prime in 1st 1
81
throw, Karina fails to get composite
in 2nd throw and Kushal gets a prime And if Raj starts the game then
in 3rd throw, in this case probability probability that Sanchita wins the
is P(A′) × P(B′) × P(A) = 1/2 × 2/3 × 1/2 game is 1 – 9/17 = 8/17.
= 1/6. 16. (a) There are two cases when they will
Case (iii) in 5 throw then similar contradict each other:
to above case probability is Case (i) Gargy is speaking truth and
Ashmeet is lying then probability is
(1/2) × (2/3) × (1/2) × (2/3) × (1/2) = 1/18
(3/4)(1/5) = 3/20.
And this process will continue and
Case (ii) Gargy is lying and Ashmeet
the required probability is
is speaking truth then probability is
(1/2) + (1/6) + (1/18) + ... ∞ (1/4)(4/5) = 1/5.
1 Required probability = 3/20 + 1/5 = 7/20.
Or required probability = 2 3
= 17. (a) There are two cases when they will
1 4 agree to each other:
1 −
3
Case (i) When both of them are
13. (a) Probability that Karina will win the
speaking truth then probability is
game is 1 – 3/4 = 1/4.
(6/10)(7/10) = 42/100.
14. (b) Probability of getting a six is 1/6.
Case (ii) When both of them are
If Sanchita starts the game then the
lying then probability is
probability that she wins is
(4/10)(3/10) = 12/100.
1
1   5   5  1  6 Required probability
           ...     6 
6  6  6  6  25 11 = 42/100 + 12/100
1
56 = 54/100 = 54%
282  Theory of Counting
18. (b) We have three cases in this question: 19. (d) Given that P(A) = 8/10 and P(A′) = 2/10
Case (i) Only A has solved the similarly P(B) = 7/10 and P(B′) = 3/10
question and B and C failed to solve Probability that none of them can
that question then probability of this solve a question is P(A′) × P(B′) =
 1   5  5 25 (2/10)(3/10) = 6/100.
case is       =
 3   7   8  168 So required probability is
Case (ii) Only B has solved the 1 – 6/00 = 94/100 or 94%
question and A and C failed to solve
that question then probability of this 20. (c) Let A be the event that 1st man will
 2  2  5 20 be alive till 75 years and B be the
case is       = event that 2nd man will be alive
 3   7   8  168
till 70 years then P(A) = 5/16 and
Case (iii) Only C has solved the P(B) = 3/7 then P(A′) = 11/16, and
question and A and B failed to solve
P(B′) = 4/7
that question then probability of this
 2  5  3 30 Probability that none of them will be
case is       = alive 35 years hence is
 3   7   8  168
So required Probability is P(A′) × P(B′) = 11/16 × 4/7 = 11/28

25 20 30 75 25 Then required probability is


+ + = = 1 – 11/28 = 17/28
168 168 168 168 56
Part B Dependent and Independent Events  283

Concept 1 Introduction

Properties of Independent events • A′ and B are independent events


Consider two independent events A and B: • For two independent events
• A and B′ are independent events P(A ∩ B) = P(A) × P(B)
• A′ and B′ are independent events • P (A ∪ B) = 1 – P(A′) P(B′)

Concept 2 Addition Theorem of Probability

If A and B be any two events in a sample From the set theory we know that:
space S of an experiment, then the probability n( A ∪ B ) = n(A) + n (B) – n ( A ∩B )
of occurrence of at least one of the events A Dividing both sides by n (S), we get
and B is given by n(A ∪ B) n(A) n(B) n(A ∩  B)
= + –
P ( A ∪ B ) = P (A) + P (B) – P (A ∩ B) n(S) n(S) n(S) n(S)

Representation Of Union Of Sets or P(A ∪ B) = P(A) + P(B) – P (A ∩ B)


Addition theorem for three events
A B Consider three events A, B and C in a sample
space S then from the addition theorem
P( A ∪ B ∪ C) = P(A) + P(B) + P(C)
– P(A ∩ B) – P(B ∩ C)
– P(A ∩ C) + P(A ∩ B ∩ C)
Proof: From set theory, we know that
n(S) = n(A) + n(B) + n(C) – n(A ∩ B)
– n( B ∩ C) – n(A ∩ C) + n ( A ∩ B ∩ C)
284  Theory of Counting
Dividing both sides by n(S), we get \ P(A ∪ B ∪ C) = P(A) + P(B) + P(C)
n( A  B  C ) n( A) n( B) n(C ) – P( A ∩ B) – P(B ∩ C)
  
n( S ) n( S ) n( S ) n( S ) – P(A ∩ C) + P(A ∩ B ∩ C).
n( A  B ) n( B  C ) If A, B, C are mutually exclusive events, then
– – A ∩ B = φ,
n( S ) n( S )
B ∩ C = φ,
n( A  C ) n( A  B  C )
–  A ∩ C = φ, A ∩ B ∩ C = φ
n( S ) n( S ) \ P(A ∪ B ∪ C) = P(A) + P(B) + P(C)

1
Concept Deviator (CD)
Ideal Time Revise your concept (IIT advance or
5 Min. tougher questions)
Per Question

1. Let E and F be two independent events. 4. The probability that X ≥ 6 given X > 3
The probability that exactly one of them equals [IIT JEE 2009]
occurs is 11/25 and the probability of
(a) 125/216 (b) 25/36
none of them occurring is 2/25. If P(T)
denotes the probability of occurrence of (c) 5/36 (d) 25/216
the event T, then [IIT JEE 2011]
5. An experiment has 10 equally likely
(a) P(E) = 4/5, P(F) = 3/5 outcomes. Let A and B be two non empty
(b) P(E) = 1/5, P(F) = 2/5 events of the experiment. If A consists
(c) P(E) = 2/5, P(F) = 1/5 of 4 outcomes, the number of outcomes
that B must have so that A and B are
(d) P(E) = 3/5, P(F) = 4/5
independent, is [IIT JEE 2008]
Direction for question number 2 to 4:
(a) 2, 4 or 8 (b) 3, 6 or 9
A fair die is tossed repeatedly until a six is
obtained. Let X denote the number of tosses (c) 4 or 8 (d) 5 or 10
required.
6. The probability that event A happens in
2. The probability that X = 3 equals one trial of an experiment is 0.4. Three
[IIT JEE 2009] independent trials of the experiment
(a) 25/216 (b) 25/36 are performed . The probability that the
(c) 5/36 (d) 125/216 event A happens at least once is-
3. The probability that X ≥ 3 equals
[IIT JEE 1980]
[IIT JEE 2009]
(a) 125/216 (b) 25/36 (a) 0.936 (b) 0.784

(c) 5/36 (d) 25/216 (c) 0.904 (d) none of these


Dependent and Independent Events  285
7. In a certain city only two news papers 11. A box contains 2 fifty paise coins, 5 twenty
A and B are published. It is known that five paise coins and certain fixed number
25% of the city population reads A and n (≥ 2) of ten and five paise coins. Five
20% reads B while 8% reads both A and coins are taken out of the box at random.
B. It is known that 30% of those who read Find the probability that the total value
A but not B look into the advertisements of these 5 coins is less than one rupee
and 40% of those who read B but not A and fifty paise. [IIT JEE 1988]
look into the advertisements while 50% (a) 1 – 10(n + 2)/n + 7C5
of those who read both A and B look (b) 10(n + 2)/n + 7C5
into the advertisements. What is the
(c) 1 – (n + 2)/n + 7C5
percentage of population who reads an
advertisement?      [IIT JEE 1984] (d) None of these
(a) 19.5% (b) 19% 12. If E and F are independent events such
that 0 < P (E) < 1 and 0 <P(F) < 1, then
(c) 21% (d) None of these
[IIT JEE 1989]
8. A and B are two independent events. The
probability that Both A and B occur is (a) E and F are mutually exclusive
1/6 and probability that neither of them (b) E and F′ (the complement of the
occurs is 1/3. Find the probability of event F) are independent
occurrence of A. [IIT JEE 1984] (c) E′ and F′ are independent
(a) 1/4 or 1/3 (b) 1/2 or 1/3 (d) P(E| F) + P (E| F′) = 1
(c) 1/5 or 1/4 (d) None of these 13. For any two events A and B in a sample
9. A students appear for tests, I, II and III. space: [IIT JEE 1991]
The student is successful if he passes (a)  A   { P ( A )   P ( B ) –1
 },
either in tests I and II or tests I and III. B  P(B)
 
The probabilities of the student passing
in tests I, II and III are p, q, are 1/2 , P(B) ≠0, is always true
respectively. If the probability that the (b) P( A ∩ B′) = P(A) – P(A ∩ B) does not
student is successful is 1/2, then hold
[IIT JEE 1986] (c) P(A ∪ B) = 1 – P(A′) P(B′), if A and B
(a) p = q = 1 (b) p = q = 1/2 are independent
(c) p = 1, q = 0 (d) p = 1, q = 1/2 (d) P(A ∪ B) = 1 – P(A′) P(B′) , if A and B
are disjoint
10. Urn A contains 6 red and 4 black balls and
urn B contains 4 red and 6 black balls. One 14. India plays two matches each with
ball is drawn at random from urn A and the West Indies and Australia. In any
placed in urn B. Then one ball is drawn at match, the probabilities of India getting
random from urn B and placed in urn A. If 0, 1 and 2 points are 0.45, 0.05 and 0.50
one ball is now drawn at random from urn respectively. Assuming that the outcomes
A, the probability that it is found to be red are independent, the probability of India
getting at least 7 points is
is [IIT JEE 1988]
[IIT JEE 1991]
(a) 327/605 (b) 314/605
(a) 0.8750 (b) 0.0875
(c) 328/605 (d) None of these
(c) 0.0625 (d) 0.0250
286  Theory of Counting
15. Let E and F be two independent events. (a) P(E) = 1/3, P(F) = 1/4
The probability that both E and F happen (b) P(E) = 1/2 , P(F) = 1/6
is 1/2 and the probability that neither E
(c) P(E) = 1/6, P(F) = 1/2
nor F happens is 1/12. Then
(d) P(E) = 1/4 , P(F) = 1/3.
[IIT JEE 1993]

Response Grid
1. a b c d 2. a b c d 3. a b c d 4. a b c d 5. a b c d
6. a b c d 7. a b c d 8. a b c d 9. a b c d 10. a b c d
11. a b c d 12. a b c d 13. a b c d 14. a b c d 15. a b c d

2
Concept Eliminator (CE)
Ideal Time Revise your concept (Maths Olympiad
Just solve it or tougher questions)

1. Two dice are thrown simultaneously what (a) 51/100


is the probability that sum of the two 99
numbers will be 5 before 7? 51   49 
(b) 
(a) 2/5 (b) 1/5  100   100 

(c) 3/5 (d) None of these


(c) 11/1010
2. There are 100 players participating in a
tournament. At the beginning player Pi (d) None of these
has i number of coins. At each stage, two
3. The probability of India winning a
players are chosen to play a game, with
the winner of the game receiving 1 coin test match against West Indies is 1/2.
from the looser. At any stage if fortune Assuming independence from west
of a player drops to 0 coins then he is Indies is 1/2. Assuming independence
eliminated from the tournament. This from match to match the probability that
process continues till only one player in a 5 match series India’s second win
having all the coins and is designated
occurs at the third test is
as winner of the tournament. What is
the probability that 51st player (having (a) 1/8 (b) 1/4
51 coins) will be the winner of the
(c) 1/2 (d) 2/3
tournament?
Dependent and Independent Events  287
4. 3 friends A, B and C play the game 5. 3 friends A, B and C play the game
“Pahle Hum Pahle Tum” in which they “Pahle Hum Pahle Tum” in which they
throw a die one after the other and the throw a die one after the other and the
one who will get a composite number one who will get a composite number
1st will be announced as winner, If A 1st will be announced as winner, then
started the game followed by B and then remaining two will continue to play for
C then what is the ratio of their winning the 2nd position. If A started the game
probabilities? followed by B and then C then what is the
probability that A will get 2nd position?
(a) 9 : 6 : 4 (b) 8 : 6 : 5
(a) 24/95 (b) 12/95
(c) 10 : 5 : 4 (d) None of these
(c) 18/95 (d) None of these

Response Grid
1. a b c d 2. a b c d 3. a b c d 4. a b c d 5. a b c d

Solutions
Concept Deviator (CD)
1. (a,d) 2. (a) 3. (b) 4. (b) 5. (d) 6. (b)
7. (a) 8. (b) 9. (c) 10. (c) 11. (a) 12. (b, c, d)
13. (a, c) 14. (b) 15. (a, d)

1. (a, d) Since the probability that exactly one (1 – P (E)) (1 – P(F)) = 2 /25
of them occurs is 11/25 so From the given options option (a)
P (EF′) + P(FE′) = 11/25 and (d) satisfy above condition.
And none of them occurring is 2/25 2. (a) Probability of getting 6 is P(6) = 1/6 and
so P (E′F′) = 2/5 probability not getting 6 is P(6′) = 5/6

Since the events are independent so In this case X =3 which is possible


P(E)P(F′) + P(F)P(E′) = 11/25 when

P(E)(1–P(F)) + P(F) (1 – P (E)) = 11/25 P(6′)P(6′)P(6) = (5/6)(5/6)(1/6)


P(E) + P(F) – 2 P (E)P(F) = 11/25 = 25/216
288  Theory of Counting
3. (b) Here X ≥ 3 hence required probability 8. (b) It is given that A and B are two
is independent events i.e P(A) . P(B) =
1/6, and P(A′) . P(B′) = 1/3
 5  2  1   5 3  1   5  2  1 
                ... Or (1 – P(A))(1 – P(B)) = 1/3
6  6  6  6  6  6 
Or  1 – P(A) – P(B) + P(A) . P(B) = 1/3
Y using sum of infinite terms of GP Or  P(A) + P(B) = 5/6 and P(A) . P(B) = 1/6
required probability.
We will get a quadratic equation and
 5  2  1  
  the value of P(A) = 1/2 or 1/3
    
 6   6   25 9. (c) As per the given condition, to be
= 
successful student must pass in test
5 36
1– I and along with that he has to pass
6
4. (b) Here we need to find probability of in at least one of the test II or test III,
X ≥ 6 given that X > 3 1 1 q
probability of this is q+ − 
2  2 2
Required probability is
which is equal to 1/2.
  5     6     7   
 5 1 5 1 5 1 
                . 
 6   6   6   6   6   6   So 1  q + 1 − q  = 1   or p(q + 1) = 1,
2 2 2 2
  3     4     5   
 5 1 5 1 5 1  from the given options option (c)
                . 
 6   6   6   6   6   6   satisfy the condition,
  5  10. (c) It is given that third draw is red, we
 5 
   have four conditions for 1st 4 draws
 6   25
   and these are RB, RR, BR, BB.
  3  36
 5  Hence the result of three draws must
  
 6   be one of the- RBR, RRR, BRR, BBR.

5. (d) If A and B are independent events Required probability is
then P(A ∩ B) = P(A) × P(B) = (4/10)
(p/10) = 2p/5 is an integer so p = 5 or 10  5 5 6  6 6 5
 11 × 11 × 10  +  10 × 11 × 10 
6. (b) Probability that even A didn’t occur
is 1 – 0.4 = 0.6.
 4 4 7  4 7 6
Probability that A didn’t occur a single + × × + × ×
 10 11 10   10 11 10 
time is (0.6 × 0.6 × 0.6) = 0.216
328
So required probability is =
605
1 – 0.216 = 0.784 11. (a) Total number of coins is n + 2 + 5 = n + 7
7. (a) Required probability is given by Sample space is n + 7C5
P(E/A) × P(A) + P(E/B) × P(B) Now we have to calculate the number
of non favorable cases:
+ P(E/A ∩ B) × P(A ∩ B)
Case (i) Two 50 paise, two 5 paise
= (30/100)(25/100) + (20/100)(40/100) and one fron n coins of ten and five paise,
+ (50/100)(8/100) = 195/100 = 19.5% number of ways is (2C2 × 5C2 × nC1).
Dependent and Independent Events  289
Case (ii) One 50 paise, five 25 paise Australia and to get 7 points in these 4
coins, number of ways is (2C1 × 5C5). matches condition must be either 4 win
Case (iii) Two 50 paise, three 25 paise or 3 win.
coins, number of ways is (2C2 × 5C3). So required probability is
(4C3)(0.05)(0.5)3 + (4C4)(0.5)4 = 0.0875
So total number of ways is
15. (a, d) It is given that P(E′ ∩ F′) = 1/2
(2C 2 × 5C
2 × nC
1 ) + (2C
1 × 5C
5 ) + (2C 2 × 5C3) Since E and F be two independent events.
= 10n + 20 = 10(n + 2). Hence E′ and F′ are also two independent
Required probability events hence-
= 1 – 10(n + 2)/(n + 7C5). P(E′ ∩ F′) = P(E′) P(F′)
12. (b, c, d) Since E and F are independent = 1/12 = (1 – P(E))(1 – P(F))
events hence statements b, c and d = 1 – P(E) – P(F) + P(E)P(F)
are correct.
= 1 – P(E) – P(F) + 1/12
13. (a, c) From the theory- option (a) and (c)
or  P(E) + P(F) = 7/12
are correct.
On solving above equations we will
14. (b) Since India plays 4 matches two
get P(E) = 1/3 and P(F) = 1/4 or P(E)
matches each with the West Indies and
= 1/4 and P(F) = 1/3.

Concept Eliminator (CE)


1. (a) 2. (d) 3. (b) 4. (a) 5. (a)

1. (a) Probability of getting 5 is 4/36, = 1/9, probability 1/n of being the winner.
Probability of getting 7 is 6/36 = 1/6 Now, suppose these n players are
divided into r teams, with team i
Probability that neither 5 nor 7 will
containing ni players, i = 1, . . . , r.
appear is 1 – 10/36 = 13/18
Then the probability that the victor
Required probability is is a member of team i is ni/n.
 1  13   1  13  13   1 
                
  . Applying to the similar logic in this
 9  18   9  18  18   9  case n = 1 + 2 + 3 + … + 100 = 100 ×

101/2 = 50 × 101= 5050.
 
  Required probability is
 1  2
  (1 / 9)  
1  13  5   51/5050
 18 
  3. (b) There are only two ways when it is
2. (d) To understand the situation properly possible- LWW and WLW. Here W
and L represents India’s win or loss.
let us assume that there are n
players, with each player initially So required probability is
having 1 coin. In this case all the  1  1  1  1  1  1 1
players have equal probability to be  2   2   2  +   2   2   2  = 4
the winner hence each player has
290  Theory of Counting
4. (a) Probability of getting a composite 2nd position and C is at 3rd position-
number is 2/6 = 1/3 From the solution of previous
question Probability that B will get
Probability that A will win the game 1st position is 6/19, now after B’s
is move C will continue then probability
1   2   2   2  1  that A will win over C is given by:
           
3  3  3  3  3   2  1   2   2   2  1 
             
 2   2   2   2   2   2  1  3  3  3  3  3  3 
                 .
3  3  3  3  3  3  3   2   2   2   2   2  1 
               .
1 3  3  3  3  3  3 

3  1 27  9
       2
8  3 19  19 2 9  2
1  9      
27 4

1 9 5  5
Probability that B will win the game 9
is
So required probability for this case
 2  1   2   2   2   2  1 
                6 2 12
3  3  3  3  3  3  3  is   .
19 5 95
 2   2   2   2   2   2   2  1 
                   . Case (ii) C is at 1st position, A is at
3  3  3  3  3  3  3  3  2nd position and B is at 3rd position-

2 From the solution of previous
9  2 27  6
       question Probability that C will get
8  9 19  19 1st position is 4/19, now after C’s
1
27 move A will continue then probability

Probability that C will win the game that A will win over C is given by:
is
1   2   2  1 
 2   2  1   2   2   2   2   2  1           
                    3  3  3  3 
3  3  3  3  3  3  3  3  3   2   2   2   2  1 
 2   2   2   2   2   2   2   2  1               .
                     ... 3  3  3  3  3 
3  3  3  3  3  3  3  3  3 

4 1
 4 27  4 3 1 9  3
 27             
4 3 5  5
8  27 19  19 1
1
27 9

So required ratio is 9 : 6 : 4. So required probability for this case
5. (a) We have to find the probability that 4 3 12
is  
A will be at 2ndpositions, there are 19 5 95
two different cases for the same- So required probability
Case (i) B is at 1st position, A is at   = 12/95 + 12/95 = 24/95
11
Conditional Probability  291

Conditional Probability

Topics Covered

 Introduction of Conditional Probability

 Properties of Conditional Probability

 Bayes’ Theorem

Part A: Topic Number of Questions


Solved Example 5
Concept Applicator 10
Concept Builder 10
Concept Cracker 10
Part B: Topic Number of Questions
Concept Deviator 10
Concept Eliminator 4
Total 49
292  Theory of Counting
Part A
Introduction of
Concept 1 Conditional Probability

Conditional Probability: Consider two events Set theory approach


A and B such that B ≠ φ, then probability of A B
occurrence of event A when B has already
occurred is called conditional probability and
is denoted by P(A/B).
Lets take some examples to understand this-
(i) Find the probability of getting a number
A∩B
greater than 4 if it is given that the outcome
is an odd number:
Here in this case sample space
S = {1,2,3,4,5,6}
Let A = the event of occurrence of a number
greater than 4 = {5,6} and n(A) = 2. Now consider two events A and B in the sample
B = the event of occurrence of an odd space, we have to find the probability P(A/B)
number = {1,3,5} and n(B) = 3. since it is given that event B has occurred so
new sample space or reduced sample space is B
Now we have to find probability of getting a
which shown as a shaded area in the diagram,
number greater than 4 if it is given that the and now we have to find the occurrence of A
outcome is an odd number i.e P(A/B) i.e (A ∩B), so required probability is-
Here when an odd number has occurred, total P(A/B) = n(A ∩ B)/n(B)
number of cases is only 3 (not 6) so we can say
By dividing n(S) in numerator and denominator
that new Sample space or reduced Sample
we will get
space is {1, 3, 5} and it has only 3 elements
 n( A  B ) 
and favorable number of cases is 1 i.e {4}.  
 n(S ) 
Hence the required probability is 
P(A/B)  n( B )  = P(A ∩ B)/P(B)
P(A/B) = n(A ∩ B)/n(B) = 1/3  
 n(S ) 
So in the above example we have seen that B
works as the sample space and A ∩ B works
Hence P(A/B) = P(A ∩ B)/P(B)
as the event.
Conditional Probability  293
Conditional Probability for independent P(A/B) = P(A ∩ B)/P(B)
Events but we know that for two independent events
If A and B are independent events, then P(A ∩ B) = P(B) ⋅ P(A)
probability of occurrence of event A is not Hence P(A/B) = P(A ∩ B)/P(B) = P(A)
affected by occurrence or non-occurrence Hence for two independent events A and B
of event B and vice versa then P(A/B) i.e
P(A/B) = P(A)
probability of occurrence of event A when B
has already occurred P(B/A) = P(B)

Properties of
Concept 2 Conditional Probability
Some Properties of Conditional If A1, A2, ... An are independent events, then
Probability P(A1∩ A2∩ ... ∩ An) = P(A1). P(A2) ... P(An)
(i) P(A/B) = P(A ∩ B)/P(B) (viii) If A and B are two independent events,
or P(A ∩ B) = P(B) ⋅ P(A/B) then P(A ∪ B) = 1 – P(A′) ⋅ P(B′)
(ii) P(B/A) = P(A ∩ B)/P(A) (ix) If A1, A2 ... An are independent events,
or P(A ∩ B) = P(A) ⋅ P(B/A) then P(A1∪ A2∪ ... ∪ An) = 1 – P(A′1) ⋅
(iii) P(A/B) may or may not be equal to P(A′2) ... P(A′n)
P(B/A) (x) If A and B are two events such that
(iv) If A and B are independent events, then B ≠ Ф, then
P(B/A) = P(B) P(A/B) + P(A′/B) = 1
(v) Two events A and B are independent if Proof: P(A/B) + P(A′/B) = P(A ∩ B)/P(B)
and only if P(A ∩ B) or P(AB) = P(A) ⋅ P(B) + P(A′ ∩ B)/P(B)
= [P(A ∩ B) + P(A′ ∩ B)]/P(B)
(vi) If A, B and C are any three independent
events, then = P(B)/P(B)
P(A ∩ B ∩ C) = P[A ∩ (B ∩ C)] = 1 [Q (A ∩ B) ∪ (A′ ∩ B) = B and A ∩ B
= P(A) ⋅ P(B ∩ C) and A′ ∩ B are mutually exclusive ]
(xi) If A and B be two events such that
= P(A) ⋅ P(B) ⋅ P(C) A ≠ Ф, then
(vii) If we generalize the result for n events P(B) = P(A). P(B/A) + P(A′). P(B/A′)
then : If A1,A2, ... An be any n events none Proof: P(A) ⋅ P(B/A) + (A′) ⋅ P(B/A′)
of which is an impossible event, then
= P(A ∩ B) + P(A′ ∩ B)
P(A1 ∩ A2 ∩ ... ∩ An)
= P[(A ∩ B) ∪ (A′ ∩ B)]
= P(A1) ⋅ P(A2/A1) ⋅ P(A3/A1A2) … [Q A ∩ B and A′ ∩ B are mutually exclusive]
P(An/A1A2 ⋅ An – 1) = P(B) [Q (A ∩ B) ∪ (A′ ∩ B) = B]
294  Theory of Counting
Example 1:  If P(A) = 1/3 and P(A ∩ B) = 1/6 Example 3:  If A and B are two mutually
then find the value of P(B/A) exclusive events with P(A) = 1/20
Solution:  We know that P(B/A) and P(B) = 1/11 then find the value of
1 P(A/B).
P A  B 6 1 Solution:  Since A and B are two mutually
  
P A 1 2 exclusive events so P(A ∩ B) = 0 so P(A/B) = 0
3
Example 4:  Two die are thrown together
Example 2:  If P(A/B) = 2/3 and P(A ∩ B) = 1/6 what is the probability that sum of
then find the value of P(B). these two number is a prime number
if it is given that both the results are
Solution:  We know that
prime numbers?
P A  B Solution:  As per the given condition outcome
P(A/B) =
P B is (2,2), (2,3), (2,5), (3,2), (3, 3), (3,5), (5, 2),
1 (5, 3) and (5,5).
P A  B 6 1 So sample space is 3 × 3 = 9 and number of
or P(B) =   favourable case is (2, 3), (2,5), (3, 2) (5,2)
P A / B 2 4
3 So required probability is 4/9.

Concept 3 Bayes’ Theorem


Bayes’ theorem is just extension of conditional event which occurs together (in conjunction)
probability, in order to understand the Bayes’ with either of Ai i. e. if A1, A2, ..., An form a
theorem lets discuss the partition of a set: partition of the sample space S and A be any
event, then from Bayes’ theorem
Partition of a Set A 
P  k 
A family of sets A1, A2, ... An is said to form a  A 
partition of a set A if following conditions are  A 
satisfied: P  
 Ak 
(i) A1, A2, ..., An are non – empty    P ( Ak ).
   
(ii) Ai ∩ Aj = φ for i ≠ j P ( A )  P  A    P ( A )    
 1   2 
(iii) A = A1 ∪ A2 ∪ … ∪ An   A1  
 
 A   A 
P    ...   P ( An )  P  
Bayes’ Theorem   
  A2   An  
Let us consider n mutually exclusive and
exhaustive events A1, A2, A3,... An and A is an Proof: Since A1, A2, ..., An form a partition of S,
Conditional Probability  295
Now, • If A1, A2, ... An form a partition of an
A = A ∩ S = A ∩ ( A1 ∪ A2∪ …∪An) event A, then
= ( A ∩ A1) ∪ (A ∩ A2) ∪…∪(A ∩ An) P ( Ak )
P(Ak/A) 
Since A1, A2, ..., An are disjoint sets hence P ( A1 )  P ( A2 )     P ( An )
(A ∩ A1), (A ∩ A2), ..., (A ∩ An) are also
disjoint. Example 5:  Mr. Susmit a famous liar is
known to speak the truth 3 out of 4
\ From addition theorem
times. His friend Hasan throws a die
P(A) = P(A ∩ A1) + P( A ∩ A2) blindfolded and asked Susmit the
+ … + P(A ∩ An) result who says that it is a six. Find the
probability that it is actually a six.
We know that P(Ak/A ) = P(Ak ∩ A)/P(A)
Solution:  Let A = the event that Susmit
P ( Ak  A )
Or P(Ak/A )  reports occurrence of 6
P ( A1  A )   P ( A2  A )
Or P(Ak/A ) =   Let B = the event of occurrence of 6 when
     P ( An  A )
 
 A  a die is thrown
 
A  And B’ = the event of non- occurrence of
 P ( Ak )   k
     6 when a die is thrown
P ( A )  P  A   P ( A )  P  A  
 1 A  2  A 
  1  2  Now From Bayes’ theorem
 
  A  
 
     P ( An )   P   A
 P  
  An  
B 
P      P ( B )  B 
A 
 A  A  
P ( B )  P     P ( B )P    
Properties of Bayes’  B   B  

Theorem We know that P(B) = 1/6 and P(B′) = 5/6


• If A1, A2, ... An form a partition of S and P(A/B′) = probability that man reports occurrence
A be any event then
of 6 when 6 has actually occurred = probability
P(A) = P(A1) ⋅ P(A/A1) + P(A2) ⋅ P(A/A2) that the man speaks the truth = 3/4
+ ... + P(An) ⋅ P(A/An) P(A/B′) = probability that man reports occurr-
• If P(A1) = P(A2) = … = P(An), ence of 6 when 6 has not actually occurred =
probability that the man tells a lie = 1/4.
P(Ak /A) = P(A/Ak)/P(A/A1) + P(A/A2)
+ … + P(A/An) Putting these values in (1), we get

• Priori and Posteriori Probabilities: 1   3 


    
The probabilities P(A1), P(A2), ..., P(A2), B  6  4  3
P    
..., P(An) which are known before the  A   1   3   5   1   8
        
experiment takes place are called priori           
 6   4   6   4  
probabilities and P(Ai/A) are called
posteriori probabilities.
296  Theory of Counting

1
Concept Applicator (CA)
Ideal Time Apply your concepts with easy and
10 Min. conceptual questions

1. If P(A) = 2/3 and P(A ∩ B) = 1/3 then find 7. Two coins are tossed, what is the
the value of P(B/A). probability of getting one Head and one
(a) 1/2 (b) 1/3 Tail if it is given that at least one Tail
(c) 1/4 (d) None of these comes up?
2. If P(A/B) = 2/3 and P(A ∩ B) = 1/3 then (a) 1/2 (b) 1/3
find the value of P(B). (c) 2/3 (d) None of these
(a) 1/2 (b) 1/3 8. Two dice are thrown, then what is the
(c) 1/4 (d) None of these probability of getting the sum of two
3. If A and B are two mutually exclusive results more than 10 if it is known that
events with P(A) = 1/5 and P(B) = 1/6 number 5 will always come on the 1st
then find the value of P(A/B). die?
(a) 1/2 (b) 1/3 (a) 1/2 (b) 1/3
(c) 1/4 (d) None of these (c) 1/6 (d) None of these
4. If P(A) = 2/5, P(B) = (3/5) and P(A ∪ B) = 9. Two dice is thrown, then what is the
4/5 then find the value of P(A/B). probability of getting 3 on at least one
(a) 1/2 (b) 1/3 die if it is given that the summation of
(c) 1/4 (d) None of these the result is 8?
5. Find the value of P(A/B) + P(A′/B) (a) 1/8 (b) 2/5
(a) 0 (b) Less than 1 (c) 1/5 (d) None of these
(c) 1 (d) None of these 10. Two dice are thrown, then what is the
6. If a die is thrown what is the probability probability of getting the sum of two
of coming up a number more than 4 if results as perfect squares if it is known
it is known that the result is an odd that number 3 will always come on the
number? 1st die?
(a) 1/2 (b) 1/3 (a) 1/2 (b) 1/3
(c) 1/4 (d) None of these (c) 1/6 (d) None of these

Response Grid
1. a b c d 2. a b c d 3. a b c d 4. a b c d 5. a b c d
6. a b c d 7. a b c d 8. a b c d 9. a b c d 10. a b c d
Conditional Probability  297

2
Concept Builder (CB)
Ideal Time Revise your concepts with questions
15 Min. medium difficulty level questions

1. If P(B) = 3/8 and P(A ∪ B) = 7/8 then find (a) 8/13 (b) 5/13
the value of P(A’/B’) (c) 7/13 (d) None of these
(a) 1/2 (b) 1/3 7. A bag contains 25 tickets, numbered
(c) 1/4 (d) None of these from 1 to 25, two tickets are drawn from
the bag without replacement, what is the
2. If P(A) = 0.4, P(A ∩ B) = 0.2 = P( A′ ∩ B′)
probability that the 2nd ticket is a perfect
then find the value of P(B/A).
square if it is know that 1st ticket was a
(a) 0.55 (b) 0.33 perfect square number?
(c) 0.67 (d) None of these (a) 1/8 (b) 1/6
3. If P(A/B) = 0.4, P(B/A) = 0.5, and P(B) = 0.6 (c) 1/4 (d) None of these
then find the value of P(A) 8. A bag contains 25 tickets, numbered
(a) 0.67 (b) 0.33 from 1 to 25, one tickets are drawn from
(c) 0.75 (d) None of these the bag what is the probability that the
number is a perfect cube if it is given
4. A bag contains 8 red and 6 black balls,
that number is not a perfect square?
two balls are taken out from the bag one
after the other without replacement then (a) 1/20 (b) 1/16
what is the probability that 2nd ball is (c) 1/21 (d) None of these
red if 1st ball was black? 9. Two dice are thrown simultaneously
(a) 8/13 (b) 5/13 what is the probability that two digit
number formed by two results is a prime
(c) 7/13 (d) None of these
number if it is given that both the results
5. Three students are selected from a group are prime number?
of 10 students. What is probability that
(a) 1/3 (b) 1/9
Hasan is selected if it is given that Adnan
is always one of the selected student? (c) 2/9 (d) None of these
(a) 7/36 (b) 11/36 10. Two dice are thrown simultaneously what
is the probability that two digit number
(c) 13/36 (d) None of these formed by two results is a perfect square
6. A bag contains 8 red and 6 black balls, number if it is given that both the results
two balls are taken out from the bag one are prime number?
after the other without replacement then (a) 1/3 (b) 1/9
what is the probability that both of them (c) 2/9 (d) None of these
are black?
Response Grid
1. a b c d 2. a b c d 3. a b c d 4. a b c d 5. a b c d
6. a b c d 7. a b c d 8. a b c d 9. a b c d 10. a b c d
298  Theory of Counting

3
Concept Cracker (CC)
Ideal Time Boost up your confidence with good
20 Min. questions

1. In Praxis Business School Kolkata, 50% 6. Abhay speaks the truth only 60% . Hasan
students like chocolate, 30% students rolls a die blindfolded and asks Abhay to
like cake and 10% like both. If a student tell him if the outcome is a ‘prime’. Abhay
is selected at random then what is the says, “YES”. What is the probability that
probability that he likes chocolates if it the outcome is really ‘prime’?
is known that he likes cake? (a) 0.5 (b) 0.75
(a) 1/3 (b) 2/5 (c) 0.6 (d) None of these
(c) 3/5 (d) None of these
7. Abhay speaks the truth only 60% . Hasan
2. In a class 30% students like tea, 20% like
rolls a die blindfolded and asks Abhay to
coffee and 10% like both tea and coffee. A
tell him if the outcome is a ‘prime’. Abhay
student is selected at random then what
says, “NO”. What is the probability that
is the probability that he likes tea if it is
the outcome is really ‘prime’?
known that he likes coffee?
(a) 1/3 (b) 1/4 (a) 0.5 (b) 0.75
(c) 1/2 (d) None of these (c) 0.6 (d) None of these
3. In a class 30% students like tea, 20% like 8. Consider and event E = E1 ∩ E2 ∩ E3 find
coffee and 10% like both tea and coffee. A the value of P(E) if P(E1) = 2/5, P(E2/E1)
student is selected at random then what =1/5 and P(E3/E1E2) = 1/10
is the probability that he does not like
tea if it is known that he likes coffee? (a) 2/125 (b) 1/125
(a) 1/2 (b) 3/4 (c) 3/125 (d) None of these
(c) 1/3 (d) None of these Direction for question number 9 and 10:
4. In a class 30% students like tea, 20% like In a college 40% students play football, 50%
coffee and 10% like both tea and coffee. A play cricket and 60% play chess, If P(A/B)
student is selected at random then what denotes the probability that a random selected
is the probability that he does not like tea student is playing football if it is given that he
if it is known that he does not like coffee? plays cricket or chess.
(a) 2/3 (b) 3/4 9. What is the maximum value of P(A/B).
(c) 1/2 (d) None of these (a) 1/3 (b) 1/2
5. What is the probability the quadratic (c) 2/3 (d) None of these
equation x2 + 2bx + (5b – 4) = 0 has
imaginary roots if – 5 < b < 5. 10. What is the minimum value of P(A/B)
(a) 2/11 (b) 3/11 (a) 1/3 (b) 1/2
(c) 1/11 (d) None of these (c) 2/3 (d) None of these

Response Grid
1. a b c d 2. a b c d 3. a b c d 4. a b c d 5. a b c d
6. a b c d 7. a b c d 8. a b c d 9. a b c d 10. a b c d
Conditional Probability  299

Solutions
Concept Applicator (CA)
1. (a) 2. (a) 3. (d) 4. (b) 5. (c) 6. (b)
7. (c) 8. (c) 9. (b) 10. (b)

1.
(a) We know that P(B/A) 7.
(c) Sample space is {TH, HT, TT} so number
1 of elements is 3, and favorable case is
P( A  B) 3 1 {HT, TH} so required probability is 2/3
  
P( A) 2 2 Alternately:
3 Let A = event of getting one Head
2. (a) We know that and one Tail
P( A  B) and B = event of getting at least 1 Tail.
P(A/B)  P(B) Then P(A) = 2/4 = 1/2 ,
1
P(B) = 3/4 ,
P( A  B) 3 1
or  P(B)    and P(A ∩ B) = 2/4 = 1/2
P( A / B) 2 2
3 We have to find
3. (d) Since A and B are two mutually P(A/B) = P(A ∩ B)/P(B)
exclusive events so P(A ∩ B) = 0 so = (1/2)/(3/4) = 2/3
P(A/B) = 0.
8. (c) In this case sample space S = {(5,1),
4. (b) Since P(A ∪ B) = P(A) + P(B) – P(A ∩ B) so (5, 2), (5,3),(5,4), (5,5) and (5,6)}.
Number of elements in the sample
 2  3  4  1
P(A ∩ B) =   +    –   = space is 6 and the favorable case is
 5  5  5  5 (5, 6) so required probability is 1/6
1 Alternately:
P( A  B) 5 1 Let A = Event of getting outcome
Then P(A/B)     
P(B) 3 3 such as sum is more than 10
5 And B = Event of getting 5 on 1st
5. (c) We know that P(A/B) + P(A′/B)= 1. die
6. (b) It is known that the number is an Then  P(B) = 1/36 = 1/6 ,
odd number so sample space is and  P(A ∩ B) = 1/36
  S = { 1, 3, 5} We have to find
And favorable case is {5} so required P(A/B) = P(A ∩ B)/P(B)
probability is 1/3. = (1/36)/(1/6) = 1/6
300  Theory of Counting
9.
(b) In this case sample space S = { (2,6), 10. (b) Here Sample Space S = {(3, 1), (3, 2),
(3, 5), (4, 4), (5, 3) and (6, 2)}. (3, 3), (3, 4), (3,5) and (3, 6)}.
So number of elements in sample So number of elements is 6.
space is 5. And the favorable case is {(3, 1) and
Now consider the favorable cases- (6, 3)}
{(3, 5) and (5, 3)} So required probability is 2/6 = 1/3
So required probability is 2/5.

Concept Builder (CB)


1. (d) 2. (d) 3. (c) 4. (a) 5. (d) 6. (d)
7. (b) 8. (a) 9. (c) 10. (b)

1. (d) We know that probability of 2nd ball being red is


P( A  B) 8/13.
P(A/B) = 5. (d) Total number of students are 10 but
P(B)
out of those Adnan is always selected
P ( A '  B ') so now we have to select 2 students
or P(A′/B′)  from 9 students and that can be done
P ( B ')
in 9C2 = 36 ways.
7
1 Number of ways such that Hasan is
1  P ( A   B ) 8 1
  selected is 8 so required probability
1  P(B) 3 5 is 8/36 = 2/9.
1
8
6. (d) Two black balls can be selected in
6C = 15 ways and two balls can
2. (d) We know that P( A′ ∩ B′) = 1– P(A∪B) 2
be selected in 14C2 = 91 ways, so
hence P (A ∪ B) = 1 – 0.2 = 0.8
required probability is 15/91.
Since P(A ∪ B) = P(A) + P(B) – P( A ∩ B)
7. (b) Sample space for the 2nd ticket is 24
or 0.8 = 0.4 + P( B) – 0.2 or P(B) = 0.6
(Since 1 ticket is already withdrawn)
Now P(B/A) = P(A ∩ B)/P(A) from 1 to 25 we have 5 perfect
= 0.2/0.4 = 1/2. squares so out of these 1 is already
3. (c) We know that drawn so favorable cases are only 4,
so required probability is 4/24 = 1/6
P( A  B)
P(A/B)   0.4  8. (a) From 1 to 25 we have 5 perfect
P(B) squares so sample space has 20
elements, and out of these only 8 is a
P( A  B) perfect cube so required probability
and P(B/A)   0.5 
P( A) is 1/20.
9. (c) As per the given condition outcome
P( A) 5
hence =   is (2,2), (2,3), (2,5), (3,2), (3, 3), (3,5),
P(B) 4 (5, 2), (5, 3) and (5,5)
Then P(A) = 0.75 So sample space is 3 × 3 = 9 and
number of favorable case is (2, 3)
4. (a) If 1st ball is black then remaining in
and (5,3)
bag is 8 red and 5 black balls then
So required probability is 2/9.
Conditional Probability  301
10. (b) As per the given condition outcome So sample space is 3 × 3 = 9 and
is (2,2), (2,3), (2,5), (3,2), (3, 3), (3,5), number of favourable case is (2, 5).
(5, 2), (5, 3) and (5,5) So required probability is 1/9.

Concept Cracker (CC)


1. (a) 2. (c) 3. (a) 4. (b) 5. (b) 6. (c)
7. (d) 8. (b) 9. (c) 10. (d)

1. (a) Let P(A) = probability that a randomly Let P(A′) = probability that a
selected student likes chocolate = 0.5 randomly selected
Let P(B) = probability that a randomly student does not like
selected student likes cake = 0.3 tea = 1 – 0.3 = 0.7
Then P(A ∩ B) = 0.1 Let P(B) = probability that a
randomly selected
Now we have to find
student likes coffee
P(A/B) = P(A ∩ B)/P(B) = 0.2
= 0.1/0.3 = 1/3 Let P(B′) = probability that a
2. (c) Let P(A) = probability that a randomly randomly selected
selected student likes tea = 0.3 student does not like
Let P(B) = probability that a coffee = 1 – 0.2 = 0.8
randomly selected student likes And P(A ∩ B) = 0.1
coffee = 0.2 So P(A ∪ B) = P(A) + P(B) – P(A ∩ B)
Then P(A ∩ B) = 0.1 = 0.3 + 0.2 – 0.1 = 0.4
Now we have to find So P(A′ ∩ B′) = 1 – P(A ∪ B) = 1 – 0.4 =
P(A/B) = P(A ∩ B)/P(B) = 0.1/0.2 = 1/2 0.6
3. (a) Let P(A) = probability that a randomly Now we have to find
selected student likes tea = 0.3. P(A′/B′) = P(A′ ∩ B′)/P(B′) = 0.6/0.8 = 3/4
Let P(A′) = probability that a randomly 5. (b) For imaginary roots D < 0 or (2b)2 –
selected student does not like tea 4(1)(5b – 4) < 0
= 1 – 0.3 = 0.7.
Or b2 – 5b + 4 < 0 or (b – 1)(b – 4) < 0
Let P(B) = probability that a randomly or 1 < b < 4
selected student likes coffee = 0.2
So required probability is 3/11
Then
6. (c) There are two cases when Abhay
P(A′ ∩ B) = P(B) – P(A ∩ B) will say ‘Yes’:
= 0.2 – 0.1 = 0.1 Case (i) The number that came out
Now we have to find is a prime and Abhay is speaking
P(A′/B) = P(A′ ∩ B)/P(B) = 0.1/0.2 = 1/2 truth, probability for this case is
P(P) × P(T)
4. (b) Let P(A) = probability that a
randomly selected Here P(P) = probability of getting a
student likes tea = 0.3 prime = 3/6 = 1/2 = 0.5
302  Theory of Counting
P(T) is probability that Abhay is 8. (b) Required probability
speaking truth and P(T) = 0.6
= P(E)
So probability for this case is
0.5 × 0.6 = 0.3. = P(E1) × P(E2/E1) × P(E3/E1E2)
Case (ii) The number that came = (2/5)(1/5)(1/10) = 1/125.
out is not a prime and Abhay is not
speaking truth, probability for this Solution for 9 and 10.
case is P(P′) × P(T′) = 0.5 × 0.4 = 0.2.
So total probability for the given
Foot Ball (40x) Cricket (50x)
case is 0.3 + 0.2 = 0.5.
New sample space is 0.5 and we
have to find the probability of case
(i) which is 0.3/0.5 = 0.6. a d
7. (d) There are two cases when Abhay g b
will say ‘No’
f
Case (i) The number that came out is
e
a prime and Abhay is not speaking
truth, probability for this case is
P(P) × P(T’).
Here P(P) = probability of getting a Chess (60x)
prime = 3/6 = 1/2 = 0.5.
P(T) is probability that Abhay is
speaking truth and P(T) = 0.6 so From the above diagram
P(T’) = 0.4. P(A/B) =
So probability for this case is 0.5 x d+g+f 40x  – a
0.4 = 0.2. =
b + c + d + e + f + g 110x  –( g + e )
Case (ii) The number that came out
is not a prime and Abhay is speaking 9. (c) For maximum value of P(A/B)
truth, probability for this case is numerator should be maximum and
denominator should be minimum
P(P’) × P(T) = 0.5 × 0.6 = 0.3.
Or required value is 40x/60x = 2/3
So total probability for the given
case is 0.3 + 0.2 = 0.5. 10. (d) For minimum value of P(A/B)
numerator should be minimum and
New sample space is 0.5 and we
denominator should be maximum
have to find the probability of case
(i) which is 0.2/0.5 = 0.4. Or required value is 0/100 = 0
Part B Conditional Probability  303

Introduction of
Concept 1 Conditional Probability
Conditional Probability: Consider two events of event B and vice versa then P(A/B) i.e
A and B such that B ≠ Ф, then probability probability of occurrence of event A when B
of occurrence of event A when B has already has already occurred
occurred is called conditional probability and P(A/B) = P(A ∩ B)/P(B)
is denoted by P(A/B). but we know that for two independent events
Hence P(A/B) = P(A ∩ B)/P(B) P(A ∩ B) = P(B) ⋅ P(A)
Hence
Conditional Probability for
independent Events P(A/B) = P(A ∩ B)/P(B) = P(A)
Hence for two independent events A and B
If A and B are independent events, then
probability of occurrence of event A is not P(A/B) = P(A)
affected by occurrence or non-occurrence P(B/A) = P(B)

Properties of
Concept 2 Conditional Probability
Some Properties of Conditional (v) Two events A and B are independent if
Probability and only if
(i) P(A/B) = P(A ∩ B)/P(B) P(A ∩ B) or P(AB) = P(A). P(B).
or P(A ∩ B) = P(B) ⋅ P(A/B). (vi) If A, B and C are any three independent
events, then
(ii) P(B/A) = P(A ∩ B)/P(A)
P(A ∩ B ∩ C) = P[A ∩ (B ∩ C)]
or P(A ∩ B) = P(A) ⋅ P(B/A)
= P(A) . P(B ∩ C)
(iii) P(A/B) may or may not be equal to
= P(A) ⋅ P(B) ⋅ P(C)
P(B/A).
(vii) If we generalize the result for n events
(iv) If A and B are independent events, then then : If A1,A2, ... An be any n events none
P(B/A) = P(B). of which is an impossible event, then
304  Theory of Counting
P(A1 ∩ A2 ∩ … ∩ An) = P(A1) ⋅ P(A2/A1) ⋅ (ix) If A1, A2 ... An are independent events,
P(A3/A1A2) … P(An/A1A2.An – 1) then P(A1 ∪ A2 ∪ …. ∪ An) = 1 – P(A′1) ⋅
If A1, A2, ... An are independent events, P(A2′) ... P(An′)
then (x) If A and B are two events such that
P(A1∩ A2 ∩ ... ∩ An) = P(A1) ⋅ P(A2) ... P(An) B ≠ Ф, then
P(A/B) + P(A′/B) = 1
(viii) If A and B are two independent events,
then P(A ∪ B) = 1 – P(A′) ⋅ P(B′) (xi) If A and B be two events such that
A ≠Ф, then
P(B) = P(A). P(B/A) + P(A′). P(B/A′)

Concept 3 Bayes’ Theorem

Bayes’ theorem is just extension of conditional A 


P  k     
probability, in order to understand the Bayes’ A
theorem lets discuss the partition of a set. A
P  
Partition of a Set P( Ak ).  Ak 
 A  
A family of sets A1, A2, ... An is said to form a  P( A1 ).  P     P( A2 ). P  A  
 A   A 
partition of a set A if following conditions are  1  2 
 
satisfied:   A  
     P( An ).P 
  
 A  
(i) A1, A2,…., An are non – empty  n
(ii) Ai ∩ Aj = φ for i ≠ j
(iii) A = A1 ∪ A2 ∪ … ∪ An
Properties of Bayes’
Bayes’ Theorem
Theorem
Let us consider n mutually exclusive and
exhaustive events A1, A2, A3, ..., An and A is an • If A1, A2, ... An form a partition of S and
event which occurs together (in conjunction) A be any event then
with either of Ai i. e. if A1, A2, ..., An form a
P(A) = P(A1) ⋅ P(A/A1) + P(A2) ⋅ P(A/A2) +
partition of the sample space S and A be any
event, then from Bayes’ theorem … + P(An) ⋅ P(A/An)
Conditional Probability  305
• If P(A1) = P(A2) = … = P(An), probabilities and P(Ai/A) are called
posteriori probabilities.
P(Ak /A) = P(A/Ak)/P(A/A1) + P(A/A2) + …
+ P(A/An) • If A1, A2, ..., An form a partition of an
event A, then
• Priori and Posteriori Probabilities:
The probabilities P(A1), P(A2), ..., P(A2), P Ak 
P(Ak/A) 
..., P(An) which are known before the P A1  P A2 
     P An 
experiment takes place are called priori

1
Concept Deviator (CD)
Ideal Time Revise your concept (IIT advance or
5 Min. tougher questions)
Per Question

1. Let X and Y be two events such that that each American man is seated
P(X|Y) = 1/2 , P(Y|X) =1/3, and adjacent to his wife is  [IIT JEE 2007]
P(X ∩ Y) =1/6 . Which of the following is
(a) 1/2 (b) 1/3
(are) correct? [IIT JEE 2012]
(a) P(X ∪ Y) = 2/3 (c) 2/5 (d) 1/5
(b) X and Y are independent 4. A person goes to office either by car,
scooter, bus or train probability of which
(c) X and Y are not independent
being 1/7, 3/7, 2/7 and 1/7 respectively.
(d) P(Xc ∩ Y) = 1/3 Probability that he reaches office late, if
2. A signal which can be green or red with he takes car, scooter, bus or train is 2/9,
probability 4/5 and 1/5 respectively, is 1/9, 4/9 and 1/9 respectively. Given that
received by station A and transmitted to he reached office in time, then what is
station B. the probability of each station the probability that he travelled by a car.
receiving the signal correctly is 3/4. If the [IIT JEE 2006]
signal received at station B is green, then (a) 1/9 (b) 1/11
the probability that the original signal
was green is      [IIT JEE 2010] (c) 2/11 (d) 1/7
(a) 3/5 (b) 6/7 5. A is targeting B, B and C are targeting
A. Probability of hitting the target by A,
(c) 20/23 (d) 9/20
B and C are 2/3,1/2 and 1/3 respectively.
3. One Indian and four American men and If A is hit then find the probability that
their wives are to be seated randomly B hits the target and C does not.
around a circular table. Then the [IIT JEE 2003] 
conditional probability that the Indian
(a) 1/2 (b) 1/5
man is seated adjacent to his wife given
(c) 2/11 (d) 1/7
306  Theory of Counting
6. A spaceship send signal to earth in respective probabilities 1/2, 1/4 and
binary logic, that means entire data is 1/4. For the ship to be operational at
sent only through two symbols ‘0’ and ‘1’. least two of its engines must function.
The probability that when 0 is sent from Let X denote the event that the ship is
the spaceship and received it correctly is operational and let X1, X2 and X3 denote
0.96 and the probability that when 1 is respectively the events that the engines
sent from the spaceship and received it E1, E2 And E3 are functioning. Which of
correctly is 0.95. If data received at the the following is (are) true?
earth is 1 then what is the probability [IIT JEE 2012]
that data sent is 0? c/X]
(a) P[X1 = 3/16
(a) 2/99 (b) 5/99
(b) P[Exactly two engines of the ship
(c) 4/99 (d) None of these
are functioning |X] = 7/8
7. Two fair dice are thrown what is the
(c) P[X|X2] = 5/16
probability that 1stdie will show a
number multiple of three if it is given that (d) P[X|X1] = 7/16
summation of two result is a multiple of 3.
10. For a biased die the probabilities for
(a) 1/6 (b) 5/12 different faces to turn up are given
(c) 7/36 (d) None of these below:
8. Two fair dice are thrown what is the
Face 1 2 3 4 5 6
probability that 1st die will show a
number multiple of three if it is given Probability 0.1 0.32 0.21 0.15 0.05 0.17
that summation of two result is not a
multiple of 3. The die is rolled and you are told that
face 1 or face 2 has turned up. Then the
(a) 1/6 (b) 5/12
probability that is face 1 is
(c) 7/36 (d) None of these
[IIT JEE 1981]
9. A ship is fitted with three engines
E1, E2 and E3. The engines function (a) 1/21 (b) 2/21
independently of each other with (c) 5/21 (d) None of these

Response Grid
1. a b c d 2. a b c d 3. a b c d 4. a b c d 5. a b c d
6. a b c d 7. a b c d 8. a b c d 9. a b c d 10. a b c d
Conditional Probability  307

2
Concept Eliminator (CE)
Ideal Time Revise your concept (Maths Olympiad
Just solve it or tougher questions)

1. Rajesh doesn’t like to study. Probability (a) 7/15 (b) 7/30


that he will study is 1/3. If he studied (c) 8/15 (d) None of these
then probability that he will fail is 1/2
3. A pair of fair dice is rolled together till
and if he didn’t study then probability
a sum of either 5 or 7 is obtained. The
that he will fail is 3/4. If in result Rajesh
probability that 5 comes before 7 is
failed then what is the probability that
________ .
he didn’t studied.
(a) 1 (b) 2/5
(a) 2/3 (b) 3/4
(c) 1/5 (d) None of these
(c) 1/3 (d) None of these
4. In a Competitive test, a candidate
2. A box contains 8 red balls and 4 white
guesses, copies or knows the answer
balls. Hasan draws 2 balls from the box
to a multiple choice question with four
without replacement. It is given that the
choices. The probability that he makes a
balls have different weights, with each
guess is 1/3 and the probability that he
red ball having weight 0.1 kg and each
copies the answer is 1/6. The probability
white ball having weight 0.2 kg. Suppose
that his answer is correct given that he
that the probability that a given ball in
copied it is 1/8. Find the probability that
the urn is the next one selected is its
he knew the answer to the question,
weight divided by the sum of the weights
given that he answered it correctly.
of all balls currently in the box then what
is the probability that at least one of the (a) 24/29 (b) 26/29
ball is white? (c) 22/29 (d) None of these

Response Grid
1. a b c d 2. a b c d 3. a b c d 4. a b c d
308  Theory of Counting

Solutions
Concept Deviator (CD)
1. (a,b) 2. ( c) 3. (c) 4. (d) 5. (a) 6. (c)
7. (a) 8. (d) 9. (b, d) 10. (c)

1. (a,b) Since P(X/Y) = P(X ∩ Y)/P(Y) = 1/2 4.


(d) Let C, S, B, T be the events of the
hence P(Y) = 2P(X ∩ Y) = 2 × 1/6 = person going by car, scooter, bus or
1/3 train respectively.
Similarly P(Y/X) = P(X ∩ Y)/P(X) = 1/3. Then as per the given information
Hence P(X) = 3P(X ∩ Y) = 3 × 1/6 = 1/2 P(C) = 1/7, P(S) = 3/7, P(B) = 2/7, and
P(T) = 1/7.
Now P(X ∪ Y) = P(X) + P(Y) – P(X∩Y)
= 1/2 + 1/3 – 1/6 = 2/3 Let L be the event of the person
reaching the office in time.
Hence option (a) is correct.
Now lets define a new event O as
Since P(X ∩ Y) = 1/6 and P(X) × P(Y) an event of the person reaching the
= (1/2)(1/3) = 1/6 office in time.
So P(X ∩ Y) = P(X) × P(Y) Then from the given information
So X and Y are independent P(O/C) = 7/9, P(O/S) = 8/9,
P(Xc ∩ Y) = (1 – 1/2)(1/3) = 1/6. P(O/B) = 5/9 and P(O/T) = 8/9
2. (c) Event G = original signal is green Probability that he will reach office
E1 = A receives the signal correct late = P(O)
       
E2 = B receives the signal correct =  1  7    3  8     2  5     8  1 
E = signal received by B is green  7 9   7 9   7 9   9 7 
Probability that he will reach office
Now total probability such that
1 7 
signal received by B is green is late if he travel by car   
7 9 
= P(GE1E2) + P [G(E1′E2′)]
+ P [E1 (G′E2′)] + P[E2(G′E1′)] Here we need to find probability of
P(C/O) = {P(O/C) × P(C)}/P(O)
P(E) = 45/80
P(G/E) = (40/5 × 16)/46/5 × 16  1 7  
   
= 20/23   7 9  
3. (c) As per the given condition if we  1 7   3 8   2 5   8 1  
                  
Fix four American couples and one  7 9   7 9   7 9   9 7  
Indian man in between any two 1
couples; we have 5 different ways 
7
in which his wife can be seated, of
which only 2 cases are favorable. 5. (a) This is an example of conditional
∴  required probability = 2/5. probability,
Conditional Probability  309
Required probability is
{P (A|BC ’)P (BC ’)}
{P (A|BC ’)P (BC ’)+  P (A|B’C )P (B’C )+  P (A|BC )P (BC )+  P (A|B’C ’)P (B’C ’)}
 1   2  
1     
=  2   3   
1
 1  2  1  1 1  1  1   2   2
(1)       (1)       (1)       (0)     
 2  3  2  3  2  3  2   3  
6. (c) If data received at earth is 1 then we have two cases:
Case (1) data sent is ‘1’ and received as ‘1’ probability in this case is 0.5(0.95)
Case (2) data sent is 0 and received as ‘1’, probability in this case is 0.5(0.04)
0.5 (0.04 )
So required probability is = 4/99.
{0.5 (0.95)+  0.5 (0.04)}
7. (a) Sum is multiple of 3 so we have following cases:
Case (i) sum is 3 then cases are (1, 2), (2, 1)
Case (ii) sum is 6 then cases are (1, 5), (2, 4), (3, 3), (4, 2) and (5, 1)
Case (iii) sum is 9 then cases are (3, 6), (4, 5), (5, 4), and (6, 3)
Case (iv) sum is 12 then only case is (6,6)
So total number of elements in sample space is n(S) = 12
Number of favorable cases are 2
So required probability = 2/12 = 1/6
8. (d) From the answer of previous question required probability is 1 – 1/6 = 5/6.
9. (b,d) We will analyze the options one by one-
 1  1  1
      1
2 4 4
(a) P[X1c/X]=  =
 1   1   3   1   1   1   1   1   3   1   1   1   8
      +       +       +       
 2 4 4 4 4 2 2 4 4 2 4 4 
 8   1   1   1  
  –       
(b) P(Exactly two engines are functioning/X) =  32   4   4   2  
8
 1   1   3   1   1   1   1   32
      +     +      
4  2 4 2 4 2 4  5
(c) P(X/X2) = =
1  1   1   1   3   1   1   1   3   8
    +      +     +     
4  2   4   2   4   2   4   2   4  
 1   1   1   1   3   1   3  
      +      +      
2  4 4 4 4 4 4  7
(d) P(X/X1) = =
1  3   3   1   1   1   3   1   3   16
  ×   +          +      
2  4   4   4   4   4   4   4   4  
Option b and d are correct.
10. (c) Here sample space is 0.1 + 0.32 = 0.42
And favourable case is 0.1, so required probability is 0.1/0.42 = 5/21.
310  Theory of Counting

Concept Eliminator (CE)


1. (b) 2. (d) 3. (b) 4. (a)

1. (b) He will fail in exam in two cases: Now, P( a sum of 5 in a single throw)
Case (i) He studied and failed,     = 4/36 =1/9
probability of this case is P( neither 5 nor 7) = 26/36 = 13/18
  (1/3)(1/2) = 1/6 Required probability

Case (ii) He didn’t studied and failed,  1   13   1   13  2  1  2


=                ... 
probability of this case is  9   18   9   18   9  5
4. (a) Let G, C, K, and A are events where
  (2/3)(3/4) = 1/2.
G = Guess answer, C = Copy answer,
So total probability is
K = Know the answer, and A = Answer
  1/6 + 1/2 = 4/6 = 2/3 correctly
Then required probability P(G) = Probability that the Candidate
Guess the answer = 1/3 (given)
  = (1/2)/(2/3) = 3/4.
P(C) = Probability that the Candidate
2. (d) Probability that 1st ball is red is Copy the answer = 1/6 (given)
8  0.1 1 P(K) = Probability that the Candidate

8  0.1   4  0.2 2 Know the answer = ?
Now, G,C and K are mutually
Probability that 2nd ball is red is
exclusive and exhaustive events
7  0.1 7
 Therefore, P(G) + P(C) + P(K) = 1
7  0.1    4  0.2 15
P(K) = 1 – 1/3 – 1/6 = 1/2.
So probability that both of them are
Say G(Candidate guesses) has
 1  7  7 occurred. As, there are four choices
red is     =
 2   15  30
out of which only one is correct, then
Required probability = 1–7/30 = 23/30. the probability that the candidate
made a guess is 1/4.
3. (b) Sum of 5 can be given by, (1,4), (4, 1),
(3,2) and (2,3) i.e 4 ways.   P(A/G) = 1/4
P( A/C) = 1/8 ( given) then
Sum of 7 can be given by ( 1,6), (6,1)
(2,5), (5, 2), (4,3) and (3,4) i.e 6 ways P( A/K) = Answer correctly that the
candidate know = 1
We can find the number of ways of
Applying Baye’s Theorem we have,
throwing neither 5 nor 7 is
P(K/A) = [ P(K).P(A/K)]/[P(G) ⋅ P(A/G) +
  36 – (4+6) = 26
P(C) ⋅ P(A/C) + P(K) ⋅ P(A/K) = 24/29
12
Binomial Probability  311

Binomial
Probability

Topics Covered

 Introduction of Binomial Theorem

 Binomial Distribution

 Expected Value

Part A: Topic Number of Questions


Solved Example 5
Concept Applicator 10
Concept Builder 10
Concept Cracker 10
Part B: Topic Number of Questions
Concept Deviator 12
Concept Eliminator 5
Total 52
312  Theory of Counting
Part A
Introduction of
Concept 1 Binomial Probability
Consider a situation in which an experiment Therefore, probability of occurrence of event
is carried out n times E with probability P exactly r times out of n
Let P = the probability of occurrence of an experiments is given by
event E in one trial P(r) = nCr (P ⋅ P ... r times) [P′ . P′ ... (n – r) times]
P′ = 1 – P = probability of non-occurrence of = nCrpr q n – r
event E in one trial.
Example 1:  A coin is tossed thrice what
Let r = number of successes i. e. number of is the probability of getting exactly 1
times event E occurs in n trials. ‘HEAD’?
Then the probability of occurrence of event E Solution:  Probability of getting a ‘HEAD’ is
exactly r times in n trials is denoted by P(r) 1/2 , out of three results exactly 1 is HEAD
and is given by so we have three cases:
P(r) = nCrPr P′ n – r Case (i) if 1st result is Head then probability
= ( r + 1)th term in the expansion is (1/2)(1/2)(1/2)
of (P + P′)n Case (ii) if 2nd result is Head then probability
Proof: Probability of occurrence of event E in is (1/2)(1/2)(1/2)
one trial = P. Case (iii) if 3rd result is Head then probability
Probability of non-occurrence of event E in is (1/2)(1/2)(1/2)
one trial = Q = P′ = 1 – P So total probability is 3/8.
Event E occurs exactly r times in n experiments Example 2:  A die is thrown 5 times, what is
means that the even E occurs r times and it the probability that a composite number
does not occur (n – r) times in n experiments. will appear exactly three times?
In other words out of n experiments event Solution:  Let us take this question to
with probability P occurred r times and event understand the Binomial Probability:
with probability P′ occurred n–r times
When a die is thrown then probability of getting
Out of n experiments event E can be selected a composite number is P(C) = 2/6 = 1/3
in nCr ways.
Then probability of not getting a composite
Since the n experiments are independent. number is P(C′) = 1–1/3 = 2/3.
QR Code / Video Link

For Smart phone/ Tablet users

Video Link for Desktop/Laptops users http://dishapublication.com/video-resources


Binomial Probability  313
Let us take a case in which 1st, 2nd and 3rd Alternately:
result is a composite number and 4th and 5th
Total number of experiments = n = 5
is not a composite numbers or the outcome
CCCC′C′ and probability in this case is Number of favorable cases = r = 3
3 2
 1   1   1   2  2  1   2 Probability of getting favorable case = P(F) = (1/3)
 3   3   3   3   3  =   3   3 
Probability of not getting favorable case
Now we have to find that how many such
cases exists, it is similar to in how many ways = P(F′) = (2/3)
we can arrange 3C and 2C′ which is equal to Required probability is
(5!)/(2!)(3!) = 5C3. 3 2
3 2
 1   2
 1   2 (5C3)(F)3(F′)2 = (5C3)    
So total number of ways is (5C3)     3 3
 3  3

Concept 2 Binomial Distribution

Consider E be an event. Example 3:  A bag contains 3 red and 2


Let P = probability of occurrence of event black balls, one ball is drawn 4 times
E in one trial. (with replacement) then what is the
And Q = probability of non-occurrence of probability that the result is 2 red and
event E in one trial = P′ = 1 – P
2 black balls?
Hence clearly P + Q = 1
Let number of times the experiment is carried Solution:  Total number of experiments = n = 4
out is n and out of these n experiments r Number of favorable cases = r = 2
number of times event E (With probability P)
occurred then probability of this is given by (Here favorable case is getting red ball)
P(r) = nCrpr q n – r Probability of getting favorable case
Consider with different values of r- = P(F) = (3/5)
P(0) = nC1qn Probability of not getting favorable case
P(1) = nC1pqn – 1
= P(F′) = (2/5)
P(2) = nC2p2qn – 2
Required probability is
P(3) = nC3p3qn – 3
… 2 2
 3  2
… (4C2)(F)2(F′)2 = (4C2)    
 5  5
P(n) = nCnpn
And P(1) + P(2) + P(3) + ... + P(n) = 1 = 6 × 36/3125 = 216/3125
314  Theory of Counting
Example 4:  A coin is tossed 8 times what Solution:  Total number of experiments = n = 5
is the probability that at least one Number of favorable cases = r = 3
‘Head” will appear?
Probability of getting favorable case
Solution:  Probability that no Tail will appear
is (1/2)8 = 1/256 = P(F) = (2/6) = (1/3)
1 255 Probability of not getting favorable case
So required probability = 1 – = = P(F′) = (2/3)
256 256
Example 5:  A die is thrown 5 times what is Required probability is
the probability that a number multiple 5×2 10
of 3 will appear exactly 4 times? (5C4)(1/3)4(2/3)1 = 5
=
3 243

Concept 3 Expected Value

The concept of “expected value” is very Expected Value = Σ[Probability (Ei) ×


important in the Theory of Probability. To Monetary Value associated with event Ei]
understand this let us take an example, say Lets take one more example to understand
a man is playing a game of “throwing a die”. the concept of expected value:
He is given ` 100 if he throws a ‘one’ and ` 500 Let us assume that in a biased game a person
if he throws a “ six” on the die and not paid will get ` 5 if he throws a ‘five’ and will get
anything if he throws any other number (i.e ` 8 for any other number, then we have to
if he throw 2, 3, 4 or 5) As per the rule of the find the expected income per throw for a large
game , he will have to pay some amount to the number of experiments.
casino each time he wants to throw the die. It is given that number ‘five’ will appear 5
Suppose he throws the die a large number times as frequently as any other number.
Here 1st we need the probability of the events
of times – say 6000 times. As the number of
involved.
times the experiment is repeated becomes very Let us assume that the probability of getting
large hence the number of times each event any number other than ‘five’ as p. The
will occur is given by theory of probability. probability of getting a five is 5p.
A “one” will appear with a probability of 1/6, Since all the events are mutually exclusive
i.e. it is expected to appear 1000 times out of a and collectively exhaustive, the sum of their
total of 6000 times the die is thrown. Similarly, probabilities must be equal to 1.
a “six” will appear 1000 times. Hence, the Hence p + p + p + p + p + 5p = 1
amount he will get in 6000 throws is 1000 × ⇒ 10p = 1  or  p = 1/10
100 + 1000 × 500 = 600000. The amount he
Hence the probability that the number ‘five’
gets per throw will be 600000/6000 = 100. appears is 5/10 = 1/2 and the probability that
So in this example we can say that the person’s any other number appears is 1/10.
expected value of this game per throw in the Hence the expected value is = (1/2)(5) + 1/10(8
long run is ` 100. + 8 + 8 + 8 + 8) = 65/10 = 6.5.
Binomial Probability  315

1
Concept Applicator (CA)
Ideal Time Apply your concepts with easy and
10 Min. conceptual questions.

1. A die is thrown 5 times, what is the (a) 108/3125 (b) 216/3125


probability that an even number will (c) 432/3125 (d) None of these
appear exactly three times?
7. A bag contains 3 red and 3 black balls, one
(a) 5/16 (b) 5/32
ball is drawn 5 times (with replacement)
(c) 5/64 (d) None of these
then what is the probability that the
2. A die is thrown 10 times, what is the
result is 3 red and 2 black balls?
probability that an odd number will
appear 6 times? (a) 5/32 (b) 5/64
(a) 210/512 (b) 105/512 (c) 5/128 (d) None of these
(c) 105/1024 (d) None of these 8. A bag contains 6 red and 4 black balls, one
3. A die is thrown 5 times, what is the ball is drawn 6 times (with replacement)
probability that a prime number will then what is the probability that the
appear exactly three times? result is 3 red and 3 black balls?
(a) 5/16 (b) 5/32
(a) 432/3125 (b) 864/3125
(c) 5/64 (d) None of these
(c) 216/3125 (d) None of these
4. A coin is tossed five times what is the
probability of getting exactly 2 ‘HEAD’ ? 9. A coin is tossed 10 times what is the
(a) 5/16 (b) 5/32 probability that exactly 5 ‘Tail” appear?
(c) 5/64 (d) None of these (a) 31/256 (b) 63/512
5. A die is thrown 6 times, what is the (c) 63/256 (d) None of these
probability that a composite number will
10. A coin is tossed 10 times what is the
appear three times?
probability that at least one ‘Tail” will
(a) 40/729 (b) 80/729 appear?
(c) 160/729 (d) None of these 1023 1
6. A bag contains 3 red and 2 black balls, one (a) (b)
1024 1024
ball is drawn 4 times (with replacement)
then what is the probability that the 511
(c) (d) None of these
result is 3 red and 1 black balls? 1024

Response Grid
1. a b c d 2. a b c d 3. a b c d 4. a b c d 5. a b c d
6. a b c d 7. a b c d 8. a b c d 9. a b c d 10. a b c d
316  Theory of Counting

2
Concept Builder (CB)
Ideal Time Revise your concepts with questions
15 Min. medium difficulty level questions
1. In a school there are 100 students out of 5. Probability that a shooter can shoot the
which 10 girls and 90 boys, school teacher target is 0.8, if tried 5 times then what is
called 1 student every day for 8 days then the probability that the target will be hit (i.e
what is the probability that there will be 3 shooter will hit the target at least once)?
girls and 5 boys in that 8 students? (a) 624/625 (b) 3124/3125
1  9
3 5 (c) 124/125 (d) None of these
(a) (8C3)     6. A pair of dice is thrown 5 times. If getting
 10   10 
a total of 9 is considered as win then
3 5
 8  9 what is the probability of getting at least
(b) (8C3)    
 10   10  4 success?
3 5 40 39
 8  9 (a) (b)
(c) (8C5)     95
95
 10   10 
(d) None of these 41
(c) (d) None of these
2. What is the probability that at least one 95
girl is selected ? 7. A bag contains 5 red, 6 black and 4 blue
balls, 5 balls are drawn with replacement
108 −  89 108 −  98 then what is the probability that red ball
(a) (b)
108 108 will appear in exactly 3 times?
98 −  88 (a) 35/243 (b) 71/243
(c) (d) None of these
8
9 (c) 40/243 (d) None of these
3. If two dice are drawn 5 times then what 8. A bag contains 5 red, 6 black and 4 blue
is the probability that sum 10 will appear balls, 5 balls are drawn with replacement
in exactly 3 times? then what is the probability that red ball
will appear at least 3 times?
(a) 6655/124416 (b) 1210/124416
(a) 7/18 (b) 2/9
(c) 605/124416 (d) None of these
(c) 5/18 (d) None of these
4. A bag has 3 red and 2 black balls, 1 9. In the above question what is the
ball is drawn from the bag 5 times with probability that at least 1 red ball will
replacement, what is the probability that appear?
the there will be 3 red and 2 black balls
in the 5 results? 32 64
(a) (b)
243 243
(a) 57/225 (b) 104/225
31
(c) 216/625 (d) None of these (c) (d) None of these
243
Binomial Probability  317
10. A die is thrown 5 times what is the 10 11
probability that a composite number will (a) (b)
243 243
appear exactly 4 times?
7
(c) (d) None of these
243
Response Grid
1. a b c d 2. a b c d 3. a b c d 4. a b c d 5. a b c d
6. a b c d 7. a b c d 8. a b c d 9. a b c d 10. a b c d

3
Concept Cracker (CC)
Ideal Time Boost up your confidence with good
20 Min. questions

1. A coin is tossed 10 times then what is the 2 10


probability that ‘Tail’ will appear at least  1   11 
(b) (12C10)    
8 times?  12   12 
(a) 7/128 (b) 7/256 1
10
 11 
2
(c) (12C10)    12 
(c) 7/512 (d) None of these  12 
2. Probability that India will win against (d) None of these
Pakistan in a cricket match is 2/3, in 5. If two dice are drawn 12 times then what
series of 5 matches what is the probability is the probability that sum will be at
that India will loose only 2 games? least 10 in exactly 10 times?
(a) 40/81 (b) 40/243 10 2
1  11 
(a) (12C3)    12 
(c) 80/243 (d) None of these  12 
3. Probability that India will win against 2
 1   11 
10
Pakistan in a cricket match is 2/3, in (b) (12C10)    
 12   12 
series of 5 matches what is the probability
10 2
that India will win the series? 1  11 
(c) (12C10)    12 
(a) 161/81 (b) 192/243  12 

(c) 172/243 (d) None of these (d) None of these


6. Probability that Priyanka will pass an
4. If two dice are drawn 12 times then what
exam is ‘k’, she appeared in 5 exams and
is the probability that sum 10 will appear
if probability that she will pass is exactly
in exactly 10 times?
4 out of 5 then find the value of ‘k’
10 2 (a) 4/5 (b) 1/5
1  11 
(a) (12C3)    12 
 12  (c) 2/5 (d) None of these
318  Theory of Counting
7. In a bulb manufacturing company it replacement, what is the probability that
is known that 10% of the bulbs are the there will be a red and b black balls
defective, what is the probability that in the (a+b) results?
out of 10 randomly selected bulbs not
( ab)!( a abb )
more than 10% is defective? (a)
( a !)(b !)( a + b)a +b
910 99
(a) (b)
1010 109 ( a + b)!( a abb )
10 (b)
9 ( a ! b !)( a + b)a +b
(c) (d) None of these
109
8. Two dice are thrown 4 times what is 2( a + b)!( a abb )
(c)
the probability the summation will be a ( a !)(b !)( a + b)a +b
prime number for exactly prime number
of times? (d) None of these
11 19 10. If ‘k’ coins are tossed together then what
(a) (b) is the probability that ‘Tail’ will appear
124 124
on Odd number of coins?
41
(c) (d) None of these  1  1 
124 (a)   (b) 
 2k   2k −1 
9. A bag has a red and b black balls, 1 ball
(c) 1/2 (d) None of these
is drawn from the bag (a+b) times with

Response Grid
1. a b c d 2. a b c d 3. a b c d 4. a b c d 5. a b c d
6. a b c d 7. a b c d 8. a b c d 9. a b c d 10. a b c d
Binomial Probability  319

Solutions
Concept Applicator (CA)
1. (a) 2. (b) 3. (a) 4. (a) 5. (c) 6. (b)
7. (d) 8. (b) 9. (c) 10. (a)

1. (a) Total number of experiments = n = 5 Required probability is


Number of favorable cases = r = 3 3 2
 1  1
Probability of getting favorable case (5C3)(F)3(F′)2 = (5C3)    
 2  2
= P(F) = (1/2) = 10/32 = 5/16
Probability of not getting favorable 4. (a) Total number of experiments = n=5
case = P(F′) = (1/2)
Number of favorable cases = r = 2
Required probability is
Probability of getting favorable case
3 2
 1  1 = P(F) = (1/2)
(5C3)(F)3(F′)2 = (5C3)    
 2  2 Probability of not getting favorable
= 10/32 = 5/16 case = P(F′) = (1/2)
2. (b) Total number of experiments Required probability is (5C2)(F)2(F′)3
= n = 10 2
 1  1
3

Number of favorable cases = r = 6 = (5C2)     = 10/32 = 5/16.


 2  2
Probability of getting favorable case 5. (c) Total number of experiments = n=6
= P(F) = (1/2) Number of favorable cases = r = 3
Probability of not getting favorable Probability of getting favorable case
case = P(F′) = (1/2)
= P(F) = (1/3)
Required probability is
Probability of not getting favorable
6 4
 1  1 case = P(F′) = (2/3)
(10C6)(F)6(F′)4 = (10C6)    
 2  2 Required probability is
= 210/1024 = 105/512 3 3
 1   2
3. (a) Total number of experiments = n = 5 (6C3)(F)3(F′)3 = (6C3)    
 3  3
Number of favorable cases = r = 3
= 20 × 8/729 = 160/729
Probability of getting favorable case
6. (b) Total number of experiments = n=4
= P(F) = (1/2)
Number of favorable cases = r = 3
Probability of not getting favorable (Here favorable case is getting red
case = P(F′) = (1/2) ball)
320  Theory of Counting
Probability of getting favorable case Probability of getting favorable case
= P(F) = (3/5) = P(F) = (6/10) = (3/5)
Probability of not getting favorable Probability of not getting favorable
case = P(F′) = (2/5) case = P(F′) = (2/5)
Required probability is Required probability is
3 1 3 3
 3  2  3  2
(4C3)(F)3(F′)1 = (4C3)     (6C3)(F)3(F′)3 = (6C3)    
 5  5  5  5
= 27 × 8/3125 = (20 × 27 × 8)/15625
= 216/3125 = 864/3125
7. (d) Total number of experiments = n=5 9. (c) Total number of experiments
Number of favorable cases = r = 3 = n = 10
(Here favorable case is getting red Number of favorable cases = r = 5
ball) Probability of getting favorable case
Probability of getting favorable case = P(F) = (1/2)
= P(F) = (3/6) = (1/2) Probability of not getting favorable
Probability of not getting favorable case = P(F′) = (1/2)
case = P(F′) = (1/2) Required probability is
Required probability is 5 5
 1  1
3 2 (10C5)(F)5(F′)5 = (10C5)    
 1  1  2  2
(5C3)(F)3(F′)2 = (5C3)    
 2  2
= 63/256
= 10/32 = 5/16 10. (a) Probability that no Tail will appear
8. (b) Total number of experiments = n=6 is (1/2)10 = 1/1024
Number of favorable cases = r = 3 So required probability
(Here favorable case is getting red 1 1023
ball) = 1 – =
1024 1024

Concept Builder (CB)


1. (a) 2. (b) 3. (c) 4. (c) 5. (b) 6. (c)
7. (c) 8. (b) 9. (d) 10. (a)

1. (a) Total number of experiments = n=8 Required probability is


Number of favorable cases = r = 3 3 5
1  9
(Here favorable case is when girl (8C )(1/10)3(9/10)5 = (8C
3) 
3  10   10 
student is called)
Probability of getting favorable case 2. (b) Total number of experiments = n=8
= P(F) = (10/100) = (1/10) Number of favorable cases = r = 0
(Here favorable case is when girl
Probability of not getting favorable
student is called)
case = P(F′) = (9/10)
Binomial Probability  321
Probability of getting favorable case Probability of getting favorable case
= P(F) = (10/100) = (1/10) = P(F) = (0.8)
Probability of not getting favorable Probability of not getting favorable
case = P(F′) = (9/10) case = P(F′) = (0.2)
Required probability is Required probability is
(8C0)(1/10)0(9/10)8 (5C0)(0.8)0(0.2)5 = 1/3125
1  9
0
98
8 So probability that the target is hit at
= (8C )     = least once is 1 – 1/3125 = 3124/3125
0  10   10 
108
6. (c) At least 4 success means a sum of 9
Required probability
occurred 4 or 5 times.
98 108 −  98 So required probability is P(4) + P(5)
= 1 − =
108 108 Total number of experiments = n = 5
3. (c) Total number of experiments = n = 5 Number of favorable cases = r = 4 or 5
Number of favorable cases = r = 3 Sum of 9 is given by (3, 6), (4, 5), (5,
Total sample space in one such trial 4), and (6, 3) so number of elements
is n(S) = 6 × 6 = 36 in favorable case is 4
Favorable cases are (4, 6), (5, 5) and Probability of getting favorable case
(6, 4) = P(F) = (4/36) = (1/9)
Probability of getting favorable case Probability of not getting favorable
= P(F) = (3/36) = (1/12) case = P(F′) = (8/9)
Probability of not getting favorable Required probability is now
case = P(F′) = (11/12) 4 1
 1   8
Required probability is P(4) = (5C4)    
 9  9
3 2
 1   11  5 0
(5C3)     = 605/124416  1   8
 12   12  And P(5) = (5C5)    
 9  9
4. (c) Total number of experiments = n = 5
4 1
Number of favorable cases = r = 3  1   8
So total probability = (5C4)    
(Here favorable case is getting red  9  9
ball) 5 0
 1   8 40 + 1 41
+ (5C5)     = =
Probability of getting favorable case  9  9 95 95
= P(F) = (3/5) 7. (c) Total number of experiments = n=5
Probability of not getting favorable Number of favorable cases = r = 3
case = P(F′) = (2/5)
(Here favorable case is getting red
Required probability is
ball)
10  27  4 216
(5C3)(3/5)3(2/5)2 =  Probability of getting favorable case
5 625
5 = P(F) = (5/15) = (1/3)
5. (b) Total number of experiments = n = 5
Probability of not getting favorable
Number of favorable cases = r = 0
case = P(F′) = (2/3)
(means he will not hit the target)
322  Theory of Counting
Required probability is 9. (d) Total number of experiments = n = 5
10  1  4 40 Number of favorable cases = r = 0
(5C3)(1/3)3(2/3)2 = 
5 243 (that means no red ball will appear)
3
8. (b) Total number of experiments = n = 5 Probability of getting favorable case
Number of favorable cases = r = 3 or = P(F) = (5/15) = (1/3)
4 or 5 (Here favorable case is getting
red ball). Probability of not getting favorable
case = P(F′) = (2/3)
Probability of getting favorable case
= P(F) = (5/15) = (1/3) Required probability is
Probability of not getting favorable 1  1  32
32
(5C0)(1/3)0(2/3)5 = 
case = P(F′) = (2/3) 3 243 5
P(3) = (5C3)(1/3)3(2/3)2 So probability that at least one red
32 211
10  1  4 40 ball is 1 – =
=  243 243
5 243
3 10. (a) Total number of experiments = n = 5
P(4) = (5C4)(1/3)4(2/3)1 Number of favorable cases = r = 3
5 1  2 10
=  Probability of getting favorable case
35 243
= P(F) = (2/6) = (1/3)
P(5) = (5C0)(1/3)5(2/3)2
1 1  4 4 Probability of not getting favorable
=  case = P(F′) = (2/3)
35 243
Required probability is Required probability is
40 10 4 54 2 52 10
+  +  = = (5C4)(1/3)4(2/3)1 = 
243 243 243 243 9 35 243

Concept Cracker (CC)


1. (a) 2. (c) 3. (b) 4. (c) 5. (d) 6. (a)
7. (d) 8. (d) 9. (b) 10. (c)

1. (a) As per the given question we have to 10


 1
find the probability of getting 8 or 9 =  
2
(45 +1 0 +1 ) = 56/1024 = 7/128
or 10 ‘Tail’
So required probability is P(8) + P(9) 2. (c) Total number of experiments = n = 5
+ P(10) Number of favorable cases = r = 3
8 2 9 1 (Here favorable case is when India
 1  1  1  1
(10C + (10C9) wins the game)
8) 
 2   2 
 2   2 
Probability of getting favorable case
10 0
 1  1 = P(F) = (2/3)
+ (10C 10) 
 2   2  Probability of not getting favorable
case = P(F′) = (1/3)
Binomial Probability  323
Required probability is Favorable cases are For summation
3 2 as 10 – (4, 6), (5, 5) and (6, 4)
 2  1 
(5C3)     = 80/243 For summation as 11 – (5, 6) and (6.5)
 3  3
For summation as 12 – (6,6)
3. (b) Total number of matches = n = 5
So total number of favorable cases is 6
India will win the series if it wins
either 3 or 4 or 5 matches. Probability of getting favorable case
In previous question we have = P(F′) = (6/36) = (1/6)
calculated the value of P(3) = Probability of not getting favorable
probability of winning 3 matches case = P(F′) = (5/6)
3 2 Required probability is
 2  1 
= (5C 3)  10 2
 3   3   1  5
(12C10)    6 
Required Probability  6
= P(3) + P(4) + P(5) 6. (a) Total number of experiments = 5
3 2 4 1 Number of favorable cases = r = 4
 2  1   2  1 
= (5C3)     + (5C4)     Probability of getting favorable case
 3  3 3 3
= P(F) = (k)
5 0
 2  1  Probability of not getting favorable
+ (5C 5) 
 3   3  case = P(F′) = (1 – k)
10  8 5  16 1  32 192 Required probability is
   
243 243 243 243 (5C4) ( k )4 (1 − k )1 = 256/625 so k = 4/5
7. (d) Total number of experiments = n = 10
4. (c) Total number of experiments = n = 12
Number of favorable cases = r = 0
Number of favorable cases = r = 10
or 1 (Here favorable case is getting
Total sample space in one such trial defective bulbs)
is n(S) = 6 × 6 = 36
Probability of getting favorable case
Favorable cases are (4, 6), (5, 5) and
= P(F) = (1/10)
(6, 4)
Probability of not getting favorable
Probability of getting favorable case
case = P(F′) = (9/10)
= P(F) = (3/36) = (1/12)
Required probability is
Probability of not getting favorable
case = P(F′) = (11/12) 0 10 1 9
1  9 1  9
Required probability is (10C 0) 
 10   10  + (10C 1) 
 10   10 
10 2
1  11  19(99 )
(12C10)    12     =
 12 
1010
5. (d) Total number of experiments = n = 12 8. (d) Total number of experiments = n = 4
Number of favorable cases = r = 10 Number of favorable cases = r = 2 or 3
Total sample space in one such trial Total sample space in one such trial
is n(S) = 6 × 6 = 36 is n(S) = 6 × 6 = 36
324  Theory of Counting
Favorable cases are For summation Probability of getting favorable case
as 10 – (4, 6), (5, 5) and (6, 4) = P(F) = a/(a + b)
For favorable cases: Probability of not getting favorable
Summation is 2 – (1,1) case = P(F′) = b/(a+b)
Summation is 3 – (1, 2) and (2, 1) Required probability is
Summation is 5 – (1,4), (2, 3), (3, 2) (4, 1)  a   b 
a b
Summation is 7 – (1,6), (2, 5), (3,4 ), (a+bCa) 
 a + b   a + b 
(4,3), (5, 2) and (6, 1)
= ( a + b)!( a abb )
Summation is 11 – (5, 6) and (6, 5)
( a !)(b !)( a + b)a +b
So total number of favorable cases is
15 10. (c) Required probability is given by P(1)
+ P(3) + P(5) + ... here P(r) represents
Probability of getting favorable case
probability of getting ‘r’ number of
= P(F) = (15/36) = (5/12) tails.
Probability of not getting favorable Now
case = P(F′) = (7/12)
1 k −1
Required probability is  1  1
P(1) = (kC1)    
2 2  2  2
 5  7
P(2) + P(3) = (4C2)     + (4C3) k
 12   12   1
= (kC1)  
 2
 5  3  7 1 6   52  72   4  53  7
     3 k −3
12  12  124  1  1
P(3) = (kC3)    
 2  2
(52  7  2)(21   1
  0)
  k
4
12  1
= (kC3)  
350  31  2

124 So required probability
9. (b) Total number of experiments = P(1) + P(3) + P(5) +…..
= n = (a + b)  1
=  k  (kC1 + kC3 + kC5 + …)
Number of favorable cases = r = a 2 
(Here favorable case is getting red
ball)  1  k −1 1
=  k  (2 ) =
2  2
Part B Binomial Probability  325

Introduction of
Concept 1 Binomial Probability
Consider a situation in which an experiment Let r = number of successes i. e. number of
is carried out n times times event E occurs in n trials.
Let P = the probability of occurrence of an Then the probability of occurrence of event E
event E in one trial exactly r times in n trials is denoted by P(r)
P′ = 1 – P = probability of non-occurrence of and is given by
event E in one trial. P(r) = nCrPr P′ n – r = ( r + 1)th term in the
expansion of (P + P′)n

Concept 2 Binomial Distribution

Consider E be an event. Consider with different values of r-


Let P = probability of occurrence of event E in P(0) = nC1qn
one trial. P(1) = nC1pqn – 1
And Q = probability of non-occurrence of event P(2) = nC2p2qn – 2
E in one trial = P′ = 1 – P
P(3) = nC3p3qn – 3
Hence clearly P + Q = 1
...
Let number of times the experiment is carried
...
out is n and out of these n experiments r
number of times event E (With probability P) P(n) = nCnpn
occurred then probability of this is given by And P(1) + P(2) + P(3) +…..+ P(n) = 1
P(r) = nCrpr q n – r
326  Theory of Counting

Concept 3 Expected value

Expected Value = Σ[Probability (Ei) × Monetary Value associated with event Ei]

1
Concept Deviator (CD)
Ideal Time Revise your concept (IIT advance or
5 Min. tougher questions)
Per Question

1. A multiple choice examination has 5 6 4


 4  2
questions. Each question has three (c) (10C6)    
alternative answers of which exactly one  5  5
is correct. The probability that a student (d) None of these
will get 4 or more correct answers just by 3. Probability that a shooter will hit
guessing is [IIT JEE 2013] the target is 1/2, What should be the
13 11 minimum number of shoots required so
(a)                              (b) that probability that the target is hit
5
3 35 twice is greater than 0.99
10 17
(c)                              (d) (a) 8 (b) 9
35 35 (c) 10 (d) None of these
2. In a large population 40% voted for 4. A lot contains 20 articles. The probability
candidate A and 60% voted for candidate that the lot contains exactly two defective
B, what is the probability that out of 10 articles is 0.4and the probability that the
students randomly selected from this lot contains exactly 3 defective articles
large population exactly 60% voted for is 0.6. Articles are drawn from the lot at
candidate B? random one by one , without replacement
6 4 and are tested till all the defective articles
 3  2  are found. What is the probability that
(a) (10C6)    
 5  5 the testing procedure ends at the twelfth
6 6 testing. [IIT JEE 1986]
 3  2 
(b) (10C4)     (a) 99/190 (b) 99/1900
 5  5
(c) 19/99 (d) none of these
Binomial Probability  327
5. An urn contain 2 white and 2 black balls. [IIT JEE 1994]
A ball is drawn at random. If it is white, (a) 1/2 (b) 2/5
it is not replaced into the urn. Otherwise (c) 1/5 (d) 2/3
it is replaced along with another ball of
10. There are four machines and it is known
the same color. The process is repeated.
that exactly two of them are faulty. They
Find the probability that the third ball
are tested, one by one, in a random order
drawn is black. [IIT JEE 1987]
till both the faulty machines are identified.
(a) 27/30 (b) 23/30 Then the probability that only two tests are
(c) 25/30 (d) none of these needed is [IIT JEE 1998]
6. A man takes a step forward with (a) 1/3 (b) 1/6
probability 0.4 and backwards with (c) 1/2 (d) 1/4
probability 0.6. Find the probability that 11. Three players A, B, and C toss a coin cyclically
at the end of eleven steps he is one step in that order (that is, A, B, C, A, B, C, A, B…)
away from the starting point. till a head shows up. Let p be the probability
[IIT JEE 1987] that coin shows a head. Let α β and ϒ be
(a) 11C 0.24)5 11C 5 respectively that A, B and C gets the first
6( (b) 7(0.24)
11C
head. Prove that β = ( 1 – p) α. Determine α,
(c ) 6( 0.2)5 (d) none of these β and ϒ (in terms of p) [IIT JEE 1998]
7. One hundred identical coins, each with (a) p/(1 – q3), p(1 – q)/{1 – (1 – p)3}
probability, p, of showing up heads are
and (p – 2p2 + p3)/ {1 – (1 – p)3}
tossed once, if 0 < p < 1 and the probability
of heads showing on 50 coins is equal to (b) p/{1 – (1 – p)3), p(1 – q)/{1 – (1 – q)3}
that heads showing on 51 coins, then the and (p – 2p2 + p3)/ {1 – (1 – p)3}
value of p is [IIT JEE 1988] (c) q/(1 – q3), p(1 – q)/{1 – (1 – p)3}
(a) 1/2 (b) 49/101 and (p – 2p2 + p3)/ {1 – (1 – p)3}
(c) 50/101 (d) 51/101 (d) p/(1 – q3), p(1 – p)/{1 – (q)3}
8. A box contains 24 identical balls of which and (p – 2p2 + p3)/ {1 – (q)3}
12 are white and 12 are black. The balls 12. An unbiased die, with faces numbered
are drawn at random from the box one at 1,2,3,4,5,6 is thrown n times and the
a time with replacement. The probability list of n numbers showing up is noted.
that a white ball is drawn for the 4th time What is the probability that among the
on the 7th draw is [IIT JEE 1994] numbers 1,2,3,4,5,6, only three numbers
appear in this list? [IIT JEE 2001]
(a) 5/64 (b) 27/32
(a) {(3 – 3.2 + 3) × C3}/ 6n
n n 6
(c) 5/32 (d) 1/2
(b) 3n × 6C3/ 6n
9. An unbiased die is tossed until a number
(c) {(3n – 3.2n + 3) × 6C3}/ 6
greater than 4 appears. The probability
that an even number of tosses is needed is (d) none of these

Response Grid
1. a b c d 2. a b c d 3. a b c d 4. a b c d 5. a b c d
6. a b c d 7. a b c d 8. a b c d 9. a b c d 10. a b c d
11. a b c d 12. a b c d
328  Theory of Counting

2
Concept Eliminator (CE)
Ideal Time Revise your concept (Maths Olympiad
Just solve it or tougher questions)

1. This question is Based on Maths What is the probability that out of 10


Olympiad. balls, 1st box will get 2, 2nd will get 3 and
India and Pakistan is playing a best 3rd box will get 5 balls?
four of 7 match series. What is the 10! 1
probability that the tournament ends up (a) (b)
(2!)(3!)(5!)(2 ) 15 2 15
in 6 matches assume that no match ends
up in a draw? 1
(c) (d) None of these
(a) 5/16 (b) 5/32 2 14
(c) 5/48 (d) None of these
4. Two fair dice are thrown till outcome is
2. This question is Based on Maths Olympiad 12. What is the probability that one has
A number selector machine can select to do 20 throws for this?
any number from 1 to 9. This machine 18 2 19
 35   1  35   1
is operated ‘n’ times, means there are n (a)     (b)    
 36  36  36  36
results. These ‘n’ results are multiplied
19
then what is the probability that the  1  35 
(c)     (d) None of these
product is divisible by 10.  36  36
n n n
 8  5  4 5. In a bulb manufacturing company
(a)   –   +   
 9  9  9 probability that a bulb is defective is
n
 8  5
n
 4
n 0.01. The company sells the bulbs in a
(b) 1 –   –   +    package of 10. After getting complains
 9  9  9
n n n
about defective bulb company decided
8 5 4 to offer a money back guarantee that
(c) 1 –   –   −   
 9  9  9 at most of the 10 from the package
(d) None of these is defective. What percentage of the
3. This question is Based on Maths Olympiad packages company has to replace?
10 balls are thrown into three boxes (a) 1% (b) 4%
namely Box 1, Box 2, and Box 3 with (c) 5% (d) None of these
respective probabilities 1/4, 1/4 and 1/2.

Response Grid
1. a b c d 2. a b c d 3. a b c d 4. a b c d 5. a b c d
Binomial Probability  329

Solutions
Concept Deviator (CD)
1. (b) 2. (a) 3. (d) 4. (b) 5. (b) 6. (a)
7. (d) 8. (c) 9. (b) 10. (b) 11. (d) 12. (a)

1.
(b) Let P(C) = probability of getting a 4. (b) Testing procedure may terminate in
correct answer is 1/3 the following two events:
From thye given condition required One – when the lot contains two
answer is defective articles.
4 1 5
 1  2   1 11 Two – when the lot has three
(5C4)     + (5C5)   =
 3  3  3 35 defective articles.
2. (a) Total number of experiments = n = 10 Number of experiments carried out
Number of favorable cases is 12.
= r = 60% or 6
Probability of getting favorable Let E be the event( testing procedure
case = P(F) = (60/100) = (3/5) (here ends at 12th testing)
we have assumed the event that E1 = lot has 2 defective articles
Candidate B got the vote)
E2 = lot has 3 defective articles
Probability of not getting favorable
Now, it can be possible that first 11
case = P(F′) = (40/100) = (2/5)
draws contains 10 non- defective
Required probability is
6 4
articles and a defective and 12th
 3  2  draw contains a defective article.
(10C6)(F)6(F′)4 = (10C6)    
 5  5 So,
3. (d) Let us suppose that number of shoots 18
C10  × 2C1
required is k then P(E/E1) = 20
× 1/9
C11
Probability that none of the time
Now, another case that first 11 draws
 1 contains 9 non- defective articles and
target was hit is  
 2n  two defective and 12th draw contains
Probability that target is hit once is a defective article. So,
 1   1  n  17
n  n−1    =  n  C9  × 3C 2
2   2  2  P(E/E1) = 20
× 1/9
C11
So required probability is
 1  n 99 Required probability
1 –  n −   n  > 0.99 = = P(E) ⋅ P(E/E1) + P(E)P(E/E2)
2  2  100
= 99/1900
or  n = 11
330  Theory of Counting
5. (b) Let, B and W be black and white starting point.
ball Required probability = P(one step
Bn = nth ball drawn is black and ahead) + P ( one step behind)
Wn = nth ball drawn is white ball Now, P (one step ahead) = the
E = third ball drawn which is black. probability that the man will be one
We can observe that, the black ball step ahead at the end of eleven steps
can be drawn in the third draw in one if he moves six steps forward and five
of the following mutually exclusive step backward = 11C6(0.4)6(0.6)5.
ways- In similar way, the probability that
1. Both first and second balls drawn are the man will be one step behind at
white and third all drawn is black. the end of eleven steps if he moves
i.e ( W1 ∩ W2) ∩ E six steps backward and five steps
forward = 11C6(0.4)5(0.6)6.
2. Both first and second balls are black
and third ball drawn is black Hence, required probability
i.e (B1 ∩ B2) ∩ E = 11C6( 0.24)5.
3. The first ball drawn is black, the
7. (d) According to question,
second ball drawn is white and the
third ball drawn is black P( X = 50) = P(X = 51)
i.e (B1 ∩ W2) ∩ E Or  100C 50(1 – p)50
50p
4. The first ball drawn is white, the = 100C51p51(1 – p)49
second ball drawn is black and the
Or  p/(1 – p) = 51/50 and p = 51/101
third ball drawn is black
i.e (W1 ∩ B2) ∩ E 8. (c) As per the given condition white ball
is drawn for the 4th time on the 7th
Therefore,
draw it is possible when exactly 3
P(E) = P[{(W1 ∩ W2) ∩ E} ∪ {( B1 ∩ B2 )
white ball appear in 1st 6 draws, so
∩ E} ∪ {( B1 ∩ W2) ∩ E} ∪
required probability is
{( W1 ∩ B2) ∩ E}
3 3
1 1 1
2 1 2 3 4 2 2  (6C3)        = 5/32
      1             2  2  2
4 3 4 5 6 4 3 9. (b) Let P(A) = probability of getting a
3  2 2  3 23 number greater than 4 is 4/6 = 1/3
       
4  4 5  4 30 then P(A′) = 2/3

6. (a) From the question we can say that Required Probability is given by A′A
the man will be one step away from + A′A′A′A + A′A′A′A′A′A + ...
the starting point either he is one
step ahead or one step behind the
Binomial Probability  331
Required Probability = P(H) P(T) + P(H)P(T)4
2  1 2  2  2  1 + P(H) P(T)7 + ...
=              
3  3  3  3  3  3 = P(T)[P(H)+ P(H)P(T)3
2  2  2  2  2  1
              + P(H) P(T)6+ ...]
3  3  3  3  3  3
2 = qa = (1 – p)a = p(1 – p)/ (1 – q3)
2 Again we have a + b + γ = 1,
..  9 
4 5 γ = 1 – a + b = 1 – [p + p(1 – p)]/
1
9 [1 – q3] = [p – 2p2 + p3]/ 1 – ( 1 – p)3
10. (b) Let P( two tests are needed) = P ( 1st
Hence,
machine is fault) × P( 2nd machine is
faulty) = 2/4 × 1/3 =1/6 a = p/(1 – q3),
b = p(1 – p)/{1 – (q)3}
11. (d) We assume q = 1 – p = probability of
getting the tail i.e P(T). and γ = (p – 2p2 + p3)/{1– (q)3}

Now, a = probability of A getting the 12. (a) Let Pn be the probability that no two
head on tossing firstly ( or more ) consecutive heads occur.

= P ( H1 or T1 T2 T3 H4 or T1 T2 T3 T4 Then Pn relates the probability that 1


T6 H7 or ….) = P (H) + P(H) P(T)3 + or no head occur where n = 1, 2, 3, ...
P(H) P(T)6 + ….. Now, n = 1. Probability P = 1( both
the cases we get less than two heads
= P(H)/{1 – P(T)3} = P/( 1 – q3)
(H,T))
Now b = probability of getting the
Now n = 2, P2 = 1 – p ( two head occur
head on tossing secondly
simultaneously) = 1 – p(HH) = 1 – p2
= P (T1 H2 or T1 T2 T3 T4 H5
Now
or  T1 T 2T 3T 4 T5 T6 H7 or ...) n ≥ 3, Pn = Pn – 1( 1 – P) + Pn – 2( 1 – P)P
332  Theory of Counting

Concept Eliminator (CE)


1. (a) 2. (b) 3. (a) 4. (b) 5. (b)

1. (a) Let India won the tournament, Probability for box 2 is (1/4)3
means India won total 4 matches Probability for box 3 is (1/2)5
out of 6 and Pakistan won only 2
matches, with the condition that And these can be arranged in
India won the last match. (10!)/(2!)(3!)(5!) ways.
So if we look at the result of 1st 5 So required probability is
matches then India won 3 and   2 3 5
10!   1   1   1  
Pakistan won 2 matches and then         
 (2!)(3!)(5!)   4   4   2  
6th match is won by India.
10!
So required probability in this case is  
(2!)(3!)(5!)(2 15 )
 1
(5C3)(2C2)  6  = 5/32
2  4.
(b) In a throw of two dice probability
A similar value we will get if that one will get 12 is 1/36 and will
Pakistan won the tournament- not get 12 is 35/36.
So required probability is As per the given condition in 1st 19
5/32 + 5/32 = 5/16 throw outcome is not 12 and 20th
throw outcome is 12 so required
2. (b) Product is divisible by 10 if and only
19
if at least one of the number selected 35
probability is    1
is 5 and at least one of the selected  36   
36
number is even.
5. (b) Probability that none of them
So required probability is 0 10
are defective is (0.1) (0.99 ) and
n n n
 8  5  4 probability that one of them are
1 –   –   +   
 9  9  9 0 10
defective is ( 10C1) (0.1) (0.99 )
(a) As per the given condition 1st box
3. Required proportion is
will get 2, 2nd will get 3 and 3rd box 0 10
will get 5 balls. 1 – (0.1) (0.99) – (10C1)
Probability for box 1 is (1/4)2 (0.1)0 (0.99)10 = 0.04 (approx)
13
On the Cards  333

On the Cards

Topics Covered

 Introduction of playing cards

 Problems related to playing cards

Part A: Topic Number of Questions


Solved Example 5
Concept Applicator 10
Concept Builder 10
Concept Cracker 10
Part B: Topic Number of Questions
Concept Deviator 5
Concept Eliminator 4
Total 44
334  Theory of Counting
Part A

Concept 1 Introduction of Playing Card

A probability chapter/book/concept is • 52 cards are equally divided into 4 suits


incomplete without a good discussion of with each suit has 13 cards.
questions based on playing cards, so I have • Out of 4 suits, 2 suits namely ‘Diamonds’
decided to dedicate a complete chapter on the and ‘Heart’ are red coloured card and
this concept. ‘Spades’ and ‘Clubs’ are black coloured
Here is some of the features of the playing cards.
cards: • A suit has 13 cards out of which 9 are
• A deck of playing cards has 52 cards. numbered (from 2 to 10) and 4 are face
• Out of 52 cards, 26 are black coloured cards (also known as Honours cards)
and 26 are red coloured Namely Ace, King, Queen and Jack.

Pictorial representation of above classification is shown below:


Tree of Cards

PACK/DECK (52)

RED (26) BLACK (26)

Suits→ Diamonds(13) Hearts(13) Spades(13) Clubs(13)

Example 1:  A card is drawn from a well shuffled pack of cards. What is the probability
that it is a Diamond suit card?
Solution:  Since out of 52 cards, 13 cards belongs to ‘Diamond’ suit so required probability is
13/52 = 1/4.
On the Cards  335
Example 2:  A card is drawn from a well Example 5:  Two cards are drawn from
shuffled pack of cards. What is the a well shuffled pack of cards (with
probability that it is a red coloured replacement). What is the probability
card? that these two cards are either black
Solution:  Since out of 52 cards, 26 cards are coloured or honors cards?
red so required probability is 26/52 = 1/2.
Solution:  Consider the 1st card. Probability
Example 3:  Two cards are drawn from of getting a black coloured card is
a well shuffled pack of cards. What is
the probability that 1st is red coloured P(B) = 26/52 = 1/2.
card and 2nd is black coloured?
Probability of getting a black an honor card is
Solution:  Since out of 52 cards, 26 cards are
red and 26 are black coloured so required P(H) = 16/52 = 4/13
probability is (26/52) × (26/51) = 13/51.
Probability of getting a black coloured honor
Example 4:  Three cards are drawn from card is
a well shuffled pack of cards (with
replacement). What is the probability that P(B ∩ H) = 8/52 = 2/13
at least one of them is black coloured?
Solution:  Since out of 52 cards, 26 cards are P(B ∪ H) = P(B) + P(H) – P(B ∩ H)
red and 26 are black coloured probability
= (1/2) + (4/13) – (2/13) = 17/26
that none of them are black coloured is (1/2)
(1/2)(1/2) = 1/8. 17 17 289
Required probability = × =
So required probability is 1 – 1/8 = 7/8 26 26 676

1
Concept Applicator (CA)
Ideal Time Apply your concepts with easy and
10 Min. conceptual questions

1. A card is drawn from a well shuffled pack (a) 1/13 (b) 1/2
of cards. Find the probability that it is (c) 1/4 (d) None of these
a jack.
4. A card is drawn from a well shuffled pack
(a) 1/13 (b) 1/26 of cards. Find the probability that it is a
(c) 2/52 (d) None of these red ace.
2. A card is drawn from a well shuffled pack (a) 1/13 (b) 1/26
of cards. Find the probability that it is a (c) 5/52 (d) None of these
black numbered card. 5. If two cards are drawn simultaneously
(a) 1/13 (b) 1/2 from a well shuffled pack of cards then
(c) 9/26 (d) None of these find the probability of both being the
King
3. A card is drawn from a well shuffled pack 4
of cards. Find the probability that it is a (a) 2/52 (b) C2 / 52 C2
club . (c) 4/52 (d) None of these
336  Theory of Counting
6. If two cards are drawn simultaneously then find the probability of both being
from a well shuffled pack of cards then numbered card.
find the probability of both being black 8 52 4 52
(a) C2 / C2 (b) C2 / C2
honors. 52
4 (c) 36 C2 / C2 (d) None of these
(a) 8
C2 /
52
C2 (b) C2 / 52 C2
9.
A card is drawn from a pack of cards the
(c) 16 C2 / 52 C2 (d) None of these probability that it is a black Queen is
7. If two cards are drawn simultaneously (a) 1/26 (b) 1/2
from a well shuffled pack of cards (c) 2/5 (d) 3/7
then find the probability of both being 10. Two cards are drawn from a well shuffled
Honours. pack of cards given that both the cards are
4 52 numbered cards. What is the probability
(a) 2/5 (b) C2 / C2
that the cards have same number and
(c) 16 C2 / 52 C2 (d) None of these different colour on them?
8. If two cards are drawn simultaneously (a) 1/35 (b) 1/70
from a well shuffled pack of cards (c) 2/35 (d) None of these

Response Grid
1. a b c d 2. a b c d 3. a b c d 4. a b c d 5. a b c d
6. a b c d 7. a b c d 8. a b c d 9. a b c d 10. a b c d

2
Concept Builder (CB)
Ideal Time Revise your concepts with questions
15 Min. medium difficulty level questions

1. If two cards are drawn at random from a 13 13 13


C1 C1 C2
pack of cards then what is the probability (a)   (b) 2
52 52 52
that one of them is a numbered card and C2 C2 C2
the other is a queen?
13 13
36 C2 C1
C1  ×   4 C1 36 4 (c) +  52 (d) None of these
(a) (b) C1  ×    C1 52
C2 C2
52
C2
36
C1 3. Two cards are drawn simultaneously from
(c) (d) None of these a pack of cards. What is the probability
52
C2 that both are black or both are Ace?
2. If two cards are drawn simultaneously
13 4
from a pack of cards then what is the C1 C1
(a) 
probability that both are diamond or 52
C2 52
C2
both are heart ?
On the Cards  337
13 4 2 8. Four cards are drawn at random, from
C2 C C2
(b) 52 +  52 2 – a well shuffled pack of cards, then what
52
C2 C2 C2
is the probability that the cards are
26 4 2
C2 C2 C2 honours of four different suits?
(c) + –
52 52 52
C2 C2 C2
16 16
C4 C1
(d) None of these (a) (b)
52 52
4. Two cards are drawn at random from a C4 C4
well shuffled pack of cards. What is the
probability that both are spades or both 256
(c) (d) None of these
are clubs? 52
C4
(a) 4/17 (b) 3/17 9. If four cards drawn at random, from a
(c) 2/17 (d) 1/17 well shuffled pack of cards, then what
5. Two cards are drawn at random from a is the probability that three of them are
well shuffled pack of cards. What is the number cards of the same colour and the
probability that both are Jack or both
4th card is a numbered card of different
are red coloured?
(a) 331/1326 (b) 7/13 colour ?
(c) 200/221 (d) 55/221 18
2.18 C 1   18 C 3 C1   18 C3
6. Two cards are drawn at random from a (a) (b) 52
well shuffled pack of cards. What is the
52
C4 C4
probability that both are Heart or neither
is an Ace? 2.18 C 2   18 C 3
(c) (d) None of these
(a) 190/221 (b) 55/221 C4 52

(c) 200/1326 (d) 201/221 10. Two cards are drawn from a well
7. If four cards drawn at random, from a well shuffled pack of cards given that both
shuffled pack of cards, then what is the the cards are numbered cards. What is
probability that all of them are honours?
the probability that the cards have same
16 16
C4 C2
(a) (b) colour on them?
52 52
C4 C4
(a) 17/35 (b) 17/70
12
C4
(c) (d) None of these (c) 34/35 (d) None of these
52
C4

Response Grid
1. a b c d 2. a b c d 3. a b c d 4. a b c d 5. a b c d
6. a b c d 7. a b c d 8. a b c d 9. a b c d 10. a b c d
338  Theory of Counting

3
Concept Cracker (CC)
Ideal Time Boost up your confidence with good
20 Min. questions

1. When three cards are drawn in succession probability that both are queens or both
from a pack of cards with replacement, are hearts?
what is the probability that the first (a) 83/26 (b) 401/663
drawn card is a diamond, second is a red
(c) 14/221 (d) 7/221
card and third an honour?
6. If three cards are drawn at random from
(a) 1/26 (b) 1/39
a well shuffled pack of cards, then what
(c) 1/52 (d) None of these is the probability that all of them are
2. When three cards are drawn simultan- from the same suit?
eously from a pack of cards, what is the 13
C3 4 ⋅13 C3
probability that one of them is a red card (a) (b)
52 52
another a spade and the third is a club? C3 C3
26 13 13
(a)
C1 × C1 × C1 3 ⋅13 C3
52
(c) (d) None of these
C3 52
C3
26
C2 × 13C1 × 13C1 7. When three cards are drawn at random
(b) from a well shuffled pack of cards, then
52
C3
what is the probability that all of them
26 26
C1 × C1 × 13C1 are from the different suits?
(c)
52 13
C3 C3
(a) 52
(d) None of these C3
3. Two cards are drawn at random from a
well shuffled pack of cards. What is the 4 ⋅ (13 C1 ⋅13 C1 13
C1 )
probability that both are club or both are (b) 52
C3
diamonds?
(a) 2/17 (b) 1/17 3 ⋅13 C3 13C1 13C1
(c) 3/17 (d) 4/17 (c)
52
C3
4. Two cards are drawn at random from a
(d) None of these
well shuffled pack of cards. What is the
probability that both are numbered card
or both are black cards? 8. If three cards are drawn at random from
a well shuffled pack of cards, then what
(a) 101/1326 (b) 401/663
is the probability that two of them are
(c) 362/663 (d) 432/663 number cards from the same suit and
5. Two cards are drawn at random from a the 3rd number card is from a different
well shuffled pack of cards. What is the suit?
On the Cards  339
(a) 8/325 (b) 4/325
4 ⋅ ( 9C2 ⋅ 27C1 ) 3 ⋅13 C313C1
(a) (b) (c) 1/65 (d) 2/65
52 52
C3 C3
10. If two cards are drawn at random from
13
C3 a well shuffled pack of cards. Given that
(c) (d) None of these
52
C3 both are red, what is the probability that
9. Two cards are drawn at random from a the cards have different honours and
well shuffled pack of cards. Given that belong to different suits?
both are red, what is the probability
(a) 9/325 (b) 18/325
that the cards have the same honour on
them? (c) 72/325 (d) 12/325

Response Grid
1. a b c d 2. a b c d 3. a b c d 4. a b c d 5. a b c d
6. a b c d 7. a b c d 8. a b c d 9. a b c d 10. a b c d
340  Theory of Counting

Solutions
Concept Applicator (CA)
1. (a) 2. (c) 3. (c) 4. (b) 5. (b) 6. (a)
7. (c) 8. (c) 9. (a) 10. (d)

1. (a) Required probability is 4/52 = 1/13. 7. (c) In one suit we have 4 honors cards so
2. (c) Since in one suit we have 9 numbered total we have 16 honors cards hence
card so number of black numbered required probability is 16 C2 / 52 C2 .
card is 18 and required probability
is 18/52 = 9/26. 8. (c) Total number of numbered cards is
3. (c) In suit Club number of cards is 13. So, 9 × 4 = 36.
required probability is 13/52 = 1/4. So required answer is option (c).
4. (b) Number of red Ace card is 2. 9. (a) Since number of black queen is 2
2 1
So required probability is = hence required probability is
52 26
2/52 = 1/26.
5. (b) Here sample space is 52 C2 and
10. (d) Given that both the cards are
favorable case is 4 C2 . So required
numbered so sample space is
probability is 4 C2 / 52 C2 .
9 × 4 = 36.
6. (a) In one suit we have 4 honors cards
so total we have 8 black honors 1stcard is anything now 2nd card
cards hence required probability is should be of the same number so
8
C2 / 52 C2 . required probability is 2/36 = 1/18.

Concept Builder (CB)


1. (a) 2. (b) 3. (c) 4. (c) 5. (d) 6. (a)
7. (a) 8. (c) 9. (a) 10. (a)

1. (a) Probability that a numbered card 2. (b) Probability that both of them are
and the other is a queen is selected
36 13
C1 × 4C1 diamond is
C2
is . 52
52 C2
C2
On the Cards  341
Probability that both of them are 6.
(a) Probability that both the cards are
13 13
C2 C2 13 × 12
heart is 52 Heart is P(H) = =
52
C2 C2 52 × 51
3. (c) Probability that both the cards are Probability that both the cards are
26 48
C2 C2 48 × 47
black is P(B) = not Ace is P(A') = =
52
52
C2 C2 52 × 51
Probability that both the cards are Probability that both the cards are
4
C2 Heart and they are not Ace is
Ace is P(A) = 12
52
C2 C2 12 × 11
P(H ∩ A') = =  
Probability that both the cards are 52 52 × 51
C2
2
C2 So required probability is
black Ace is P(A ∩ B) = 52
C2 P(H∪A′) = P(H) + P(A′) – P(H ∩ A’)
So required probability is 13 × 12 48 × 47 12 × 11
P(A ∪ B) = P(A) + P(B) – P(A ∩ B) = +  − 
52 × 51 52 × 51 52 × 51
26 4 2
=
C2
+
C2

C2 12 (13 + 188 − 11)
52 52 52 =
C2 C2 C2 52 × 51
4. (c) Probability that both the cards are 12 × 190 190
= =  
13 × 12 1
spades is = 52 × 51 221
52 × 51 17 7. (a) Since total number of honors card
Probability that both the cards are is 16 hence required probability is
13 × 12 1 16
C4
clubs is =
52 × 51 17 52
C4
So required probability is 2/17.
8. (c) Here sample space is 52 and
5. (d) Probability that both the cards are C4
26
C2 26 × 25 favorable case is 4 × 4 × 4 × 4 = 256.
red is P(R) = =
52 256
C2 52 × 51
So required probability is 52
Probability that both the cards are C4
4 52
C2 4×3 9. (a) Here sample space is C4 and
Jack is P(J) = 52 =
C2 52 × 51 18
favorable case is 2. C1    C3 hence 18

Probability that both the cards are red


2
C2 2 ×1 2.18 C1   18 C3
Jack is P(R ∩ J) = =   required answer is
52
52
C2 52 × 51 C4
10. (a) Sample space is 36C and favorable
So required probability is 2
P(R ∪ J) = P(R) + P(J) – P(R ∩ J) case is 2 × (18C 2) so required prob-
26 × 25 4×3 2 ×1
= +  −    ability is
52 × 51 52 × 51 52 × 51
325 + 6 − 1 55 2 × 18 × 17 17
2 × (18C2)/( 36C2) = =
= =   36 × 35 35
26 × 51 221

342  Theory of Counting

Concept Cracker (CC)


1. (a) 2. (a) 3. (c) 4. (c) 5. (c) 6. (c)
7. (c) 8. (d) 9. (d) 10. (a)

1. (a) It is an example of independent Probability that both the cards are


event. 26
C2 26 × 25
black is P(B) = =
Probability that 1st card is a diamond 52
C2 52 × 51
is 13/52 = 1/4
Probability that both the cards are
Probability that 2nd card is a red is
numbered and they are black is
26/52 = 1/2
18
Probability that 3rd card is a honors C2 18 × 17
P(N ∩ B) = =  
is 16/52 = 4/13 52 52 × 51
C2
Required probability is (1/4)(1/2) So required probability is
(4/13) = 1/26. P(N ∪ B) = P(N) + P(B) – P(N ∩ B)
36 × 35 26 × 25 18 × 17
2. (a) Sample space for this case is 52 C3 and = +  −   
52 × 51 52 × 51 52 × 51
favorable case is 26 C1 × 13C1 × 13C1
1604 401
so required probability is = =
2652 663

26 13 13 5. (c) Probability that both the cards are
C1 × C1 × C1
52
C3 queens is

4
C2 4×3
3. (a) Probability that both of them are P(Q) = =
52 52 × 51
13 C2
C213 × 12 1
club is = = Probability that both the cards are
52
C2 52 × 51 17
heart is
Probability that both of them are
13
13 C2 13 × 12
13 × 12C2 1 P(H) = =
diamonds is = = 52
C2 52 × 51
52 52 × 51 17
C2
Probability that both the cards are
So required probability is 2/17. queens and they are heart is 0.
4. (b) Probability that both the cards are
So required probability is
numbered is
4×3 13 × 12 14
36 + =
C2 36 × 35 52 × 51 52 × 51 221
P(N) = =
52 52 × 51
C2
On the Cards  343
52 52
6. (b) In this case sample space is C3 (d) In this case sample space is C3 and
8.
13
and favorable case is 4 ⋅ C3 so favorable case is 12 ⋅ ( 9 C2 ⋅ 27C1 )
so required probability is
4 ⋅13 C3
required probability is . 12 ⋅ ( 9 C2 ⋅ 27
C1 )
52
C3
52
C3
7. (b) In this case sample space is 52
C3 and 9. (b) Sample space is 26C2 and number of
favorable case is 4.
13 13 13
favorable case is 4 ⋅ ( C1 ⋅ C1· C1 )
Required probability is
so required probability is
(4 × 2)/(26 × 25) = 4/325.
10. (d) Required probability is
4 ⋅ (13 C1 ⋅ 13C1 · 13
C1 )
. (8/26)(3/25) = 12/325.
52
C3
344  Theory of Counting
Part B
Introduction of
Concept 1 Playing Cards
Here is some of the features of the playing • Out of 4 suits, 2 suits namely ‘Diamonds’
cards: and ‘Heart’ are red colored card and
• Deck of playing cards has 52 cards. ‘Spades’ and ‘Clubs’ are black colored
• Out of 52 cards, 26 are black coloured cards.
and 26 are red coloured. • A suit has 13 cards out of which 9 are
• 52 cards are equally divided into 4 suits numbered (from 2 to 10) and 4 are face
with each suit has 13 cards. cards (also known as Honours cards)
Namely Ace, King, Queen and Jack.

Pictorial representation of above classification is shown below:

Tree of Cards

PACK/DECK (52)

RED (26) BLACK (26)

Suits→ Diamonds(13) Hearts(13) Spades(13) Clubs(13)


On the Cards  345

1
Concept Deviator (CD)
Ideal Time Revise your concept (IIT advance or
5 Min. tougher questions)
per question

1. Two cards are drawn from set of 52 cards, [A  FULL HOUSE consists of 3  cards of
what is the probability that the 2nd card the same kind (eg, 3 Kings) and 2 cards
is Jack if it is given that both the cards of another kind (eg, 2 Aces)]
are honors cards?
6 4
(a) (b)
(a) 1/6 (b) 3/16 4165 4165
(c) 1/4 (d) None of these 3
(c) (d) None of these
2. Three cards are randomly selected, 4165
without replacement, from a deck of 52
5. Cards are drawn one by one at random
playing cards. What is the probability that
from well shuffled full pack of 52 playing
the first card selected is a club given that
cards until 2 aces are obtained for the
the second and third cards are clubs.
first time. If N is the number of aces
(a) 11/60 (b) 11/50 required to be drawn, then which one of
(c) 11/40 (d) None of these the following is correct if 2 ≤ n ≤ 50.
3. What is the probability of randomly [IIT JEE 1983]
drawing five cards from a deck has (51 − n)(52 − n)(n − 1)
exactly one Ace? (a)
13 × 17 × 49 × 50
(a) 0.3127 (b) 0.299
(b)
(51 − n)(52 − n)
(c) 0.273 (d) None of these 13 × 17 × 49 × 50
4. What is the probability of getting a (51 − n)(52 − n)(n − 1)
“FULL HOUSE” in five cards drawn in (c)
13 × 17 × 49
a poker game from a standard pack of
(d) None of these
52-cards?

Response Grid
1. a b c d 2. a b c d 3. a b c d 4. a b c d 5. a b c d
346  Theory of Counting

2
Concept Eliminator (CE)
Ideal Time Revise your concept (Maths Olympiad
Just solve it or tougher questions)

1. An ordinary deck of 52 playing cards 3. If 3 identical cards are coloured both the
is randomly divided into 4 groups of 13 sides such that the first card are colored
cards each. What is the probability that red, both sides of the second card are
each group has exactly 1 jack? colored black, and one side of the third
39 × 26 39 × 26 × 13 card is colored red and the other side
(a) (b)
51 × 50 51 × 50 × 49 black. The 3 cards are mixed up, and 1
39 × 26 × 11 card is randomly selected and put down
(c) (d) None of these on the ground. If the upper side of the
51 × 50 × 49
chosen card is colored black, what is the
2. In an examination that has multiple
probability that the other side is colored
choice questions with m options, a
red?
student either know the answer or guess
the answer. While guessing an option (a) 1/2 (b) 2/3
each option is of equal probability of (c) 1/3 (d) None of these
being correct. What is the conditional 4. In a poker game what is the probability
probability that a student knew the of getting a “pair” in five cards?
answer to a question given that he or she [A  pair  consists of 2  cards of the same
answered it correctly? It is known that kind (eg, 2  Kings) and 3  cards that are
p be the probability that the student different from the kind of the pair (eg,
knows the answer of a question. different from Kings) and that are all
mp p different from each other.]
(a) (b)
{1 + (m − 1)p} {1 + (m − 1)p} (a) 0.321 (b) 0.422
m (c) 0.329 (d) None of these
(c) (d) None of these
{1 + (m − 1)p}

Response Grid
1. a b c d 2. a b c d 3. a b c d 4. a b c d
On the Cards  347

Solutions
Concept Deviator (CD)
1. (c) 2. (b) 3. (b) 4. (a) 5. (a)

1. (c) Since both the cards are honors cards the deck and there are 13  different
hence sample space is 4 × 4 = 16. kinds of cards, so the total number
Here we have two cases for the 2nd of combinations possible of 2  cards
cards being a jack is 6  ×  13  =  78. There are 4  ways
Case (i) 1st card is a jack and 2nd card to choose 3  cards of the same kind
 4  3 from 4  cards of the same kind, but
is a jack the probability is    
 16   15  because the 3-of-a-kind suit must be
Case (ii) 1st card is not a jack and different from the 2-of-a-kind suit,
2nd card is a jack the probability is the possible combinations of this is,
 12   4  4 × 12 = 48.
   
16 15
Required probability is So total number of ways is
 4   3   12   4  1
    +      = 48 × 78 = 3744
16 15 16 15 4
Sample space is 52C5 = 2,598,560
2. (b) Required probability = P(all the 3
cards are Club)/P((2nd and 3rd cards Required Probability
are Club)
3744 6
 13 C 3  = =
 52  2598560 4165
 C 3 
= 13 5. (a) Since as per the given condition Nth
 C 2 
 52  draw gives 2nd Ace that means in 1st
 C 2  (N-1) draw only one Ace will appear

=
{(13 × 12 × 11) × (52 × 51)}= 11 and next card is Ace, so required
{(13 × 12 ) × (52 × 51 × 50)} 50 probability

3. (b) Required probability is =
{4 (48!)(n − 1)!(53 − n)!}× 3
(4C1)(48C4)/(52C5) = 0.299 (approx.) {52! (n − 2 )! (50 − n)!} 53 − n
(51 − n)(52 − n)(n − 1) 
4. (a) There are 6 ways to select 2 cards of =
13 × 17 × 49 × 50
the same kind from the 4  cards in
348  Theory of Counting

Concept Eliminator (CE)


1. (b) 2. (a) 3. (c) 4. (b)

1. (b) Probability that 1st group has only p mp


one jack is =
 4!  48!  
  1 
 p +   (1 − p )
{1 + (m − 1)p}
     m 

( C1 )( C12 )  3!   (12!)(36!) 
4 48
52
= 3. (c) Let the three cards represent as
C13  52!  (RR), (BB), (BR)
 
 (13!)(39!) In this case we have to find the
conditional probability
 4!  (39 × 38 × 37 × 13) 
=  P (BR|B)
 3!   (52 × 51 × 50 × 49) 
= (39 ×  38 ×  37) /(51 ×  50 ×  49)  {P (B|BR)P (BR)}
=
 P (B|BB)P (BB)+  P (B|BR )P (BR )
Similarly probability that 2nd group  
has a jack is: +  P (B|RR )P (RR ) 
 3!   36!    1  1
       
( C 1 )( C 12 )  2!  (12!)(24!) 
3 36
=
2 3
=
1
=   1  1  1 3
39  39!   1 
C 13
  (1)  +      +  (0 )  
3 2 3 3 
 (13!)(26!)

Alternately:
 3!  Let the three cards represent as
=   {(26 × 25 × 13) /(39 × 38 × 37)}
 2! (RR), (BB), (BR)
= (26 × 25) /(38 × 37)  If one face is red then we have

following cases-
Similarly probability that 3rd group
has a jack is 13/25 Front side Back Side
Then remaining 4th group must have Black Black
a jack Black Black
Required probability is Black Red
39  × 38 × 37 26 × 25 13 Out of three possible cases only
× ×
51 × 50  × 49 38 × 37 25 one case is favorable so required
39 × 26 × 13 probability is 1/3
= 4. (b) Total number of ways of selecting 2
51 × 50 × 49
cards (6 × 13) and that for 3rd, 4th
2. (a) He can answer the question correctly and 5th card is 48, 44 and 40 ways,
in two cases: so total number of favourable cases
Case (1): Probability that student is (78 × 48 × 44 × 40).
knows the answer = p. Sample space is 52C5 = 2,598,560
Case (2): Probability when he made Required Probability
a guess = (1 – p)/m.
78 × 48 × 44 × 40
So required probability is = = 0.422 (Approx.)
2598560
14
Probability Miscellaneous  349

Probability
Miscellaneous

Topic Covered

 Revision of Permutation and Combination with Probability

Part A: Topic Number of Questions

Concept Applicator 15
Concept Builder 15
Concept Cracker 20
Part B: Topic Number of Questions
Concept Deviator 25
Concept Eliminator 7
Total 82
350  Theory of Counting
Part A
So finally we arrive at the end and the to the same concept and that is concepts of
concluding chapter of this book, here we counting.
will see many problems of permutation and In this chapter we will discuss the questions
combination can be converted to a problem of previous chapters after converting them to
of probability after all both of them belongs the questions of probability.

1
Concept Applicator (CA)
Ideal Time Apply your concepts with easy and
15 Min. conceptual questions

1. If 11 players be selected out of 20 players 6. In a class of 10 students 6 are boys


then what is probability that Sachin and and 4 are girls, if class teacher selects
Sourav are always rejected? student/s for a project then what is the
(a) (18C11)/(20C11) (b) (18C09)/(20C11) probability that the group has at least 1
(c) ( 19C10)/(20C11) (d) None of these boy and 1 girl?
2. If 5 distinct alphabets are selected from (a) 945/1023 (b) 1022/1023
English alphabets then what is the (c) 512/1023 (d) None of these
probability that no vowels is selected?
7. From a system of 10 points that form a
(a) ( 26C5) /(25C5) (b) ( 21C5) /(26C5)
polygon of 10 sides one line is selected at
(c) ( 20C5) /(26C5) (d) None of these random what is the probability that this
3. Out of 6 apples, 5 mangoes and 4 bananas line is a diagonal?
one or more fruits are selected then what
(a) 5/9 (b) 7/9
is the probability that at least one mango
is always selected? (c) 4/9 (d) None of these
(a) 175/209 (b) 120/209 8. If 4 volumes of English and 6 volume of
(c) 119/209 (d) None of these Maths book are arranged on a bookshelf
then what is the probability that all the
4. Out of 6 apples, 5 mangoes and 4 bananas
one or more fruits are selected then what English books and all the Maths books
is the probability that at least one fruit are together?
of each type is always selected? (a) (4!)(6!)/(10!) (b) 2(4!)(6!)/(10!)
(a) 175/209 (b) 120/209 (c) (6! +4!)/(10!) (d) None of these
(c) 119/209 (d) None of these 9. If ‘n’ boys and ‘n’ girls, can be arranged in a
5. In an examination a student has to straight line then what is the probability
write 6 papers and has to pass in each that no two boys are together?
paper to pass the exam then what is the
2(n !)2 (n !)2
probability that a student is failed in the (a) (b)
exam? 2n ! 2n !
(a) 31/64 (b) 64/65 4(n !)2
(c) (d) None of these
(c) 63/64 (d) None of these 2n !
Probability Miscellaneous  351
10. If 10 students can be seated around a Direction from question number 13 to 15:
circular table having 10 seats then what Letters of the word CREATIVITY is arranged
is the probability that two students A to form 10 letter word then:
and B are never together?
(a) 7/9 (b) 2/9 13. What is the probability that the word
(c) 4/9 (d) None of these formed starts with letter C?
11. If 7 boys and 8 girls are seated around a (a) 1/10 (b) 1/12
circular table then what is the probability
that all the boys do not sit together? (c) 1/4 (d) None of these

(a)
(7!)× (8!) (b)
7! + 8! 14. What is the probability that the word
14! 14! formed starts with letter C and ends
with Y?
(c)
{(14!) – 7!
( )(8!)}
(d) None of these
14! (a) 1/10 (b) 1/12
12. A round table conference is to be held (c) 1/4 (d) None of these
between 20 delegates of 20 countries
then what is the probability that 2 15. What is the probability that the word
particular delegates Mr. A and Mr. B formed has all the vowels are together?
will sit together?
(a) 1/10 (b) 1/12
(a) 1/19 (b) 2/19
(c) 3/19 (d) None of these (c) 1/4 (d) None of these

Response Grid
1. a b c d 2. a b c d 3. a b c d 4. a b c d 5. a b c d
6. a b c d 7. a b c d 8. a b c d 9. a b c d 10. a b c d
11. a b c d 12. a b c d 13. a b c d 14. a b c d 15. a b c d

2
Concept Builder (CB)
Ideal Time Revise your concepts with questions
25 Min. medium difficulty level questions

1. If 10 students are selected from a group of (c) (20C10 – 18C8 – 1)/(20C10)


20 students then what is the probability (d) None of these
that Rajesh and Dinesh are not selected 2. If one or more students are selected from
together? a group of 6 students then what is the
(a) (20C10 – 19C9 – 18C8 )/(20C10) probability that at least 4 students are
(b) ( 20C10 – 18C8 + 1)/(20C10) selected?
352  Theory of Counting
(a) 21/64 (b) 20/63 8. If 10 students of different height are
(c) 22/63 (d) None of these arranged in a straight line then what
is the probability that the tallest and
3. In a board exam there are two sections
shortest are never together?
each section has 5 questions. As per
the given condition a candidate has (a) 3/5 (b) 1/5
to answer any 6 questions out of 10 (c) 4/5 (d) None of these
questions. What is the probability that a 9. In how many ways 5 boys and 5 girls can
student answered 6 questions such that be seated in a row so that boys and girls
not more than 4 questions selected from are alternate?
one section?
(a) 10/21 (b) 16/21
2 (5!)(5!) (5!)(5!)
(a) (b)
10! 10!
(c) 20/21 (d) None of these
4. From 8 boys and 4 girls a team of 5 is to be 2 (5! +  5 !)
(c) (d) None of these
formed. In how many ways can this be done 10!
so as to include at least 1 girl in the team? 10. What is the probability that A is always
(a) (12C5 – 8C5 + 1 /(12C5) ahead of B when 10 students are arranged
(b) (12C5 – 8C5/(12C5) in a row?
(c) (12C5 – 8C5 – 1/(12C5) (a) 3/5 (b) 1/5
(d) None of these (c) 4/5 (d) None of these
5. There are 6 questions in an examination, a 11. If 5 distinct Apples and 4 distinct
student has to answer at least three question bananas are arranged in a row then
to pass the exam, what is probability that a what is the probability that no 2 apples
student will fail the exam? are together?
(a) 5/16 (b) 31/64
(a) 0 (b)
(5!)(4!)
(c) 21/32 (d) None of these
9!
6. If 5 Indians, 6 Americans and 4 Russians
be seated in a row then what is the (c)
{(9!) – 4!
( )(5!)}
(d) None of these
probability that all persons of the same 9!
nationality sit together? 12. If 20 students can be seated around a
3!5!6!4! 5!6!4! circle then what is the probability that 3
(a) (b)
15! 15! students A, B, C are always together?

(c)
(14!) (d) None of these
(a) 6/19 (b) 2/19

15! (c) 1/57 (d) None of these
7. If 10 boys and 8 girls are arranged in a 13. In an international convention, 5
straight line then what is the probability participants from each of USA, China
that two girls Seema and Reema are
and Russia were arranged around a circle
together?
then what is the probability that no two
(a)
(16)(17!)(2!) (b)
(16!)(17)(2!) participants from USA are together?
18! 18!
(a) (9!)(10P5)/14! (b) (9!)(10C5)/(14!)
(c)
(16!)(17!)(2!) (d) None of these (c) (3!)(5!)(5!)(5!)/(14!) (d) None of these
18!
Probability Miscellaneous  353
14. In a family gathering 4 couples were to 15. Letters of the word INTERMEDIATE
sit around a circular table then what is are arranged then what is the probability
the probability that no two males as well that in a randomly selected word no
as no couple is together? vowel is between two consonants?
(a) 6/(7!) (b) 12/(7!) (a) 1/11 (b) 1/33
(c) 18/(7!) (d) None of these (c) 1/66 (d) None of these

Response Grid
1. a b c d 2. a b c d 3. a b c d 4. a b c d 5. a b c d
6. a b c d 7. a b c d 8. a b c d 9. a b c d 10. a b c d
11. a b c d 12. a b c d 13. a b c d 14. a b c d 15. a b c d

3
Concept Cracker (CC)
Ideal Time Boost up your confidence with good
40 Min. questions

1. In Independence Day party every student (a) (10C2 × 8C2 )/(2 × 10C2 × 10C2)
gave a gift to every other student. Total (b) (10C2 × 8C2)/( 10C2 × 10C2)
number of gift distributed is 240 if a (c) (10C2 × 6C2)/(20C4)
gift is selected at random then what is
(d) None of these
the probability that this gift belongs
to Rakesh (received by a participant 4. In a group of 10 students comprising 6
Rakesh)? boys and 4 girls, what is the probability
that if a team of 5 is to be formed then
(a) 1/15 (b) 1/16 number of girls is always more than the
(c) 1/18 (d) None of these number of boys?
2. If teams of 11 players is to be made from (a) (4C3 × 6C2) × (4C4 × 6C1) /(10C5)
25 players then what is the probability (b) (4C3 × 6C1 + 4C4 × 6C1 )/(10C5)
that the players Rajesh, Rajni and Riju (c) (4C3 × 6C2+ 4C4 × 6C1)/(10C5)
are not selected together? (d) None of these
(a) ( 25C11– 22C8 )/(25C11) 5. In a function 10 singers were selected to
(b) ( 25C11 – 22C11)/(25C11) perform, but at a time only 3 singers can
(c) ( 25C11– 25C8)/(25C11) perform with a condition that no same
group of 3 singers performed twice, then
(d) None of these
in a randomly selected performance what
3. In a group picnic 10 married couples is the probability that particular singer
planned to play mixed doubled game in Gunjan will perform?
tennis. In a game, what is probability (a) (10C2 )/(10C3) (b) (9C2)/(10C3)
that no husband wife pair is participating
together ? (c) (9C1 × 8C1)/(10C3) (d) None of these
354  Theory of Counting
6. Total 16 people participated in a picnic, 6 18!
peoples are to be selected for a work then (c) –1
what is the probability that only one
(5!)(6!)(7!)
from Mr & Mrs A is selected similarly (d) None of these
one from Mr & Mrs B and Mr & Mrs C is 11. If 2 Indians, and 2 Americans are
selected for the work? arranged in a row then what is the
probability that no two participants from
(a) 960 /(16C6) (b) 480/(16C6)
the same country are together?
(c) 120/(16C6) (d) None of these
(a) 1/3 (b) 1/2
7. If 2 identical rooks are placed on a (c) 1/6 (d) None of these
chessboard what is the probability that
12. There are 2 Indians, 2 Americans and
they are not in attacking positions, if
2 Russians in a conference, if they are
rooks can attack only in a same row or in
arranged in a straight line then what is
a same column? the probability that no two participants
(a) 4/9 (b) 5/9 from the same country are together?
(c) 7/9 (d) None of these (a) 1/3 (b) 1/2
8. If 10 students are arranged in a row then (c) 1/6 (d) None of these
what is the probability that A is ahead of 13. Total 7 boys and 6 girls are selected
B and who in turn is ahead of C ? to represent their school in science
(a) 1/5 (b) 1/6 conference and they are arranged in a
line then what is the probability that 3
(c) 1/3 (d) None of these
girls are together and other three are
9. There are 100 students with roll number also together but these two group of girls
1 to 100. If 10 students are selected are not together?
out of 100 students and their names
are listed down according to their roll (a)
(5040)(8!) (b)
(2520)(8!)
numbers in increasing order then what 13! 13!
is the probability that rank 50 will be at
(c)
(1260)(8!) (d) None of these
6th position? 13!
(a) (49P5)(50P4) /(100P10) 14. A group of students comprising 3 girls
(b) (49C 5)(
50C
4 )/(100P10)
and 5 boys went for a picnic. During a
(c) (49!)(50!)/(100P10) game they arranged in a circle then what
is the probability that each boy has one
(d) None of these girl on at least one side?
10. If 5 identical apples, 6 identical bananas (a) 1/21 (b) 1/14
and 7 identical oranges are arranged in a (c) 1/7 (d) None of these
straight line then what is the probability
15. Rishita has 5 distinct red and 5 distinct
that at least one fruit is separated from
white flowers with that she made garland
similar fruits?
then what is the probability that the
(a)
(5!)(6!)(7!) garland has no flowers of same colour
18! are together?

(b)
(5!)(6!)(7!)(18) (a) 24(5!)/(9!) (b) 12(5!)/(9!)
18! (c) 6(5!)/(9!) (d) None of these
Probability Miscellaneous  355
16. In Praxis Business school during a 18. There are 40 distinct pearl, out of which
meeting professors from different 20 are of blue coloured and remaining
department were to sit around a circular 20 are of red coloured, a necklace is
table. If number of professors from formed from these pearls then what is
Marketing, Finance, Operations, and the probability that no two beads of same
HR are 4, 5, 6 and 7 in any order. What colour are together?
is the probability that all the professors
from the same department are together? (a)
{(40!) – 2 (19!)(20!)}
39!
(a)
(2!)(4!)(5!)(6!)(7!) {(39!) – 2 (19!)(20!)}
21! (b)
39!
(b)
(3!)(4!)(5!)(6!)(7!) (19!)(20!)
(c)
21! 2 × 39!

(c)
(4!)(5!)(6!)(7!) (d) None of these
21! 19. Letters of the word MATHEMATICS are
(d) None of these arranged in all the possible ways, and a
17. In Praxis Business school during a word is selected randomly then what is
meeting professors from different probability that letter C is between S and
department were to sit around a circular H (these three letters are not necessary
table. If number of professors from together)?
Marketing, Finance, Operations, and (a) 1/5 (b) 1/6
HR are 5 each. What is the probability
(c) 1/3 (d) None of these
that all the professors from the same
department are together and HOD of 20. Letters of the word MATHEMATICS are
any department is exactly between the arranged in all the possible ways, , and
professors of that department? a word is selected randomly then what
is probability that letter C is exactly
(a)
(3!)(4!)4 (b)
(4!)4 between S and H ?
19! 19!
(a) 1/5 (b) 1/6
(c)
(4 )(4!)4 (d) None of these (c) 1/3 (d) None of these
19!

Response Grid
1. a b c d 2. a b c d 3. a b c d 4. a b c d 5. a b c d
6. a b c d 7. a b c d 8. a b c d 9. a b c d 10. a b c d
11. a b c d 12. a b c d 13. a b c d 14. a b c d 15. a b c d
16. a b c d 17. a b c d 18. a b c d 19. a b c d 20. a b c d
356  Theory of Counting

Solutions
Concept Applicator (CA)
1. (a) 2. (b) 3. (a) 4. (b) 5. (c) 6. (a)
7. (b) 8. (b) 9. (a) 10. (a) 11. (c) 12. (b)
13. (a) 14. (d) 15. (d)

1. (a) This question is based on question 9.


(a) This question is based on question
number 7 of CA of chapter 2. number 14 of CA of chapter 4.
Required answer is option (a) Here sample space is (2n!) so
2. (b) This question is based on question 2
2 (n !)
number 10 of CA of chapter 2. required probability is
2n !
Required answer is option (b) Required answer is option (a).
3. (a) This question is based on question 10. (a) This question is based on question
number 11 and 12 of CA of chapter 2. number 3 of CA of chapter 5.
Required answer is option (a) Required probability
4. (b) This question is based on question {(9!) – (2!)(8!)} 7
number 11 and 12 of CA of chapter 2. = =
9! 9
Required answer is option (b) Required answer is option (a).
5. (c) This question is based on question 11. (c) This question is based on question
number 14 of CA of chapter 2. number 7 of CA of chapter 5.
Required answer is option (c), Required answer is option (c).
12. (b) This question is based on question
6. (a) This question is based on question
number 13 of CA of chapter 5.
number 16 and 17 of CA of chapter 2.
Required answer is option (b).
Required answer is option (a) 13. (a) This question is based on question
7. (b) This question is based on question number 4 of CA of chapter 6.
number 15 of CA of chapter 3. Here sample space is (10!)/(2!)(2!)
Number of diagonals is 35, and Required probability is
number of sides is 10 so required  9! 
probability is 35/45 = 7/9  
 (2!)(2!) 1
Required answer is option (b) =
 10!  10
8. (b) This question is based on question  
number 8 and 9 of CA of chapter 4.  (2!)(2!)
Required answer is option (b), Required answer is option (a)
Probability Miscellaneous  357
14. (d) This question is based on question 15. (d) This question is based on question
number 5 of CA of chapter 6. number 6 of CA of chapter 6.
Required probability is
Required probability is
 8! 

 (2!)(2!)

1 {6(7!)} =
1
= 30
 10!  90  10! 
 
 
 (2!)(2!)  (2!)(2!)

Required answer is option (d). Required answer is option (d).

Concept Builder (CB)


1. (d) 2. (c) 3. (c) 4. (b) 5. (c) 6. (a)
7. (b) 8. (c) 9. (a) 10. (d) 11. (b) 12. (c)
13. (a) 14. (b) 15. (c)

1. (d) This question is based on question 8. (c) This question is based on question
number 6 of CB of chapter 2. number 8 of CB of chapter 4.
Required answer is option (d). Required probability is 8(9!)/(10!)
2. (c) This question is based on question = 4/5
number 8 of CB of chapter 2.
Required answer is option (c).
Required answer is option (c).
3. (c) This question is based on question 9. (a) This question is based on question
number 11 of CB of chapter 2. number 9 of CB of chapter 4.
Here sample space is Required answer is option (a).
10C = (10!)/(6!)(4!)
6 10. (d) This question is based on question
= (7 × 8 × 9 × 10)/(24) = 210 number 13 of CB of chapter 4.
Required answer is 200/210 = 20/21. Required probability is 1/2.
4. (b) This question is based on question
Required answer is option (d).
number 12 of CB of chapter 2.
Required answer is option (b). 11. (b) This question is based on question
number 15 of CB of chapter 4.
5. (c) This question is based on question
number 19 of CB of chapter 2. Required answer is option (b).
Here sample space is 26 = 64 12. (c) This question is based on question
Required answer is option (c). number 2 of CB of chapter 5.
6. (a) This question is based on question Required answer is option (c).
number 3 of CB of chapter 4.
13. (a) This question is based on question
Required answer is option (a).
number 10 of CB of chapter 5.
7. (b) This question is based on question
number 6 of CB of chapter 4. Required probability is (9!)(10P5)/(14!)
Required answer is option (b). Required answer is option (a).
358  Theory of Counting
14. (b) This question is based on question Required probability is
number 14 of CB of chapter 5.  (5!)(7!) 
Required answer is option (b).  
 2   (5!)(7!)(2!)(3!)(2!)
15. (c) This question is based on question  12!  (2)(12!)
number 8 of CB of chapter 6.  
 (2!)(3!)(2!) 
The word INTERMEDIATE has 2 120  6  2
I’s, 3 E’s 2 T’s and remaining are 
single letters so sample space is 8  9  10  11  12
12! 1

(2!)(3!)(2!) 66

Concept Cracker (CC)

1. (b) 2. (a) 3. (b) 4. (c) 5. (b) 6. (a)


7. (c) 8. (b) 9. (a) 10. (d) 11. (a) 12. (a)
13. (b) 14. (c) 15. (a) 16. (b) 17. (a) 18. (c)
19. (b) 20. (b)

1. (b) This question is based on question case is given by answer of question


number 3 of CC of chapter 2. number 15.
Required answer is option (b). Required answer is option (b).
2. (a) This question is based on question 6. (a) This question is based on question
number 9 of CC of chapter 2. number 23 of CC of chapter 2.
Required answer is option (a). Required answer is option (a).
3. (b) This question is based on question 7. (c) This question is based on question
number 11 of CC of chapter 2. number 20 of CC of chapter 3.
Here sample space is selecting 2 male Here sample space is (64 × 63)/(2!)
and 2 female persons out of 10 males Required probability is
and 10 females (in 10C2 × 10C2 ways)
(64 × 49)/(64 × 63) = 7/9
and then arranging mixed double in
(in 2 ways) so required sample space Required answer is option (c).
is 2 × 10C2 × 10C2. 8. (b) This question is based on question
Required answer is option (b). number 2 of CC of chapter 4.
4. (c) This question is based on question Required answer is option (b).
number 13 of CC of chapter 2. 9. (a) This question is based on question
Required answer is option (c). number 7 of CC of chapter 4.
5. (b) This question is based on question Required answer is option (a).
number 14 and 15 of CC of chapter 2. 10. (d) This question is based on question
Sample space is given by answer of number 9 of CC of chapter 4.
question number 14, and favorable Required answer is option (d).
Probability Miscellaneous  359
11. (a) This question is based on question 15. (a) This question is based on question
number 13 of CC of chapter 4. number 9 of CC of chapter 5.
Required probability is 8/24 = 1/3 Required probability is 24(5!)/(9!).
Required answer is option (a).
Required answer is option (a).
16. (b) This question is based on question
12. (a) This question is based on question number 11 of CC of chapter 5.
number 14 of CC of chapter 4. Required answer is option (b).
Required probability is 2(5!)/6! = 1/3 17. (a) This question is based on question
number 12 of CC of chapter 5.
Required answer is option (a).
Required answer is option (a).
13. (b) This question is based on question
18. (c) This question is based on question
number 16 of CC of chapter 4.
number 14 of CC of chapter 2.
(2520)(8!) Required answer is option (c).
Required probability
13! 19. (b) This question is based on question
Required answer is option (b).
number 1 of CC of chapter 6.
14. (c) This question is based on question Required answer is option (b).
number 8 of CC of chapter 5.
20. (b) This question is based on question
Required probability is 720/(7!) = 1/7 number 2 of CC of chapter 6.
Required answer is option (c). Required answer is option (b).
360  Theory of Counting
Part B
1
Concept Deviator (CD)
Ideal Time Revise your concept (IIT advance or
5 Min. tougher questions)
Per Question

1. In a multiple choice question there are 5. Suppose the probability for A to win
four alternative answers, of which one game against B is 0.4. If A has an option
or more is correct. A candidate will get of playing either “best of 3 games” or
marks in the question only if he ticks a “ best of 5 games” match against B,
which option should A choose so that
all the correct answers. The candidate
his probability of winning is higher? (No
decides to tick answers at random. If he
game ends in a draw) [IIT JEE 1989]
allowed up to three chances to answer
(a) Best of 3 (b) Best of 5
the question, find the probability that we
(c) Equal probability (d) None of these
will get marks in the question.
[IIT JEE 1985] 6. A is a set containing n elements. A subset
P of A is chosen at random. The set A is
(a) 1/5 (b) 3/5 reconstructed by replacing the elements of P.
(c) 2/5 (d) 1 A subset Q of A is again chosen at random.
Find the probability that P and Q have no
1   3 p 1 – p 1 – 2 p common elements. [IIT JEE 1989]
2. If , and are the
3 4 2 n n
 1  3
probabilities of the three mutually (a)   (b)  
 4  4
exclusive events, then the set of all n
values of p is [IIT JEE 1986]  2
(c)   (d) None of these
 3
(a) –1/3 ≤ p  ≤ 1 / 2 (b) 1/3 ≤ p  ≤ 1 / 2
7. Three faces of a fair die are yellow, two
(c) 1/4 ≤ p  ≤ 1 / 2 (d) None of these faces red and one blue. The die is tossed
3. The probability that at least of the events three times. The probability that the colours
A and B occur is 0.6. If A and B occur yellow, red and blue appear in the first,
simultaneously with probability 0.2 then second and the third tosses respectively, is
__________. [IIT JEE 1992]
P(A′) + P(B′) is [IIT JEE 1987]
(a) 1/12 (b) 1/24
(a) 0.4 (b) 0.8
(c) 1/36 (d) None of these
(c) 1.2 (d) None of these 8. An unbiased die with faces marked, 1, 2, 3,
4. For two given events A and B, P(A ∩ B) 4, 5 and 6 is rolled four times. Out of four
is [IIT JEE 1988] face values obtained, the probability that
the minimum face value is not less than 2
(a) Not less than P(A) + P(B) – 1 and the maximum face value is not greater
(b) Not greater than P(A) + P(B) than 5 is then, [IIT JEE 1993]

(c) Equal to P(A) + P(B) – P(A ∪ B) (a) 16/81 (b) 1/81

(d) Equal to P(A) + P(B) + P(A ∪ B) (c) 80/81 (d) 65/81


Probability Miscellaneous  361
9. Numbers are selected at random, from (a) 91/792 (b) 101/792
the two digit numbers 00, 01, 02,….., 99 (c) 193/792 (d) None of these
with replacement. An event E occurs if
13. Let 0 < P(A) < 1, 0 < P(B) < 1 and P(A ∪ B)
and only if the product of the two digits of
= P(A) + P(B) – P(A) P(B). Then
a selected number is 18. If four numbers
[IIT JEE 1995]
are selected, find the probability that the
event E occurs at least 3 times. (a) P(B – A) = P(B) – P(A)
[IIT JEE 1993] (b) P(A′ ∪ B′) = P(A′) + P(B′)
97 98
(a) (b) (c) P(( A ∪ B)′) = P(A′) P(B′)
25 4 25 4
(d) P(A|B) = P(A)
151
(c) (d) None of these
254 14. For the three events A, B and C, P
10. If two events A and B such that (exactly one of the events A or B occurs)
P(A′) = 0.3, P(B) = 0.4 and P(A ∩ B′) = 0.5 = P(exactly one of the events B or C
then P(B|(A ∪ B′)) = [IIT JEE 1994] occurs) = P(exactly one of the events B
(a) 1/2 (b) 3/4 or C occurs) = P(exactly one of the events
(c) 3/5 (d) None of these C or A occurs) = p and P( all the three
events occur simultaneously) = p2, where
11. Let A, B, C be three mutually independent
0 < p < 1/2 . Then the probability of at
events. consider the two statements S1
and S2. [IIT JEE 1994] least one of the three events A, B and C
occurring is [IIT JEE 1996]
S1 : A and B ∪ C are independent
S2 : A and B ∩ C are independent 3 p  2 p2 p  +  3 p 2
(a) (b)
Then 2 4
2
(a) Both S1 and S2 are true p  +  3 p 3 p  +  2 p 2
(c) (d)
(b) Only S1 is true 2 4
15. Three numbers are chosen at random
(c) Only S2 is true without replacement from {1, 2, ..., 10}.
(d) Neither S1 nor S2 is true The probability that the minimum of the
12. An unbiased coin is tossed. If the result chosen numbers is 3, or their maximum
is head, a pair of unbiased dice is rolled is 7, is: [IIT JEE 1997]
and the number obtained by adding the (a) 7/40 (b) 9/40
number on the two faces is noted. If the (c) 13/40 (d) 11/40
result is a tail, a card from a well shuffled
16. If p and q are chosen randomly from
pack of eleven cards numbered 2, 3, 4, …
the set {1, 2, 3, 4, 5, 6, 7, 8, 9, 10} with
12 is picked and the number on the card
replacement determine the probability
is noted. What is the probability that the
that the roots of the equation x2 + px q = 0
noted number is either 7 or 8?
[IIT JEE 1994] are real. [IIT JEE 1997]
362  Theory of Counting
(a) 61/100 (b) 62/100 1 white and 3 black balls, one ball is
drawn at random, then the probability
(c) 63/100 (d) 64/100 that 2 white and 1 black ball will be
Direction for question number 17 and 18: drawn is        [IIT JEE 1998]
Sixteen players S1, S2, ..., S16 play in a (a) 13/32 (b) 1/4
tournament. They are divided into eight pairs at (c) 1/32 (d) 3/1
random. From each pair a winner is decided on the 22. A fair coin is tossed repeatedly. If the
basis of the game played between the two players.
tail appears on first four tosses, then the
Assume that all the players are of equal strength.
probability of the head appearing on the
17. Find the probability that the player S1 is fifth toss equals [IIT JEE 1998]
among the eight winners. [IIT JEE 1995]
(a) 1/3 (b) 1/6
(a) 1 (b) 1/2
(c) 1/2 (d) 1/4
(c) 1/4 (d) none of these
23. If E and F are events with P(E) ≤ P(F)
18. Find the probability that exactly one of and P(E ∩ F) > 0, then [IIT JEE 1998]
the two players S1 and S2 is among the
eight winners. [IIT JEE 1995] (a) occurrence of E ⇒ occurrence of F
(a) 1/15 (b) 2/15 (b) occurrence of F ⇒ occurrence of E
(c) 4/15 (d) none of these (c) non-occurrence of E ⇒ non-occurr
19. If E′ and F′ are complementary events ence of F
of E and F respectively and if 0 < P(E), (d) none of the above implications holds
P(F) < 1, then [IIT JEE 1998] 24. If A, B and C are three events such
(a) P(E|F) + P(E′|F′) = 1 that P(B) = 3/4 , P(A ∩ B ∩ C′) = 1/3 and
(b) P(E|F) + P(E|F′) = 1 P(A′ ∩ B ∩ C′) = 1/3, then P(B ∩ C) is equal to:
(c) P(E′|F) + P(E|F′) = 1 [IIT JEE 2003]
(d) P(E|F′) + P(E′|F′) =1 (a) 1/12 (b) 1/6
20. Seven white balls and three black balls are (c) 1/15 (d) 1/9
randomly placed in a row. The probability 25. Two numbers are selected randomly
that no two black balls are placed adjacently from the set A = {1, 2, 3, 4, 5, 6} without
equals [IIT JEE 1998] replacement one by one. The probability
(a) 1/2 (b) 7/15 that minimum of the two numbers is less
(c) 2/15 (d) 1/3 than 4 is [IIT JEE 2003]
21. If from each of the three boxes containing (a) 1/15 (b) 4/15
3 white and 1 black, 2 white and 2 black, (c) 1/5 (d) 4/5

Response Grid
1. a b c d 2. a b c d 3. a b c d 4. a b c d 5. a b c d
6. a b c d 7. a b c d 8. a b c d 9. a b c d 10. a b c d
11. a b c d 12. a b c d 13. a b c d 14. a b c d 15. a b c d
16. a b c d 17. a b c d 18. a b c d 19. a b c d 20. a b c d
21. a b c d 22. a b c d 23. a b c d 24. a b c d 25. a b c d
Probability Miscellaneous  363

2
Concept Eliminator (CE)
Ideal Time Revise your concept (Maths Olympiad
Just solve it or tougher questions)
1. The probabilities that a student passes in at random and is tossed twice. The first
Mathematics, physics and chemistry are time it shows head and the second time
m, p and c, respectively. Of these subjects, it shows tail. What is the probability that
the student has a 75% chance of passing the coin drawn is fair?
in at least one, a 50% chance of passing (a) 9m/(m – n) (b) 1/m + n
in exactly two. Which of the following (c) 9m/(8 N + m) (d) none of these
relations are true?
5. Three of the six vertices of regular
(a) p + m + c = 19/20 hexagon are chosen at random. The
(b) p + m + c = 27/20 probability that the triangle with these
(c) pmc = 1/10 three vertices is equilateral, equals
(d) pmc = 1/4 (a) 1/2 (b) 1/5
2. Eight players P1, P2, ... P8 play a knock – (c) 1/10 (d) 1/20
out tournament. It is known that whenever 6. Susmit met his fast friend Ricky. Susmit
the players Pi and Pj play, the player Pi will asked Ricky, How many children do you
win if i < j. Assuming that the players are have? Ricky replied 2, further Ricky
paired at random in each round, what is added that one of his son born on Sunday
the probability that the player P4 reaches then what is the probability that he has
the final? two sons?
(a) 35/105 (b) 36/105 (a) 1/2 (b) 14/27
(c) 1/75 (d) none of these (c) 13/28 (d) 13/27
3. An urn contains m white and n black 7. In a game show “Kaun banega Dus Crore
balls. A ball is drawn at random and is Pati” The host Mr. Kabir Khan gave the
put into the urn along with k additional guest Mr Rajesh a choice of three doors:
balls of the same colour as that of the Behind one door is a new shining car;
ball drawn. A ball is again drawn at behind the others, nothing. Mr. Rajesh
random. What is the probability that the pick a door, say No.  1, and the host,
ball drawn now is white? Mr. Kabir Khan who knows what’s
(a) m/(m + n) (b) (m – n)/(m + n) behind the doors, opens another door,
(c) 1/(m + n) (d) none of these say No.  3, which has nothing. He then
says to Mr Rajesh, “Do you want to pick
4. A box contains N coins, m of which are door No. 2?” What he should do now to
fair and rest are biased. The probability of win the car?
getting a head when a fair coin is tossed
is 1/2 , while it is 2/3 when a biased coin (a) 2/3 (b) 1/3
is tossed. A coin is drawn from the box (c) 1 (d) 0

Response Grid
1. a b c d 2. a b c d 3. a b c d 4. a b c d 5. a b c d
6. a b c d 7. a b c d
364  Theory of Counting

Solutions
Concept Deviator (CD)
1. (a) 2. (a) 3. (c) 4. (a, c) 5. (a) 6. (b)
7. (c) 8. (a) 9. (a) 10. (b) 11. (a) 12. (c)
13. (c) 14. (a) 15. (d) 16. (b) 17. (b) 18. (d)
19. (a,d) 20. (b) 21. (a) 22. (c) 23. (d) 24. (a)
25. (d)

1. (a) Lets first find the total number of ways: On solving these in-equations we
4C 4C 4C 4C 24 1 1
1+ 2 + 3 + 4 = – 1 = 15 will get –  p .
3 2
out of 15 only one is correct. 3. (c) Given that
He will get marks in three cases- P(A  B) = 0.6, P( A  B) = 0.2
Case (i) correct answer in 1st chance Now,
1/15. P( A ') + P( B′) = {1 – P(A)} + {1 – P(B)}
Case (ii) Correct answer in 2nd = 2 – {P(A) + P(B)}
chance ( 14/15)(1/14). = 2 – { P(A ∪ B) + P(A  B ) }
Case (iii) Correct answer in 3rd = 2 – (0.6 + 0.2) = 1.2.
chance ( 14/15)(13/14)(1/13). 4. (a, c) We know, P(A ∪ B) = P(A) + P(B)
So required probability is – P(A  B ) ,
Hence option (a) and (c) are correct.
1 14   1   14  13   1  1
             5. (a) Consider both the cases separately:
15 15  14  150  14  13  5
Case (i): Best if 3 games required
1  3p 1  p 1  2p
2. (a) Since, , , are probability is (3C2)(0.4)2(0.6)1 + (3C3)
4 4 2
mutually exclusive events. (0.4)3(0.6)0 = 0.288 + 0.064 = 0.352.

Therefore, all these will lie between Case (ii): Best if 5 games required
1  3p 1 p probability is (5C3)(0.4)3(0.6)2 +
0 to 1 i.e. 0   1, 0   1, (5 C )(0.4)4(0.6)1 + ( 5C )(0.4)5(0.6)0
4 4 4 5
1  2p = 0.31744.
0   1
2
Hence best of 3 is the correct option.
Probability Miscellaneous  365
6. (b) Since set A contains n elements so NowP(B/A ∪ B′)
its power set has 2n elements so Set = P(B ∩ (A ∪ B′))/P(A ∪ B′)
P can be chosen in (2n) ways, and Q
= (0.7 – 0.5)/0.8 = 1/4.
can be selected in (2n) ways.
11. (a) Since all the three are independent
So P and Q can be selected in
events hence both statement S1 and
(2 )(2 )   4 ways
n n n
S2 are true.
Now number of disjoint sets is 12. (c) Consider the two cases of getting 7
n and 8 separately,
  r 0
( n Cr )2n  r  (1  2)n  3n
Getting 7:
n n
3  3 1   6  1   1  1 1
Required probability is =    .
n            
4  4 2 36
      2 11 12 22
7. (c) Required probability is
11  6 17
 
(3/6)(2/6)(1/6) = 1/36 132 132

8. (a) Probability of getting favorable case
Getting 8:
is (4/6) then required probability is
1   5  1   1  5 1
 4  4 16            
   2 36
      2 11 72 22
6  81 55  36 193
 
9. (a) Since product of two digit number is 792 792
18 so selected numbers must be one So required probability is
among 29, 36, 63, 92. 17 91 193
  .
So probability of getting favorable 132 792 792
case is (4/100) = 1/25. 13. (c) From the given information we can
conclude that P(A ∩ B) = P(A)P(B)
Out of 4 outcomes numbers favorable
hence A and B are independent
events will be at least 3 times. Hence
events, hence A′ and B′ are also
required probability is
independent
 1  3  24   1  4 97
= 4          4 So P(A ∪ B)' = P(A′ ∩ B′) = P(A′)P(B′)
 25   25   25  25
10. (b) From the given information 14. (a) P(Exactly one of A or B occurs)
P(A) = 1 – 0.3 = 0.7 = P(A) + P(B) – 2P(A ∩ B) = p
Since P(A ∪ B′) Similarly P(C) +P(B) – 2P(C ∩ B)
= P(A) + P(B′) – P(A ∩ B′) = P(A) + P(C) – 2P(A ∩ C) = p
= 0.7 + 0.6 – 0.5 = 0.8 Hence 2[P(A) + P(B) +P(C)
366  Theory of Counting
– P(A ∩ B) – P(B ∩ C) – P(C ∩ A)] = 3p q p No. of pairs
Since P(A ∩ B ∩ C) = p2 10 1,2,3,4,5,6 6
Hence P(at least one of A, B and C) 9 1,2,3,4,5 5
= [P(A) + P(B) + P(C) – P(A ∩ B) 8 1,2,3,4,5 5
– P(B ∩ C) – P(C ∩ A)] + P(A ∩ B ∩ C) 7 1,2,3,4,5 5
6 1,2,3,4 4
3p 3 p  2 p2
=   p2  5 1,2,3,4 4
2 2
4 1,2,3 3
15. (d) Let P(3) = Probability that the
3 1,2,3 3
minimum of the number is 3
2 1,2 2
1 7
{ ( C1 )  ( C2 )} 7 1 1 1
= 
{(10 C3 )} 40 Total 38
Let P(7) = Probability that the So number of cases not favorable is
maximum of the number is 7 100 – 38 = 62.

{(1 C1 )   ( 6 C2 )} 1 And required probability is


= 
{(10 C3 )} 8 62/100 = 0.62
17. (b) Let P(S1 to be among the eight winners)
Probability that the minimum of the
= P(S1 being a pair) × P(S1 wins)
number is 3 and maximum of the
number is 7 = 1 × 1/2 = 1/2
18. (d) We can say that if S1 and S2 are in the
{(1 C1 )  (1 C1 )  (3 C1 )} 1
=  same pair then exactly one wins.
{(10 C3 )} 40
Now, If S1 and S2 are in two pairs
So required probability
separately then we can say that one
7 1 1 11 among the two (i.e S1 and S2 )
= + – =
40 8 40 40
Will be winner among the eight. If
16. (b) Given equation is x   px  q    0
2
S1 wins and S2 loses or S1 loses and
Roots of this equation is real if S2 wins. Now the probability if S1
and S2 being in the same pair and
p2  4q
one wins = P( same pair) × P( anyone
Sample space for (p, q) is win in the pair)
10 × 10 = 100 Probability of S1 , S2 being the same
Number of cases which is not pair = n(E) /n(S), where n(E ) is the
favorable is i.e. ( p   4q ) in the
2 number of ways in which 16 persons
following cases: can be divided in 8 pairs.
Probability Miscellaneous  367
Therefore, two black ball put together is equal
14 ! 16 ! to the number of ways of choosing 3
n(E ) = and n(S) = ,
(2!)7 .7! (2!)8 .8! place out of eight places. Now this
can be done in 8C3 ways therefore our
(14!)(2!)8 . 8 ! required probability = 7/15.
now  P(same pair) =
(2!)7 . 7 !. (16 )!
21. (a) Let W be for white ball and B for
= 1/15 black balls.
The probability of any one winning in we can say, P(2 white and 1 black
the pairs of S1 S2 = ( certain event) = 1 ball) = P(W1 W2 B3 or W1 B2 W3 or B1
The pairs of S1, S2 being in two pairs W2W3)
separately and S1 wins, S2 + the = 3/4 × 2/4 × 3/4 + 3/4 × 2/4 × 1/4 +
probability of S1 S2 being in two pairs 1/4 × 2/4 × 1/4 = 13/32
separately and S1loses and S2 wins 22. (c) Let E be the event that the 5th toss
14 ! results in head which is independent
7
(2!) .7 ! of the event that the 1st 4 tosses
= {1 – } × 1/2 × 1/2
16 ! result in tails.
(2!)8 .8!
Therefore, required probability is 1/2.
14 ! 23. (d) As it is given that P( E) ≤ P (F) ⇒ E
(2!)7 .7 ! ⊆ F and P( E ∩ F) > 0 ⇒ E ≠ F,
+{1– } × 1/2 × 1/2 = 7/15
16 !
Option (a )and option( b)can be obtain
(2!)8 .8!
from second relation and option (c)
Therefore, required probability
can be obtain from 1st relation.
= 1/15 + 7/15 = 8/15
24. (a) Since P(B ∩ C) = P(B) – P(A ∩ B ∩ C′) –
19. (a,d) Option (a) and (d) are correct.
P(A′ ∩ B ∩ C′) = 3/4 – 1/3 – 1/3 = 1/12
20. (b) As we can find out, The number of
25. (d) Total number of ways is (2!)(6C2) = 30
ways of placing 3 black balls without
10C Not favourable cases are 6 so favour-
any restriction is 3. As we have
able cases are 30 – 6 = 24.
total 10 places to put 10 balls in a row.
Now the number of ways in which no Required probability = 24/30 = 4/5.

Concept Eliminator (CE)


1. (c) 2. (b) 3. (a) 4. (c) 5. (c) 6. (d)
7. (a)
368  Theory of Counting
1. (c) This question is based on principle From (i) we get,
of inclusion and exclusion 1 – (m + p + c) + (pm + pc + cm) – pcm
Let X , Y and Z be the events that the   = 0.25 ...(v)
student passes in Maths , Physics Now from (iii), (iv) and (v) we get,
and chemistry.
p + m + c = 1.35 = 27/20
P(X) = m, P(Y) = p and P(Z) = c
and pmc = 1/10
and P(passing in at least one)
2. (b) As P1, P2, ... P8 (the eight Players) the
= P(X ∪ B ∪ C) = 0.75 [ given]
number of ways they can be paired
Now, 1 – P( X′ ∩ Y′ ∩ Z′) = 0.75, in four pairs = 1/4! [ (8C2)(6C2)(4C2)
P(X) = 1 – P (X′) and P(X ∪ B ∪ C)′ (2C2)] ways = 105 ways.
= P(X ∩ Y ∩ Z ) Now, at least two players certainly reach
⇒ 1 – P(X′) P(Y′) P(Z′) = 0.75 the second round in between P1 , P2 and
X, Y and Z are independent event P3, P4 can reach in the final, if exactly
therefore X′, Y′ and Z′ are also two players play against each other
independent. in between P1 , P2 and P3and remaining
player will play against one of players
1 – (1 – m)(1 – p)(1 – c) = 0.75
from P 5, P6, P7, P8 and P4 plays against
⇒  (1 – m)(1 – p)(1 – c) = 0.25  ...(i) one of the remaining three from P5 to P8.
also P( passing exactly in one subject) Now this can be possible in
= 0.4 3C
2 × 4C1 × 2C1 = 36 ways.
⇒ P( X ∩ Y ∩ Z′ ∪ X ∩ Y′ ∩ Z ∪ X′ Required probability = 36/105.
∩ Y ∩ Z) = 0.4 3. (a) Say that the first ball drawn is
⇒ P( X ∩ Y ∩ Z′) ∪ P( X ∩ Y′ ∩ Z) ∪ white
P(X′ ∩ Y ∩ Z) = 0.4 W1 ⋅ P(W1) = (m/(m + n)
pm – pmc + pc – pmc + mc – pmc = 0.4 Ball drawn in the first draw is black
...(ii) is B1 ⋅ P(B1) = n/(m + n)
the second ball drawn is white W2.
Again P(passing at least in two
subjects) = 0.5 P(W2/W1) = (m + k)/(m + n + k)

⇒ P(X ∩ Y ∩ Z′ ) ∪ P(X ∩ Y′ ∩ Z) ∪ and P(W2/B1) = m/(m + n + k)


P(X′ ∩ Y ∩ Z) ∪ P( X ∩ Y ∩ Z) = 0.5 Then required probability

⇒  (pm + pc+ mc) – pcm = 0.5  ...(iii) P(W2) = P(W1) ⋅ P(W2/W1)

From (ii) we get + P(B1) ⋅ P(W2/B1)

  (pm + pc+ mc) – 3pcm = 0.4  ...(iv) = m/(m + n)


Probability Miscellaneous  369
4. (c) Let A be the event that the coin Case (ii) 2nd children is boy and born
tossed twice, comes head at first on Sunday then sample space is
time and tail at second and B be the
1st Children (boy) 2nd Total 7
event that coin drawn is fair. Children cases
Required probability (Boy)
P(B/A) = P(A/B) ⋅ P(B)/{P(A/B) ⋅ P(B) Born on Sunday Born on
Sunday
+ P(A/B′) P(B′)} Born on Monday
= 9m/(8N + m) Born on Tuesday
5. (c) Three vertices of a regular hexagon
Born on Wednesday
can be chosen in 6C3 ways = 20 ways.
Born on Thursday
When we draw a regular hexagon,
we can see that only two triangles Born on Friday

can be formed. Born on Saturday


Then required probability = 2/20 = 1/10
1st Children (boy) 2nd Total 7
6. (d) Here we need to find sample space Children cases
1st and then we have to calculate the (Girl)
favourable cases: Born on Sunday Born on
Case (i) 1st children is boy and born Sunday
Born on Monday
on Sunday then sample space is
Born on Tuesday
1st 2nd Children (Boy) Total 7
Children cases Born on Wednesday
(boy)
Born on Thursday
Born on Born on Sunday
Sunday Born on Friday
Born on Monday
Born on Tuesday Born on Saturday
Born on Wednesday
Now what is total sample space is it
Born on Thursday
7 + 7 + 7 + 7 = 28 NO ITS NOT.
Born on Friday
Born on Saturday Be careful and see the repetition if
both the children are boy and born
Total 7 2nd Children Total 7
cases (Girl) cases on Sunday (As shown by dark cells
Born on Born on Sunday in the above table) we have counted
Sunday Born on Monday the same thing in 1st as well as 4th
Born on Tuesday table.
Born on Wednesday So sample space is 28 – 1 = 27
Born on Thursday
And favourable cases is 14 – 1 = 13
Born on Friday
Born on Saturday So required probability is 13/27.
370  Theory of Counting
7. (a) This is a famous Monty Hall Problem (Named after the host of a game show).

The correct answer is yes he should switch to door no 2.

To understand the situation let us list down all the possibilities:

Door No. 1 Door No. 2 Door No. 3 Result

Case (i) CAR Empty Empty Switch and loose

Case (ii) Empty CAR Empty Switch and win

Case (iii) Empty Empty CAR Switch and win

Case (iv) CAR Empty Empty Stay and win

Case (v) Empty CAR Empty Stay and loose

Case (vi) Empty Empty CAR Stay and loose

So probability of winning if Mr Rajesh switch is 2/3 and if not then 1/3


15
Puzzles  371

Puzzles

Topics Covered

 Puzzles

 Miscellaneous Type of Puzzles

Part A: Topic Number of Questions

Concept Applicator 10
Concept Builder 10
Concept Cracker 25
Part B: Topic Number of Questions
Concept Deviator 11
Concept Eliminator 10
Total 66
372  Theory of Counting
Part A
Puzzles is an integral part of any aptitude test There are more than 1000 types of puzzles
exam. These are basically combination of one with 1000 different concepts so lets discuss
or more concepts of logical reasoning. few good and selected puzzles.

1
Concept Applicator (CA)
Ideal Time Apply your concepts with easy and
15 Min. conceptual questions

1. A group of students is organized in two (i) North (ii) West


rows, crossing each other. If a student is (iii) South (iv) East
tenth from all ends, how many students
are there in the group? (a) (i) or (ii) (b) (i), (ii) or (iii)
(a) 27 (b) 47 (c) (ii) or (iii) (d) (iii) or (iv)
(c) 37 (d) 39 5. In a row at a bus stop, A is 9th from the
right and B is 7th from the left. They
2. A grocer sells half of the eggs that he has
and another half an egg to Anurag. Then both interchange their positions. If there
he sells half of the balance eggs and. are 20 people in the row, what will be the
another half an egg to Deepak. Then he new position of B from the left’?
sells half of the balance eggs and another (a) 11th (b) 12th
half an egg to Shivani. In the end he is (c) 13th (d) 10th
left with just 7 eggs and he claims that
6. In a game show, participants are asked
he never broke an egg. How many eggs
to build two towers of different designs
did he start with?
using plastic bricks. These plastic
(a) 66 (b) 63 bricks are of two types: Red and Blue.
(c) 33 (d) 68 Participants are given a fixed number of
3. A hunter has just returned after a day’s bricks to build these towers. The height
bird shooting. He was asked how many of each tower is measured and 2 points
birds he had in his bag. He answered, are awarded for every 10 centimetres of
“They are all sparrows but six,” all first building and 1 point is awarded for
pigeons but six, and all doves but six. every 10 centimetres of second building.
How many birds did he have in his bag?
One participant has 4 red and 3 blue
(a) 18 (b) 9 bricks left with him. If he wants to
(c) 27 (d) None of these increase the height of his first tower, he
4. Arun started walking positioning his would need 1 red and 2 blue bricks for
back towards the sun. After some time, every 10 cm. For the second tower, to
he turned left, then turned right and raise the height he would need 2 red and
then towards the left again. In which 1 blue bricks, for every 10 cm. What is
direction is he going now? the best option for him now?
Puzzles  373
(a) Raise the height of first tower by rupees more besides. She found only one
20 cm rupee with her at the end. How much
(b) Raise the height of second tower by money Ms. Rani had before entering the
20 cm office on the annual day?
(c) Raise the heights of first and (a) ` 40 (b) ` 36
second towers by 10 cm & 20 cm (c) ` 25 (d) ` 42
respectively
9. On 1st  March, Timon arrived in a new
(d)  None of the above
city and was looking for a place to stay.
7. There are two cups, one containing
He met a landlady who offered to rent
orange juice and one containing an equal
her apartment at a reasonable price
amount of lemonade. One teaspoon of
hut wanted him to pay the rent on a
the orange juice is taken and mixed with
the lemonade. Then a teaspoon of this daily basis. Timon had a silver bar of 31
mixture is mixed back into the orange inches, and an inch of the silver bar was
juice. Is there more lemonade in the exactly equal to a day’s rent. He agreed
orange juice or more orange juice in the to pay an inch of the silver bar towards
lemonade? the daily rent. Timon wanted to make the
(a) More orange juice in the lemonade minimum number of pieces of the silver
(b) More lemonade in the orange juice bar hut did not want to pay any advance
rent. How many pieces did he make ?
(c) Equal amount of each juice between
the two cups (a) 5 (b) 8
(d) None of the above (c) 16 (d) 20
8. All employees have to pass through 10. Little Pika who is five and half years old
three consecutive entrance doors to has just learnt addition. However, he does
enter into the office and one security not know how to carry. For example, he
guard is deployed at each door. These can add 14 and 5, but he does not know
security guards report to the manager how to add 14 and 7. How many pairs of
about those who come to office after 10 consecutive integers between 1000 and
AM. Ms. Rani is an employee of this
2000 (both 1000 and 2000 included) can
office and came late on the annual day.
Little Pika add?
In order to avoid report to the manager
she had to pay each security guard half (a) 150 (b) 155
of the money she had in her purse and 2 (c) 156 (d) 258

Response Grid
1. a b c d 2. a b c d 3. a b c d 4. a b c d 5. a b c d
6. a b c d 7. a b c d 8. a b c d 9. a b c d 10. a b c d
374  Theory of Counting

2
Concept Builder (CB)
Ideal Time Revise your concepts with questions
25 Min. medium difficulty level questions

1. Prof. Bee noticed something peculiar P A Y


while entering the quiz marks of his
M E
five students into a spreadsheet. The
spreadsheet was programmed to calculate R E A L
the average after each score was entered. M O N E Y
Prof. Bee entered the marks in a random
order and noticed that after each mark 3. The letter ‘Y’ should be
was entered, the average was always an (a) 0
integer. In ascending order, the marks (b) 2
of the students were 71, 76, 80, 82 and (c) 3
91. What were the fourth and fifth marks (d) None of the above
that Prof Bee entered? 4. There are nine letters and ten digits. The
(a) 71 and 82 (b) 71 and 76 digit that remains unutilized is:
(c) 71 and 80 (d) 76 and 80 (a) 4
2. In the figure, number in any cell is (b) 3
obtained by adding two numbers in the (c) 2
cells directly below it. For example, 9 in (d) None of the above
the second row is obtained by adding the 5. The resulting value of ‘MONEY’ is
two numbers 4 and 5 directly below it. (a) 10364
The value of X – Y is (b) 10563
68 (c) 10978
Y + 29 (d) None of the above
Directions for Questions 6 and 7:
Five horses, Red, White, Grey, Black and
9
Spotted participated in a race. As per the
Y 4 5 2 X rules of the race, the persons betting on the
winning horse get four times the bet amount
(a) 2 (b) 3 and those betting on the horse that came in
second get thrice the bet amount. Moreover,
(c) 4 (d) 5 the bet amount is returned to those betting
Directions for Questions 3 to 5: Substitute on the horse that came in third, and the
different digits (0, 1, 2, ... 9) for different rest lose the bet amount. Raju bets ` 3000,
letters in the problem below, so that the ` 2000, ` 1000 on Red, White and Black horses
corresponding addition is correct and it results respectively and ends up with no profit and no
in the maximum possible value of MONEY. loss.
Puzzles  375
6. Which of the following cannot be true? 9. Each family in a locality has at most two
(a) At least two horses finished before adults, and no family has fewer than 3
spotted children. Considering all the families
(b) Red finished last together, there are more adults than
(c) There were three horses between boys, more boys than girls, and more girls
black and spotted than families. Then the minimum possible
(d) There were three horses between number of families in the locality is
white and red (a) 4 (b) 5
7. Suppose, in addition, it is known that
(c) 2 (d) 3
Grey came in fourth. Then which of the
following cannot be true? 10. My bag can carry no more than ten
(a) Spotted came in first books.I must carry at least one book each
(b) Red finished last of management, mathematics, physics
(c) White came in second and fiction. Also, for every management
(d) Black came in second book I carry I must carry two or more
fiction books, and for every mathematics
8. Three Englishmen and three Frenchmen
work for the same company. Each of book I carry I must carry two or more
them knows a secret not known to others. physics books. I earn 4, 3, 2 and 1 points
They need to exchange these secrets for each management, mathematics,
over person-to-person phone calls so that physics and fiction book, respectively, I
eventually each person knows all six carry in my bag. I want to maximise the
secrets. None of the Frenchmen knows points I can earn by carrying the most
English, and only one Englishmen knows appropriate combination of books in my
French. What is the minimum number bag. The maximum points that I can
of phone calls needed for the above earn are:
purpose?
(a) 20 (b) 21
(a) 5 (b) 10
(c) 9 (d) 15 (c) 22 (d) 23

Response Grid
1. a b c d 2. a b c d 3. a b c d 4. a b c d 5. a b c d
6. a b c d 7. a b c d 8. a b c d 9. a b c d 10. a b c d
376  Theory of Counting

3
Concept Cracker (CC)
Ideal Time Boost up your confidence with good
40 Min. questions

1. Five persons with names P, M, U, T 3. Eighty kilograms (kg) of store material


and X live separately in any one of the is to be transported to a location
following: a palace, a hut, a fort, a house 10 km away. Any number of couriers can
or a hotel. Each one likes two different be used to transport the material. The
colours from among the following: blue, material can be packed in any number of
black, red, yellow and green. U likes red units of 10, 20, or 40 kg. Courier charges
and blue. T like black. The person living are ` 10 per hour. Couriers travel at the
in a palace does not like black or blue. P speed of 10 km/hr if they are not carrying
likes blue and red. M likes yellow. X lives any load, at 5 km/hr if carrying 10 kg,
in a hotel. M lives in a: at 2 km/hr if carrying 20 kg and at 1 km/hr
(a) hut (b) palace if carrying 40 kg. A courier cannot carry
more than 40 kg. of load. The minimum
(c) fort (d) house
cost at which 80 kg of store material can
2. There are ten animals–two each of lions, be transported to its destination will be:
panthers, bison, bears, and deer–in a zoo.
(a) ` 180 (b) ` 160
The enclosures in the zoo are named X, Y,
Z, P and Q and each enclosure is allotted (c) ` 140 (d) ` 120
to one of the following attendants:
Direction for Questions 4 and 5:
Jack, Mohan, Shalini, Suman and Rita.
Two animals of different species are In a game played by two people there were
housed in each enclosure. A lion and a initially N match sticks kept on the table. A
deer cannot be together. A panther cannot move in the game consists of a player removing
be with either a deer of a bison. Suman either one or two matchsticks from the table.
attends to animals from among bison, The one who takes the last matchstick loses.
deer, bear and panther only. Mohan Players make moves alternately. The player
attends to a lion and a panther. Jack who will make the first move is A. The other
does not attend to deer, lion or bison. X, player is B.
Y, and Z are allotted to Mohan, Jack and
Rita respectively. X and Q enclosures 4. The smallest value of N (greater than 5)
have one animal of the same species. Z that ensures a win for B is
and P have the same pair of animals. (a) 7 (b) 6
The animals attended by Shalini are: (c) 10 (d) 8
(a) bear and bison 5. The largest of N (less than 50) that
(b) bison and deer ensures a win for B is
(c) bear and lion (a) 46 (b) 47
(d) bear and panther (c) 48 (d) 49
Puzzles  377
6. Consider the following set of three A maximum number of volunteers are involved
statements: in the FR project. Among them, the number
(i) There are three statements in this of volunteers involved in FR project alone
set. is equal to the volunteers having additional
involvement in the ER project.
(ii) Two of them are not true.
The number of volunteers involved in the
(iii) Your IQ score will go up by 20 if you
ER project alone is double the number of
learn to play game Z.
volunteers involved in all the three projects.
For these three statements to be
17 volunteers are involved in the TR project.
consistent:
The number of volunteers involved in the TR
(a) Only statements (ii) and (iii) must
project alone is one less than the number of
be true.
volunteers involved in ER project alone.
(b) Statement (iii) may be either true or
Ten volunteers involved in the TR project are
false
also involved in at least one more project.
(c) Statements (ii) and (iii) must be
false.
8. Based on the information given above,
(d) Statement (iii) must be false. the minimum number of volunteers
7. Three married couples on a journey come involved in both FR and TR projects, but
to a river where they find a boat that not in the ER project is:
cannot carry more than two persons at (a) 1 (b) 3
a time. An additional condition is that a
(c) 4 (d) 5
lady cannot be left on either bank where
other men are present, without her 9. Which of the following additional
husband. information would enable to find the
exact number of volunteers involved in
(a) These people will not be able to cross
various projects?
the river.
(a) Twenty volunteers are involved in
(b) They will be able to cross the river in
FR.
9 steps.
(b) Four volunteers are involved in all
(c) They will be able to cross the river in
the three projects.
10 steps.
(c) Twenty three volunteers are involved
(d) They will be able to cross the river
in exactly one project.
in 13
(d) No need for any additional
Directions for Questions 8-11 on the basis information.
of the information given below: 10. After some time, the volunteers who
Help Distress (HD) is an NGO involved in were involved in all the three projects
providing assistance to people suffering were asked to withdraw from one project.
from natural disasters. Currently, it has 37 As a result, one of the volunteers opted
volunteers. They are involved in three projects: out of the TR project, and one opted out
Tsunami Relief (TR) in Tamil Nadu, Flood of the ER project, while the remaining
Relief (FR) in Maharashtra, and Earthquake ones involved in all the three projects
(ER) in Gujarat. Each volunteer working with opted out of the FR project. Which of the
Help Distress has to be involved in at least following statements, then, necessarily
one relief work project. follows?
378  Theory of Counting
(a) The lowest number of volunteers is 15. How many of the cubes have at least 2
now in TR project. faces painted?
(b) More volunteers are now in FR (a) 104 (b) 144
project as compared to ER project. (c) 120 (d) None of these
(c) More volunteers are now in TR Directions for Questions 16 - 20:
project as compared to ER project.
In a family, Ishita is the granddaughter of
(d) None of the above Ashima. Deepika is the mother of Hannah.
11. After the withdrawal of volunteers, as Chetan is the son of Anil. Rohini is the mother
indicated in previous Question, some of Ishita. Deepika is the sister of Vivek and
new volunteers joined the NGO. Each Chetan. Nilesh has two children, Gauri and
one of them was allotted only one project Hannah. Elesh is the only grandson in the
in a manner such that, the number of family. Chetan is not married. Rohini is the
volunteers working in one project alone daughter-in-law of Anil.
for each of the three projects became 16. Who is married to Rohini?
identical. At that point, it was also found
that the number of volunteers involved in (a) Anil (b) Nilesh
FR and ER projects was the same as the (c) Chetan (d) Vivek
number of volunteers involved in TR and 17. Who is the daughter of Anil?
ER projects. Which of the projects now
has the highest number of volunteers? (a) Gauri (b) Hannah

(a) ER (c) Ishita (d) Deepika

(b) FR 18. Who is the son-in-law of Ashima?

(c) TR (a) Elesh (b) Vivek


(d) cannot be determined (c) Nilesh (d) Anil
19. How many Children does Deepika have?
Direction for Questions number 12 to 15:
(a) One (b) Two
216 cubes of similar size are arranged in the
(c) Three (d) None
form of a bigger cube (6 cubes on each side, i.e.,
6 × 6 × 6) all the exposed surfaces are painted. 20. Who is the father of Ishita?
12. How many of the cubes have 0 faces (a) Nilesh (b) Vivek
painted? (c) Elesh (d) Anil
(a) 64 (b) 125 Direction for Questions 21 to 25 are based on
(c) 27 (d) None of these the following:
13. How many of the cubes have 2 faces Sourav’s Fish Salon serves a special Friday
painted? night seafood banquet consisting of seven
(a) 144 (b) 125 courses - hilsa, pomfret, Indian shrimp, rahu,
kingfish, lobster, and bhetki. Diners are
(c) 96 (d) None of these
free to select the order of the seven courses,
14. How many of the cubes have at most according to the following conditions:
2 faces painted?
The kingfish is served sometime after rahu.
(a) 208 (b) 144 Exactly one course should be served between
(c) 210 (d) None of these the pomfret and the Indian shrimp.
Puzzles  379
The lobster is served some time before the 23. Which one of the following would make
pomfret. it possible to determine the EXACT
The kingfish is served either fifth or sixth. ordering of the courses?
The hilsa is served second. (a) Pomfret is the fourth course served
21. Which one of the following sequences (b) Indian shrimp is the fifth course
would make for an acceptable banquet? served
(a) rahu, hilsa, lobster, bhetki, pomfret, (c) Kingfish is the sixth course served
kingfish, Indian shrimp (d) Lobster is the first course served
(b) rahu, hilsa, bhetki, pomfret, kingfish, 24. If kingfish is the sixth course served, then
which one of the following CANNOT be
Indian shrimp, lobster
true?
(c) lobster, hilsa, pomfret. rahu,
(a) Rahu is the fifth course served
kingfish, Indian shrimp, bhetki
(b) Indian shrimp is the seventh course
(d) obster, hilsa, rahu. kingfish. pomfret, served
bhetki, Indian shrimp (c) Pomfret is the fifth course served
22. If kingfish is the fifth course served, then (d) Lobster is the third course served
which one of the following MUST BE
25. If Bhetki is the third course served, which
true? one of the following MUST BE true?
(a) Pomfret is the third course served (a) Pomfret is the fourth course served
(b) Indian shrimp is the fourth course (b) Kingfish is the fifth course served
served (c) Rahu is the first course served
(c) Bhetki is the seventh course served (d) Indian shrimp is the seventh course
(d) Lobster is the first course served served

Response Grid
1. a b c d 2. a b c d 3. a b c d 4. a b c d 5. a b c d
6. a b c d 7. a b c d 8. a b c d 9. a b c d 10. a b c d
11. a b c d 12. a b c d 13. a b c d 14. a b c d 15. a b c d
16. a b c d 17. a b c d 18. a b c d 19. a b c d 20. a b c d
21. a b c d 22. a b c d 23. a b c d 24. a b c d 25. a b c d
380  Theory of Counting

Solutions
Concept Applicator (CA)

1. (c) 2. (b) 3. (b) 4. (b) 5. (b) 6. (d)


7. (c) 8. (b) 9. (a) 10. (b)

1. (c) Suppose, the student is standing on 3. (b) Let the number of sparrows, pigeons
the centre as shown in the diagram. and doves be a, b and c respectively.
Then from the given condition a + b,
b + c = 6, and c + a = 6 on solving
9
we will get a = b = c = 3. The total
number of birds is 9.
4. (b) Arun’s back was positioned towards
9 the sun.Thus, depending on the time
(10) 9 of the day. Let say arun’s back is the
east direction.
W
9

S N
Hence, the total number of students
in the group = (9 + 9 + 9 + 9 + 1) = 37.
2. (b) Its better to solve this type of East      E
question from the end. As per the Finally he is going to south direction.
given condition after selling half an Now, assume that Arun’s back in the
egg more than half the number of south direction
eggs to Shivani, the grocer was left E
with 7 eggs. Therefore, before selling
eggs to Shivani the grocer had
(7 + 1/2)2 i.e., 15 eggs.

So, after selling half an egg more than
half the number of eggs to Deepak, the Finally he is going to west direction.
grocer was left with 15 eggs. Now, assume that Arun’s back in the
west direction.
Therefore, before sellings eggs to
Deepak, the grocer had (15 + 1/2)2 N S
i.e., 31 eggs. W
Similarly, before selling eggs to So, finally he is going in the North
Anurag (i.e at the start) the grocer direction.So, he can go either in
had (31 + 1/2) 2 i.e., 63 eggs. North or the South direction.
Puzzles  381
5. (b) After they interchange their (1/2x + 2) + 1/2 (1/2x – 2) + 2 +
positions, B is 9th from the right. 1/2 ( x/4 – 3 ) + 2 + 1 = x
Also, there are 20 people in the row. After solving, x = ` 36.
So, B’s position is 12th from the left.
9. (a) The parts should be made of lengths:
6. (d) As per the given conditions, if we
1, 2, 4, 8, 16. On day 1 he will give
have 4 red and 3 blue bricks then
first piece
the best possible option is to raise 10
cm for both the towers, so 3 points On day 2 he will take first piece back
will be awarded. Now evaluate the and will give second piece of length 2.
options one by one. Similar pattern continues
Option (a)–2 red and 4 blue bricks, Thus minimum parts = 5.
but it is not a possible case hence 10. (b) He can add 30 numbers in each
ruled out. group
Option (b) gives as 2 points, so it 1000 – 1099, 1100 – 1199,
is less then what can be achieved.
1200 – 1299, 1300 – 1399,
Option (c) since as per the given
condition we requires 5 red bricks 1400 – 1499, i.e., 150 numbers.
hence it is not possible. He can also add 
7. (c) Equal amount of each juice between 1099 + 1100, 1199 + 1200
the two cups. 1299 + 1300, 1399 + 1400
1499 + 1500 and 1999 + 2000 i.e.
8. (b) Let at first Ms. Rani had x rupees
6 more numbers
with her.Then, equation will be
Total numbers = 156.

Concept Builder (CB)

1. (c) 2. (c) 3. (d) 4. (d) 5. (d) 6. (d)


7. (c) 8. (c) 9. (d) 10. (c)

1. (c) We can solve this question by 68


elimination. Here we need to find
the last 2 numbers. As per the Y + 29
given condition after putting the 3
numbers average is an integer hence 9 7 X+2
the sum of three numbers must be
Y 4 5 2 X
divisible by 3.
Continue filling the boxes from the
Option (a)—in this case the 1st 3
bottom, we get the above one.
numbers are 76, 80 and 91 and their
sum 241 and it is not divisible by 68
3 hence it is ruled out. Similarly, Y + 29
option (b), and (d) are eliminated.
16 X + 9
Option (c) is correct.
9 7 X+2
2. (c) We have two variable here and if
we know the numbers at the bottom Y 4 5 2 X
most row then we can calculate the Proceeding upward we get the above
numbers for any row. one. Then, Y + 29 = 16 + X + 9 or X – Y = 4.
382  Theory of Counting
3. (d) Summation of ‘E’ and ‘L’ must give Let red got 3rd rank then his return
zero in the end so that ‘y’ comes as it = 3000
is in the fourth row. (E + L =10) The Now he can end up with No-Profit-
correct assignment of digits is No-Loss if lose his amount on other
725 two horses, i.e., white or black come
either 4th or 5th rank. This is a
16
possible condition.
9624 Let red got 4th or 5th rank in that
10365 case he will lose ` 3000.
4.
(d) From the solution of last question. So his gain from other two should be
5.
(d) From the solution of last question. ` 3000.
6.
(d) The maximum amount that he has That is possible in two cases
Case (i) white got 2nd rank then
invested is ` 3000, on red horse, so
return = 3 × 2000 = 6000.
let us start eliminating from Red
And black got either 4th or 5th rank.
horse,
Case (ii) white got 3rd rank then
Let red got 2nd rank then his return
= 3 × 3000 = 9000 return = ` 2000
No matter what rank White and And black got 1st rank then return
Black take he cannot end up with = 4000
No-Profit-No-Loss We can sum up this result in tabular
Hence this condition ruled out. form

Amount Invested Rank in case (i) Rank in case (ii) Rank in case (iii)
Red 3000 3 4/5 4/5
White 2000 4/5 2 3
Black 1000 2 4/5 1
Now from this table we can conclude Let us assume that the three English
that option (d) is correct. men are E1, E2, E3 and the Frenchmen
7. (c) If grey came 4th then case (ii) from are F1, F2, F3 respectively. Let us
the table is not possible, and white assume that E2 is the only English
cannot come 2nd. man who knows French. Since our
8. (c) aim is to use the minimum number
1 2 of calls, and there is only one possible
communicator (i.e., E2) between the
two groups, all the information to be
exchanged between the two groups
E1 E2 E3
can be done with a single call from
6 7 F2 to E2. Before this E2 should get
the secrets from E1 and E3 and F2
5 should get the secrets from F1 and F3.
Hence four calls are used till now,
two in each group. (Shown in the
8 9 diagram, from 1–4).
F1 F3
After this, E2 calls up F2 and both
F2 exchange the secrets of their
groups.
3 4
Puzzles  383
Now E2 and F2 knows all the secrets 10. (c) For every Mathematics book I have
o. So now E2 uses two calls (to E1 to carry 2 more physics books and
and E3 calls 6 and 7 shown in the points earned with the combination
diagram) to convey the French
of these three books are 4 + 2 × 1
secrets. Same is the case with F2.
= 6 points (for 3 books) For every
Total minimum number of phone
calls = (2 + 2) + 1 + (2 + 2) = 9 (as management book I have to carry 2
shown in the diagram). more fiction books and points earned
9. (d) Since: with this combination I will earn
Adults (A) > Boys (B) > Girls (G) > 3 + 2 × 2 = 7 points (for 3 books).
Families (F) So to maximize the points I will
Going by the options, when we take carry 2 sets of management books
the min. no. of families as 2, then the and one set of mathematics book
max.possible no. of adults can be 4. and total points earned is 7 × 2 + 6 =
Since in any family there should be at 20 (for 9 books), now as mathematics
least 2 boys and a girl, which means and management book I cannot
4 boys in 2 families, and violates our select as these books come in a set of
condition of Adults > Boys.
3 books so the 10th book I will select
When we take min. no of families physics book and total points earned
as 3, then all the conditions are
= 22 + 2 = 24.
satisfied.

Concept Cracker (CC)


1. (b) 2. (c) 3. (b) 4. (a) 5. (d) 6. (b)
7. (b) 8. (c) 9. (a) 10. (b) 11. (a) 12. (a)
13. (c) 14. (a) 15. (b) 16. (d) 17. (d) 18. (c)
19. (b) 20. (b) 21. (a) 22. (d) 23. (b) 24. (a)
25. (c)

1. (b) From the table we have eliminated


Palace Hut Fort House Hotel
all four except M for palace, hence M
will stay in palace.
P (Blue & × 2. (c) Write down the information given
Red)
L → –D (Lion and Dear cannot be
M (Yellow) together)
U (Red or × P → –D and –Bs (A Panthor cannot
Blue) be with a Dear or a Bison)
T (Black) × Suman → – L
X × × × × √
Mohan → L + P
Jack → P or B
384  Theory of Counting
Hence we can short out following Solution for question number 4 to 5
table
Students please note that the best way to
Keeper Animal Enclosure answer this question is by finding generally
what would ensure a win for B. If B has to win,
Mohan L + P (given) X (given)
A has to pickup the last matchstick. This can
Jack B+P Y (given) be forced upon A if there are 2 or 3 matchsticks
Shalini B+L Q left on the table when it is B’s turn. As then,
B could pickup 1 or 2 matchsticks and force
Suman D+B P
upon A to pickup the last one. For this to
Rita D+B Z (given) happen there should always be odd number of
matchsticks initially. Eg. If there are 7 match
Since Z and P have the same pair of
sticks initially any of the following combinations
animals, the other 3 enclosures viz.,
X, Y and Q must contain the other 3 will leave either 2 or 3 matchsticks on the table
species within themselves. So apart when it is B’s turn.
from Lion and Panther, Bear must
be third species in X, Y, Q. A B B A
Jack gets Bear and Panther as he 1 2 3 4 5 6 7
doesn’t handle lion (given) that
leaves. Q with Lion and Bear; which A B B A A
goes to Shalini as Suman doesn’t 1 2 3 4 5 6 7
handle lion (given).
3. (b) When it carries 10kg, then speed is A A B A
5 kmph so time taken 2 hr, and cost 1 2 3 4 5 6 7
for 2 hr is ` 20, so if we send 8 couriers
together then it will transfer 80 kg
A B B A A
and costs 20 × 8 = 160.
When it carries 20 kg, then speed is 2 1 2 3 4 5 6 7
kmph so time taken 5 hr, and cost for Hence the smallest value of N
5 hr is ` 50, so if we send 4 couriers (greater than 5) to ensure a win for
together then it will transfer 80 kg
and costs 50 × 4 = 200. B is 7. Also the largest value of N (less
When it carries 40 kg, then speed than 50) to ensure a win for B is 49.
is 1kmph so time taken 10 hrs, and 4. (a) 5. (d)
cost for 10 hrs is ` 100, so if we send
2 courier together then it will 6. (b)
transfer 80 kg and costs 100 × 2 = 200. 7. (b) Let the three couples be represented
There are some more combination as follows.
but all of them has to be more than
` 160, hence ` 160 is the minimum. (M1, F1), (M2, F2), (M3, F3)
Puzzles  385
The following table represent the steps.

Set Up People on the People People Travelling People on the


Side Travelled From Other Side Other Side
1 M1 M2 M3 F3 F1 F2 — —
2 M1 M2 M3 F3 — F2 F1
3 M1 M3 F3 M2 F2 — F1
4 M1 M3 F3 — F1 M2 F2
5 M1 F1 M3 F3 — M2 F2
6 M1 F1 — F3 M2 F2 M3
7 F3 M1 F1 — M2 F2 M3
8 F3 — M3 M1 F1 M2 F2
9 — M3 F3 — M1 F1 M2 F2

So, 9 steps are required. Since 17 volunteers are involved in


the TR project and 10 of them are
Answers from 8 – 11
involved in at least one other project,
8. (c) FR project has the maximum 7 of them are involved in TR project
number of volunteers. Let number of alone.
volunteers involved in all the three Hence 2g – 1 = 7 or g = 4.
projects be g. Then the number of
Now it is given that the number
people involved in ER project alone of volunteers in FR alone is the
will be 2g, and number of volunteers same as those who have additional
in TR alone will be 2g – 1. involvement in ER hence b = d + 4
Now total of TR is 17 hence a + c = 6.
TR = 17 FR Total volunteer are 37 hence 8 + 7 +
4 + a + b + c + d = 37 or a + b + c + d
2g – 1 = 7 a b=8
= 18, but a + c = 6 hence b + d =12,
but b = d + 4, hence 2d + 4 = 12 or d = 4,
b = 8.
g=4
Since, FR = 16 + a has a maximum
number of volunteers hence it must
c d=4

be more than ER = 16 + c or TR = 17.


Hence 16 + c > 16 + a, or c > a but
2g = 8
a + c = 6.
Hence minimum possible value of
c = 4.
ER
386  Theory of Counting
9. (a) If it is known that twenty volunteers 11. (a) Since the addition of volunteers was
are involved in FR i.e., 16 + a , then such that they were allotted to only
we can find the value of ‘a’ and ‘b’ one projects each and the number
and hence the number of volunteers of volunteers working in one project
in each project. alone for each of the three projects
10. (b) The new Venn diagram becomes became equal, let us take it as x.
Here, the number of volunteers in
TR = 17 FR
(FR+ ER) and (TR + ER) will not be
affected.Given (FR + ER) = (TR + ER).
Hence (4 + 1 + 0) = c + 2 or c = 3 and
a+1 hence a = 3
2g–1 = 7 b=8
g=4 Also FR = (a + 1) + x + (4 + 1) = x+ 9
ER = (c + 2) + x + (4 + 1) = x + 10
c+2 d=4+1=5
TR = (c + 2) + x + (a + 1) = x + 9
Hence ER has the highest number of
volunteers.
2g = 8
Solution from 12-15

ER Since total number of cubes is


If one volunteer withdraw of the 216 =  63 hence in the formula we
TR project among the four who are will substitute n =6
involved in all the projects, he will be
12. (a) Number of the cubes with 0 faces
working only on FR and ER, So FR
3
+ ER becomes 4 + 1 = 5. Similarly, painted is (6 − 2) =  43 =  64 .
when one opts out of ER project,
13. (c) Number of the cubes with 2 faces
FR + TR becomes a + 1 and when
2
two opt out of FR project among the painted is 6 (6 − 2) =  6   × 1
  6 = 96 .
four, TR + ER becomes c + 2. 14. (a) At most 2 faces painted means
From the previous question 3 < a ≤ 6. number of cubes with 0 face painted
FR can be 18, 19 or 20. + number of cubes with 1 face
Also, since 0 ≤ b ≤ 3, ER can be 15, painted + number of cubes with 2
16, 17. face painted = 64 + 48 + 96 = 208.
Now the possibilities are (FR, ER, 15. (b) At least 2 faces painted means
TR) = (18, 17, 16) or (19, 16, 16) or number of cubes with 2 face painted
(20, 15, 16). + number of cubes with 3 face
We can conclude that FR > ER. painted = 96 + 8 = 104.
Puzzles  387
Tree of question 16 - 20

Anil Ashima

Vivek Chetan
Rohini Deepika Nilesh

Eshita Hannah
Elesh Gauri

16. (d) 17. (d) 18. (c) 19. (b) 20. (b)

Solution from 21 to 25 21. (a) Option (b) is wrong as lobster has to


before pomfret, option (c) is wrong
There are 7 courses - conditions are as
as there are two fishes in between
follows:
Pomfret and indian shrimp and (d)
1st condition: The kingfish is served is wrong because kingfish should be
sometime after rahu. in 5th and 6th .
2nd condition: Exactly one course should be 22. (d) From the given condition only
served between the pomfret and the Indian possible is lobster serve in first.
shrimp.
23. (b)
3rd condition: The lobster is served some 24. (a) If rahu is the 5th course served then
time before the pomfret. pomfret and indianshrimp on the
4th condition: The kingfish is served either 1st and 3rd place but lobster should
fifth or sixth. come before pomfret
5th condition: The hilsa is served second. 25. (c)
388  Theory of Counting
Part B
1
Concept Deviator (CD)
Ideal Time Revise your concept (IIT advance or
5 Min. tougher questions)
Per Question

1. Hasan and Susmit were describing the have to take out before you’re guaranteed
result of the test conducted at Pioneer to have at least one matching pair?
Career Kolkata. There were three (a) 4! (b) 2/20
contestants RAM, SHYAM and JODU. (c) 3 (d) 4
Hasan reported that RAM won the 4. Hasan plays a solo dice game where, on
competition, while SHYAM came in second. each turn, a normal pair of dice is rolled.
The score is calculated by taking the
Susmit reported that JODU won the product, rather than the sum, of the two
competition, while RAM came in second. numbers shown on the dice.
In fact, neither Hasan nor Susmit had On a particular game, the score for the
given a correct report of the results of second roll is five more than the score
the science exhibition. Each of them had for the first; the score for the third roll
given one correct statement and one false is six less than that of the second; the
statement. What was the actual placing score for the fourth roll is eleven more
of the three contestants? And who got 1st than that of the third; and the score
rank? for the fifth roll is eight less than that
of the fourth. What was the score for
(a) Jodu (b) Shyam each of these five throws respectively?
(c) Ram (d) None of these (a) 10,15,9,12,20 (b) 10,15,9,20,12
2. Amit shares his experience of a family (c) 10,9,20,15,12 (d) 10,20,15,12,9
reunion where the following people 5. In a promotional offer, a soap company
were present: one grandfather, one offers a soap free of cost in return of 5
grandmother, two fathers, two mothers, wrappers of the soap. If Anushree has
four children, three grandchildren, 241 wrappers, then how many soaps she
one brother, two sisters, two sons, can get free of cost?
two daughters, one father-in-law, one (a) 48 (b) 60
mother-in-law, and one daughter-in-law.
(c) 65 (d) None of these
But not as many people attended as it
6. Two trains travel toward each other on
sounds. How many were there, and who
the same track, beginning 100 miles
were they?
apart. One train travels at 40 miles per
(a) 4 (b) 10 hour; the other travels at 60 miles an
(c) 7 (d) None of these hour. A bird starts flying at the same
3. Your Shoes drawer contains 10 pairs of location as the faster train, flying at
white socks and 10 pairs of black socks. a speed of 200 miles per hour. When it
If you’re only allowed to take one socks reaches the slower train, it turns around,
from the drawer at a time and you can’t flying the other direction at the same
see what color of socks you’re taking until speed. When it reaches the faster train
you’ve taken it, how many socks do you again, it turns around — and so on. This
Puzzles  389
process continues till both the trains Figure out that maximum number.
collide, how far will the bird have flown? 9. 16 playing cards are given to a player.
(a) 100 miles (b) 200 miles The cards are four Aces, four Kings,
(c) 450 miles (d) None of these four Jacks and four Queens. He is told
7. One Sunday morning, Harsh was to make a 4 × 4 grid of these cards such
returning from office. He over heard a
that no two same cards should be in
conversation between his neighbor’s kids
the same line horizontally, vertically or
Kid 1: How old are you? diagonally. The player tried but couldn’t
Kid 2: I was eight years old two days ago do it, and then he is given some Joker
and next year I shall be eleven. cards and is told to achieve the above by
After listening to this reply the 1st kid replacing some of the cards with a Joker
was puzzled. such that the Joker can come in line with
any other card without a conflict. If for
Can you explain how Harsh helped the
each joker card he has to pay ` 10 then
1st kid by figuring out the 2nd kid’s reply?
what is his actual income if he receive
8. A farmer goes to the forest everyday to ` 100 after completion of the task?
collect woods for burning purpose. One
10. Can you make 120 with 5 zeros and
day he found 7 straight sticks which can
mathematical operators?
be placed in different orientations to
make squares and right angled triangles. 11. If distance between 1st and 5th
He then realizes that the product of the stations, when all stations are spaced
number of squares and the number of equidistantly, is 10 km, then what will
right angled triangles is maximum. be the distance of 17th station from 1st
station?

Response Grid
1. a b c d 2. a b c d 3. a b c d 4. a b c d 5. a b c d

2
Concept Eliminator (CE)
Ideal Time Revise your concept (Maths Olympiad
Just solve it or tougher questions)

1. A king called all his sons and gave them 2. A king called all his sons and gave them
a certain number of gold coins such that certain number of gold coins such that
no two sons have the same number of no two sons have same number of gold
gold coins. If the king has 7 sons then coins. If king has 200 coins then find
what could be the minimum number of the maximum number of sons that king
gold coins with king. has.
(a) 32 (b) 63 (a) 9 (b) 8
(c) 127 (d) None of these (c) 7 (d) None of these
390  Theory of Counting
3. This question is based on Maths of the 500 bottles is poisonous and even
Olympiad: a single drop of this drink can kill any
Five students A, B, C D and E took part one in 10 minutes. Mr. A wants to find
in a contest. There are two predictions out the poisonous bottle with the help
for the result. of rats (means he can give drop/drops
of cold drink to rats). On minimum how
Prediction 1: Contestant would finish
many rats he has to do this check to find
in the order ABCDE. This prediction out the poisonous bottle if has only 15
was very poor and no contestant finished minutes to do this check?
in the position predicted and no two
students predicted to finish consecutively 6. From movie Chhole (Next part of
actually did so. Sholay): The main villain Thakkar
send his best 50 commandos under the
Prediction 2: Contestant would finish
leadership of Thamba to village Ramgarh.
in the order DAECB, this prediction
These 50 commandos got defeated by
was better, exactly two of the contestant
heroes Kai and Keeru. When these 50
finished in the positions predicted and
commandos came back to Thakkar then
two disjoint pairs of students predicted
he got irritated and as a punishment he
to finish consecutively actually did so.
told all these 50 commandos to participate
Determine the order in which the in the game called “In a line”.
contestant finished and find the order
of A. As per the rule of the game, each of the
participant has to stand in a line one after
(a) 2 (b) 3
the other, then Thakkar will put a cap on
(c) 4 (d) 5 each of the commando (Thakkar has only
4. This question is based on Maths two types of Hats, Red colored and Green
Olympiad: colored and it is known to everyone).
In how many ways 2 ‘+’ and 2 ‘–’ signs are They stand in such a way that the last
filled in to 4 × 4 cell where each cell can commando can see the colour of hat of all
contain maximum 1 character such that the 49 commandos ahead of him and so
each row and column can not contain on. But the commando who is standing in
same sign? the front cannot see the colour of hat of
any commando. Then Thakkar will ask
the colour of hat of commandos starting
from the last commando, if a commando
will say the correct colour then he will
be kept alive otherwise will be killed
silently (means all the commandos can
(a) 3960 (b) 3340 listen to his previous commandos but
cannot make it out whether his answer
(c) 3255 (d) None of these
was correct or not).
5. Continuation of Dhoom 2: At the end
Thakkar gave these commandos 1 hour
of Dhoom 2, Mr. A and Sunahri started a
to devise a plan and save themselves if
Dhaba and named it “Dhaba E Dhokha”.
they can.
One day Mr. A received 500 bottles of
cold drink. He came to know that one What should be the strategy these
50 commandos should make to save
Puzzles  391
maximum number of commandos and with small lock is safe. Can you help don
maximum how many commandos are in sending this confidential file? Assume
definitely safe? that they have sufficient time and safety
7. Akbar and Birbal: of the documents is more important.
Akbar made a park named Mughal Park 9. Topic—DHOOM 2:
that has 99 wells. Water in these wells is Mr. A in DHOOM 2 has planned to rob
poisonous and makes a person falls asleep/ an ancient museum that has two golden
feeling drowsiness. But the poisonous chains of length 200m each hanging from
effect of a well can be neutralized by the central hall. The height of central
another well whose number is less than hall is 200m. He has one knife with him
the 1st well. (If someone has taken water with which he can cut the chain. The
of well number 75 then its effect can be two chains are hanging through a hook
neutralized by water of any well from 1 attached to the ceiling of hall at 1 ft from
to 74). But well number 1 can be utilized each other. Being a good gymnast Mr A
by only the king Akbar. can jump from 50m height. What is the
Once Akbar challenged Birbal for a maximum length of the chain that Mr. A
competition, the condition was that can take with him?
both of them will bring a glass full of 10. Aaj ka Mahabharat:
water collected from any one well, then As time changes so do human and
exchange it and drink it. Akbar was sure humanity; and also the Pandavas.
that he will win as he has special power Aaj ka Pandavas have 100 gold coins
of using well 1.Birbal the most intelligent with them and they have to divide it
person will devise a plan so that he can among themselves. All the Pandavas are
save himself. What was his plan? extremely intelligent and greedy.
8. Topic- DON 3- Don in Need Yudhisthir being the eldest among them
In DON 3, The Don wants to send some proposes a way of distribution of 100 gold
coins, and then all the 5 Pandavas will
confidential file to his friend Roma in a
vote for or against the proposal. If half
briefcase. On the other hand Bardhan or more of the total member vote for the
and his associate are always eager to proposal then that proposition is accepted
know the plans of Don, hence Don cannot by all, if Yudhisthir fails to obtain
send anything in unlocked briefcase. The support of at least half of the members
problem is that Don is in UK but Roma (including himself) then remaining 4 will
is in Delhi, hence they have different set kill the loser Yudhisthir (After all its
of locks and keys (that means the keys Aaj ka Mahabharat), and then this
of locks that Don has is with Don only process is continued by next eldest among
and not with Roma and vice versa). Don Pandava i.e Bhima and so on.
cannot send anything in an unlocked What is the maximum number of coins
briefcase as it may have access to spies of Yudhisthir can keep without risking
Bardhan, while the locked briefcase even his life?

response Grid
1. a b c d 2. a b c d 3. a b c d 4. a b c d
392  Theory of Counting

Solutions
Concept Deviator (CD)
1. (a) 2. (c) 3. (c) 4. (b) 5. (b) 6. (b)

1. (a) Jodu won; Shyam came in second; Now finally she has 5 wrappers from
Ram came in third. this she can get 1 soap.
2. (c) Total number of peoples is 7, (actually So total number of soaps that she
a grand father is also a father). will receive is 48 + 9 + 2 + 1= 60.
3. (c) The correct answer is 3. In the worst 6. (b) Since the trains are 100 miles apart,
case, the first two socks you take and the trains are traveling toward
out will consist of one black and one each other at 40 and 60 mph, the
white shoes. The next shoes you trains will collide in one hour. The
take out is guaranteed to match one bird will have been flying for an
or the other. hour at 200 miles per hour at that
4. (b) Following results satisfy the point, so the bird will have traveled
conditions. 200 miles.
10 is the score for the first roll. 7. The kid’s birthday must fall on
15 is the score for the second roll. 31st Dec and conversation must
9 is the score for the third roll. have happened on 1st Jan (say 1st
Jan 2014), now see the following
20 is the score for the fourth roll. illustration:
12 is the score for the fifth roll.
5. (b) Since from 240 wrappers she will get Day Age of Kid
240/5 = 48 soaps now she has 1 extra Two days before 8
wrapper. (30th Dec 2013)
From 48 soaps she will get 48 Yesterday (31st Dec 2013) 9 (Birthday)
wrappers and from that she will get Today (1st Jan 2014) 9
[49/5] = 9 soaps and she will have 4
This year (31st Dec 2014) 10
extra wrappers.
Again from 9 soaps she will get 9 Next Year (31st Dec 2015) 11
wrappers so she will have 9+4 = 13 8. The following configuration gives us
wrappers maximum value of the product.
From 13 wrappers she will get [13/5]
= 2 soaps and 3 extra wrappers.
Puzzles  393
King, Queen and Jack respectively
One of the possible configurations is
as shown below:
J JKR A K
A K Q J
Q J JKR A
JKR A K Q
So he has to use 3 JKR cards hence
actual income is 100 – 30 = 70
Number of squares = 5 (4 small ones
and a bigger one formed by 4 smaller 10. (0! + 0! + 0! + 0! + 0!)! = 5! = 120
ones).
11. Let distance between two stations
Number of right angled triangles = 9 = x km
(The line that intersects all the six
lines forms a right angled triangle Then distance between 1st & 5th
with every set of perpendicular stations = 4x km
lines)
4x = 10; x = 2.5.
Product = 5 × 9 = 45.
9. Let us represent Joker by JKR and So, the distance between 1st and
let A, K, Q and J represent Ace, 17th stations = 16x = 16 × 2.5 = 40 km.

Concept Eliminator (CE)


1. (d) 2. (a) 3. (b) 4. (a)

1. (d) The numbers are of Fibonacci series Case (ii) – (DA)(CB)E but in this
1, 2, 3, 5, 8, 13, and 21 so total sequence C is at 3rd place which
number of coins is 53. is mentioned in prediction 1 hence
ruled out.
2. (a) Continuation of Fibonacci series 1,
2, 3, 5, 8, 13, 21, 34, 55, 89… Case (iii): (AE)D(CB) but in this
sequence A is at 1stplace which is
Summation of 1st 9 numbers is 142 mentioned in prediction 1 hence
and that of 1st 10 numbers is 231 ruled out.
So king has 9 sons. Case (iv) E(DA)(CB) is the correct
3. (b) Since 2nd prediction is more accurate sequence and rank of A is 3rd
and in that there are 4 consecutive 4. (a) Let us consider 1st the ‘+’ sign, we
pairs these are (DA), (AE), (EC), and can select two columns in 4C2 = 6
(CB), out of these four two of them ways and we can place two ‘+’ signs
are correct pairs so possible cases in (4 × 3) = 12 ways so two ‘+’ signs
are: can be arranged in 72 ways.
Case (i) – (DA)B(EC) but in this Similarly two ‘–’ sign can be arranged
sequence (AB) pair is together which in 72 ways
is mentioned in prediction 1 hence
ruled out.
394  Theory of Counting
Now we have to exclude the cases If only rat C die means bottle 3 is
in which both the signs are in same poisonous
cell. If only rat A and B die means bottle
When both the ‘–’ sign occupy the 4 is poisonous.
cell of ‘+’ sign then number of ways If only rat A and C die means bottle
is 72. 5 is poisonous.
When one of the ‘–‘ sign occupy the If only rat B and C die means bottle
cell of ‘+’ sign then number of ways 6 is poisonous
is 4 × 4 × 72 = 16 × 72.
If all three die means bottle 7 is
So required number of ways poisonous
= 72 × 72 – 72 – 16 × 72 = 55 × 72 = 3960.
If none of them die means bottle 8 is
5. Mr. A will do this check on only 9 poisonous
rats, as with the binary logic with
Now let us see why it works:
the help of 9 rats he can check 29 = 512
Let us assume that we have n rats
To understand the situation let us
for experiment:
assume that number of rats available
Number of bottles that is given to
is 3 then on how many bottles we can
none of the rats is nC0[ = 1 and bottle
check.
8 in above example]
1 2 3 4 5 6 7 8
Number of bottles that is given to
A √ X X √ X √ √ X
one of the rats is nC1[3C1 = 3 and
B X √ X √ √ X √ X
bottle 1, 2 and 3 in above example]
C X X √ X √ √ √ X
Number of bottles that is given to
As per the table bottle 1 (of course a
two of the rats is nC2[ 3C2 = 3 and
drop of bottle 1) is given to only rat
bottle 4, 5 and 6 in above example]
A, a drop of bottle 2 is given to rat B,
Similarly….
a drop of bottle 3 is given to rat C,
a drop of bottle 4 is given to each of Number of bottles that is given to all
rat A and B and so on a drop of bottle of the rats is nCn[ 3C3 = 1 and bottle
7 is given to all the three rats and 7 in above example]
none of them get bottle 8. So total number of bottles is
nC nC nC nC n = 2n.
If only rat A die means bottle 1 is 0+ 1+ 2 + ... + n = (1 + 1)
poisonous. 6. Out of 50 commandos 49 are
If only rat B die means bottle 2 is definitely safe.
poisonous. They will device a plan such that the
last commando (50th commando)
Puzzles  395
will see how many red coloured hats in size so we can put more than one
are in front of him if it is even then lock on the hook of briefcase)
he will sought (answer) red and if Step 3: Now Don will remove his
its odd then he will answer green. lock (still that briefcase has one pad
In this way only last means 50th lock whose key is with Roma) and
commando’s life is at stake. So 49 send it back to Roma.
commandos are definitely safe.
Step 4: Now finally Roma will remove
If 50th commando answered Red the lock and get the document.
means number of red coloured caps
9. Follow the following steps –
in front of him is even, now 49th
Step 1: he will reach on the top of
commando will count the number of
one rope and cut the other.
red coloured caps in front of him, if
it is even then he is wearing green Step 2: He will come down to the
coloured cap otherwise red coloured floor.
cap and so on. Step 3: He will join both the ropes
7. Akbar will bring water from well 1 together. (Hanging end of one rope
because Akbar will think that there with any one end of the rope that
is no antidote for poison of well 1. he cut), so now he has a single rope
with 400 m length.
Birbal can bring the water of any
well, since he knows Akbar can save Step 4: Now he will move up keeping
himself by drinking the water of one end of the rope with him.
well 1. Step 5: He will insert the loose end
So, Birbal will drink the water of of the combined rope it to the hook
well 99 before drinking the water (Similar to a thread in a needle)
brought by Akbar. In this case, the keeping the other rope intact with
water brought by Akbar will act as the hook.
an antidote. Step 6: Now he will cut the 2nd
8. There are 4 steps: rope and the final situation will be
like entire 400 m of rope is passing
Step 1: 1st don will send the brief-
through a hook 200 m up and 200 m
case to Roma with a small pad lock
down (i.e loose ends of the rope is
on it. He will keep the keys of that
hanging near ground) .
lock with himself.
Step 7: Finally he will come down
Step 2: After receiving the briefcase
holding both the ropes and pull down
Roma will put one more small pad
the entire rope.
lock on the briefcase and send it back
to Don. (Since pad locks are smaller
396  Theory of Counting
So Mr. A will take away entire 400m (The junior most). Since Sahdeba
of ropes with him. (the junior most) knows if he does
10. The five Pandavas are Yudhisthir, not vote for Arjuna, then Arjuna will
Bhima, Arjuna, Nakula and Sahdeba. loose and the batton will be passed
To understand this question try to to Nakula who is not going to give
understand the situation with a him any thing, so obviously Sahdeba
group of 2 persons then 3 persons is going to vote for Arjua.
and so on. If there were 4 persons (Bhima,
If there were 2 persons, then the Arjuna, Nakula and Sahdeba),
1st person being the most senior, Bhima would give 1 coin to Nakul,
he would just vote for himself and and Nakul knows if he does not vote
that would be 50% of the vote, so if for Bhima, then the batton will be
number of persons are two then the passed to Arjuna and he gets nothing,
senior person will keep entire 100 so obviously Nakula is going to vote
gold coins. for Bhima.

If there were 3 persons (Arjuna, Now in case of 5 persons, Arjuna (3rd)


Nakula and Sahdeba), then 1st and Sahdeba (5th) will obviously
person (Arjun) has to take at least better vote for the Yudhisthir, or
1 more vote, or convince at least one they face choosing nothing or risking
other person to join in his plan. In death.
this case Arjuna would take 99 gold So Yudhisthir will keep 98 gold coins
coins and give 1 coin to Sahdeba with him and will pass on 1 coins to
each Arjuna and Sahdeba.

You might also like